Target 2024 Current Affairs Revision Test Compilation 2024_compressed

You might also like

Download as pdf or txt
Download as pdf or txt
You are on page 1of 413

www.shankariasacademy.

com 2
www.iasparliament.com

CHENNAI |SALEM| MADURAI | COIMBATORE DELHI | BANGALORE | THIRUVANANTHAPURAM


www.shankariasacademy.com 2
www.iasparliament.com

Total number of questions directly reflected from IAS Parliament (including Target
22
2023 series)

Number of questions directly reflected from the Target Series 2023 15

Total number of questions partially reflected from IAS Parliament 20

CHENNAI |SALEM| MADURAI | COIMBATORE DELHI | BANGALORE | THIRUVANANTHAPURAM


www.shankariasacademy.com 3
www.iasparliament.com

INDEX

TEST – I ......................................................................................................4
Answer Key - Test I ..................................................................................................................................... 19

TEST – II ...................................................................................................48
Answer Key - Test - II .................................................................................................................................. 65

TEST – III ................................................................................................ 105


Answer Key – Test - III .............................................................................................................................. 123

TEST – IV ................................................................................................ 166


Answer Key - Test - IV............................................................................................................................... 182

TEST – V ................................................................................................. 225


Answer Key - Test - V ................................................................................................................................ 242

TEST – VI ................................................................................................ 284


Answer key - Test - VI ............................................................................................................................... 301

TEST – VII ............................................................................................... 341


Answer Key - Test - VII ............................................................................................................................. 358

BONUS QUESTIONS ............................................................................... 397


Answer Key - BONUS QUESTIONS ......................................................................................................... 403

CHENNAI |SALEM| MADURAI | COIMBATORE DELHI | BANGALORE | THIRUVANANTHAPURAM


www.shankariasacademy.com 4
www.iasparliament.com

TEST – I

1. Consider the following statements: c. WHO’s awareness campaign towards


eliminating breast cancer
Statement-I: Mahabali frog is endemic to Western
Ghats. d. A political wave in Latin America towards left-
wing governments
Statement-II: It has very large hind legs which allows
it to leap more distance compared to other frogs.

Select the correct answer using the codes given below: 4. Consider the following statements with respect to
Sagol Kangjei:
a. Both Statement-I and Statement-II are correct
and Statement-II is the correct explanation for 1. It is a unique traditional form of theatre native
Statement-I to Manipur.

b. Both Statement-I and Statement-II are correct 2. It is part of the UNESCO intangible cultural
and Statement-II is not the correct explanation for heritage of humanity list.
Statement-I
Which of the above statement(s) is/are incorrect?
c. Statement-I is correct but Statement-II is
incorrect a. 1 only

d. Statement-I is incorrect but Statement-II is b. 2 only


correct
c. Both 1 and 2

d. Neither 1 nor 2
2. Which of the following best describes the
term ‘Virovore’?

a. Organism that feeds on viruses 5. Consider the following statements with respect to
Asian Pacific Postal Union (APPU)
b. Viruses that are frozen in the Arctic permafrost
1. It is a restricted union of Universal Postal
c. Heterotrophs that obtain nutrients by Union (UPU), a specialized agency of the
consuming detritus United Nation (UN).

d. Producing living young instead of eggs from 2. It is headquartered in Bangkok, Thailand.


within the body
3. India is one of the members of Asian Pacific
Postal Union (APPU).

3. ‘Pink tide’, sometimes seen in the news recently, How many of the above statement(s) is/are correct?
refers to?
a. Only one
a. A women movement against the compulsory
hijab in Iran b. Only two

b. Farmer protest against the green policies of c. All three


European Union
d. None

CHENNAI |SALEM| MADURAI | COIMBATORE DELHI | BANGALORE | THIRUVANANTHAPURAM


www.shankariasacademy.com 4
www.iasparliament.com

6. Consider the following statements: a. Only one

Statement-I: Yellow Band Disease occurs in Corals b. Only two


and is not curable.
c. Only three
Statement-II: It is so named as it turns the corals
yellow before destroying them. d. All four

Select the correct answer using the codes given below:

a. Both Statement-I and Statement-II are correct 9. Which of the following is called as the “Pyramids of
and Statement-II is the correct explanation for Assam”?
Statement-I
a. Borpatra Pukhuri
b. Both Statement-I and Statement-II are correct
and Statement-II is not the correct explanation for b. Assamese Gamosa
Statement-I
c. Charaideo Maidams
c. Statement-I is correct but Statement-II is
incorrect d. Maa Kamakhya Corridor

d. Statement-I is incorrect but Statement-II is


correct
10. Consider the following pairs:

Art Form Location


7. Consider the following statements with respect to the
1. Shumang Leela – West Bengal
International Thermonuclear Experimental Reactor
(ITER): 2. Ottanthullal – Karnataka
1. It is the world’s largest experimental reactor 3. Oppana – Odisha
facility housed in China.
How many of the above pair(s) is/are correctly
2. India is one of the partner countries of ITER. matched?
3. It aims to produce electricity through nuclear a. Only one
fission reaction.
b. Only two
How many of the above statements is/are correct?
c. All three
a. Only one
d. None
b. Only two

c. All three
11. Consider the following statements with respect to the
d. None Vibrant Village Programme (VVP)

1. It is a centrally sponsored scheme for


development of villages on the northern border.
8. Which of the following is/are the natural causes for
land subsidence in Joshimath city? 2. It will cover the border areas of Himachal
Pradesh, Uttarakhand, Arunachal Pradesh,
1. Located along the tectonic fault line and falls
Sikkim and Ladakh.
within Seismic Zone-I category.
3. The Scheme will not overlap with the Border
2. Sitting atop a glacial moraine that are laid down
Area Development Programme of Ministry of
by ancient landslides.
Home Affairs.
3. Undercutting by river currents of Alaknanda
How many of the above statement(s) is/are correct?
and Dhauliganga.
a. Only one
4. Unplanned construction activities and
population pressure. b. Only two
Select the correct answer using the codes given below: c. All three

CHENNAI |SALEM| MADURAI | COIMBATORE DELHI | BANGALORE | THIRUVANANTHAPURAM


www.shankariasacademy.com 5
www.iasparliament.com

d. None 2. Salim Ali Bird Sanctuary is located on the


Island of Chorao in the Mahadayi River.

3. Kalasa-Banduri Nala project is the construction


12. “Pineapple Express”, sometimes seen in the news of dam to divert water from the Mahadayi
recently, is an example of? River.

a. Pyroclastic flow How many of the statement(s) given above is/are


correct?
b. Atmospheric River
a. Only one
c. Upwelling in Oceans
b. Only two
d. Thermohaline Circulation
c. All three

d. None
13. Consider the following statements with respect to
BharOS

1. BharOS is an indigenous mobile operating 16. Consider the following pairs:


system (OS) developed by the National
Informatics Centre (NIC). Defence Exercises Partner countries

2. BharOS is an Android Open-Source Project 1. Veer Guardian – Japan


based operating system with no Google apps or
services. 2. Cyclone – I – Egypt

Which of the statement(s) given above is/are correct? 3. Varuna – United Kingdom

a. 1 only How many of the above pair(s) is/are correctly


matched?
b. 2 only
a. Only one
c. Both 1 and 2
b. Only two
d. Neither 1 nor 2
c. All three

d. None
14. India is a part of which of the following Multilateral
Export Control Regimes (MECR)?

1. Australia Group 17. Which of the following region is best known for
“Zardalu Mango”?
2. Nuclear Suppliers Group
a. Bihar
3. Wassenaar Arrangement
b. Telangana
4. Missile Technology Control Regime
c. Uttar Pradesh
Which of the above statement(s) is/are correct?
d. Andhra Pradesh
a. All except 2

b. All except 2 and 4


18. At the commencement of the first session after each
c. All except 1 and 2 general election to the Legislative Assembly and at the
commencement of the first session of each year, the
d. All except 2 and 3 Governor shall address the Legislative Assembly and
inform the Legislature of the causes of its summons. It
is described in which of the following articles of the
Indian Constitution?
15. Consider the following statements with respect to
River Mahadayi a. Article 71
1. It rises in the Western Ghats from the Bhimgad b. Article 123
Wildlife Sanctuary and drains at Arabian Sea.

CHENNAI |SALEM| MADURAI | COIMBATORE DELHI | BANGALORE | THIRUVANANTHAPURAM


www.shankariasacademy.com 6
www.iasparliament.com

c. Article 143 22. Which of the following is/are the criterion followed
for specification of a community as scheduled tribe?
d. Article 176
1. Distinctive culture

2. Geographical isolation
19. Consider the following statements with respect to
Protection of Children from Sexual Offences (POCSO) 3. A declining or stagnant population
Act, 2012
4. Pre-agricultural level of technology
1. It was enacted in response to India’s ratification
of the UN Convention on the Rights of the Child Select the correct answer using the codes given below:
in 1992.
a. Only one
2. The act is gender neutral and is applicable to
both boys and girls. b. Only two

3. A victim must report an offence within one year c. Only three


of the abuse has occurred.
d. All four
How many of the statement(s) given above is/are
correct?

a. Only one 23. Consider the following statements with respect to


Sixth Schedule of the Indian Constitution.
b. Only two
1. It contains provisions pertaining to the
c. Only three administration of tribal areas in the States of
Assam, Meghalaya, Tripura, and Mizoram.
d. None
2. Tribal advisory councils (TACs) are formed
under this Schedule to deal with welfare and
advancement of scheduled tribes.
20. Which of the following is India’s first Constitution
literate district? 3. Under this schedule, the President is
empowered to reorganize boundaries of the
a. Kollam, Kerala tribal areas.

b. Mahe, Puducherry How many of the statement(s) given above is/are


correct?
c. Mawlynnong, Meghalaya
a. Only one
d. Nagpur, Maharashtra
b. Only two

c. All three
21. Consider the following statements with respect to
Schengen Zone d. None

1. It was signed in 1985 in response to the


exigencies of the developing Cold War.
24. Consider the following statements regarding
2. Both EU and non-EU countries can become Stockholm Conference:
members of Schengen Zone.
Statement-I: Also known as the United Nations
3. United Kingdom is a non-EU and non- Conference on Environment and Development
Schengen Country. (UNCED), the Stockholm Conference on environmental
issues was held in 1992.
How many of the statement(s) given above is/are
correct? Statement-II: One of the major results of the
Stockholm conference was the creation of the United
a. Only one Nations Environment Programme (UNEP).
b. Only two Select the correct answer using the codes given below:
c. All three

d. None

CHENNAI |SALEM| MADURAI | COIMBATORE DELHI | BANGALORE | THIRUVANANTHAPURAM


www.shankariasacademy.com 7
www.iasparliament.com

a. Both Statement-I and Statement-II are correct How many of the statement(s) given above is/are
and Statement-II is the correct explanation for correct?
Statement-I
a. Only one
b. Both Statement-I and Statement-II are correct
and Statement-II is not the correct explanation for b. Only two
Statement-I
c. All three
c. Statement-I is correct but Statement-II is
incorrect d. None

d. Statement-I is incorrect but Statement-II is


correct
28. Consider the following pairs:

Animal Sports States


25. “Senna spectabilis”, sometimes seen in the news
recently, refers to? 1. Kambala – Karnataka

a. An invasive plant species that causes food 2. Bulbul Fight – Rajasthan


shortage for the wildlife population.
3. Dhirio Bull Fight – Tamil Nadu
b. An amphibian that has the ability to retain
How many of the pair(s) given above is/are correctly
juvenile or larval traits as adults.
matched?
c. An insectivorous fish that has a high breeding
a. Only one
capacity.
b. Only two
d. An emerging multidrug-resistant fungus causing
life-threatening outbreaks. c. Only three

d. All four
26. Consider the following statements with respect to
Article 19 of the Indian Constitution
29. Which of the following countries is/are member(s)
1. It can be enforced only against the State and not of the Organisation of Islamic Cooperation (OIC)
against any private individuals.
1. India
2. Suspension of this right is permissible only
during war or external aggression. 2. Israel
How many of the above statements is/are correct? 3. Russia
a. 1 only 4. USA
b. 2 only Select the correct answer using the codes given below:
c. Both 1 and 2 a. Only two
d. Neither 1 nor 2 b. Only three

c. Only four
27. Consider the following statements with respect to d. None
Seismic Zone map of India

1. Seismic Zonation map of India is published by


the National Centre for Seismology under the 30. Which of the following best describes the term
Ministry of Earth Sciences. “Girmitya”?
2. India is classified into five seismic zones with a. A granary or a commercial depot, chiefly of grain
Zone V being the most active region. for sale
3. National Disaster Management Authority b. A public notary who attests deeds by affixing
(NDMA) is the nodal agency for monitoring his/her seal
earthquakes in and around the country.

CHENNAI |SALEM| MADURAI | COIMBATORE DELHI | BANGALORE | THIRUVANANTHAPURAM


www.shankariasacademy.com 8
www.iasparliament.com

c. Overseas allowances given to the members of 3. Madrid International Plan of Action is related
British Indian Army to the Ageing.

d. Indentured labourers transported from British How many of the statement(s) given above is/are
India to work in other colonies correct?

a. Only one

31. How First Information Report (FIR) differ from b. Only two
Chargesheet?
c. All three
1. FIR is a preliminary report whereas
Chargesheet is a final report. d. None

2. FIR can be prepared only in the case of


cognizable offences, whereas Chargesheet can
be filed for non-cognizable offences as well. 35. They are nomadic tribes living in the high-altitude
plains of eastern Ladakh. They rear pashmina goats
3. Chargesheet is a public document, whereas FIR from which they get wool for making the world-famous
is not publicly accessible. Pashmina wool.

Select the correct answer using the codes given below: Identify the tribe mentioned in the above passage.

a. Only one a. Padari

b. Only two b. Gujjars

c. All three c. Changpa

d. None d. Bakarwal

32. Etalin Hydroelectric Power Project, sometimes 36. Consider the following statements with respect to
seen in the news, is located in? Voice of Global South Summit, 2023

a. Goa 1. South Africa hosted the first edition of Voice of


Global South Summit.
b. Karnataka
2. The theme for the event is “Unity of voice, Unity
c. Himachal Pradesh of purpose”.

d. Arunachal Pradesh Which of the statement(s) given above is/are correct?

a. 1 only

33. Project-75(I), sometimes seen in the news, is b. 2 only


associated with?
c. Both 1 and 2
a. Scramjet Technology
d. Neither 1 nor 2
b. Iron Dome Technology

c. Cryogenic propulsion system


37. Which of the following countries is/are part of either
d. Air-Independent Propulsion Technology Golden Triangle or Golden Crescent?

1. Pakistan

34. Consider the following statements with respect to 2. Thailand


the World Social Report, 2023
3. Laos
1. It is published by the International Labour
Organization (ILO). 4. Vietnam

2. The theme of the report is ‘Leaving No One Select the correct answer using the codes given below:
Behind in an Aging World’.
a. All except 3

CHENNAI |SALEM| MADURAI | COIMBATORE DELHI | BANGALORE | THIRUVANANTHAPURAM


www.shankariasacademy.com 9
www.iasparliament.com

b. All except 4 41. Union Budget 2023-24 set aside an assistance of Rs.
5,300 crore for implementing the Upper Bhadra
c. All except 1 and 4 Project. It is a major lift irrigation scheme under
implementation in which of the following states?
d. All four
a. Odisha

b. Jharkhand
38. Consider the following pairs:
c. Karnataka
Rivers Flowing States
d. Uttarakhand
1. Siyom – Assam

2. Bharathpuzha – Kerala
42. Which of the following is incorrect regarding the
3. Umkar – Meghalaya UN Commission for Social Development (CSocD)?
How many of the pair(s) given above is/are correctly a. It is a functional body within the UN Department
matched? of Economic and Social Affairs (UN-DESA).
a. Only one b. It is in charge of the follow up and
implementation of the Copenhagen Declaration and
b. Only two Programme of Action.
c. All three c. Members are elected based on equitable
geographical distribution for four-year terms.
d. None
d. India has been elected as the Chair for 62nd
Session of CSocD in 2023.
39. Munroe Thuruthu Island is slowly subsiding under
water. The island is located in?
43. Consider the following statements with respect to
a. Kerala
Article 112
b. Lakshadweep
1. The word “budget” has been mentioned in
c. West Bengal Article 112 of the Indian Constitution.

d. Andaman & Nicobar 2. As per the Article, it shall be the duty of the
Union Minister of Finance to lay the statement
of estimated receipts and expenditure before
both Houses of the Parliament.
40. Consider the following statements with respect
toTrans-fats. Which of the above statement(s) is/are correct?

1. Trans-fats are a form of unsaturated fat occurs a. 1 only


in both natural and artificial forms.
b. 2 only
2. It is commonly found in packaged foods to
increase the shelf life of food items. c. Both 1 and 2

3. World Health Organization called for the global d. Neither 1 nor 2


elimination of industrially produced trans-fat
by 2030.
44. Seven priorities, termed ‘Saptarishi’, was adopted in
How many of the statement(s) given above is/are
the Union Budget 2023-24 to guide the country towards
correct?
‘Amrit Kaal’. Which of the following comes under the
a. Only one Saptarishi’s?

b. Only two 1. Youth Power

c. All three 2. Green Growth

d. None 3. Women Welfare

CHENNAI |SALEM| MADURAI | COIMBATORE DELHI | BANGALORE | THIRUVANANTHAPURAM


www.shankariasacademy.com 10
www.iasparliament.com

4. Finance Sector b. A person-to-merchant mobile payment solution


by National Payments Corporation of India (NPCI)
Select the correct answer using the codes given below:
c. A centrally sponsored scheme of Ministry of
a. Only one Education to establish over 14,500 schools in India

b. Only two d. The label launched by the government to provide


foodgrains and other essential goods to the middle
c. Only three class at subsidized rates
d. All four

48. Consider the following statements:


45. Consider the following statements with respect to Statement-I: Sickle cell anaemia is an inherited blood
Mahila Samman Savings Certificate disorder that causes the body to produce abnormally
shaped red blood cells.
1. It is a small savings scheme for women above 18
years of age. Statement-II: National Sickle Cell Anaemia
Elimination Mission aims to eliminate sickle cell
2. The minimum deposit amount is Rs.100 and anaemia by 2030.
the maximum deposit amount is Rs.2 lakh.
Select the correct answer using the codes given below:
3. This scheme has a floating interest rate which is
subject to revision every quarter. a. Both Statement-I and Statement-II are correct
and Statement-II is the correct explanation for
How many of the above statement(s) is/are incorrect? Statement-I
a. Only one b. Both Statement-I and Statement-II are correct
and Statement-II is not the correct explanation for
b. Only two
Statement-I
c. All three
c. Statement-I is correct but Statement-II is
d. None incorrect

d. Statement-I is incorrect but Statement-II is


correct
46. Consider the following statements with respect to
Urban Infrastructure Development Fund (UIDF)

1. The Fund was established through shortfalls in 49. Consider the following pairs:
Priority Sector Lending.
Dams Rivers
2. The fund will be utilized to create urban
1. Bhakra – Sutlej
infrastructure in Tier 1 & Tier 2 cities.
2. Pong – Chenab
3. It will be managed by the National Housing
Bank (NHB). 3. Pandoh – Beas
How many of the above statement(s) is/are correct? 4. Thein – Ravi
a. Only one How many of the pair(s) given above is/are correct?
b. Only two a. Only one
c. Only three b. Only two
d. None c. Only three

d. All four
47. “BharatSHRI”, often seen in the news, refers to?

a. A digital epigraphy museum set up by the 50. Consider the following statements with respect to
Archaeological survey of India (ASI) Circuit breakers

CHENNAI |SALEM| MADURAI | COIMBATORE DELHI | BANGALORE | THIRUVANANTHAPURAM


www.shankariasacademy.com 11
www.iasparliament.com

1. It is a regulatory measure that temporarily 3. Sair-e-Gulfaroshan – Jammu and Kashmir


suspends forex transactions between any two
countries. How many of the pair(s) given above is/are correctly
matched?
2. In India, the Securities and Exchange Board of
India is responsible for the formulation and a. Only one
monitoring of the circuit breaker mechanism.
b. Only two
Which of the statement(s) given above is/are correct?
c. All three
a. 1 only
d. None
b. 2 only

c. Both 1 and 2
54. Consider the following pairs:
d. Neither 1 nor 2
Types of waves Passes through

1. P Waves - Only through liquids


51. Consider the following statements:
2. S Waves - Only through solids
They are regarded as “Tigers of rivers”. Its presence
indicates good water quality and ecosystem balance. 3. Surface waves - Solid, Liquid and Gas
India is home to 3 of the 13 species found worldwide.
How many of the pair(s) given above is/are correctly
Identify the species that has been described above. matched?

a. Otters a. Only one

b. Beavers b. Only two

c. Fishing cats c. All three

d. Water Birds d. None

52. Consider the following statements with respect to 55. Consider the following statements:
Gross Domestic Climate Risk Index
1. Common law is the body of law that is created
1. It was released by German watch, a global by judges through their written opinions, rather
organisation specialising in climate risk than through statutes or Constitutions.
analysis for regions, banks and companies.
2. India is one among the country that follows
2. Aggregate Damage Ratio (ADR) computed by common law.
the index is highest for Assam followed by Bihar
and Tamil Nadu. Which of the statement(s) given above is/are correct?

Which of the statements given above is/are correct? a. 1 only

a. 1 only b. 2 only

b. 2 only c. Both 1 and 2

c. Both 1 and 2 d. Neither 1 nor 2

d. Neither 1 nor 2
56. Climate Tipping points are critical thresholds in a
system that, when exceeded, can lead to a significant
53. Consider the following pairs: change in the state of the system, often with an
understanding that the change is irreversible.
Festivals Location
1. Collapse of Greenland Ice Sheet
1. Siang Unying – Assam
2. Die-off of low latitude coral reefs
2. Attukal Pongala – Kerala

CHENNAI |SALEM| MADURAI | COIMBATORE DELHI | BANGALORE | THIRUVANANTHAPURAM


www.shankariasacademy.com 12
www.iasparliament.com

3. Abrupt thawing of Boreal Permafrost 2. Until 1964, economic survey was presented as a
part of the Union Budget.
4. Abrupt loss of Barrents Sea Winter Ice
3. Once prepared, the Survey is approved by the
5. Collapse of Labrador Sea subpolar gyre President of India.

How many of the above are referred to as climate How many of the statement(s) given above is/are
tipping points? correct?

a. Only two a. Only one

b. Only three b. Only two

c. Only four c. All three

d. All five d. None

57. Operation Sadbhavana, often seen in the news, is 60. “Chitwan Declaration”, sometimes seen in the
associated with? news, is associated with?
a. Humanitarian initiative by Indian Army to aid a. Phasing out HFCs
people affected by terrorism.
b. Rhino conservation
b. Operation of Central Bureau of Investigation
(CBI) to crackdown cyber criminals c. Access and Benefit sharing

c. Operation to help fleeing Hindus and Sikhs from d. Transportation of hazardous chemicals
the Islamic Republic of Afghanistan.

d. Operation to evacuate the Indian citizens amidst


the 2022 Russian invasion of Ukraine. 61. Which of the following statement is incorrect with
respect to “Muscat Ministerial Manifesto”?

a. It recognised the need to address the impact of


58. Consider the following statements with respect to Anti-Microbial Resistance (AMR) in humans as well
Roche Limit as animals

1. It is the minimum distance to which a large b. It aims to reduce the total amount of
satellite can approach its primary body, without antimicrobials used in the agri-food system by 30-
being torn apart by tidal forces. 50% by 2030 from the current level

2. Roche limit is not applicable to planets in Solar c. India has not adopted the Muscat Manifesto
System. Declaration

Which of the statement(s) given above is/are correct? d. It is in line with implementation of One Health
action for controlling the spread of Anti-Microbial
a. 1 only Resistance

b. 2 only

c. Both 1 and 2 62. A recent report found that Thwaites Glacier has lost
nearly 50 billion tons of ice since 1940, contributing 4%
d. Neither 1 nor 2 to the global sea level rise each year. Thwaites glacier is
located in?

a. Arctic
59. Consider the following statements with respect to
Economic Survey b. Antarctica
1. It is an annual report prepared by the c. Siberian Plateau
Department of Economic Affairs (DEA) under
the supervision of the Chief Economic Advisor d. Himalayan cold desert
(CEA).

CHENNAI |SALEM| MADURAI | COIMBATORE DELHI | BANGALORE | THIRUVANANTHAPURAM


www.shankariasacademy.com 13
www.iasparliament.com

63. Consider the following statements with respect to Select the correct answer using the codes given below:
the Just Energy Transition Partnership (JETP):
a. Only two
1. It is a mechanism for multilateral financing by
developed countries to support an energy b. Only three
transition in developing countries.
c. Only four
2. It was launched at the COP 21 of UNFCCC, also
known as the Paris Climate Agreement. d. All five

Which of the statement(s) given above


is/are incorrect?
67. Consider the following pairs:
a. 1 only
Art Forms Type
b. 2 only
1. Sursingar – Folk dance
c. Both 1 and 2
2. Parai – Stringed Instrument
d. Neither 1 nor 2
3. Mandolin – Woodwind Instrument

How many of the above pairs are correct?


64. Which of the following statements best describes the
term Ejiao, often seen in the news? a. Only one

a. Drug made from Magic Mushrooms b. Only two

b. Endangered Species of bird indigenous to China c. All three

c. Health-related product made from donkey hides d. None

d. Phenomenon related to the formation of


Northern Lights
68. Consider the following statements with respect to
Visva-Bharati University

65. Which of the following best describe the term 1. It was founded by Debendranath Tagore in
“Quasicrystals”? Santiniketan, West Bengal.

a. The constituent atoms are arranged in a fixed 2. It received the UNESCO World Heritage
pattern inscription to become the world’s first living
heritage university.
b. The constituent atoms are arranged in random
pattern 3. It was recognised as an Institution of National
Importance by an Act of Parliament.
c. The constituent atoms are arranged in a pattern
that periodically repeats itself How many of the above statement(s) is/are correct?

d. The constituent atoms are arranged in a pattern a. Only one


that repeats itself at irregular, yet predictable,
b. Only two
intervals
c. All three

d. None
66. Which of the following celestial bodies in our Solar
System are recognized as dwarf planets?

1. Eris 69. Consider the following Parliamentary Privileges:


2. Ceres 1. Right to hold secret session
3. Pluto 2. Right to punish for its contempt
4. Haumea 3. Freedom of speech in Parliament
5. Makemake 4. Right of publication of its proceedings

CHENNAI |SALEM| MADURAI | COIMBATORE DELHI | BANGALORE | THIRUVANANTHAPURAM


www.shankariasacademy.com 14
www.iasparliament.com

5. Privilege of freedom from civil arrest c. A form of reproduction in which an egg can
develop into an embryo without being fertilized by
How many of the above privileges are mentioned in the a sperm
Constitution of India?
d. A type of white blood cell that surrounds and
a. Only two kills microorganisms, removes dead cells, and
stimulates the action of other immune system cells
b. Only three

c. Only four
73. Recent study has highlighted accelerated breeding
d. All five in mosquitoes due to presence of which of the following
chemicals in urban drains?

a. Bisphenol A
70. Consider the following statements with respect to
PARAKH b. Vinyl chloride
1. It was launched as part of the implementation c. Polymetallic Nodules
of the National Education Policy (NEP), 2020.
d. PFAS (Per- and polyfluorinated alkyl
2. It is affiliated to the Ministry of Education and substances)
works under the National Council of
Educational Research and Training (NCERT).

Which of the statement(s) given above is/are correct? 74. Consider the following statements with respect to
World Sustainable Development Summit (WSDS)
a. 1 only
1. It aims to mobilize opinion-makers to address
b. 2 only issues concerning sustainable development and
climate change.
c. Both 1 and 2
2. It is organized by the United Nation
d. Neither 1 nor 2 Environment Programme (UNEP).

3. Act4Earth is an initiative launched at WSD


Summit 2022.
71. Consider the following statements with regard to
Munich Security Conference Which of the statement(s) given above is/are correct?
1. It is an annual conference held since 1963 for a. Only one
debating international security policy.
b. Only two
2. The conference initially focused on military
issues and was later expanded to issues such as c. All three
climate change and migration.
d. None
How many of the above statement(s) is/are correct?

a. 1 only
75. Consider the following statements with respect to
b. 2 only Seismic Gaps
c. Both 1 and 2 1. It is a section of an active fault which has the
potential to produce significant earthquakes.
d. Neither 1 nor 2
2. Assam gap and the Kashmir gap are the only
two seismic gaps of Himalayas in India.
72. Which of following best explains the term Which of the above statement(s) is/are correct?
“Necrophagy”?
a. 1 only
a. It is the feeding behaviour of an organism that
eats carrion from another animal that it did not kill b. 2 only
b. A process by which a cell breaks down and c. Both 1 and 2
destroys old, damaged, or abnormal proteins and
other substances in its cytoplasm

CHENNAI |SALEM| MADURAI | COIMBATORE DELHI | BANGALORE | THIRUVANANTHAPURAM


www.shankariasacademy.com 15
www.iasparliament.com

d. Neither 1 nor 2 d. All four

76. Consider the following statements: 79. Consider the following statements

Statement-I: Participatory Notes (P-Notes) allow Statement-I: Geomagnetic induced currents (GIC) in
non-registered investors to invest in the Indian market. are a potential threat to power system stability and
availability.
Statement-II: P-notes are among the group of
investments considered to be Offshore Derivative Statement-II: GIC is caused due to the phenomenon
Investments (ODIs). called as Geomagnetic Storm (GMS).

Select the correct answer using the codes given below: Select the correct answer using the codes given below:

a. Both Statement-I and Statement-II are correct a. Both Statement-I and Statement-II are correct
and Statement-II is the correct explanation for and Statement-II is the correct explanation for
Statement-I Statement-I

b. Both Statement-I and Statement-II are correct b. Both Statement-I and Statement-II are correct
and Statement-II is not the correct explanation for and Statement-II is not the correct explanation for
Statement-I Statement-I

c. Statement-I is correct but Statement-II is c. Statement-I is correct but Statement-II is


incorrect incorrect

d. Statement-I is incorrect but Statement-II is d. Statement-I is incorrect but Statement-II is


correct correct

77. Antakya and Sanliurfa are sometimes seen in the 80. In the Union Budget 2023-2024, a scheme was
news. These are ancient cities located in? introduced for a settlement scheme for disputes, where
the arbitral award is under challenge in a court. In this
a. Syria respect the government has unveiled a draft dispute
settlement scheme? Which of the following is the
b. Turkey scheme?
c. Ukraine a. Vivad se Vishwas
d. Afghanistan b. Pradhan Mantri Mudra Yojana

c. Jan Suraksha Scheme


78. Consider the following statements with respect to d. E-Dispute Resolution Scheme
Urban 20 (U20)

1. It is a city diplomacy initiative.


81. Consider the following statements with respect to
2. It is an initiative of the Ministry of Housing and One World TB Summit
Urban Affairs.
1. It was organised by India and World Health
3. The objective is to provide core infrastructure Organisation (WHO).
and give a decent quality of life to 20 urban
cities in India. 2. It was organised under India’s vision of ‘One
Earth, One Health’.
4. It facilitates a productive dialogue between the
national and local governments. 3. The initiative ‘TB Free Panchayat’ was launched
in this Summit.
How many of the above statements is/are correct?
How many of the statement(s) given above is/are
a. Only one correct?
b. Only two a. Only one
c. Only three b. Only two

CHENNAI |SALEM| MADURAI | COIMBATORE DELHI | BANGALORE | THIRUVANANTHAPURAM


www.shankariasacademy.com 16
www.iasparliament.com

c. All three d. It is a 10-year plan to put world’s 46 most


vulnerable countries back on track to achieving the
d. None UN Sustainable Development Goals (SDGs).

82. Consider the following statements with respect to 85. Consider the following statements with respect to
International Court of Justice (ICJ) the Rules for Registration and Regulation of Foreign
Lawyers and Foreign Law Firms in India, 2022:
1. It is the official court of UN, established
through Rome Statute. 1. Foreign lawyers will not be permitted to appear
before any Indian courts and tribunals and
2. Only states that are members of United Nation cannot practise Indian law.
(UN) or International Court of Justice (ICJ) can
become parties to ICJ. 2. Foreign lawyers are allowed to advice on Indian
and international laws.
Which of the statement(s) given above is/are correct?
Which of the above statement(s) is/are incorrect?
a. 1 only
a. 1 only
b. 2 only
b. 2 only
c. Both 1 and 2
c. Both 1 and 2
d. Neither 1 nor 2
d. Neither 1 nor 2

83. Consider the following statements with respect to


Whip 86. Consider the following statements regarding Basic
Animal Husbandry Statistics
1. It is mentioned neither in the Constitution of
India nor in the Rules of the House nor in a 1. It provides production estimates of 5 Major
Parliamentary Statute. Livestock Products (MLPs) such as Milk, Egg,
Meat, silk and Wool.
2. In India, all parties can issue whips to their
members. 2. It is released by Department of Animal
Husbandry and Dairying under the Ministry of
Which of the above statements is/are correct? Agriculture & Farmers' Welfare.
a. 1 only 3. According to the report, Rajasthan tops in both
the wool and milk production.
b. 2 only
How many of the above statements is/are correct?
c. Both 1 and 2
a. Only one
d. Neither 1 nor 2
b. Only two

c. Only three
84. Which of the following best describes the ‘Doha
Programme of Action (DPoA)’? d. All four
a. It is a WTO trade negotiation, which aims to
lower trade barriers all around the world and
allowing for more global trade. 87. Bharat 6G Vision Document aims to roll out India’s
6G project in 2 phases from 2023 to 2030. It was
b. It provides the framework for the establishment, prepared by?
maintenance and termination of diplomatic
relations on a basis of consent between independent a. NITI Aayog
sovereign States.
b. Technology Innovation Group on 6G
c. It is a defence deal between Australia, the UK and
the US to help Australia deploy nuclear-powered c. Indian Space Research Organisation
submarines in the Pacific region.
d. Ministry of Science and Technology

CHENNAI |SALEM| MADURAI | COIMBATORE DELHI | BANGALORE | THIRUVANANTHAPURAM


www.shankariasacademy.com 17
www.iasparliament.com

88. Consider the following statements regarding Indo- a. A ‘Citizen’s Charter’ by Greater Hyderabad
Pacific Oceans Initiative Municipal Corporation for new ward offices to
redress public grievances in a fixed time frame
1. It aims to restore maritime stability and
strengthen international governance in the b. To address and resolve gender gaps in physics
Indo-pacific region. education and research in India

2. It is an initiative of the Government of India c. An integral aspect of the New Education Policy
and builds upon the Security and Growth for All (NEP) 2020 focused on Early Childhood Care and
in the Region (SAGAR) initiative. Education

Which of the above statements is/are correct? d. Providing a platform for urban leaders to
exchange knowledge and ideas, fostering innovative
a. 1 only solutions to urban challenges

b. 2 only

c. Both 1 and 2 92. Consider the following statements with respect to


Osaka Blue Vision
d. Neither 1 nor 2
1. It was agreed at the G20 Osaka Summit, 2019.

2. It aims to reduce additional pollution by marine


89. The term Carrageenan, sometimes seen in the news plastic litter to zero by 2050.
recently, refers to?
Which of the statements given above is/are correct?
a. It is an exotic seaweed species native to Indo-
Pacific Ocean. a. 1 only

b. It is a doping substance that has been banned by b. 2 only


World Anti-doping Agency.
c. Both 1 and 2
c. It is a marine horseshoe crab living in shallow
coastal waters on soft sandy or muddy bottom. d. Neither 1 nor 2

d. It is a substance extracted from seaweeds for


thickening or emulsifying agent in food products.
93. Consider the following statements with respect to
Marburg virus disease (MVD)

90. Consider the following statements with respect to 1. Marburg and Ebola diseases are both similar,
Asiatic Lions rare diseases, although caused by different
viruses.
1. Asiatic lions are larger than African lions.
2. Human-to-human transmission of Marburg
2. It is categorised as Endangered in IUCN Red virus can occur through direct contact.
List.
Which of the above statement(s) is/are incorrect?
3. Barda Wildlife Sanctuary in Gujarat was
approved as the second home for Asiatic lions. a. 1 only

How many of the statements given above is/are correct? b. 2 only

a. Only one c. Both 1 and 2

b. Only two d. Neither 1 nor 2

c. All three

d. None 94. PM MITRA Scheme, often seen in the news, was


launched by?

a. Ministry of MSME
91. Hyderabad Charter, sometimes seen in the news
recently, is associated with? b. Ministry of Textiles

c. Ministry of Heavy Industries

CHENNAI |SALEM| MADURAI | COIMBATORE DELHI | BANGALORE | THIRUVANANTHAPURAM


www.shankariasacademy.com 18
www.iasparliament.com

d. Ministry of Commerce and Industry Select the correct answer using the code given below:

a. Only two

95. Mission Sahbhagita, sometimes seen in the news, is b. Only three


associated with?
c. Only four
a. Rail Safety
d. All five
b. Green Shipping

c. Wetland conservation
99. Non-Steroid Anti-Inflammatory Drugs (NSAID) are
d. Malnutrition free India highly toxic to vultures. Which of the following drug(s)
fall under NSAID?

1. Diclofenac
96. Which one of the following best describes the
term Gravitational Lensing? 2. Aceclofenac

a. An astronomical phenomenon that makes 3. Tolfenamic


celestial objects appear to move
4. Meloxicam
b. A theory that the universe expanded
exponentially fast for a fraction of a second after the Select the correct answer using the codes given below:
Big Bang
a. Only one
c. A visual effect that occurs when a massive
celestial body bends the path of light around it b. Only two

d. A hypothetical scenario which describes how the c. Only three


universe will end as a result of the Big Bang
d. All four

97. Consider the following:


100. Which of the following best describes the term
It is a mini-lateral group operating in the Indian Ocean “Island Rule”?
region with India, Sri Lanka, Maldives and Mauritius as
its member countries. It provides an opportunity for a. It refers to a phenomenon that occurs when a city
India to address its own strategic concerns in the Indian experiences much warmer temperatures than
Ocean while providing an opportunity for the island and nearby rural areas.
littoral nations to address their own challenges.
b. They are new regulations introduced by the
Identify the forum using the above passage Pacific Islands Forum aimed at fostering
sustainable development across island nations.
a. Coral Triangle Initiative
c. It refers to a hypothesis that mammals change
b. Colombo Security Conclave their size on islands depending on resources
available in their environment.
c. Indo-Pacific Oceans Initiative
d. It states that non-native species are more likely
d. Pacific Basin Economic Council to become established and have major ecological
impacts on islands than on continents.

98. With reference to Millets, consider the following:

1. Water intensive

2. High Gluten content

3. High antioxidant activity

4. High tolerance to drought

5. Low requirement of chemical fertilizers

CHENNAI |SALEM| MADURAI | COIMBATORE DELHI | BANGALORE | THIRUVANANTHAPURAM


www.shankariasacademy.com 19
www.iasparliament.com

Answer Key - Test I

1 2 3 4 5 6 7 8 9 10

C A D C C A A C C D

11 12 13 14 15 16 17 18 19 20

C B B A C B A D B A

21 22 23 24 25 26 27 28 29 30

B B A D A B D A D D

31 32 33 34 35 36 37 38 39 40

A D D B C B D B A B

41 42 43 44 45 46 47 48 49 50

C A D C C B A C C B

51 52 53 54 55 56 57 58 59 60

A B A A C D A A B B

61 62 63 64 65 66 67 68 69 70

C B B C D D D B A C

71 72 73 74 75 76 77 78 79 80

A A A B A B B B A A

81 82 83 84 85 86 87 88 89 90

B B C D C A B C D A

91 92 93 94 95 96 97 98 99 100

B C D B C C B B B C

 It is considered to be common, but it is hard to


EXPLANATION find due to its fossorial behaviour.
1. c  Fossorial animals are organisms that have
adapted successfully to the lifestyle of living
Mahabali Frog (Nasikabatrachus sahyadrensis) underground by
digging burrows
A proposal to declare a species of purple frog as
and tunnels.
Kerala’s official amphibian has been kept in abeyance
by the Kerala Wildlife Advisory Board.
 It buries itself all
through the year
 Mahabali Frog is also known as Purple
and surfaces
Frog/Pignose frog, which is endemic to
only one day to
Western Ghats.
breed.
 These frogs prefer loose, damp and well-
 The frog was
aerated soil close to ponds and ditches or
named after the
streams.
mythical King Mahabali, who visits his people
 Unlike other frogs, it has very short hind legs, once a year from the underneath world.
which does not allow it to leap from one spot
 IUCN status - Near Threatened
to another.

CHENNAI |SALEM| MADURAI | COIMBATORE DELHI | BANGALORE | THIRUVANANTHAPURAM


www.shankariasacademy.com 20
www.iasparliament.com

2. a UNESCO’s INTANGIBLE CULTURAL


HERITAGE OF HUMANITY
Virovore
1. Kutiyattam, Sanskrit theatre (2008)
Researchers claimed to have found the 1st known
Virovore. 2. Ramlila, the traditional performance of the
Ramayana (2008)
 Virovore is an organism that eats and
survive on viruses. 3. Tradition of Vedic chanting (2008)

 They found that a species of Halteria can eat 4. Ramman, religious festival and ritual theatre of the
huge numbers of infectious chloroviruses. Garhwal Himalayas (2009)

 Halteria is a microscopic ciliate that lives in 5. Novruz, Nowrouz, Nooruz, Navruz, Nauroz, Nevru
freshwater worldwide. (2009)

 Ciliates are single-celled organisms with 6. Chhau dance (2010)


minuscule hairs.
7. Kalbelia folk songs and dances of Rajasthan (2010)
 Chloroviruses are made up of nucleic acids, a lot
of nitrogen, and phosphorous. 8. Mudiyettu, ritual theatre and dance drama of Kerala
(2010)
3. d
9. Buddhist chanting of Ladakh: recitation of sacred
Pink Tide Buddhist texts in the trans-Himalayan Ladakh region,
Jammu and Kashmir, (2012)
 Pink tide or “the turn to the left” is a political
wave and turn towards left- 10. Sankirtana, ritual singing, drumming and dancing
wing governments in Latin America. of Manipur (2013)

 This political wave was against the right-wing 11. Thatheras of Jandiala Guru, Punjab (2014)
and neo-liberalism that had concentrated
12. Yoga (2017)
power and wealth in the hands of the elite.
13. Kumbh Mela (2017)
4. c
14. Durga Puja in Kolkata (2021)
Sagol Kangjei
15. Garba of Gujarat (2023)
 An indigenous sport of Manipur, which is a
predecessor to Modern polo. 5. c
 Players ride horses, specifically the Manipur Asian Pacific Postal Union (APPU)
Ponies.
India takes over the leadership of the Asian Pacific
 It is believed that the Manipur King Kanba Postal Union (APPU) for a tenure of 4 years.
invented the game in the 14th century BC.
 Dr. Vinaya Prakash Singh takes over the charge
 In the 1850s, the British observed Manipuri’s of Secretary General of the APPU.
sport and they later formed a polo club in
Silchar.  This is the 1st time an Indian is leading an
international organization in the postal sector.
 In Meitei culture, polo is a sport of the gods
where the deity carries a bamboo stick in his  APPU – It is an intergovernmental
hands just like a polo player. organization with 32-member countries of the
Asian-Pacific region.
 The Manipur Pony is one of 5 recognised equine
breeds of India.  Headquarters - Bangkok, Thailand.

 Sagol Kangjei is not listed under the UNESCO  Aim - To extend, facilitate and improve postal
intangible cultural heritage of humanity list relations between member countries.

 It is a restricted union of Universal Postal


Union (UPU), a specialized agency of the
United Nation (UN).

CHENNAI |SALEM| MADURAI | COIMBATORE DELHI | BANGALORE | THIRUVANANTHAPURAM


www.shankariasacademy.com 21
www.iasparliament.com

 Restricted unions promote further cooperation and stone, it's not the main rock. It lies on an
among Posts from a specific region and are ancient landslide.
governed by Article 9 of the UPU Constitution.
 The report added that undercutting by river
 There are currently 19 restricted unions, currents of Alaknanda and
covering most of the world's regions. Dhauliganga are also playing their part in
bringing landslides.
6. a
 Land subsidence - It is the sinking of the
Yellow Band Disease ground because of underground material
movement.
 It affects Corals, and is named for the
colour it turns corals before
destroying them.

 It was first spotted decades ago and has caused


widespread damage to reefs in the Caribbean.

 There is no known cure for yellow band disease.

 When the coral is infected with this disease, it


just dies.

 Overfishing, pollution and rising water


temperatures may be making the reefs more
vulnerable to yellow-band disease.

 One of the main challenges in this disease is


its impact cannot be reversed, unlike the
effects of coral bleaching.

7. a
9. c
International Thermonuclear Experimental
Reactor (ITER) Charaideo Maidams
 It is the world’s largest experimental fusion Assam’s Charaideo Maidam is India's nomination for
reactor facility in France. UNESCO's world heritage site status in the cultural
category for 2023.
 Aim – To prove the viability of nuclear
fusion as an energy source.  The Charaideo maidams, located in Assam,
represents the late medieval (13th-19th century
 Members - China, the European CE) mound burial tradition of the Tai Ahom
Union, India, Japan, Korea, Russia and the community in Assam.
United States.

 It will not produce electricity, but it will resolve


critical scientific and technical issues in order to
take fusion to the point where industrial
applications can be designed.

8. c

Joshimath Land Subsidence

 Joshimath is located in Zone V (high-risk


area) of India’s seismic zone map.  They are actually burial grounds of Ahom Kings
and Queens.

 Therefore, Charaideo Maidams are considered


 Joshimath sits on top of a glacial moraine, as the Ahom equivalent of the ancient Egyptian
which is a mound of debris left by a glacier. pyramids and famously called as the
"Pyramids of Assam".
 According to Mishra Committee
report, 1976 Joshimath is a deposit of sand  Ahoms - Ahom dynasty was founded by Chao
Lung Siu-Ka-Pha in 1253.

CHENNAI |SALEM| MADURAI | COIMBATORE DELHI | BANGALORE | THIRUVANANTHAPURAM


www.shankariasacademy.com 22
www.iasparliament.com

 Charaideo was the first capital of the Ahom  The VVP will not be overlap with Border
dynasty. Area Development Programme (BADP)
of Ministry of Home Affairs.
 The Ahom rule lasted for about 600 years until
the British annexed Assam in 1826. 12. b

 Currently, there are 3 World Heritage Sites in Atmospheric River


the northeast, but none of them in the category
of cultural heritage.  These are relatively long, narrow regions in the
atmosphere, like rivers in the sky, that
1. Kaziranga National Park, Assam - Natural transport most of the water vapor from
heritage site the tropics to the northern latitudes.

2. Manas Wildlife Sanctuary, Assam - Natural  Atmospheric rivers are part of the Earth's ocean
heritage site water cycle, and are tied closely to both water
supply and flood risks.
3. Khangchendzonga National Park, Sikkim -
Mixed heritage site  It produces intense rainfall or snowfall when it
encounters mountains or local atmospheric
10. d dynamics.
Art forms  A well-known example of a strong atmospheric
river is called the Pineapple Express.
 Ottanthullal (Thullal) – It is recite-and-dance
art-form of Kerala.  Pineapple express is a ribbon of water vapour
that begins in Hawaii, where warm conditions
 The costume and makeup of the performer are help evaporate ocean waters into the
similar to that of a Kathakali artist. atmosphere and can wallop the U.S. and
Canada's West Coasts with heavy rainfall and
 Margam Kali – It is a popular art form snow.
in Kerala and is believed to have evolved from
Kalaripayattu.  Upwelling - The rising of subsurface waters in
the open ocean and along coastlines.
 Oppana – It is a traditional song and dance
performance popular among the Mappila  Subsurface water that rises to the surface as a
community of Malabar in Kerala. result of upwelling is typically colder, rich in
nutrients, and biologically productive.
 Shumang Leela – It translates to “courtyard
performance”, is a traditional form of theatre  Pyroclastic flow - It is a dense, fast-moving
in Manipur. flow of solidified lava pieces, volcanic ash, and
hot gases. It occurs as part of certain volcanic
11. c
eruptions.
Vibrant Village Programme (VVP)
 Thermohaline circulation - It is the
 It is a centrally sponsored scheme that movement of ocean currents due to differences
in temperature and salinity in different regions
aims for the development of villages on
of water.
the northern border, thus improving the
quality of life of people living in
identified border villages.

 The objective of the programme is to create


sufficient incentives for people to stay on in the
selected villages.

 It was announced in the Union Budget 2022-


23 to 2025-26 and launched by the Ministry of
Home Affairs.

 It will cover the border areas of Himachal


Pradesh, Uttarakhand, Arunachal
Pradesh, Sikkim and Ladakh.

 Vibrant Village Action Plans will be created by


the district administration with the help of
Gram Panchayats.

CHENNAI |SALEM| MADURAI | COIMBATORE DELHI | BANGALORE | THIRUVANANTHAPURAM


www.shankariasacademy.com 23
www.iasparliament.com

13. b  It rises in the Western Ghats, from


the Bhimgad Wildlife Sanctuary in
BharOS Belagavi district of Karnataka.

 It is an indigenous mobile operating system  Flowing westward, it enters Goa. Over two-
(OS), like Android or iOS. thirds of the river’s stretch lies in Goa (76km).
It is known as River Mandovi in Goa.
 It is developed by JandK Operations Private
Limited, a non-profit organization incubated  River Mahadayi joins the Arabian Sea at
at IIT Madras and funded by the Department Panaji, Goa.
of Science and Technology.
 Salim Ali Bird Sanctuary, an estuarine
Features of BharOS mangrove habitat, is located on western tip of
the Island of Chorao along the Mandovi River,
 BharOS is an AOSP (Android Open-Source Goa.
Project) based operating system with no
Google apps or services.  Tributaries of Mahadayi include, Kalasa Nala,
Surla Nala, Haltar Nala, Poti Nala, Mahadayi
 BharOS would offer Native Over the Air Nala, Pansheer Nala, Bail Nala, Andher Nala,
(NOTA) updates and No Default Apps (NDA) etc.
setting.
 The variations in the terrain have led to
 BharOS will use Private App Store Services formation of various water falls such as
(PASS) system, which will examine and curate the Doodhsagar, Vajra poha, etc.
the apps that are safe for the users.
 The Kalasa-Banduri Nala project revolves
14. a around the construction of barrages on the
Kalasa and Banduri streams of the Mahadayi
Multilateral Export Control Regime (MECR)
River.
 Currently, there are 4 Multilateral Export
 It is a disputed project between Karnataka
Control Regime (MECR) and India is member
and Goa.
of 3 groups except NSG.

 Nuclear Suppliers Group (NSG) – It aims


to prevent the proliferation of nuclear weapons
by controlling the export of nuclear materials
and technologies.

 Missile Technology Control Regime


(MTCR) – It aims to limit the spread of
ballistic missiles and other unmanned delivery
systems that could be used for chemical,
biological, and nuclear attacks.

 Australia Group – It aims to prevent the


spread of chemical and biological weapons by
controlling the export of related materials and
technologies.
16. b
 Wassenaar Arrangement – It aims to
promote regional and international security Defence Exercises of India
and stability by controlling the export of
conventional weapons and dual-use goods and  Veer Guardian 2023 – It is bilateral Air
technologies. exercise between Indian Air Force (IAF)
and Japan Air Self Defence Force (JASDF).
15. c
 Varuna – It is a Bilateral Naval Exercise
Mahadayi River between India and France.
 River Mahadayi spreads across the districts  Cyclone – I – It is the 1st joint exercise
of Goa, Karnataka and Maharashtra. between the special forces of the Indian Army
and the Egyptian Army.

CHENNAI |SALEM| MADURAI | COIMBATORE DELHI | BANGALORE | THIRUVANANTHAPURAM


www.shankariasacademy.com 24
www.iasparliament.com

 Tarkash 2023 – It is the 6th edition of  Privacy- The Act forbids the disclosure of the
Counter Terrorism Exercise between India’s victim’s identity in any form of media unless
NSG (National Security Guard) and US Special authorised by the special courts established by
Operations Forces (SOF). the Act.

 TROPEX – It is Indian Navy’s major maritime  Procedure- It also defined the procedure for
exercise that is conducted biennially. reporting of cases, including a provision for
punishment for failure to report a case or false
 AMPHEX 2023 – It is a biennial Tri-Services complaint.
Exercise.
 Child-friendly manner- The procedures for
17. a recording of the statement of a child by the
police and court should be done in a child-
Zardalu Mango friendly manner.
 It is a GI Tagged product from Bhagalpur 20. a
of Bihar.
Constitution Literate District
 Features – Unique fragrance, light yellow
skin, and sweetness. Kollam in Kerala is India’s first Constitution literate
district.
 Other GI products from Bihar – Katarni rice,
Shahi litchi, Magahi paan, Makhana and  The Citizen is a Constitution literacy campaign
Mircha Rice. jointly launched by the Kollam district
panchayat, District Planning Committee and
18. d the Kerala Institute of Local Administration
(KILA).
Article 176
 The ambitious campaign involved 2,200
 It says that at the commencement of the first trainers called senators.
session after each general election to the
Legislative Assembly and at the  The senators visited schools, offices, auto
commencement of the first session of each year, stands and tribal councils to spread awareness.
the Governor shall address the Legislative
Assembly.  Kollam will be officially declared India’s first
Constitution literate district by Chief Minister
19. b of Kerala.
The Protection of Children from Sexual 21. b
Offences (POCSO) Act, 2012
Schengen Area
 It was enacted in consequence to India’s
ratification of the UN Convention on the Rights In January 2023, Croatia joins the Europe's Schengen
of the Child in 1992. Area, a zone with no internal border controls,
including passport checks.
 It is the first comprehensive law in the country
dealing specifically with sexual abuse of  Schengen agreement – A treaty, signed in
children. 1985 in Schengen, by France, Germany,
Belgium, Luxembourg and Netherlands and
 Aim- To address offences of sexual exploitation came into force in 1995.
and sexual abuse of children
 Schengen is a village in Luxembourg,
 Gender neutral- The Act defines a child as bordering France and Germany.
“any person” under the age of 18.
 Aim – To abolish internal border
 Non-reporting- If a person in charge of an checks and to provide a single set of rules for
institution who fails to report the commission controls at the external borders in Schengen
of a sexual offence will face punishment area.
excluding children.
 Membership – It has allowed both EU and
 Time limit- A victim may report an offence at non-EU countries.
any time, even years after the abuse has
occurred.  23 of 27 EU countries are parties except
Cyprus, Ireland, Romania and Bulgaria.

CHENNAI |SALEM| MADURAI | COIMBATORE DELHI | BANGALORE | THIRUVANANTHAPURAM


www.shankariasacademy.com 25
www.iasparliament.com

 Non-EU States like Iceland, Norway, Particularly Vulnerable Tribal Groups (PVTGs)
Switzerland and Liechtenstein are members of
Schengen zone.  The central government has
recognized 75 tribal communities as
 United Kingdom (UK) is a non-EU and Particularly Vulnerable Tribal Groups (PVTGs)
non-Schengen Country. based on the recommendations of the Dhebar
Commission.

 The criteria for identifying Particularly


Vulnerable Tribal Groups are:

1. Pre-agricultural level of technology

2. Low level of literacy

3. Economic backwardness

4. A declining or stagnant population

23. a

Sixth Schedule of the Indian Constitution

 According to Article 244 of the Indian


Constitution, the Sixth Schedule consists of
provisions for the administration of tribal areas
in Assam, Meghalaya, Tripura and
Mizoram.

Schengen Visa  Passed by the Constituent Assembly in 1949, it


seeks to safeguard the rights of tribal
 It is issued for short stays under 90 days. population through the formation
of Autonomous District Councils (ADC) and
 Benefits – Freedom to travel to other Regional Councils.
European nations without border checks.
 Each autonomous district and regional council
 In March, 2024, Kosovo secured visa-free consist of not more than 30 members, of which
access to the Schengen zone in Europe. 4 are nominated by the governor and the rest
via elections.
22. b
 All of them remain in power for a term of 5
Scheduled Tribes in India years. The Bodoland Territorial Council,
however, is an exception.
 The term 'Scheduled Tribes' first appeared
in Article 366 of the Constitution of India.  Fifth Schedule provides for establishment of
a Tribes Advisory Council (TAC) in any
 The criterion followed for specification of a State having Scheduled Areas.
community, as scheduled tribes are:
 The Governors of these states are empowered
o Indications of primitive traits to reorganize boundaries of the tribal areas.

o Distinctive culture  Acts passed by Parliament and state legislatures


may or may not be levied in these regions unless
o Geographical isolation the President and the governor gives approval,
with or without modifications in the laws.
o Shyness of contact with the community at
large, and  The ADCs are empowered with civil and
judicial powers, can constitute village courts
o Backwardness. within their jurisdiction to hear trial of cases
involving the tribes.
 This criterion is not spelt out in the
Constitution but has become well established 24. d
by the Lokur Committee in 1965.
Stockholm Conference, 1972

CHENNAI |SALEM| MADURAI | COIMBATORE DELHI | BANGALORE | THIRUVANANTHAPURAM


www.shankariasacademy.com 26
www.iasparliament.com

 United Nations Conference on the Human  Article 19 – It guarantees freedom of speech


Environment or the Stockholm Conference and expression is a right generally invoked
was held in Stockholm, Sweden in 1972. against the state.

 It was the first United Nations conference that  Article 19 can be suspended only when
focused on international environmental issues. emergency is declared on the grounds of war
or external aggression (i.e., external
 The participants adopted a series of principles emergency) and not on the ground of armed
for sound management of the environment rebellion (i.e., internal emergency).
including the Stockholm Declaration and
Action Plan for the Human Environment. ARTICLE 19

 One of the major results of the Stockholm RIGHTS RESTRICTIONS


conference was the creation of the United
Nations Environment Programme  Article  These rights are not
(UNEP). 19 provides for absolute and
the right to the reasonable
Rio Conference (Earth Summit), 1992 restrictions includes
1. Freedom of speech
 United Nations Conference on Environment and expression 1. Sovereignty and integrity of
and Development (UNCED), also known as India
the Rio Conference or the Earth Summit held 2. Assemble
in Rio de Janeiro, 1992. peaceably and 2. Security of the state
without arms
 It was at this Summit where the United Nations 3. Friendly relations with
Framework Convention on Climate Change 3. Form associations foreign states
(UNFCCC) and the Convention on Biological or unions
Diversity (CBD) were opened for signature, 4. Public order
while the negotiation of the United Nations 4. Move freely
Convention to Combat Desertification throughout the 5. Decency or morality
(UNCCD) was called for in the Summit outcome territory of India
6. Contempt of court,
- Agenda 21.
5. Reside and settle defamation
 This is why these three sister conventions later in any part of the
collectively became known as "the Rio territory of India 7. Incitement to an offence
Conventions".
6. Practice any
25. a profession, or to
carry on any
 Senna spectabilis (calceolaria shower) is an occupation, trade
invasive plant species known for its prolific seed or business
production and rapid growth that displaces the
native plant species. 27. d

 Gambusia fish is an insectivorous fish that is Seismic Zone Map


found mostly in fresh and brackish, and
The crisis in Joshimath, Uttarakhand, where
occasionally marine waters and has a high numerous houses exhibit significant or minor cracks,
breeding capacity. underscores the critical significance of seismic zone
mapping in India.
 Axolotls are paedomorphic or neotenic
aquatic salamanders that has the ability to  A seismic zone is an area where there is a high
retain juvenile or larval traits. probability of earthquakes due to the area’s
geology.
 Candida auris is an emerging multidrug-
resistant fungus causing life-threatening
 Seismic zonation involves dividing areas based
outbreaks.
on expected ground motion.
26. b
 It assesses the hazards related to earthquakes in
Ambit of Article 19 such areas to provide inputs for safer
constructions and other practices.
The Supreme Court in its judgment said that the
fundamental rights under Article 19 & 21 of the  The first national seismic zoning map of India
Constitution can be enforced against private was compiled by the Geological Survey of India
individuals & entities. in 1935.

CHENNAI |SALEM| MADURAI | COIMBATORE DELHI | BANGALORE | THIRUVANANTHAPURAM


www.shankariasacademy.com 27
www.iasparliament.com

 Seismic Zonation map of India was given Camel racing Rajasthan


by Bureau of Indian Standards (BIS).
Bulbul fight Assam
 It categorised India into four seismic
zones from Zone II to Zone V, with zone V Andhra Pradesh,
being the most vulnerable and seismically Cock fighting Chattisgarh and
active region. Jharkhand

 National Centre for Seismology 29. d


(NCS) under Ministry of Earth Sciences is the
nodal agency for monitoring earthquakes in Organisation of Islamic Cooperation (OIC)
and around the country.
 It is formerly the Organisation of the Islamic
 According to the Journal of the International Conference.
Society for the Prevention and Mitigation of
Natural Hazards, almost 65% of India falls  Established in - 1969
in high to very high seismic zones.
 Headquarters - Jeddah, Saudi Arabia

 Aim - To strengthen cooperation and solidarity


between the Member States, to protect the
rights and interests of the Islamic World.

 Membership - 57 states (Syria’s membership


suspended in 2012)

 Members includes Palestine, Afghanistan,


Pakistan, Bangladesh, Saudi Arabia, etc.

 India is home to the world's 3rd largest


Muslim population, next only to Indonesia and
Pakistan. Yet, India is not a member of the
OIC.

 Non-members includes United States of


America and Israel.

 Russia is one of the five observer country in


OIC.

30. d

Girmityas or Jahajis

28. a  They are indentured labourers from British


India transported to work on plantations in
Animal Sports States other countries.

Jallikattu Tamil Nadu  Girmitya countries – The countries where


these indentured Indian labourers were settled.

Rekla Tamil Nadu  It includes Fiji, Mauritius, South Africa, East


Africa, the Malay Peninsula, Caribbean and
Bail Gadi Shariat Maharashtra South America (Trinidad and Tobago, Guyana
and Suriname).
Kambala Karnataka
 Girmitya – It was coined by Mahatma Gandhi
Maramadi Bull Surfing Kerala who referred to himself as 1st Girmitya.

Dhirio bull fight Goa 31. a

Buffalo fight during Charge Sheet and FIR


Assam
Bhogali Bihu
The Supreme Court upheld that the chargesheets are
not ‘public documents’ and denied their free public
access.

CHENNAI |SALEM| MADURAI | COIMBATORE DELHI | BANGALORE | THIRUVANANTHAPURAM


www.shankariasacademy.com 28
www.iasparliament.com

First Information  River Dibang is a tributary of the


Charge Sheet
Report (FIR) Brahmaputra River which flows through the
states of Arunachal Pradesh and Assam.
It is a preliminary
report recorded by an on-  The Idu Mishmi, an animist tribe in the
It is a final report of an
duty officer from an Dibang valley, will be directly affected by this
investigation officer
aggrieved person either in Hydroelectric Power Project.
writing or made orally.
33. d
It is filed in Police Station It is filed in Courts
Project 75 (India)
It is not a public
It is a public document  Project 75 – India signed Project 75 to
document
construct 6 Scorpene class submarines over 30
Unless, the Police Officer years with technology transfer from France.
is permitted by an order of
the jurisdictional  Project 75I – It is a follow-up to Project 75
A FIR is prepared only in Magistrate to investigate and improves upon the design and technology
the case of cognizable the non- cognizable of its predecessor.
offences, and hence there is offence, the Police
no FIR prepared for a non- Officer does not get  It aims to procure diesel-electric attack
cognizable offence. jurisdiction to submarines with fuel cells and Air-
investigate the matter and Independent Propulsion System
file a final report or the (AIP) for the Indian Navy.
charge sheet.
Project 75 vs Project 75-I
32. d
 The conventional diesel-electric submarines
Etalin Hydroelectric Power Project such as the Scorpene, under Project 75, come
with improved stealth features such as
The Etalin hydroelectric power project in Arunachal advanced acoustic absorption techniques, low
Pradesh has been scrapped in its present form and the radiated noise levels, long-range guided
Forest Advisory Committee have asked Arunachal torpedoes, tube-launched anti-ship missiles,
Pradesh to review it. sonars and sensor suites.

 The Etalin Hydroelectric Project (EHEP) is a  However, as electrical batteries power them,
joint venture of the Jindal Power Limited and they need to surface every 48 hours to be
the Hydropower Development Corporation recharged.
of Arunachal Pradesh.
 The AIP technology in Project 75I allows the
 It is located in Etalin village in the Dibang submarines to stay submerged for up to
Valley district of Arunachal Pradesh. two weeks.

 The project is a combination of two run-of-the-  These submarines may even be larger in size
river schemes and involves the construction of compared to the ones under Project 75.
concrete gravity dams on
the Tangon and Dri rivers. 34. b

 Major tributaries of Dibang River includes Dri, The World Social Report, 2023
Mathun and Talon.
 The World Social Report 2023 explores the
economic and social implications of the ageing
of the human population.

 The report is published by the UN Department


of Economic and Social Affairs (UN DESA).

 Theme of the report - Leaving No One


Behind in an Aging World.

 Ageing refers to the process by which the


proportion of older individuals in a society
increases over time.

CHENNAI |SALEM| MADURAI | COIMBATORE DELHI | BANGALORE | THIRUVANANTHAPURAM


www.shankariasacademy.com 29
www.iasparliament.com

 By 2050, the number of persons aged 65 years 3. Global South Science and Technology
or older is expected to double, surpassing 1.6 Initiative
billion accounting for more than 16% of the
global population. 4. Global South Young Diplomats Forum

Madrid International Plan of Action (MIPAA), 5. Global South Scholarships


2002
37. d
 It is a comprehensive list of commitments for
United Nations Member States. Golden Crescent

 It focuses on three priority directions  The Golden Crescent and the Golden Triangle
are distinct geographical confluences in Asia
o Older persons and development that allow drug cartels to smuggle their
merchandise across borders due to the
o Advancing health and well-being into favourable geography.
old age and
 The Golden Crescent is an Opium Producing
o Ensuring enabling and supportive Region in Asia, comprising Afghanistan,
environments. Iran, and Pakistan.

35. c  The Golden Triangle, lies to India’s east,


encompassing the intersection
Pashmina Shawls between Burma, Thailand, Vietnam and
Laos.
 Pashmina comes from an animal
fibre Cashmere, derived from
the Changthangi goat of Ladakh.

 Changthang is the land of the nomads, located


east of Leh, about 14600 m above sea level and
winter temperature can drop to −40 degree
Celsius.

 Changpa, a nomadic tribe, who rear


changthangi goats in harsh climates.

 From the goats, they get the wool for making


the world-famous Pashmina wool.

 Pashmina shawls are GI tagged for the region


of Jammu & Kashmir.

36. b 38. b

Voice of Global South Summit  The Kole Wetlands – It is a Ramsar site and
an Important Bird Area, spread between the
India hosted the virtual conference of the 1st ever banks of Chalakudy River in the south
Voice of Global South Summit. and Bharathapuzha River in the north
in Kerala.
 Theme - ‘Unity of Voice, Unity of Purpose’
 A farmer from War tribe (sub-tribe of Khasi) in
 It envisages bringing together countries of the Meghalaya created a Living root bridge – It
Global South to share their perspectives and was created over Umkar River near
priorities on a common platform. Cherrapunjee in the state of Meghalaya.

 A total of 125 nations took part in the digital  Siyom Bridge – It is a strategically important
conference. bridge over the Siyom River in Arunachal
Pradesh constructed by the Border Roads
 Indian Prime Minister also announced five new Organisation (BRO).
initiatives by India in the summit
39. a
1. Arogya Maitri (Wellness Friendship)
Munroe Thuruthu
2. Global South Center of Excellence

CHENNAI |SALEM| MADURAI | COIMBATORE DELHI | BANGALORE | THIRUVANANTHAPURAM


www.shankariasacademy.com 30
www.iasparliament.com

 It is a string of 8 islets at the confluence of the  India is among the top implementers in the
Ashtamudi Lake and the Kallada River. middle-income countries category.

 It is located in Kollam district, Kerala.  India and TFA - Food Safety and Standards
Authority of India (FSSAI) monitors Trans
 It was named after Colonel John Munroe, the Fatty Acids (TFAs) in food products.
British Resident of erstwhile Travancore State.
 FSSAI has capped the amount of TFA in food
 It is slowly going under water, and its fleeing products to 2% from 2022.
residents join the swelling ranks of climate
refugees. 41. c

40. b Upper Bhadra Lift Irrigation Project

Transfat In Budget 2023-2024, Union Government has


announced that the central assistance will be given to
A new status report from the World Health Upper Bhadra Project for irrigation and drinking
Organisation (WHO) has found that 5 billion people water in the central Karnataka.
globally remain unprotected from harmful trans-fat.
 Objective – To provide sustainable irrigation
 Trans-fats or trans-fatty acids (TFA), are a form facility during kharif season.
of unsaturated fat.
o 1st stage – Lifting water from Tunga
 TFA occurs in both natural and artificial forms. River to existing Bhadra reservoir.

 Naturally-occurring trans-fat come from o 2nd stage – Lifting water from Bhadra
ruminants (cows and sheep). reservoir to Tunnel near Ajjampura, in
Tungabhadra sub-basin of Krishna
 Industrially produced trans-fat (also called basin.
industrially produced trans-fatty acids) by
hydrogenation of vegetable oils to make them Tungabhadra River
more solid.
 It is a tributary of River Krishna
 It is commonly found in packaged foods, baked
goods, cooking oils and spreads.  It is formed by the confluence of 2 rivers,
the Tunga River and the Bhadra.
 It is done to increase shelf life of food items and
for use as an adulterant as they are cheap.  It flows through the states of Karnataka,
Andhra Pradesh and Telangana and was also
 Health Impacts - Increases the risk of heart known by its ancient name was Pampa.
disease and death.
 Both Tunga & Bhadra Rivers originate on the
 Trans-fats are responsible for up to 500 000 eastern slopes of the Western Ghats.
premature deaths from coronary heart disease
each year globally.

 WHO on TFA - In 2018, WHO called for the


global elimination of industrially produced
trans-fat by 2023.

 WHO established specific criteria to follow


‘Best-practices’ in trans-fat elimination policies
and limit industrially produced trans-fat in all
settings.

 There are 2 best-practice policy alternatives:

1. Mandatory national limit of 2 grams of


industrially produced trans-fat per 100
grams of total fat in all foods.
42. a
2. Mandatory national ban on the
production or use of partially UN Commission for Social Development
hydrogenated oils (a major source of (CSocD)
trans-fat) as an ingredient in all foods.

CHENNAI |SALEM| MADURAI | COIMBATORE DELHI | BANGALORE | THIRUVANANTHAPURAM


www.shankariasacademy.com 31
www.iasparliament.com

India has been elected as the Chair of the 62nd Session 45. c
of the Commission for Social Development in 2023.
Mahila Samman Savings Certificate
 Along with India as the Chair, the commission
also elected North Macedonia, Dominican  It is a one-time new small savings scheme
Republic, and Luxembourg as Vice-Chairs. for women and girl child available for two
years, from April 2023-March 2025.
CSocD
 It was announced by the Union government in
 It is a Commission within the UN Economic the Budget 2023- 24.
and Social Council (ECOSOC).
 The minimum deposit amount under the
 Its main responsibility is to provide advice to scheme is Rs.1,000 in multiples of rupees one
the UN on social development issues. hundred and the maximum deposit amount
is Rs.2 lakh.
 The CSocD meets every year at the United
Nations Headquarters in New York.  This scheme has a fixed interest rate of
7.5% p.a. compounded quarterly.
 Role – It is tasked with monitoring and
carrying out the Copenhagen Declaration  It also has partial withdrawal options wherein
and Programme of Action adopted in an account holder can withdraw up to 40% of
World Summit for Social Development in the account balance after one year from the
Copenhagen in 1995. account opening date.

 Members are elected based on equitable 46. b


geographical distribution for four-
year terms. Urban Infrastructure Development Fund
(UIDF)
43. d
 Union Budget 2023-24 has announced the
Article 112 setting up of the UIDF through use of priority
sector lending shortfall for creating urban
 A budget is an estimation of revenue and infrastructure in Tier 2 and Tier 3 cities.
expenses over a specified future period of time.
 It aims to supplement the efforts of the State
 In the Constitution of India, the budget is Governments for urban infrastructure
referred as the Annual Financial development works implemented through
Statement (Article 112). Public/State Agencies, Municipal Corporations
and Urban Local Bodies, by providing a stable
 Article 112 - The President shall in respect and predictable source of financing.
of every financial year cause to be laid before
both the Houses of Parliament a statement of  The Fund will be managed by the National
the estimated receipts and expenditure of the Housing Bank.
Government of India.
 It has been operationalized with an initial
44. c corpus of Rs.10,000 crore.

Seven priorities of Budget 2023-24 47. a

 The seven priorities, termed Saptarishi, were BharatSHRI


adopted in the Union Budget for FY 2023-24 to
guide the country towards 'Amrit Kaal', thus An announcement was made in Union Budget Speech
providing a blueprint for an empowered and 2023-2024 to setup Bharat Shared Repository of
inclusive economy. Inscriptions (BharatSHRI).

 It is a digital epigraphy museum, with


digitization of 1 lakh ancient inscriptions in the
first stage.

 It was setup by the Archaeological Survey


of India (ASI) at Hyderabad.

 No separate budget has been allocated for the


project and the expenditure is incurred through
the allocated funds to ASI.

CHENNAI |SALEM| MADURAI | COIMBATORE DELHI | BANGALORE | THIRUVANANTHAPURAM


www.shankariasacademy.com 32
www.iasparliament.com

 ASI under the Ministry of Culture is the o Thein (Ranjitsagar) on the river Ravi.
custodian of the epigraphic wealth of the
nation.

 PM SHRI is a centrally sponsored scheme of


Ministry of Education to establish over 14,500
schools in India.

48. c

Sickle cell anaemia

 Sickle cell disease (SCD) is an inherited group


of blood disorders that is genetic in nature.

 Transmission - Usually transferred from the


parents to the child during birth i.e. both 50. b
parents can be carriers of SCDs.
Circuit Breakers
 Characteristics - A modification in the shape
 It is a regulatory measure that
of the red blood cell into a crescent shape.
temporarily suspends trading on an
exchange.
 The sickle cells die prematurely, resulting in a
chronic lack of red blood cells (anaemia), often
 Circuit breakers are pre-defined percentage
called sickle-cell anaemia.
values that check for uncontrolled movement
in a security or index in either direction.
 Effects - This condition leads to poor blood
oxygen levels and blood vessel blockages.
 It can cap how much the value of a stock can
fall in a single day/trading session and in
 Treatment - Medication, blood transfusions
doing so, create a more stable market overall.
and rarely a bone-marrow transplant are done.
 In India, the Securities and Exchange
 In the Union Budget 2023-24, National
Board of India (SEBI) is responsible for the
Sickle Cell Anaemia Elimination
formulation and monitoring of the circuit
Mission was announced to eliminate Sickle
breaker mechanism.
Cell Anaemia by 2047.
51. a
 Casgevy, the 1st therapy based on gene editing
technology Crispr-Cas9 for sickle cell disease Otters
and thalassaemia has been approved in UK.
 Otters are regarded as ‘Tigers of Rivers’.

 Their presence indicates good water quality


and ecosystem balance.

49. c

Dams in Indus River

 India has already constructed several storages


works along the eastern rivers of Indus River
system.

o Bhakra Dam on the Sutlej River

o Pong and Pandoh Dam on the


Beas River

CHENNAI |SALEM| MADURAI | COIMBATORE DELHI | BANGALORE | THIRUVANANTHAPURAM


www.shankariasacademy.com 33
www.iasparliament.com

 In the rivers, otters are what the tigers are in  ‘Phool Walon ki Sair’ or Sair-e-Gulfaroshan
forests. When they return, it means that there is (walk of the flowers) – It is an annual
enough to eat in the river and the ecosystem community celebration in Mehrauli, New
health is improving. Delhi.

 Diversity – Of the 13 species of otters, 3  Mehrauli is considered to be the oldest city of


are in India and 5 are present in Asia. Delhi and is the oldest area of the metropolis to
be continuously inhabited.
Otters in India (IUCN Status)
54. a
 Asian Small clawed otter - Vulnerable
Earth Quake
 Smooth coated otter - Vulnerable
A major earthquake struck Turkey and Syria killing
 Eurasian Otter - Near Threatened more than 2600 people and flattening thousands of
buildings.
52. b
Important terminologies
Gross Domestic Climate Risk Report
 Fault - A fracture in the rocks that make up the
 Index – It calculates the physical climate risk Earth’s crust
to build environments such as buildings and
properties.  Epicenter - The point at the surface of the
Earth above the focus
 Physical risk refers to vulnerability from 8
climate change events like heat waves, coastal  Plates - Massive rocks that make up the outer
flooding, extreme wind, etc. layer of the Earth’s surface and whose
movement along faults triggers earthquakes
 Released by - Cross Dependency
Initiative (XDI), a global organisation  Seismic waves - Waves that transmit the
specialising in climate risk analysis for regions, energy released by an earthquake
banks and companies.
 Focus (Hypocenter) - The point within the
 Aggregated Damage Ratio (ADR) signifies the Earth where an earthquake rupture starts
total amount of damage a region’s-built
environment would sustain in 2050 and a high Types of energy waves
ADR signifies more peril.
 P waves or primary waves – They are the
 Assam, Bihar and Tamil Nadu had the first waves to be detected.
highest ADR among Indian States.
 These are compressional waves that push and
 Assam, in particular, would witness the pull as they move through Solids and fluids.
maximum increase of climate risk, rising up to
330% by 2050 as compared to 1990.  S waves or secondary waves - They are the
next waves to be detected.
53. a
 These waves move only through solids.
 Siang Unying Festival – It is an important
festival of Adi community was celebrated  They move up and down or side to side,
in Arunachal Pradesh. perpendicular to the direction in which the
wave is moving.
 It is celebrated to mark beginning of Adi
community’s new year i.e., arrival of spring  Surface waves - It follow P and S waves and
season and to strengthen the bond among travel along the surface of the earth and thus
community. cause the most damage.

 Padams, Milangs, Komkars, Minyongs and  Surface waves can be characterized as


Pasis collectively call themselves as Adi
meaning hill people. o Love waves - Faster and move the
ground from side to side, and
 Attukal Pongala – It is held in Attukal
Temple, Thiruvananthapuram in Kerala. o Rayleigh waves - Roll like waves on
the surface of oceans and lakes
 It is the largest congregation of women for a
festival in the world.

CHENNAI |SALEM| MADURAI | COIMBATORE DELHI | BANGALORE | THIRUVANANTHAPURAM


www.shankariasacademy.com 34
www.iasparliament.com

 The system – in this case the forest – may


remain ‘tipped’ even if the temperature falls
below the threshold again.

 Categories - These elements broadly fall into


three categories

1. Cryosphere

2. Ocean-atmosphere and

3. Biosphere

 Over the years, scientists have identified a host


of different climate tipping points across the
Earth system. They are:

 The threshold would be reached in the event of


increase in global temperatures ranging from 2
degree to 4 degree Celsius.

55. c

Common law

 It is the body of law that is created by


judges through their written opinions, rather
than through statutes or constitutions
(statutory law).

 UK and the Commonwealth


countries, including India, are common law
countries.

 In Civil law countries, codified legislation


prevails but these distinctions are not as
obvious as they appear.

 In reality, many countries combine elements of


common and civil law systems.

56. d

Climate Tipping Points 57. a

 According to IPCC, tipping points are ‘critical Operation Sadbhavana


thresholds in a system that, when exceeded, can
lead to a significant change in the state of the  Operation Sadbhavana (Goodwill) is unique
system, often with an understanding that the humane initiative undertaken by Indian
change is irreversible.’ Army.

 In essence, climate tipping points  The operation is taking place in the State of
are elements of the Earth system in which Jammu & Kashmir.
small changes can kick off reinforcing
loops that ‘tip’ a system from one stable state  The operation aims to address aspirations of
into a profoundly different state. people affected by scrooge of terrorism,
sponsored and abetted by Pakistan.
 Example - A rise in global temperatures
because of fossil fuel burning, further down the  The key focus areas of the operation include:
line, triggers a change like a rainforest infrastructure development, health &
becoming a dry savannah. sanitation, woman & youth empowerment,
community development projects, and
 This change is propelled by self- education among others.
perpetuating feedback loops, even if what
was driving the change in the system stops.

CHENNAI |SALEM| MADURAI | COIMBATORE DELHI | BANGALORE | THIRUVANANTHAPURAM


www.shankariasacademy.com 35
www.iasparliament.com

58. a  The countries that signed the declaration


are India, Bhutan, Indonesia, Malaysia,
Roche Limit and Nepal (Rhino range countries).

 Roche limit is the minimum distance to  The participants agreed to achieve at least a 3%
which a large satellite (like moon) can approach annual growth rate of Greater One?Horned,
its primary body (like Earth) without being torn Javan and Sumatran Rhinos.
apart by tidal forces.
61. c
 Inside Roche limit, the large satellite (moon
here) will disperse and form rings around the Muscat Ministerial Manifesto
primary body (Earth here).
Over 30 countries adopted the Muscat Ministerial
 The Roche limit is not a sharp boundary. Manifesto at the 3rd Global High-Level Ministerial
Conference on Antimicrobial Resistance held in Muscat
 The Roche limit doesn’t just exist between the in 2022.
Earth and the moon and it is applicable to
any planet and the celestial bodies  India had adopted the Muscat Manifesto
around it. Declaration.

 For instance, in Saturn, the beautiful rings that  Manifesto – Need to accelerate political
you see around the planet are within the Roche commitments in the implementation of One
limit and therefore, there are no moons in that Health action for controlling the spread of
area. AMR.

 It also recognised the need to address the


impact of AMR not only on humans but
also on animals, and in areas of
environmental health, food security and
economic growth and development.

 It has proposed to achieve 3 targets.

o Target 1 - Reduce the total amount of


antimicrobials used in the agri-food
59. b system at least by 30-50% by
2030 from the current level.
Economic Survey
o Target 2 - Zero use of medically
 The Economic Survey is a detailed annual important antimicrobials for human
report of the state of the national economy in medicine in animals for non-veterinary
the financial year that is coming to a close. medical purposes or in crop production
and agri-food systems for non-
 It is prepared by the Economic Division of phytosanitary purposes.
the Department of Economic Affairs
(DEA) under the guidance of the Chief o Target 3 - Ensure that ACCESS group
Economic Advisor (CEA). antibiotics are at least ≥60% of overall
antibiotic consumption in humans by
 Once prepared, the Survey is approved by 2030.
the Union Finance Minister.
 It called on the 4-way alliance formed by the
 The first Economic Survey was presented for United Nations to provide the necessary
1950-51 and until 1964, it was presented sectoral technical support and guidance to
along with the Budget. implement these goals and procedures.
60. b  4-way alliance – World Health Organisation
(WHO), Food and Agriculture Organisation
Chitwan Declaration for Asian Rhinos
(FAO), the World Organisation for Animal
Conservation 2023
Health (WOAH) and the United Nations
The 3rd Asian Rhino Range Countries Meeting held in Environment Programme (UNEP).
Chitwan, Nepal adopted the Chitwan declaration.
62. b
 It is an agreement to manage the population of Thwaites glacier
greater one-horned, Javan, and Sumatran
rhinos.

CHENNAI |SALEM| MADURAI | COIMBATORE DELHI | BANGALORE | THIRUVANANTHAPURAM


www.shankariasacademy.com 36
www.iasparliament.com

Thwaites Glacier has lost 50 billion tons of ice since International Partners Glasgow Financial
1940. Group Alliance for Net Zero

 Thwaites glacier, also known as Doomsday Comprises of Japan, the


Multilateral and national
glacier, is one of the fastest-retreating glaciers USA, Canada, Denmark,
development banks and
in Antarctica and is already a major France, Germany, Italy,
finance agencies such as
contributor to global sea level rise. Norway, the EU, and the
HSBC and Citi Bank
UK
 Melting of thwaites glacier had already
contributed 4% to global sea level rise each 64. c
year.
Ejiao
 Thwaites is important for Antarctica as it slows
the ice behind it from freely flowing into the The demand for ejiao has led to a shortage of donkeys
ocean. in China and increasingly worldwide.

 Ejiao – An ancient health product that is made


from collagen extracted from donkey hides
mixed with herbs and other ingredients.

 Use – It’s believed to have properties that


strengthen the blood, stop bleeding and
improve the quality of both vital fluids and
sleep.

65. d

Quasicrystals

Scientists have discovered a new type of quasicrystal,


one with 12-fold symmetry, in Nebraska, USA.

 Solids – The constituent atoms are confined to


63. b a fixed arrangement.
Just Energy Transition Partnership (JETP)  Crystals – The atoms are arranged in a pattern
that periodically repeats itself.
 Aim - It is a key mechanism for multilateral
financing by developed countries to support  Quasicrystals – The atoms are arranged in a
an energy transition in developing pattern that repeats itself at irregular, yet
countries. predictable, intervals.
 Launched at – COP - 26 in Glasgow with  It simply represents an incremental deviation
the support of the United Kingdom (UK), the from the natural order.
United States (US), France, Germany, and the
European Union (EU).  They are found in meteorites or the debris from
nuclear blasts.
 Beneficiaries - South Africa, Indonesia, and
Vietnam are the first three countries to receive 66. d
funding.
Dwarf planet
 Funding - JETP funding can go through
grants, loans, or investments. Astronomers are puzzled by a dwarf planet in our solar
system, about half the size of Pluto, with unusual rings,
 The donor pool includes the International according to a new study.
Partners Group (IPG) and the Glasgow
Financial Alliance for Net Zero (GFANZ)  The dwarf planet orbits the Sun beyond
Working Group. Neptune.

 International Astronomical Union (IAU) – It


sets definitions for planetary science.

 Dwarf planet – It is a celestial body that:

o Has to orbit around a star, but is not


itself a satellite.

CHENNAI |SALEM| MADURAI | COIMBATORE DELHI | BANGALORE | THIRUVANANTHAPURAM


www.shankariasacademy.com 37
www.iasparliament.com

o Has enough mass to assume a nearly  Parai – It is


round shape under its own gravity. a percussion instrument
is made of neem wood and
o Has not cleared the buffalo hide and its weight
neighbourhood around its orbit ranges from 1 kg to 5 kg.
 Unlike major planets, these bodies are not  It is played with 2 sticks a
massive enough to have swept up smaller short thick one and a long
nearby bodies by gravitational attraction; they thin one with the drum
thus failed to grow larger. balanced on the elbow.

 Pluto was reclassified from a planet to a dwarf  Karakattam – An ancient folk dance
planet in 2006 because it is not gravitationally of Tamil Nadu.
dominant in its neighbourhood.
 The performers wear colourful saris and dance
 There are now five known celestial bodies in our with a pot (karakam) on their head, to invoke
Solar System that qualify as dwarf Mariamman, the goddess of rain.
planets: Pluto, Eris, Ceres, Makemake
and Haumea.  Perini Odissi – Perini Natyam is a dance
dedicated to Lord Shiva that was quite popular
 Plutoids – A category within dwarf planets, during the Kakatiya dynasty.
which are dwarf planets that are farther from
the Sun than Neptune. 68. b

 All the dwarf planets except Ceres are Visva-Bharati University


plutoids, because of its location in the asteroid
belt. The Visva-Bharati University has received the
UNESCO World Heritage inscription to become
the world’s first living heritage university.

 Visva-Bharati University was founded


by Rabindranath Tagore in 1921, located in
Santiniketan, West Bengal.

 In 1951, the institution was given the status of


Central University and ‘An Institution of
National Importance’ by the Visva-Bharati
Act, 1951.

 The President of India is the Visitor of the


University, the Governor of West Bengal is the
Rector, and the Prime Minister of India acts as
the Chancellor.
67. d
 The President of India appoints the Vice-
 Sursingar - A stringed musical instrument,
chancellor of the University.
similar to the sarod, but is older and produces
deeper notes. 69. a
 It is made of Parliamentary Privileges
wood and has a
gourd attached  It refers to the powers, privileges and
to a hollow immunities enjoyed by Houses of Parliament
wooden handle with a metal fingerboard. and their members in the performance of their
duties.
 Mandolin -
A stringed instrument,  Article 105 specifically deals with
usually with 8 strings that parliamentary privileges.
are plucked with a pick,
similar to a lute. o Freedom of speech in the parliament -
There shall be freedom of speech in Parliament
 It is a moderately sized,
smaller than the Veena, o No member of Parliament shall be liable to any
Sitar, or guitar. proceedings in any court in respect of anything

CHENNAI |SALEM| MADURAI | COIMBATORE DELHI | BANGALORE | THIRUVANANTHAPURAM


www.shankariasacademy.com 38
www.iasparliament.com

said or any vote given by him in Parliament or Billionaire investor George Soros has recently made a
any committee thereof. controversial speech at the Munich Security
Conference (MSC) linking Adani and Indian Prime
o Right of publication of its proceedings - Minister.
No person shall be so liable in respect of the
publication by or under the authority of either  MSC is an annual conference on global
House of Parliament of any report, paper, votes security issues held in February every year,
or proceedings at Munich, Germany.

 In other respects, the powers, privileges and  It was founded by Ewald-Heinrich von Kleist at
immunities shall be defined from time to time the peak of the Cold War (1947-1991).
by Parliament through law.
 Started in 1963, the conference initially
 Some other privileges of the members of focused on military issues and was
parliament defined by the law made by the attended by western countries.
parliament are:
 Its main objective is to display a united front in
o Privilege of freedom from civil their struggle against Soviet communism.
arrest and molestation i.e. he cannot be
imprisoned within a period of 40days before  After cold war, MSC expanded its agenda by
and 40 days after the session of parliament. including issues such as climate change and
migration.
o Right to disallow to publication of its Reports
and proceedings (The SC held that the  It also started to invite leaders from eastern
publication of expunged portion of speech nations, including Russia, India and China.
constituted a breach of the privilege of the
house) Munich Security Conference (MSC), 2023

o Right to hold secret session.  For the first time in 20 years, Russia and Iran
were not invited to the MSC 2023 conference
o Right to exclude stranger from its proceedings. following the ongoing Russia-Ukraine
Crisis and brutal suppression of protests by
o Right to regulate internal affairs (Article-122 of women in Iranian cities.
the Indian constitution prohibits courts not to
inquire into proceedings of parliament)  India participated in the MSC 2023.
o Right to punish for its contempt 72. a
70. c Omorgus Khandesh
Performance Assessment, Review, and Analysis Scientists have discovered a new Indian beetle species
of Knowledge for Holistic Development Omorgus Khandesh.
(PARAKH)
 It belongs to Trogidae family and thus, there are
 PARAKH is India's 1st national assessment a total of 14 extant species of this family in
regulator. India.
 It will work on setting norms, and guidelines for  Features – They are sometimes called hide
student assessment and evaluation for all beetles as they tend to cover their body under
recognised school boards in the country. the soil and hide.
 It will bring an equivalence in terms of the  They are not photogenic; they are usually black
academic standards among learners. or grey and encrusted in dirt.
 It was established within the education survey  It is necrophagous and is, therefore, also
division of the National Council of called a keratin beetle.
Educational Research and Training
(NCERT).  Necrophagy is the feeding behaviour of an
organism that eats carrion from another animal
 It has been launched as part of the that it did not kill.
implementation of the National Education
Policy (NEP)-2020.  Significance - It is important for forensic
science as it helps detect the time of death of an
71. a animal or human.
Munich Security Conference (MSC)

CHENNAI |SALEM| MADURAI | COIMBATORE DELHI | BANGALORE | THIRUVANANTHAPURAM


www.shankariasacademy.com 39
www.iasparliament.com

Other Important terms  Impacts - It is associated with an increased


risk of a rare form of liver cancer, as well as
 Autophagy – It is a process by which a cell primary liver cancer, brain and lung cancers,
breaks down and destroys old, damaged, or lymphoma, and leukaemia.
abnormal proteins and other substances in its
cytoplasm.  PFAS (Per- and polyfluorinated alkyl
substances) - New study has found alarming
 Parthenogenesis – It is a form of levels of toxic PFAS, also known as “forever
reproduction in which an egg can develop into chemicals” in Norwegian Arctic ice which may
an embryo without being fertilized by a sperm. affect wildlife.

 Macrophage – A type of white blood cell that  They are large chemical family of over 4,700
surrounds and kills microorganisms, removes highly persistent chemicals that don't occur in
dead cells, and stimulates the action of other nature.
immune system cells.
 Impacts - They hardly degrade in natural
73. a environment and have been found in blood of
people and wildlife all round world. It can be
Bisphenol A (BPA) toxic to both humans and wildlife.
A study has highlighted the accelerated breeding in 74. b
mosquitoes due to presence of Bisphenol A (BPA) in
urban drains. World Sustainable Development Summit
(WSDS)
 Usage - Bisphenol A is primarily used in
production of polycarbonate plastics.  It is an annual flagship multistakeholder
convening, which was previously called
 It is also used in shatterproof windows, as Delhi Sustainable Development
eyewear, water bottles, and epoxy resins. Summit.
 Impacts - It can leach into food and beverages  Instituted in - 2001.
as well, exposing adults to ill effects of it such as
high blood pressure, diabetes and  Aim - To mobilize opinion-makers to address
cardiovascular disease. issues concerning sustainable development
and climate change.
 It can disrupt the endocrine system by
interfering with hormones and affects brain  It is only independent international Summit on
and prostate gland of foetuses, infants and sustainable development and environment,
children. based in the Global South.

 Vinyl chloride - It is a colourless industrial  Organised by - TERI (the Energy and


gas primarily used to manufacture polyvinyl Resources Institute).
chloride (PVC), a form of plastic fashioned as
pipes and wire coatings, among other similar  TERI - It is an independent, multi-
applications. dimensional organization, with
expertise in research, policy, consultancy and
implementation.

 Main Partners - Ministries of Earth Sciences,


New and Renewable Energy and Environment,
Forests, and Climate Change (MoEFCC).

 Act4Earth Initiative is launched


during WSDS 2022 to fast-track meeting of
global goals on climate and sustainable
development

75. a

Seismic Gaps

 Seismic Gaps - It is a section of an active fault


which has the potential to produce
significant earthquakes.

CHENNAI |SALEM| MADURAI | COIMBATORE DELHI | BANGALORE | THIRUVANANTHAPURAM


www.shankariasacademy.com 40
www.iasparliament.com

 Any large or longstanding gap in displacement 78. b


is considered to be the fault segment most likely
to suffer future earthquakes. Urban 20 (U20)

 There 3 Seismic gaps in Himalayas in India:  It is a city diplomacy initiative, one of the
Engagement Groups of G20.
o Assam Gap which was formed
between the 1934 Bihar-Nepal and  Launched in – 2017, at the One Planet
1950 Assam earthquake ruptures, has Summit in Paris.
the potential to cause at least 3 great
earthquakes.  Functions – It provides a platform for cities
from G20 countries to facilitate discussions
o Central Gap formed between 1905 on various important issues of urban
Kangra and 1934 Bihar-Nepal development.
earthquakes, has the potential to cause
3 great earthquakes.  It includes climate change, social inclusion,
sustainable mobility, affordable housing, and
o Kashmir Gap that lies west of financing of urban infrastructure and propose
the1905Kangra earthquake rupture, collectives.
this has potential to cause at least 2
great earthquakes.  It facilitates a productive dialogue between
the national and local governments.
76. b
79. a
Participatory Notes (P-Notes)
 Earth’s magnetosphere – An area around
 P-Notes – They are financial instruments Earth controlled by its magnetic field which
required by investors or hedge funds to invest protects its inhabitants from most of the
in Indian securities without having to register particles emitted by sun.
with the Securities and Exchange Board of
India (SEBI).  Geomagnetic Storm (GMS) – It is
a disturbance in earth’s magnetosphere.
 They are considered as Offshore Derivative
Investments (ODIs).  Caused by – Massive exchange of energy from
solar wind into space environment surrounding
 Any dividends or capital gains collected from the Earth.
the securities go back to the investors.
 Categorised between G1 and G5, the latter being
 It allows non-registered investors to the strongest.
invest in the Indian market.
 Effects of GMS – It leads to formation
 They are popular investments due to the of bright red auroras.
investor remaining anonymous.
 It disrupts navigations systems like GNSS
77. b and create harmful geomagnetic induced
currents (GIC) in power grid and pipelines.
Ancient cities in Turkey
 It damages satellite electronics and expose
 Antakya is a city in south-central astronauts to higher levels of radiation.
Turkey, large parts of which have been
reduced to rubble due to an earthquake. 80. a

 It was once the ancient city of Antioch which Vivad se Vishwas


rivalled Alexandria as a major centre of early
Christianity and was a key staging point on the The Finance Ministry released the draft of Vivad se
Silk Road. Vishwas II (Contractual Disputes) scheme.

 Sanliurfa in Turkey, formerly Urfa the  Vivad se Vishwas is a voluntary settlement


Glorious, is the home of the world's oldest scheme to bring to an end the long-festering
known megalithic structures. contractual disputes of government agencies.

 It is situated at Gobekli Tepe (Potbelly Hill), a  It is the one-time settlement scheme circulated
UN World Heritage Site, in Southeastern for stakeholders’ feedback.
Anatolia.

CHENNAI |SALEM| MADURAI | COIMBATORE DELHI | BANGALORE | THIRUVANANTHAPURAM


www.shankariasacademy.com 41
www.iasparliament.com

 It contains a draft agreement between the  Initiatives – TB-free Panchayat, Shorter


litigating parties to bring finality to the TB Preventive Treatment (TPT) and Family-
contractual dispute settlement. centric care model for TB

 It is also proposed to be implemented through  Theme of 2023 World TB Day – ‘Yes! We


an online functionality on the Government e- can end TB!
Marketplace (GeM), portal.
 Leading the Way India TB Report 2023 by
 Offer settlement – For all disputes with Ministry of Health and Family Welfare was also
Central government agencies, including released during the event.
autonomous bodies, public sector banks and
financial institutions, central public sector  India’s status – It contributes the highest
firms, union territories and the National Capital number of TB cases in the world and is
Territory of Delhi and their agencies. infamously referred as the ‘TB Capital of
World’.
 Organisations like metro corporations, where
the Centre has a 50% stake, can choose to opt 82. b
out of the scheme at their discretion after
securing board approvals. International Court of Justice (ICJ)

81. b ICJ ICC

One World TB Summit Year 1946 2002

The ‘One World TB Summit’ was held in 2023 on the The Hague, The Hague,
HQ
occasion of the World TB Day (March 24). Netherlands Netherlands

 Organized by – Union Ministry of Health Independent but


Relation
and Family Welfare (MoHFW) and Official court of UN can receive cases
with UN
the ‘Stop TB Partnership’. from UNSC

 It was organised under India’s vision of ‘One Statute ICJ Statute Rome Statute
Earth, One Health’.
Individuals either
Only states that are
ratified member of
Parties either member of
ICC or accepted
UN or ICJ or both
jurisdiction of ICC

Contentious Criminal
Jurisdiction Jurisdiction and Prosecution of
Advisory opinions individuals

Settle the legal Trial criminal


dispute i.e. offenses
sovereignty, i.e. genocide,
Subject
boundary disputes, crimes against
matter
maritime disputes, humanity, war
trade, natural crimes, crimes of
resource aggression

Instituted by Appeal
Appeal No
chamber

India Member Not a member

83. c

Office of Whip

 Term ‘whip’ – It is derived from the old


British practice of “whipping in” lawmakers to
follow the party line.

CHENNAI |SALEM| MADURAI | COIMBATORE DELHI | BANGALORE | THIRUVANANTHAPURAM


www.shankariasacademy.com 42
www.iasparliament.com

 Provisions – It is mentioned neither in The 5th UN Conference on the Least Developed


the Constitution of India nor in the Rules of Countries (LDC5) concluded with adoption of the ‘Doha
the House nor in a Parliamentary Statute. Political Declaration’.

 The office of Whip is a Parliamentary  The 2nd part of LDC5 conference focuses on
convention. implementing the Doha Programme of Action
(DPoA).
 In parliamentary parlance, a whip may refer to
both  Doha Programme of Action – It is a 10-
year plan (2022-2031) to put world’s 46
o A written order to members of a most vulnerable countries back on track to
party in the House to abide by a certain achieving the UN Sustainable Development
direction, and Goals (SDGs).

o A designated official of the  6 key focus areas – Eradicating poverty,


party who is authorised to issue such a leveraging science & innovation, supporting
direction. structural transformation, enhancing
international trade, addressing climate change
 Position – In India, all parties can issue whips and mobilizing international support.
to their members
 5 key deliverables – An online university, a
 Parties appoint a senior member from among graduation support package, a food stock
their House contingents to issue whips. holding solution, an investment support centre
and a crisis mitigation and resilience building
 This member is called a chief whip, and he/ she mechanism.
is assisted by additional whips.
 Doha Development Round – It is a WTO
 Function - A whip may require that party trade negotiation whose goal was to lower
members be present in the House for an trade barriers all around the world, allowing for
important vote, or that they vote only in a more global trade.
particular way.
 It is also known as the Doha Development
 Degree of importance of Whip - Whips can Agenda.
be of varying degrees of seriousness and can be
inferred from the number of times an order  Vienna Convention on Diplomatic
is underlined. Relations (1961) – It provides the framework
for the establishment, maintenance and
 Usually, to inform party termination of diplomatic relations on a basis
A one-line
members of a vote. of consent between independent sovereign
whip
States.
Underlined  Allows them to abstain in case
once they decide not to follow the  AUKUS Deal – It is a 2021 defence
party line. deal between Australia, the UK and the US to
help Australia deploy nuclear-powered
A two-line  Directs them to be present submarines in the Pacific region.
whip during the vote.
85. c
Underlined  Abstention from voting invites
Twice more scrutiny from party. Foreign lawyers in India

The Bar Council of India (BCI) has allowed foreign


 Strongest whip. lawyers and law firms to practise in India and notified
in the official gazette.
A three-line  Employed on important
whip occasions such as the second  BCI notified the Rules for ‘Registration and
reading of a Bill or a no- Regulation of Foreign Lawyers and Foreign
Underlined confidence motion. Law Firms in India, 2022’.
Thrice
 Places an obligation on  Rules – The foreign lawyers and firms can
members to toe the party line. now set up offices in India and practice
transactional and corporate work on a
84. d reciprocal basis.
Doha Political Declaration

CHENNAI |SALEM| MADURAI | COIMBATORE DELHI | BANGALORE | THIRUVANANTHAPURAM


www.shankariasacademy.com 43
www.iasparliament.com

 They are entitled to register with BCI to practise in 2021 by Department of Telecommunications
in India if they are entitled to practise law in (DoT) under Ministry of Communications.
their home countries.
 TIG-6G – It has members from various
 Foreign lawyers will only be allowed to advice Ministries/Departments, research and
on international laws. development institutions, academia,
standardization bodies, Telecom Service
 They will not be permitted to appear before Providers and industry/
any Indian courts and tribunals and cannot
practise Indian law.  Aim – To design, develop and deploy 6G
network technologies that provide ubiquitous
 2018 Bar Council of India vs A.K. Balaji - intelligent and secure connectivity for high
Supreme Court judgment forbade foreign quality living experience for the world.
law firms or foreign lawyers from
practising in litigation or non-  India will launch a 6G Mission that holistically
litigation in India. combines all associated technologies,
supported by an adequate financial backup.
86. a
 Implemented in – 2 phases, the 1st one from
Basic Animal Husbandry Statistics, 2022 2023 to 2025 and the 2nd one from 2025 to
2030.
 It provides production estimates of 4 Major
Livestock Products (MLPs) Milk, Egg, Meat  Significance – It will provide an R&D
and Wool. platform to start-ups, researchers, industry and
other broadband wireless applications in India
 It also contains information like livestock like e-Governance, smart cities, rural
population as per latest 20th Livestock Census, Broadband or other Digital India initiatives
import and export data of livestock etc. under Atmanirbhar Bharat.

 Released by – Department of Animal 88. c


Husbandry and Dairying under Ministry of
Fisheries, Animal Husbandry & Dairying. Indo-Pacific Oceans Initiative

 Implemented by – Animal Husbandry Italy had decided to join in the Indo Pacific Ocean
Statistics (AHS) Division of Department of Initiative.
Animal Husbandry & Dairying (DAHD).
 It is an open, non-treaty-based
 Livestock Census (LC) & Integrated Sample initiative for countries to work together for
Survey (ISS) are the 2 components of this cooperative and collaborative solutions to
statictics. common challenges in the region.

 Findings – Per-capita availability of milk  Genesis – It is an initiative of the


(2021-22) is 444 gram/day. Government of India and builds upon the
“Security and Growth for All in the Region”
o Top milk producing States – (SAGAR) initiative.
Rajasthan, UP, MP.
 It was announced by India at the 14th East Asia
o Top meat producing States – Summit (EAS) held in Bangkok, Thailand in
Maharashtra, UP, West Bengal. 2019.

o Top wool producing states/UTs –  Aim – To restore maritime stability and


Rajasthan, J&K, Gujarat. strengthen international governance in this
strategically vital region.
o Top egg producing States – Andhra
Pradesh, Tamil Nadu, Telangana.  Knowledge Partners – Indian Council of
World Affairs (ICWA) among others.
87. b
 7 pillars – Maritime security, Maritime
Bharat 6G Vision ecology, Maritime resources, Capacity building
& Resource sharing, Disaster Risk Reduction
India released “Bharat 6G Vision” document which
Management, Science, Technology & Academic
eyes 6G services rollout by 2030 and launched the 6G
Cooperation and Trade, Connectivity &
research and development test bed.
Maritime Transport.
 Prepared by – Technology Innovation
Group on 6G (TIG-6G) that was constituted

CHENNAI |SALEM| MADURAI | COIMBATORE DELHI | BANGALORE | THIRUVANANTHAPURAM


www.shankariasacademy.com 44
www.iasparliament.com

 Partners – It includes all major dialogue Gujarat government has proposed a 2nd home for
partners of ASEAN and Japan, Australia and Asiatic lions in Gujarat at Barda Wildlife Sanctuary
the US. (BWS).

 Significance – It would also be an important  Centre has given in-principle approval to


contribution to the UN Decade of Ocean make Barda as 2nd home for Asiatic
Science for Sustainable Development being Lions in Gujarat as proposed by Project
observed from 2021-2030. Lion.

89. d  Project Lion – A landscape ecology-based


conservation of Asiatic Lion in Gujarat by
Kappaphycus alvarezii integrating conservation and eco-development.

The coral reefs of Valai Island in the Keelakarai group  It was launched in 2020 on lines of Project
and Krusadai Island of Mandapam group were dying Tiger and Project Elephant.
due to stress from Kappaphycus invasion.
 Asiatic Lions – They are slightly smaller than
 Kappaphycus alvarezii is a red alga African lions.
(seaweed) and its actual live colour is green or
yellow. o Scientific Name – Panthera leo leo.

 Habitat – Native to the Indo-Pacific region.  Habitat – They are found only in India, in 5
protected areas in Gujarat.
 It has been widely introduced and cultivated in
tropical regions for commercial purposes. o Gir National Park, Gir Sanctuary, Pania
Sanctuary, Mitiyala Sanctuary, and
 Significance – Used in the production of an Girnar Sanctuary.
industrially lucrative polymer called
Carrageenan.  Conservation status

o Carrageenan – A substance o IUCN Red List – Vulnerable


extracted from red and purple (Recategorised in 2024)
seaweeds, used as a thickening
or emulsifying agent in food o Wildlife (Protection) Act 1972 –
products. Schedule I

 Impact – It attaches to the coral colonies and o CITES – Appendix I


starts forming a thick gelatinous unbreakable
mat over them, eventually smother and kill 91. b
corals.
Hyderabad Charter
 Seagrass beds, which are Dugong’s prime
 The Hyderabad Charter was drafted by the
foraging grounds are likely to be impacted.
Gender in Physics Working Group of the Indian
Horseshoe crab species Physics Association in 2019, during a national
conference organised at the University of
 Scientists have urged Odisha government to Hyderabad.
immediately come up with a robust protection
mechanism for horseshoe crabs.  The Hyderabad Charter aims to address and
resolve gender gaps in physics
 4 extant species – American, tri-spine, education and research in India.
coastal and marine
 It is an excellent and thorough roadmap for
 In India - 2 species (coastal horseshoe crab physicists across India to follow and implement
and the mangrove horseshoe crab) to create a level playing field for all.

 Odisha is their largest habitat in India.  The gender gap in physics is higher than any
other Science fields.
 It is in the Schedule IV of the Wildlife
Protection Act 1972, under which the catching  Fundamental Principles of Hyderabad
and killing of a horseshoe crab is an offence. Charter

90. a o People of all genders have equal


potential to excel in all aspects of
Asiatic Lions physics practice.

CHENNAI |SALEM| MADURAI | COIMBATORE DELHI | BANGALORE | THIRUVANANTHAPURAM


www.shankariasacademy.com 45
www.iasparliament.com

o Utilizing the talents of all is essential with surfaces and materials (e.g. bedding,
for the physics enterprise to achieve its clothing) contaminated with these fluids.
full potential.
 Symptoms – Illness caused by Marburg virus
92. c begins abruptly, with high fever, severe
headache and severe malaise. Muscle aches and
Osaka Blue Ocean Vision pains are a common feature.
 Osaka Blue Ocean Vision was agreed at the G20  Treatment - There is currently no available
Osaka Summit, 2019 treatment for MVD.
 It aims to reduce additional pollution by  Supportive care, including providing adequate
marine plastic litter to zero by 2050 hydration, pain management and treatment of
through a comprehensive life-cycle approach. symptoms as they arise under professional care
remains the safest and most effective way to
 This includes reducing the discharge of manage MVD.
mismanaged plastic litter by improved waste
management and innovative solutions. 94. b

 It is in line with SDG 14, Conserve and PM MITRA Scheme


sustainably use the oceans, seas and marine
resources.  Pradhan Mantri Mega Integrated Textile Regi
on and Apparel.
 Plastic waste makes up 80% of all marine
pollution.  PM MITRA scheme was launched in 2022.

93. d  PM MITRA Parks is inspired by the 5F


vision of Hon’ble Prime Minister - Farm to
Marburg virus disease (MVD) Fibre to Factory to Fashion to Foreign.

 Marburg virus disease (MVD), earlier known as  It will help in creating world-class industrial
Marburg hemorrhagic fever, is a severe, often infrastructure which would attract large scale
fatal hemorrhagic fever. investment including FDI and encourage
innovation and job creation within the sector.
 It is a highly virulent disease that causes
haemorrhagic fever, with a fatality ratio of up to  The Ministry of Textiles will oversee the
88%. execution of PM MITRA Parks.
 It is in the same family as the virus that causes  The Parks are to come up in states such as Tamil
Ebola virus disease. Nadu, Telangana, Gujarat, Karnataka, Madhya
Pradesh, Uttar Pradesh and Maharashtra.
 Marburg and Ebola diseases are both
similar, rare diseases, although caused  A Special Purpose Vehicle owned by Centre
by different viruses. and State Government will be set up for each
park which will oversee the implementation of
 It was first detected in 1967 after simultaneous the project.
outbreaks in Marburg and Frankfurt in
Germany and in Serbia.  The Ministry of Textiles will provide financial
support in the form of Development Capital
 Subsequently, outbreaks and sporadic cases Support up to Rs. 500 crore per park.
have been reported in Angola, Democratic
Republic of the Congo, Kenya, South Africa. 95. c

 Human infection with Marburg virus disease Mission Sahbhagita


initially results from prolonged exposure to
mines or caves inhabited by Rousettus bat  Aim - To promote the wise use and
colonies. participatory conservation of wetlands.

 Human-to-human transmission of  It was launched by the Ministry of


Marburg virus can occur through direct Environment, Forest and Climate Change
contact. (MoEF&CC) on the occasion of Azadi Ka Amrit
Mahotsav.
 It can spread through broken skin or mucous
membranes with the blood, secretions, organs  ‘Save Wetlands Campaign’ is part of this
or other bodily fluids of infected people, and Mission.

CHENNAI |SALEM| MADURAI | COIMBATORE DELHI | BANGALORE | THIRUVANANTHAPURAM


www.shankariasacademy.com 46
www.iasparliament.com

 National Plan for Conservation of  Cosmic inflation is a theory that the universe
Aquatic Ecosystems (NPCA) – Is a expanded exponentially fast for a fraction of a
centrally-sponsored scheme that manages and second after the Big Bang.
conserves wetlands and lakes in India.
 The Big Crunch is a hypothetical scenario for
 NPCA was formed in 2015 by combining the the end of the universe. It's based on Einstein's
National Wetlands Conservation Programme Theory of General Relativity and describes how
and the National Lake Conservation Plan. the universe will end as a result of the Big Bang.

 Wetlands Rejuvenation Programme – 97. b


Aims to undertake systematic rejuvenation of
selected wetlands on the basis of well-defined. Colombo Security Conclave (CSC)

 The Wetlands Rejuvenation Programme is part  Colombo Security Conclave (CSC) is a mini-
of the National Plan for Conservation of Aquatic lateral group operating in the Indian Ocean
Ecosystems (NPCA). region.

 Mission LiFE – An India-led global mass  Member countries – India, Sri Lanka,
movement to nudge individual and community Maldives and Mauritius.
action to protect and preserve the environment.
 Observers – Bangladesh and the Seychelles.
96. c
 The small group provides an opportunity for
Gravitational Lensing India to address its own strategic concerns in
the Indian Ocean while providing an
Astronomers have discovered one of the biggest black opportunity for the island and littoral nations to
holes ever found due to Gravitational lensing. address their own challenges.

 Gravitational Lensing is the phenomenon  The CSC countries are all members of the two
where a foreground object (a galaxy or a black region-wide Indian Ocean groupings,
hole) bends the light from a more distant the Indian Ocean Rim Association
object behind it. (IORA) and the Indian Ocean Naval
Symposium (IONS).
 It warps the space around the massive
foreground object, distorting the image of a  In March 2022, the group adopted an agenda of
background object into rings, arcs or multiple 5 pillars, that includes:
points of light.
1. Maritime safety and security
 The foreground object acts as a gravitational
lens and the distant object in the background is 2. Countering terrorism and radicalisation
magnified.
3. Combating trafficking and transnational
 Gravitational lenses help reveal dark matter in organised crime
galaxies.
4. Cybersecurity and protection of critical
 Einstein ring is an example of gravitational infrastructure and technology
lensing.
5. Coral Triangle Initiativeassistance and
 Aberration of light is an astronomical disaster relief
phenomenon that makes celestial objects
98. b
appear to move. Gravitational lensing is the
study of how light rays bend from their straight Millets
path due to the curvature of spacetime.
 Millets are small-seeded grasses belonging to
the botanical family Poaceae.

 They constitute an important source of food


and fodder for millions of resource-poor
farmers and play a vital role in ecological and
economic security of India.

 They are good source of minerals like iron, zinc,


and calcium.

CHENNAI |SALEM| MADURAI | COIMBATORE DELHI | BANGALORE | THIRUVANANTHAPURAM


www.shankariasacademy.com 47
www.iasparliament.com

 They are gluten-free and have a low of Diclofenac, a drug used to treat livestock
glycemic index, making them ideal for which is toxic to vultures.
people with celiac disease or diabetes.
 When the vultures eat the carcass of an animal
 These millets are also known as "coarse cereals" recently treated with Diclofenac, a non-
or "cereals of the poor". steroidal anti-inflammatory drug (NSAID) it
causes visceral gout.
 Millets are highly adaptive to a wide range
of ecological conditions.  Visceral gout is a disease where uric acid
crystals accumulate on organs, particularly the
 Indian millets are a group of nutritiously rich, kidneys and resulted in kidney failure.
drought tolerant and mostly grown in the arid
and semi-arid regions of India.  After the findings, the government banned the
use of Diclofenac in 2008.
 Indian Millets are nutritionally superior to
wheat and rice as they are rich in protein,  Fatal four NSAIDs Vultures – Diclofenac;
vitamins and minerals. Aceclofenac; Nimesulide; Ketoprofen

 India is among the top 5 exporters of millets in  Alternatives - Tolfenamic and Meloxicam
world. were considered safe for vultures.

 India is the largest producer as well as the 100. c


largest exporter of cereal products in the world.
Island Rule
 The main millet-growing states in India are
Rajasthan, Maharashtra, Karnataka, Andhra  Island Rule is a hypothesis that
Pradesh, and Madhya Pradesh. mammals change their size on islands
depending on resources available in
Significance their environment.

 Less Water intensive – They are rain-fed  It is the tendency of mammal populations on
grains which have low requirements of water islands to have body sizes that are different
and fertility when compared to other popular from their mainland counterparts.
cereals.
 Specifically, large mammals seem to shrink
 Anti-oxidant property – Millets with “referred to as dwarfism, while small mammals
phytates, polyphenols, tannins, anthocyanins, appear to enlarge known as gigantism.
phytosterols and pinacosanols present in it
having important role in aging and metabolic  It states that small mammals like rats may
diseases. evolve and turn bigger in size over time when
they are isolated on an island with more
 All millets possess high antioxidant resources and fewer predators.
activities.
 By the same token, larger mammals like
 Lower Cholestrol – Niacin in millet can help elephants begin to develop smaller bodies if
lower cholesterol. they happen to become isolated in an island
where the availability of resources is scarce.
99. b

Vultures

 Vultures play a very important role in our


ecosystem as the primary removers of
carrion.

 Vultures constitute the natural animal disposal


system and keep our environment clean.

 The decrease in Vulture population may result


in an increase in population of rats and feral
dogs (main scavenging species in the absence of
vultures) and the spread of diseases.

 Drastic decrease of vulture population in the


recent decades was notably due to the use

CHENNAI |SALEM| MADURAI | COIMBATORE DELHI | BANGALORE | THIRUVANANTHAPURAM


www.shankariasacademy.com 48
www.iasparliament.com

TEST – II
1. Consider the following statements regarding b. 2 only
disqualification of Member of Parliament (MP)
c. Both 1 and 2
1. Constitution does not prescribe any grounds for
d. Neither 1 nor 2
disqualification of an MP.

2. An MP convicted of any offence and sentenced


to imprisonment for not less than 2 years shall 4. Consider the following statements with respect to
be disqualified with immediate effect. Aravalli Green Wall Project
Which of the statement(s) given above is/are correct? 1. It aims to create green belt buffer around
Aravalli Mountain range covering the states of
a. 1 only
Haryana, Rajasthan, Gujarat and Delhi.
b. 2 only
2. The project involves planting native species of
c. Both 1 and 2 trees along with rejuvenating surface water
bodies such as ponds, lakes and streams.
d. Neither 1 nor 2
Which of the statement(s) given above is/are correct?

a. 1 only
2. Consider the following statements regarding
Systemically Important Banks (SIBs) b. 2 only

1. SIBs are perceived as banks that are ‘Too Big To c. Both 1 and 2
Fail’ (TBTF).
d. Neither 1 nor 2
2. India has 3 Domestic Systemically Important
Banks (D-SIBs).
5. Consider the following statements with respect to
3. No Indian bank is listed as Global Systemically
World Happiness Report
Important Banks (G-SIBs).
1. It is based on people's own assessment of their
4. In India, Reserve Bank of India (RBI) classifies
happiness, as well as economic and social data.
a bank as D-SIBs.
2. It is released by United Nations Sustainable
How many of the above statements is/are correct?
Development Solutions Network (UN-SDSN)
a. Only one along with other partners.

b. Only two Which of the statement(s) given above


is/are incorrect?
c. Only three
a. 1 only
d. All four
b. 2 only

c. Both 1 and 2
3. Consider the following statements regarding
Manyam Rebellion: d. Neither 1 nor 2

1. The major cause of this rebellion is the


enactment of Madras Forest Act of 1882 which
6. Consider the following statements with respect to
banned the collection of minor forest produce
Multi-Party Interim Appeal Arbitration Arrangement
by the tribal communities.
(MPIA)
2. It was led by Alluri Sitharama Raju, who was
1. It is an alternative system for resolving WTO
honoured with the title, “Manyam Veerudu”
disputes that are appealed in the absence of a
(Hero of the Jungle).
functioning WTO Appellate Body.
Which of the statement(s) given above is/are correct?
2. India is a member of MPIA since 2020.
a. 1 only
Which of the statement(s) given above is/are correct?

CHENNAI |SALEM| MADURAI | COIMBATORE DELHI | BANGALORE | THIRUVANANTHAPURAM


www.shankariasacademy.com 49
www.iasparliament.com

a. 1 only b. Only two

b. 2 only c. All three

c. Both 1 and 2 d. None

d. Neither 1 nor 2

11. Consider the following pairs:

7. ‘Suspension of Operations (SoO) Agreement’, often Exercises with India Countries


seen in the news, is a tripartite ceasefire agreement
among Government of India,? 1. LA Perouse – France

a. Government of Assam and Bodo community 2. Konkan – United Kingdom

b. Government of Chhattisgarh and Naxal groups 3. DUSTLIK – Kazakhstan

c. Government of Manipur and Kuki insurgent How many of the pair(s) given above is/are correctly
groups matched?

d. Government of Nagaland and Naga insurgent a. Only one


groups
b. Only two

c. All three
8. Desawari is a GI tagged betel leaf variety cultivated
in? d. None

a. Bihar

b. Karnataka 12. Consider the following statements with respect to


Dragonfly Mission
c. Tamil Nadu
1. It was launched by the European Space Agency
d. Madhya Pradesh (ESA) under its New Frontiers Program.

2. It aims to explore the chemistry and habitability


of Jupiter’s largest moon, Titan.
9. Global Greenhouse Gas Monitoring Infrastructure
(GGMI) was launched by? Which of the statement(s) given above is/are correct?

a. IQ Air a. 1 only

b. UN Environment Programme b. 2 only

c. World Meteorological Organization c. Both 1 and 2

d. Jointly by a and b d. Neither 1 nor 2

10. Consider the following statements with respect to 13. Consider the following pairs:
the New India Literacy Programme (NILP)
Places in News Location
1. It is a Central sector scheme with a goal of
reaching 5 crore non-literates. 1. Vanuatu Island – Ecuador

2. All the non-literate above 15 years of age can 2. Mount Merapi – Italy
avail the benefits of the scheme.
3. Solomon Islands – Indian Ocean
3. Volunteerism forms the primary method for
teaching and learning within the scheme. How many of the above pairs are correct?

How many of the statement(s) given above is/are a. Only one


correct?
b. Only two
a. Only one

CHENNAI |SALEM| MADURAI | COIMBATORE DELHI | BANGALORE | THIRUVANANTHAPURAM


www.shankariasacademy.com 50
www.iasparliament.com

c. All three 17. Consider the following statements with respect to


UN 2023 Water Conference
d. None
1. It is only the 2nd UN Conference dedicated to
water.

14. Consider the following statements with respect to 2. Water Action Decade (2018-2028) is one of 5
Parvatmala Project themes of this conference.
1. It is the National Master Plan for Multi-modal Which of the statement(s) given above
Connectivity. is/are incorrect?

2. It is also known as National Ropeway a. 1 only


Development Programme.
b. 2 only
3. It was launched by the Ministry of Road
Transport and Highways. c. Both 1 and 2

4. It aims to decongest mountainous areas where d. Neither 1 nor 2


road and train connectivity cannot be upgraded
beyond a limit.

How many of the above statement(s) is/are correct? 18. Global Education Monitoring Report is published
by?
a. Only one
a. World Bank
b. Only two
b. World Economic Forum
c. Only three
c. United Nations Development Program
d. All four
d. United Nations Educational, Scientific and
Cultural Organization

15. Which of the following is incorrect regarding


Green Tug Transition Programme (GTTP)?
19. Consider the following statements:
a. It is an initiative of Ministry of Environment,
Forest and Climate Change. Statement-I: Angel taxes are taxes on funds raised by
startups if they exceed the Fair Market Value (FMV) of
b. It has been set for initial Green Tugs to start the company.
working in all major ports by 2025.
Statement-II: Angel taxes aims to discourage
c. It is in line with India’s Mission LiFE movement laundering of unaccounted money via unlisted firms
and to become carbon neutral by 2070. disguised as capital investments.

d. It will play a crucial role towards achievement of Select the correct answer using the codes given below:
UN’s Sustainable Development Goal (SDG 14).
a. Both Statement-I and Statement-II are correct
and Statement-II is the correct explanation for
Statement-I
16. Which of the following best describes the term
“Tashkent Treaty”? b. Both Statement-I and Statement-II are correct
and Statement-II is not the correct explanation for
a. It is a military alliance between US and Eastern Statement-I
European countries.
c. Statement-I is correct but Statement-II is
b. It is a military alliance of Eurasian states with its incorrect
headquarters in Russia.
d. Statement-I is incorrect but Statement-II is
c. It is an accord signed by India and Pakistan to correct
end the 17-day war in 1965.

d. It is a treaty signed between Russia and Ukraine


for a temporary ceasefire for humanitarian 20. Consider the following statements about National
assistance in war affected areas in Ukraine. Platform for Disaster Risk Reduction (NPDRR)

CHENNAI |SALEM| MADURAI | COIMBATORE DELHI | BANGALORE | THIRUVANANTHAPURAM


www.shankariasacademy.com 51
www.iasparliament.com

1. It is a multi-stakeholder national platform 23. Consider the following statements with respect to
established in 2013. Central Bureau of Investigation (CBI)

2. It will be chaired by the Union Home Minister. 1. It is a statutory body established under the
Delhi Special Police Establishment Act, 1946.
3. To review the National Disaster Management
Policy is one of the objectives of NPDRR. 2. The superintendence of CBI related to
investigation of offences under the Prevention
How many of the statement(s) given above is/are of Corruption Act, 1988 lies with the Central
correct? Vigilance Commission (CVC).

a. Only one 3. Supreme Court and High Courts can order CBI
to investigate a crime anywhere in the country
b. Only two without the consent of the respective State.
c. All three How many of the above statements is/are correct?
d. None a. Only one

b. Only two
21. Consider the following statements c. Only three
Statement-I: Prepaid Payment Instruments (PPIs) d. All four
can only be used in Indian rupees.

Statement-II: Prepaid payment instruments are


payment instruments that facilitate purchase of goods 24. Consider the following with respect to G7 grouping
and services against the value stored on such
instruments. 1. Build Back Better World (BBBW) is an initiative
of G7 to provide an alternative to China’s Belt
Select the correct answer using the codes given below: and Road initiative (BRI).

a. Both Statement-I and Statement-II are correct 2. G7 has pledged to end new plastic pollution by
and Statement-II is the correct explanation for 2040.
Statement-I
Which of the above statement(s) is/are correct?
b. Both Statement-I and Statement-II are correct
and Statement-II is not the correct explanation for a. 1 only
Statement-I
b. 2 only
c. Statement-I is correct but Statement-II is
incorrect c. Both 1 and 2

d. Statement-I is incorrect but Statement-II is d. Neither 1 nor 2


correct

25. Consider the following statements with respect to


22. Consider the following statements regarding Lab-Grown Meat
Minerals Security Partnership (MSP)
1. It is also called as cell-based meat.
1. It is a United States led initiative to catalyse
public and private investment in responsible 2. Stem cells are being used in the process of
critical minerals supply chains globally. producing lab-grown meat.

2. India is a member in this partnership. 3. It is considered vegan, since it does not involve
killing of animals.
Which of the statement(s) given above is/are correct?
How many of the statements given above are correct?
a. 1 only
a. Only one
b. 2 only
b. Only two
c. Both 1 and 2
c. All three
d. Neither 1 nor 2
d. None

CHENNAI |SALEM| MADURAI | COIMBATORE DELHI | BANGALORE | THIRUVANANTHAPURAM


www.shankariasacademy.com 52
www.iasparliament.com

d. Rising temperature due to global warming is one


of the major reasons for this new threat to human
26. Which of the following is/are the applications beings.
of Gum Arabic?

1. Providing texture for gum candies


29. First census of waterbodies in India was released by
2. Preventing sugar crystallization the Ministry of Jal Sakthi. Consider the following
statements in this regard.
3. Encapsulation agent in medications
1. Water bodies in rural areas are higher than
4. Shoe polish and ceramic glazing Urban areas.
Select the correct answer using the codes given below: 2. Man-made water bodies are more in numbers
than the natural water bodies.
a. All except 2
3. Privately owned waterbodies are more than the
b. All except 3 government-owned.
c. All except 4 How many of the above statement(s) are correct?
d. All four a. Only one

b. Only two
27. Consider the following statements regarding c. Only three
CLEANaction
d. None
1. It was launched at COP26 of UNFCCC to ensure
that the impact on nature by clean energy
systems is fully accounted for prior to any
installation. 30. Consider the following pairs

2. Birdlife International and the Worldwide Fund Portal Purposes


for Nature (WWF) are founding members of
this forum. 1. Abhilekh Patal – Online system for tracing seeds

Which of the statement(s) given above is/are correct? 2. Yuva – Promoting young start-ups

a. 1 only 3. SATHI – Digital repository of Historical records

b. 2 only How many of the above pair(s) are correct?

c. Both 1 and 2 a. Only one

d. Neither 1 nor 2 b. Only two

c. All three

28. The 1st case of Chondrostereum purpureum d. None


infecting human beings has been reported from
Kolkata. Which of the following is incorrect with
respect to Chondrostereum purpureum?
31. Consider the following statements with respect to
a. It is a plant fungus that causes silver leaf disease the UN Statistical Commission?
in plants, particularly those in the rose family.
1. It is the highest decision-making body for
b. It is a red algae that infects the leaves and international statistical activities.
branches of another plant, can be fatal for the plant
if not quickly treated. 2. The members are elected by the United Nations
Economic and Social Council on the basis of an
c. It has the capacity to infect human cells by equitable geographical distribution.
evading the process of ‘phagocytosis’ which means
‘cell eating’. 3. India has been elected to this commission from
the Asia-Pacific States group for a 4-year term.

How many of the statement(s) given above is/are


correct?

CHENNAI |SALEM| MADURAI | COIMBATORE DELHI | BANGALORE | THIRUVANANTHAPURAM


www.shankariasacademy.com 53
www.iasparliament.com

a. Only one 1. It facilitates micro credit access up-to Rs. 50


lakhs.
b. Only two
2. MUDRA is a Non-Banking Financial Institution
c. All three with the RBI.

d. None 3. These loans can be given by Commercial Banks


except Regional Rural Banks (RRBs) and Small
Finance Banks.
32. ‘Operation Kaveri’, sometimes seen in the news, How many of the statements given above are correct?
refers to?
a. Only one
a. An operation to evacuate Indian nationals from
Ukraine. b. Only two
b. An operation to evacuate Indian nationals from c. All three
Sudan.
d. None
c. An operation to provide maritime safety to
Indian ships in Red Sea.

d. An operation to assist Myanmar which has been 36. Consider the following pairs with reference to
devastated by Cyclone Mocha. Geographical Indications (GI) Products in India

GI Products States

33. Consider the following pairs: 1. Basholi Painting – Rajasthan

Reports Published by 2. Manamadurai Pottery – Tamil Nadu

1. Rule of law Index - WMO 3. Mircha Rice – Meghalaya

2. State of the Global Climate - World Justice Project How many of the pair(s) given above is/are correctly
matched?
3. State of world's Children - UNICEF India
a. Only one
How many of the pair(s) given above is/are correctly
matched? b. Only two

a. Only one c. All three

b. Only two d. None

c. Only three

d. None 37. Consider the following pairs:

Terms Relevance

34. With reference to tribal communities in India, ‘Reh’ 1. Kuttamperoor – River in Kerala
and ‘Ke-Meh-Ha’ are two important festivals celebrated
by? 2. Hakki Pikki – Tribal community in Karnataka

a. Gond 3. Uttaramerur – Inscription in Tamil Nadu

b. Jarawa How many of the pair(s) given above is/are correctly


matched?
c. Idu Mishmis
a. Only one
d. Kattunayakan
b. Only two

c. All three
35. Consider the following statements with respect to
PM MUDRA Yojana d. None

CHENNAI |SALEM| MADURAI | COIMBATORE DELHI | BANGALORE | THIRUVANANTHAPURAM


www.shankariasacademy.com 54
www.iasparliament.com

38. Which of the following is/are the eligibility for d. It celebrates the movement of the Sun,
National Party Status? underlining the concept of ‘One Sun, One Earth’.

1. It should secure at least 2% of the seats in the


Lok Sabha, with its candidates having been
elected from at least 3 states. 41. ‘Stockholm Agreement’, sometimes seen in the
news, refers to?
2. Having representation of at least one Member
of Parliament for every 25 members or any a. It is a UN brokered agreement signed by Yemen
fraction allotted to the state in the Lok Sabha. to free conflict related detainees.

3. Being 'recognized' as a state party in at least b. It is an agreement between factions of Northern


four states. Ireland, and governments of Britain and Ireland, to
end 30 years of violence known as ‘The Troubles’.
4. Attaining a minimum of 6% vote share in at
least four states during the preceding Lok c. It is a trade agreement proposed by World Trade
Sabha or Assembly elections. Organisation to facilitate investment flows.

Select the correct answer using the codes given below: d. A free trade agreement (FTA) that provides
foreign investors with the right to access an
a. Only one international tribunal to resolve investment
disputes.
b. Only two

c. Only three
42. Consider the following statements with respect to
d. All four Goldilock zones

1. Goldilock zone is the range of orbital distances


from a star at which liquid water can exist on
39. Consider the following statements with respect to the surface of a planet, without water boils away
UN Convention against Torture (UNCAT) or freezes.
1. It is an international human rights treaty 2. Smaller and cooler stars have their goldilocks
adopted by the UN General Assembly in 1984. zone much closer in, while bigger hotter stars
have their Goldilocks Zones further out.
2. It forbids member states to transport people to
any country where there is reason to believe Which of the statement(s) given above is/are correct?
they will be tortured.
a. 1 only
3. India has ratified the convention in 2002.
b. 2 only
Which of the above statement(s) is/are correct?
c. Both 1 and 2
a. Only one
d. Neither 1 nor 2
b. Only two

c. Only three
43. Consider the following statements
d. None
1. Software as a medical device (SaMD) is a
software that controls the device remotely.
40. Which of the following statements 2. Software in a medical device (SiMD) is a
is/are incorrect with respect to Guardian Ring software that functions completely
Program? independent of existing medical devices.
a. It is a unique innovative programme envisaged Which of the above statement(s) is/are correct?
for International Yoga Day, 2022.
a. 1 only
b. It aims to raise awareness on the importance of
protection of Earth. b. 2 only

c. It is a collaborative exercise between 79 countries c. Both 1 and 2


and United Nations organisations.
d. Neither 1 nor 2

CHENNAI |SALEM| MADURAI | COIMBATORE DELHI | BANGALORE | THIRUVANANTHAPURAM


www.shankariasacademy.com 55
www.iasparliament.com

44. Which of the following big cats are covered under d. Neither 1 nor 2
the International Big Cats Alliance (IBCA)?

1. Tiger
47. Consider the following pairs
2. Cheetah
Important Sites States
3. Lion
1. Pattanam – Kerala
4. Cougar
2. Siju Cave – Madhya Pradesh
5. Clouded leopard
3. Paigah Tombs – Telangana
Select the correct answer using the codes given below:
How many of the pair(s) given above is/are correctly
a. Only two matched?

b. Only three a. Only one

c. Only four b. Only two

d. All five c. All three

d. None

45. Consider the following pairs:

Missions Objectives 48. Consider the following pairs

1. POEM - NASA’s atmospheric observatory Rivers Drains in

2. Hakuto R - Private lunar mission of Japan 1. Danube River - Black Sea

3. JUICE - Mission to study Jupiter by the ESA 2. Rhine River - Pacific Ocean

4. Dhawan II - India’s first privately developed engine 3. Loire River - Atlantic Ocean

How many of the pair(s) given above is/are correctly 4. Po River - Adriatic Sea
matched?
How many of the pair(s) given above is/are correctly
a. Only one matched?

b. Only two a. Only one

c. Only three b. Only two

d. All four c. Only three

d. None

46. Consider the following statements:

1. India Justice Report is released by the NITI 49. Consider the following pairs:
Aayog to track the performance of States in
effectively delivering justice. GI Products Regions

2. World Development Report is an annual report 1. Cumbum Grapes – Tamil Nadu


published by the UN Development Program
(UNDP). 2. Aranmula Mirror – Andhra Pradesh

Which of the above statement(s) is/are correct? 3. Ganga – Madhya Pradesh

a. 1 only How many of the pair(s) given above is/are correctly


matched?
b. 2 only
a. Only one
c. Both 1 and 2
b. Only two

CHENNAI |SALEM| MADURAI | COIMBATORE DELHI | BANGALORE | THIRUVANANTHAPURAM


www.shankariasacademy.com 56
www.iasparliament.com

c. All three b. It is a subunit of United Nations Peace keeping


forces, working in Ukraine.
d. None
c. It is an international terrorist organisation
responsible for the civil war in France.

50. Consider the following statements with respect to d. It is a multilateral export control regime on
the Pacific Trash Vortex export controls for conventional arms and dual-use
goods and technologies.
1. The entire trash is bounded by the North Pacific
Subtropical Gyre.

2. It is located between California and Japan, and 54. Consider the following pairs:
formed due to diverging ocean currents.
1. Logistic Performance Index (LPI) - World Bank
Which of the statement(s) given above is/are correct?
2. Logistics Ease Across Different States - NITI Aayog
a. 1 only
Which of the pair(s) given above
b. 2 only is/are incorrectly matched?

c. Both 1 and 2 a. 1 onlys

d. Neither 1 nor 2 b. 2 only

c. Both 1 and 2

51. Consider the following with respect to C+C5 Summit d. Neither 1 nor 2

1. It denotes the summit of China and the five


Central Asian countries.
55. Consider the following with respect to National
2. The C+C5 summit, 2023 was held at Shaanxi Policy for Rare Diseases, 2022
Province, China.
1. The policy was formulated by the Ministry of
Which of the above statement(s) is/are correct? Health and Family Welfare.

a. 1 only 2. According to WHO, rare disease is a lifelong


disease or disorder with a prevalence of 1 or
b. 2 only less, per 1000 population.
c. Both 1 and 2 3. The policy categorises rare diseases in to three
groups with group 1 requires lifelong treatment.
d. Neither 1 nor 2
How many of the statement(s) given above is/are
correct?
52. LOCKBIT, sometimes seen in the news, refers to? a. Only one
a. A ransomware that targets macOS devices b. Only two
b. A protection software used to safeguard c. All three
important credentials
d. None
c. A newly invented technology to trace the lipid
nanoparticles in mRNA

d. A public ledger that records bitcoin transactions 56. Consider the following pairs:
in the virtual data
Villages Famous for

1. Suketi – Asia’s oldest fossil park


53. Which of the following best describes the
term “Wagner Group”? 2. Modhera – 1st solar powered village

a. It is a private Russian paramilitary unit of armed 3. Raini village – Chipko movement


force.
4. Bishnoi – Living in harmony

CHENNAI |SALEM| MADURAI | COIMBATORE DELHI | BANGALORE | THIRUVANANTHAPURAM


www.shankariasacademy.com 57
www.iasparliament.com

How many of the pair(s) given above is/are correctly 5. Tourism


matched?
Select the correct answer using the codes given below
a. Only one
a. Only two
b. Only two
b. Only three
c. Only three
c. Only four
d. All four
d. All five

57. “HydroSOS” is an operational mechanism of


assessing the current hydrological status and its likely 60. Consider the following statements with respect to
near-future outlook for all areas. Which of the following United Nations Permanent Forum on Indigenous Issues
organisation has launched this system? (UNPFII)

a. Indian Meteorological Department 1. It is a high-level advisory body to the UN


Economic and Social Council (ECOSOC).
b. United Nations Environment Program
2. It is the only UN body that deals the issues with
c. Intergovernmental Panel on Climate Change indigenous peoples.

d. World Meteorological Organisation Which of the statement(s) given above is/are correct?

a. 1 only

58. Consider the following statements with respect to b. 2 only


the National Commission for Protection of Child Rights
(NCPCR) c. Both 1 and 2

1. It has been constituted under the Commission d. Neither 1 nor 2


for Protection of Child Rights (CPCR) Act,
2005.

2. It can initiate suo motu action on non- 61. Consider the following statements with respect to
implementation of laws providing for National Capital Territory of New Delhi
protection and development of children.
1. Article 239 AA confers upon special status to
3. Under the provisions of the Right to Education Delhi.
(RTE) Act, 2009 it can inquire into complaints
relating to child's right to free and compulsory 2. It was inserted by the 69th Constitutional
education. amendment act based on the recommendations
of Balakrishnan Committee.
How many of the statements given above are correct?
3. The Constitution lays down that the strength of
a. Only one the Council of Ministers for Delhi shall not be
more than ten percent of the total number of
b. Only two members in the Assembly.

c. All three How many of the statement(s) given above is/are


correct?
d. None
a. Only one

b. Only two
59. Which of the following are the contributors of
marine litter? c. Only three

1. Plastics d. None

2. Fishing gear

3. Untreated municipal sewage

4. Shipbreaking yards

CHENNAI |SALEM| MADURAI | COIMBATORE DELHI | BANGALORE | THIRUVANANTHAPURAM


www.shankariasacademy.com 58
www.iasparliament.com

62. Match the following: b. Only two

Launched c. All three


Missions Objectives
by
d. None
1. TROPICS a. p. To study astronomical
Mission Roscosmos X-ray sources

2. XPoSat q. To improve forecasting 65. Consider the following statements:


b. ISRO
Mission hurricanes and typhoons
Statement - I: Carbon Border Adjustment Mechanism
r. To study soil and (CBAM) will encourage cleaner industrial production in
3. Luna 25 c. NASA non-European Union (non-EU) countries.
atomosphere of moon
Statement - II: CBAM is the European Union’s (EU’s)
Select the correct answer using the codes given below:
tool to put a fair price on the carbon emitted during the
a. 1-c-p; 2-b-q; 3-a-r production of goods that are entering the EU.

b. 1-a-p; 2-b-q; 3-c-r Select the correct answer using the codes given below:

c. 1-b-q; 2-c-p; 3-a-r a. Both Statement-I and Statement-II are correct


and Statement-II is the correct explanation for
d. 1-c-q; 2-b-p; 3-a-r Statement-I

b. Both Statement-I and Statement-II are correct


and Statement-II is not the correct explanation for
63. Consider the following statements: Statement-I

Statement-I: India Meteorological Department (IMD) c. Statement-I is correct but Statement-II is


is the national meteorological agency of India, incorrect
established before India’s Independence.
d. Statement-I is incorrect but Statement-II is
Statement-II: IMD is one of the six tropical cyclone correct
Regional Specialized Meteorological Centres (RSMCs).

Select the correct answer using the codes given below:


66. ANZUS Treaty was an agreement signed in 1951 to
a. Both Statement-I and Statement-II are correct protect the security of the Pacific. Which of the
and Statement-II is the correct explanation for following countries are part of this treaty?
Statement-I
1. USA
b. Both Statement-I and Statement-II are correct
and Statement-II is not the correct explanation for 2. India
Statement-I
3. China
c. Statement-I is correct but Statement-II is
incorrect 4. Australia

d. Statement-I is incorrect but Statement-II is 5. New Zealand


correct
Select the correct answer using the codes given below:

a. All except 2
64. Which of the following are the advantages of
b. All except 2 and 3
invoicing in Indian Rupee for international trade
settlements? c. All except 3, and 5
1. More competitive exports d. All five
2. Mitigate exchange rate risks

3. Reduction in demand for forex reserves 67. ‘Meiogyne Arunachalensis’, sometimes seen in the
news, refers to?
Select the correct answer using the codes given below:
a. A newly discovered tree species in Arunachal
a. Only one
Pradesh

CHENNAI |SALEM| MADURAI | COIMBATORE DELHI | BANGALORE | THIRUVANANTHAPURAM


www.shankariasacademy.com 59
www.iasparliament.com

b. A hornbill species that goes extinct in Arunachal 71. The waterway component of the Kaladan
Pradesh Multimodal Transit Transport project (KMTTP) passes
through the Kaladan River. In India, Kaladan flows in
c. A rare species of mushroom endemic to which of the following states?
Arunachal Pradesh
1. Mizoram
d. A medicinal plant that increases metabolism in
humans found in Arunachal Pradesh. 2. Tripura

3. Manipur

68. Global Greenhouse Gas Watch (G3W) which aims to 4. Nagaland


establish internationally coordinated monitoring of
greenhouse gas fluxes is and initiative of? Select the correct answer using the codes given below:

a. Climate Watch a. 1 only

b. UN Environment Programme b. 1 and 2 only

c. Climate and Clean Air Coalition c. 1, 2 and 3 only

d. World Meteorological Organisation d. 1, 2, 3 and 4

69. The Global Buddhist Summit held in 2023 has the 72. Consider the following statements with respect to
potential to leverage India’s soft power to connect the Trachoma
Buddhist population around the world. The summit was
organised by? 1. It is a bacterial eye disease caused by Chlamydia
trachomatis.
1. Union Ministry of Culture
2. Blindness from trachoma is irreversible.
2. Union Ministry of External Affairs
3. India has eliminated trachoma in 2014.
3. International Buddhist Confederation
How many of the statements given above are correct?
4. Nalanda University
a. Only one
Select the answer using the code given below:
b. Only two
a. 1 only
c. Only three
b. 1 and 3 only
d. None
c. 1, 2 and 3 only

d. 1, 2, 3 and 4
73. Consider the following spaced based observatories:

1. Thirty Meter Telescope


70. Consider the following statements with respect to
World Press Freedom Index 2. Spitzer Space Telescope

1. It is an annual ranking of countries published 3. Hubble Space Telescope


by the International Press Institute (IPI)
4. Chandra X-Ray Observatory
2. India ranks higher than all its border countries.
5. Compton Gamma-Ray Observatory
Which of the above statement(s) is/are correct?
Which of the following is/are the part of the NASA's
a. 1 only Great Observatories Program?

b. 2 only a. All except 1

c. Both 1 and 2 b. All except 5

d. Neither 1 nor 2 c. All except 1 and 2

CHENNAI |SALEM| MADURAI | COIMBATORE DELHI | BANGALORE | THIRUVANANTHAPURAM


www.shankariasacademy.com 60
www.iasparliament.com

d. All five 77. Consider the following statements with respect to


the Forum for India-Pacific Islands Cooperation
(FIPIC)

74. Consider the following pairs with respect to Drying 1. It is a multinational grouping established in
Lakes across World 2014 for cooperation between India and 14
Pacific Islands Countries.
Lakes Regions
2. FIPIC Summit, 2023 was held in Port Moresby,
1. Aral Sea – Central Asia Papua New Guinea.
2. Dead Sea – South-Western Asia Which of the statement(s) given above is/are correct?
3. Xinkai Lake – Western Asia a. 1 only
4. Toshka Lakes – North-Eastern Africa b. 2 only
How many of the pair(s) given above is/are correctly c. Both 1 and 2
matched?
d. Neither 1 nor 2
a. Only one

b. Only two
78. Consider the following statements with respect to
c. Only three the Indian Council for Cultural Relations (ICCR)
d. All four 1. It is an autonomous body under the
administrative control of the Ministry of
Culture.
75. Consider the following passage: 2. Jawaharlal Nehru, the former Prime Minister of
India, is the first President of this council.
Dating back 3,200 years, this civilization was situated
along the river Tamirabharani. Adichanallur, Korkai, 3. Based on the recommendations of Asoka Mehta
and Sivakalai are the sites associated with this Committee, ICCR assumed control of all
civilization. cultural delegation activities previously
overseen by the Department of Culture.
Identify the civilization using the above passage.
How many of the statement(s) given above is/are
a. Vedic Civilization
correct?
b. Keezhadi Civilization
a. Only one
c. Indus Valley Civilization
b. Only two
d. Porunai Civilization
c. Only three

d. None
76. Consider the following statements with respect to
the Purana Qila
79. Consider the following statements with respect to
1. It was built by Humayun as a part of his new
Periyar Tiger Reserve
city ‘Dinpanah’ in 16th century.
1. It spreads only in the Southern state of Tamil
2. It is situated on the South-eastern part of the
Nadu.
present city of New Delhi.
2. It consists of semi- evergreen and moist
Which of the above statement(s) is/are incorrect?
deciduous forests.
a. 1 only
3. Paliyans, Mala Pandarams and Ulladans are
b. 2 only some of the tribes found in the reserve.

c. Both 1 and 2 How many of the statement(s) given above is/are


correct?
d. Neither 1 nor 2
a. Only one

CHENNAI |SALEM| MADURAI | COIMBATORE DELHI | BANGALORE | THIRUVANANTHAPURAM


www.shankariasacademy.com 61
www.iasparliament.com

b. Only two 3. LIBOR and Mumbai Interbank Forward


Outright Rate (MIFOR) ceased to be a
c. Only three representative benchmark from 2023.

d. None How many of the statement(s) given above is/are


correct?

a. Only one
80. Consider the following statements with respect to
Self-Reliant India (SRI) Fund b. Only two
1. It is a SEBI registered, category-II Alternative c. All three
Investment Fund.
d. None
2. It aims to make capital investments in micro,
small and medium enterprises (MSMEs).

3. It is managed by the SBICAP Ventures (SVL). 83. In India, Civil Liability for Nuclear Damage Act,
2010 was enacted to keep in line with which of the
How many of the statement(s) given above is/are following conventions?
correct?
1. Nuclear Security Group
a. Only one
2. Indo-US Civil Nuclear Agreement
b. Only two
3. Vienna Convention on Civil Liability for
c. All three Nuclear Damage

d. None 4. Convention on Supplementary Compensation


for Nuclear Damage

Select the answer using the code given below:


81. Consider the following statements with respect to
Madden-Julian Oscillations (MJO) a. 1 and 2 only

1. It is a westward moving system of wind, cloud b. 3 and 4 only


and pressure that brings rain as it circles
around the equator. c. 2, 3 and 4 only

2. Indian Ocean Dipole (IOD) and El Nino remain d. 1, 2, 3 and 4 only


over their respective positions, while MJO is a
traversing phenomenon.

Which of the statement(s) given above is/are correct? 84. The initiative on Critical and Emerging
Technologies (iCET) is a defence partnership with India
a. 1 only and?

b. 2 only 1. Israel

c. Both 1 and 2 2. France

d. Neither 1 nor 2 3. United Kingdom

4. United States of America

82. Which one of the following is relevant to the concept Select the correct answer using the codes given below:
of Secure Overnight Financing Rate (SOFR)?
a. 4 only
1. SOFR is a benchmark overnight interest rate
and reference rate for dollar-denominated b. 2 and 3 only
derivatives and loans.
c. 1 and 4 only
2. It was established as an alternative to London
Interbank Offered Rate (LIBOR) d. 1, 2, 3 and 4

CHENNAI |SALEM| MADURAI | COIMBATORE DELHI | BANGALORE | THIRUVANANTHAPURAM


www.shankariasacademy.com 62
www.iasparliament.com

85. Which of the following statements regarding 88. Which of the following best describes the
Leatherback Turtles is/are correct? term, “Laundromat Countries”?

1. It is the largest sea turtle species in the world. a. A group of countries which serves as an
alternative supply chain against China to avoid its
2. It is the only species of sea turtle that lack scales monopoly.
and a hard shell.
b. A group of countries where labour and
Select the answer using the code given below: production cost are lower than the rest of the world.
a. 1 only c. A group of countries where natural resources are
abundant but have less economic growth than
b. 2 only countries with fewer natural resources.
c. Both 1 and 2 d. A group of countries that buys Russian oil and
sell processed products to European countries, thus
d. Neither 1 nor 2 sidestepping European sanctions against Russia.

86. Consider the following statements with respect to AI 89. Consider the following statements with respect to
Research, Analytics and knowledge Assimilation the Mpox Disease
(AIRAWAT)
1. It is a zoonotic disease caused by the
1. It is a cloud computing platform for Big Data monkeypox virus.
analytics with advanced AI processing
capabilities. 2. It is caused by a virus which also causes
smallpox and chicken pox disease.
2. It is based on the recommendations of the
National Strategy for Artificial Intelligence 3. It can spread from person to person or
(NSAI). occasionally from animals to people.
3. It works under the aegis of Ministry of Science How many of the statement(s) given above is/are
and Technology. correct?
How many of the statement(s) given above is/are a. Only one
correct?
b. Only two
a. Only one
c. All three
b. Only two
d. None
c. Only three

d. None
90. Which of the following sectors are eligible for green
deposits scheme according to the recent RBI
Framework for green deposits?
87. Which of the following best describes the
term “Zombie Lending”? 1. Green Building
a. It refers to a higher foreign portfolio investment 2. Nuclear Energy
in the budding MSMEs.
3. Direct waste incineration projects
b. It refers to the practice of providing credit to
entities that do not have the capability to repay. 4. Hydropower plants smaller than 25MW
c. It is a type of money laundering process that 5. Sustainable Water and waste management
refers to the practice of lending to a shell company
and diverting its funds. Select the correct answer using the code given below:

d. It is an interest-free instruments that can be a. Only two


purchased by companies to support political
parties. b. Only three

c. Only four

CHENNAI |SALEM| MADURAI | COIMBATORE DELHI | BANGALORE | THIRUVANANTHAPURAM


www.shankariasacademy.com 63
www.iasparliament.com

d. All five 3. Scheme of Fund for Regeneration of Traditional


Industries

4. PM’s Employment Generation Programme


91. Consider the following statements:
5. Panchayat Empowerment and Accountability
1. Bihan Mela is a seed festival organized to Incentive Scheme
promote the cultivation of traditional crops
among the Kondh tribe. Select the correct answer using the codes given below:

2. It is held annually in the state of Madhya a. Only three


Pradesh.
b. Only four
Which of the statement(s) given above is/are correct?
c. All five
a. 1 only
d. None
b. 2 only

c. Both 1 and 2
95. Consider the following statements with respect to
d. Neither 1 nor 2 the Arab League

1. Membership in the Gulf Cooperation Council


(GCC) is a prerequisite for joining this league.
92. Consider the following pairs:
2. It is headquartered at United Arab Emirates
In News Regions (UAE).

1. Kolak River – Madhya Pradesh 3. India is a member of Arab League.


2. Channapattana Toys – Andhra Pradesh How many of the statements given above are correct?
Which of the pair(s) given above is/are correctly a. Only one
matched?
b. Only two
a. 1 only
c. All three
b. 2 only
d. None
c. Both 1 and 2

d. Neither 1 nor 2
96. Consider the following statements with respect to
the World Health Organization (WHO)

93. Sanchar Saathi Portal, sometimes seen in the news, 1. It is a specialized agency of the United Nations
is an initiative of? (UN) that is responsible for international public
health.
a. Department of Commerce
2. WHO receives funding only on voluntary
b. Department of Co-operation contributions from Member States and other
partners.
c. Department of Telecommunications
3. The State of the World’s Children Report is
d. Department of Investment and Public Asset published annually by the World Health
Management Organization (WHO).

How many of the statement(s) given above is/are


correct?
94. Which of the following schemes are implemented by
the Khadi and Village Industries Commission (KVIC)? a. Only one
1. Project BOLD b. Only two
2. Project RE-HAB c. All three

d. None

CHENNAI |SALEM| MADURAI | COIMBATORE DELHI | BANGALORE | THIRUVANANTHAPURAM


www.shankariasacademy.com 64
www.iasparliament.com

97. Consider the following statements with respect to c. All three


the Animal Welfare Board of India (AWBI)
d. None
1. It is a statutory body established by the Wildlife
Protection Act, 1972.

2. It works under the Ministry of Fisheries, 100. Consider the following statements:
Animal Husbandry and Dairying.
1. Neonatal Mortality Rate (NMR) refers to the
3. It is headed by the Prime Minister. number of deaths of infants aged below 1 year
per 1,000 live births.
How many of the statement(s) given above is/are
correct? 2. Maternal Mortality Rate (MMR) is the number
of maternal deaths during a given time period
a. Only one per 1,000 live births during the same time
period.
b. Only two
3. A baby who dies after 28 weeks of pregnancy,
c. All three but before or during birth, is classified as a
stillbirth.
d. None
How many of the statement(s) given above is/are
correct?
98. Consider the following statements with respect to a. Only one
the Group of 77
b. Only two
1. It is the largest intergovernmental organization
of developed countries in the United Nations. c. All three
2. The South Summit serves as the supreme d. None
decision-making body of the Group of 77.

3. India currently holds the Chairmanship of the


G-77 in New York for the year 2024.

How many of the statement(s) given above is/are


correct?

a. Only one

b. Only two

c. All three

d. None

99. Consider the following statements with respect to


UN Economic and Social Commission for Asia and the
Pacific (ESCAP)

1. It is one of the 5 regional commissions of the


United Nations.

2. It works under the jurisdiction of the United


Nations Economic and Social Council.

3. India is a member to the council.

How many of the statement(s) given above is/are


correct?

a. Only one

b. Only two

CHENNAI |SALEM| MADURAI | COIMBATORE DELHI | BANGALORE | THIRUVANANTHAPURAM


www.shankariasacademy.com 65
www.iasparliament.com

Answer Key - Test - II

1 2 3 4 5 6 7 8 9 10

B D C C D A C D C B

11 12 13 14 15 16 17 18 19 20

B D D C A B D D A C

21 22 23 24 25 26 27 28 29 30

B C B C B D C B C A

31 32 33 34 35 36 37 38 39 40

C B A C A A C C B B

41 42 43 44 45 46 47 48 49 50

A C D C C D B B B A

51 52 53 54 55 56 57 58 59 60

C A A B B D D C D A

61 62 63 64 65 66 67 68 69 70

C D B C A B A D B D

71 72 73 74 75 76 77 78 79 80

A B A C D D C A B C

81 82 83 84 85 86 87 88 89 90

B C B A C B B D B A

91 92 93 94 95 96 97 98 99 100

A D C B A A A B C A

CHENNAI |SALEM| MADURAI | COIMBATORE DELHI | BANGALORE | THIRUVANANTHAPURAM


www.shankariasacademy.com 66
www.iasparliament.com

EXPLANATION o However, in 2013 ruling in ‘Lily


Thomas v Union of India’, the SC
1. b struck down Section 8(4) of the
RPA as unconstitutional.
Disqualification of Members of Parliament
o SC also spelt that when an appellate
Mr. Rahul Gandhi has been disqualified as an MP Court stays the conviction and
according to Section 8 (3) of RPA, 1951 and it was later sentence, the disqualification will be
revoked. lifted and the membership will be
restored to the disqualified MP.
Conditions of Disqualifications
2. d
a. Through Constitutional Grounds
D-SIBs (safety of Indian Banks)
 Article 102(1) – It prescribes grounds for
disqualification of members of Parliament. The failure of Silicon Valley Bank and Signature Bank
They are: in the US raises questions on the safety of depositors'
wealth in India.

 SIBs are perceived as banks that are ‘Too


Big To Fail (TBTF)’.

 In simple terms, it refers to those banks which


if fail would have a significant impact on the
economy.

Domestic Systemically Important Banks (D-


SIBs)
 Article 102(2) – It states that a person shall
 Classified by – RBI
be disqualified for being a member of
either House of Parliament if s/he is
 At present, SBI, ICICI Bank,
so disqualified under the 10th Schedule.
and HDFC Bank D-SIBs in India.
b. Through Representation of the People Act
 Mandate – These banks have to earmark
(RPA), 1951
additional capital and provisions to safeguard
 It deals with Elections, disqualifications and their operations.
offences related to Parliament and State
 Requirement - Banks classified as D-SIBs are
Legislatures.
subjected to additional Common Equity Tier 1
 Section8 (1) – Offences such as promoting (CET1) capital requirements in addition to
enmity between two groups, bribery, and undue the Capital Conservation Buffer.
influence or personation at an election.
 The additional CET1 requirement is
 Section 8(2) – Offences that deal with provisioned as a % of Risk Weighted Assets.
hoarding or profiteering, adulteration of food
Status of D-SIBs (As of Dec, 2023)
or drugs and for conviction and sentence of at
least six months for an offence under any CET1 as % of
provisions of the Dowry Prohibition Act. Since Buckets Banks
RWAs
 Section 8(3) – A person convicted of any 5 - 1%
offence and sentenced to imprisonment for not
less than 2 years shall be disqualified from State Bank of
the date of such conviction and shall continue 2015 4 0.80%
India
to be disqualified for a further period of 6
years since his/her release. 3 - 0.60%
 Section 8(4) – Disqualification takes 2017 2 HDFC 0.40%
effect only “after 3 months have elapsed” from
the date of conviction. 2016 1 ICICI 0.20%
o Within that period, lawmakers could D-SIBs are placed across these five buckets based on
file an appeal against the sentence the level of their importance, with bucket 5
before the High Court. representing the most important D-SIBs.

CHENNAI |SALEM| MADURAI | COIMBATORE DELHI | BANGALORE | THIRUVANANTHAPURAM


www.shankariasacademy.com 67
www.iasparliament.com

Global Systemically Important Banks (G-SIBs) green corridors across the country to combat
land
 Listed by – Financial Stability Board degradation
(FSB) of G20 in consultation with Basel and
Committee on Banking Supervision (BCBS) and desertification.
Swiss national authorities.
 Aim – To
 G-SIBs – It includes JP Morgan, Citibank, green the area
HSBC, Bank of America, Bank of China, around the
Barclays, BNP Paribas, Deutsche Bank, and Aravalli Hill
Goldman Sachs. No Indian bank is on the G- Range in 4
SIB list. states.

3. c  Aravalli hills landscape spans over 6 million


hectares of land in 4 states.
Rampa Rebellion & Alluri Sitharama Raju
 Coverage – Haryana, Rajasthan,
The Telugu movie ‘RRR’ was inspired by the lives of Gujarat and Delhi for roughly 1,400-km
Indian freedom fighters Alluri Sitharama Raju and long and 5-km wide green belt buffer.
Komaram Bheem.
 Activities – Planting native species of trees
 Rampa Rebellion – It is also called as and shrubs on degraded land.
Manyam Rebellion held in Godavari agency of
Madras Presidency, British India.  Agroforestry and pasture development.
 Cause - Madras Forest Act of  Benefits
1882 banned the collection of minor forest
produce and tribal people were forced into o Increase green cover and improve soil
labour by the colonial government. fertility & water availability.

 It restricted the free movement of the tribal o Protect NCR of Delhi from sand and
communities and prohibiting them from dust storms and pollution.
engaging in Podu agricultural system.
o Sequester carbon and conserve
 Podu is a traditional system of cultivation also biodiversity.
known as Shifting cultivation, whereby
different areas of jungle forest are cleared by o Build resilience to climate change.
burning each year to provide land for crops.
o Enhance water conservation,
 Alluri Sitharama Raju – In August 1922, he agricultural production and farm
launched the Rampa Rebellion against the incomes in the Degraded most
British. degraded land of India.

 He used guerrilla warfare along with his army


of tribal people to fight against the British.

 He was honoured for his valour and fiery spirit


with the title, “Manyam Veerudu” (Hero of
the Jungle).

 On his 125th birth anniversary our Prime


Minister unveiled a 30-ft bronze statue of him
at Bhimavaram, in the West Godavari district,
Andhra Pradesh.

4. c

Aravalli Green Wall Project

India had launched the Aravalli Green Wall Project, a


major green initiative to revive Aravallis.

 The Aravalli Green Wall Project is part of the


Union Environment Ministry's vision to create

CHENNAI |SALEM| MADURAI | COIMBATORE DELHI | BANGALORE | THIRUVANANTHAPURAM


www.shankariasacademy.com 68
www.iasparliament.com

 Aravallis – It stand as the only barrier  Membership – Any member can join MPIA
stopping the expansion of desert land towards (India not a member yet) by notifying the
north-west India. Dispute Settlement Body (composed of
representatives of all WTO Members).
 Threat – Rampant mining, dumping of waste
and encroachments.  Dispute resolution – It will supersede the
previous appeal processes and also apply to
5. d future disputes between members.

World Happiness Report 2023 7. c


On the occasion of ‘International Day of Happiness’, Suspension of Operations (SoO) Agreement
annually observed on March 20, ‘World Happiness
Report’ unveiled its 11th edition this year. The Manipur government withdrew from the
Suspension of Operations (SoO) agreement.
Click here to read World Happiness Report, 2024
 It was a ceasefire agreement.
 It is based on people's own assessment of
their happiness, as well as economic and social  Signed in – 2008.
data.
 Primary objective – To initiate a political
 1st published in – 2012, and published annually dialogue with Kuki insurgent groups and meant
since then. ending violence and hostilities from all sides.

 It assigns a happiness score based on an  Signatories – The Centre, the Manipur


average of data over a 3-year period. Government with 2 Kuki umbrella
groups
 2023 Rankings - The 2023 report ranked 137
countries on the list. o 17 groups under the Kuki National
Organisation (KNO)
 For the 6th year running, Finland was named
the world's happiest country followed by o 8 groups under the United People’s
Denmark and Iceland. Front (UPF)

 Israel occupied fourth position, up 5 spots from  There are nearly 30 Kuki insurgent groups in
last year. Manipur, of which 25 are under SoO
Agreement.
 Unhappiest – War-scarred Afghanistan has
occupied the bottom spot (137th) on the table  Tenure – 1 year, is extendable according to the
since 2020. progress of its implementation.

 India – Ranked 126th on the report (136th in  Agreement – The militant cadres are to be
2022). confined in designated camps identified by the
Government.
 India’s Neighbours – Nepal, China,
Bangladesh, Sri Lanka and Pakistan fared  Security forces, including state and central
better than India. forces, are not to launch any operations, nor can
the underground groups (UG).
6. a
 The signatories of UPF and KNO shall abide by
Multi-Party Interim Appeal Arbitration the Constitution of India, the laws of the land
Arrangement (MPIA) and the territorial integrity of Manipur.

Japan became the latest country to join Multi-Party  As a rehabilitation package, the UG cadres
Interim Appeal Arbitration Arrangement (MPIA). living in the designated camps are given a
monthly stipend of Rs 5000.
 Formed in – 2020
 Monitoring by – A committee called the Joint
 It is an alternative system for resolving WTO Monitoring Group (JMG), with representatives
disputes that are appealed by a member in from all the signatories.
the absence of a functioning WTO
Appellate Body.  Manipur – Kuki’s make up about 30% of
population, while rest belong to Meitis (non-
 WTO members can resort to MPIA under tribals) and Nagas.
Article 25 of WTO Dispute Settlement
Mechanism (DSM).

CHENNAI |SALEM| MADURAI | COIMBATORE DELHI | BANGALORE | THIRUVANANTHAPURAM


www.shankariasacademy.com 69
www.iasparliament.com

8. d  Development – It will build on and expand


WMO’s longstanding activities in GHG
Mahoba’s Desawari monitoring, implemented as part of the Global
Atmosphere Watch (GAW) and the Integrated
The Mahoba Desawari variety has been particularly Global Greenhouse Gas Information System
affected by extreme weather events and rising input (IG3IS).
costs.
 Significance – A step towards trying to fill a
 It is a betel leaf variety that is cultivated in key gap in the fight against climate change,
Mahoba district of Madhya Pradesh. standardised & real-time tracking of
greenhouse gases.
 Recognition – It received GI tag in 2021
World Meteorological Organization (WMO)
 Uniqueness – Their size is larger than other
betel varieties and they have a unique aroma  An intergovernmental organization and
and lesser fibre with a mild bitterness and sweet a specialised agency of the United Nations for
taste. meteorology (weather and climate).
 They are crisper, brighter in colour and have a  Membership – 193 Member States and
glossy shine. Territories.
 Medicinal properties – It acts as a natural  Genesis – It originated from the International
cleaning agent for the mouth, quenches thirst, Meteorological Organization (IMO), the roots
improves digestion and also prevents of which were planted at the 1873 Vienna
malnutrition. International Meteorological Congress.
 Cultivation – It is highly susceptible to  Established by the ratification of the WMO
climatic fluctuations and so its cultivation is Convention on 23 March 1950, WMO became
done in a controlled environment in a the
conservatory called as Bareja.
 Headquarters – Geneva, Switzerland
 A Bareja is made up of locally available material
like bamboo and other stalks that can withstand  Its supreme body is the World Meteorological
strong winds and storms. Congress.

 Authoor betel leaves – It is from Tamil 10. b


Nadu which received GI certificate in 2023.
New India Literacy Programme (NILP)
 ‘Chakkai’, ‘Maaththu’, ‘Raasi’ and ‘Sanna
Ragam’ are also betel leaf varieties from  The New India Literacy Programme (NILP)
Authoor, Tamil Nadu. provides basic education, numeracy, life skills,
skill development, livelihood opportunities,
9. c and continuing education to non-literates.

Global Greenhouse Gas Monitoring  The program is based on the National


Infrastructure Education Policy-2020 Sustainable
Development Goals and aims to achieve 100%
Global Greenhouse Gas Monitoring Infrastructure literacy by 2030.
(GGMI) was launched.
 NILP is Centrally Sponsored Scheme that
 Launched by – World Meteorological aims to cover a target of 5.00 crore non-literates
Organization (WMO). in the age group of 15 years and above.
 Aim – To provide better ways of measuring  All the non-literate above 15 years of age
planet-warming pollution and help inform can avail the benefits of the scheme.
policy choices.
 The Scheme has 5 components that includes:
 It will integrate space-based and surface-
based observing systems to clarify o Foundational Literacy and Numeracy
uncertainties about where GHG emissions end
up. o Critical Life Skills

 It is embodied by WMO’s 60-year-old World o Vocational Skills Development


Weather Watch and its acclaimed Global
Atmosphere Watch. o Basic Education

CHENNAI |SALEM| MADURAI | COIMBATORE DELHI | BANGALORE | THIRUVANANTHAPURAM


www.shankariasacademy.com 70
www.iasparliament.com

o Continuing Education Titan

 The beneficiaries under the scheme are  It is the second largest moon in our solar
identified through door-to-door survey on system (after Jupiter’s Ganymede).
Mobile App by the surveyors in the States/UTs.
 It is larger than Mercury, and has lakes, rivers,
 The scheme is mainly based on clouds, and rain of methane on the surface.
volunteerism for teaching and learning.
 Titan is around 10-times farther than the earth
 The scheme is based on technology and from the sun, and as such, its surface
implemented predominantly through online temperature is around -179º C.
mode.
13. d
 The teaching learning material and resources
have been made available on DIKSHA platform Vanuatu Island
of NCERT and can be accessed through the
mobile-apps. A group of 16 countries led by Vanuatu, seeks an
advisory opinion from the International Court of
 Further, other modes like TV, Radio, Samajik Justice (ICJ) on the issue of climate change.
Chetna Kendra etc. are also to be used for
dissemination of Foundational Literacy and  It is a Melanesian Island nation located in
Numeracy. the South Pacific Ocean.

11. b  It is a Y-shaped archipelago of 83 islands in


north-east of Australia and a part of the
 LA Perouse – It is a biennial exercise Australasia Eco zone.
conducted by the French Navy to improve
maritime awareness and security in the Indian Solomon Islands
Ocean and Asia-Pacific region.
China boosts South Pacific influence with a contract to
 The exercise includes 7 nations which are develop a key port in the Solomon Islands.
India, Australia, Canada, The United States,
 A country in Melanesia in the south-
France, Japan, The United Kingdom
western Pacific Ocean.
 DUSTLIK – It is the name of a joint military
exercise between the Indian Army and  It is an archipelago of tropical volcanic islands
Uzbekistan Army that takes place every two and coral atolls.
years.
 It achieved independence from the British as a
 Konkan – Is an annual bilateral maritime republic in 1978.
exercise between the Indian Navy and the
 Honiara is the capital of and the largest city.
Royal Navy of United Kingdom.

12. d

Dragonfly Mission

 Dragonfly is a NASA mission to explore the


chemistry and habitability of Saturn's largest
moon, Titan.

 It is a part of NASA's New Frontiers


Program which aims to further understand
the Solar System.

 Dragonfly will send an autonomously-operated


rotorcraft to visit dozens of sites on Titan.
Mount Merapi
 It will investigate the moon’s surface and
shallow subsurface for organic molecules and Merapi volcano spews hot clouds and ash in new
possible biosignatures. eruption and halted tourism.

 Dragonfly is scheduled to launch in 2026 and  Location – Indonesia.


arrive at Titan in 2034.

CHENNAI |SALEM| MADURAI | COIMBATORE DELHI | BANGALORE | THIRUVANANTHAPURAM


www.shankariasacademy.com 71
www.iasparliament.com

 Other volcanoes in Indonesia – Mount Semeru,  It is a digital platform to bring 16 ministries for
Mount Bromo, Mount Sinabung, Mount Ili integrated planning and coordinated
Lewotolok. implementation of infrastructure projects.

15. a

Green Tug Transition Programme (GTTP)

India aims at becoming ‘Global Hub for Green Ship’


building by 2030 with launch of Green Tug Transition
Programme (GTTP)

 Tug Boats or Tugs are small but powerful ships


which are used to pull or push other large
ships for maneuvering or salvage purposes.

 Launched by – Ministry of Ports,


Shipping and Waterways.
14. c
 Programme – It will start with ‘Green
Parvatmala Project Hybrid Tugs’, which will be powered by
Green Hybrid Propulsion systems, and
The detailed feasibility studies completed for 8 projects subsequently adopting non-fossil fuel solutions
for implementation of Ropeways under Parvatmala like (Methanol, Ammonia, and Hydrogen).
program.
 Targets – It has been set for initial Green Tugs
 Parvatmala means garlanding mountains. It is to start working in all major ports by 2025.
also known as ‘National Ropeway
Development Programme’  At least, 50% of all Tugs are likely to be
converted into Green Tugs by 2030.
 Aim – To decongest mountainous
areas where road and train connectivity  Benefits – Reduce emission as country moves
cannot be upgraded beyond a limit. towards achieving sustainable development.

 Announced in – 2022-23 Union Budget.  In line with India’s Mission


LiFE movement and to become carbon neutral
 Nodal Ministry – Ministry of Road by 2070.
Transport and Highways.
 Helps in achievement of UN’s SDG 14, to
 Implementing Agency - The National sustainably manage and protect marine &
Highways Logistics Management coastal ecosystems from pollution,
Limited (NHLML). conservation & sustainable use of ocean-based
resources.
 NHLML is a 100% owned SPV of National
Highways Authority of India (NHAI).  The event also saw the inauguration of India’s
first Centre of Excellence in Green Port &
 It has aimed to complete construction of 8 Shipping (NCoEGPS) in Gurugram.
ropeways in FY 2022-2023.
 NCoEGPS – It is a result of collaboration
 Activities – It will construct ropeways in between Ministry of Ports, Shipping &
mountainous terrains of India under the Public Waterways and The Energy and Resources
Private Partnership (PPP). Institute (TERI).
PM Gati Shakti 16. b
 It is the National Master Plan for Multi- Collective Security Treaty Organisation (CSTO)
modal Connectivity.
Armenia accused that Collective Security Treaty
 Launched by – DPIIT, Ministry of Commerce Organisation (CSTO) failed to protect it amid a
and Industry standoff with Azerbaijan over Nagorno-Karabakh.

 Aim – To provide integrated and seamless  CSTO – It originates from the conclusion of the
connectivity for movement of people, goods Collective Security Treaty, signed in Tashkent
and services. (Uzbekistan) in 1992 (a year after dissolving of
Warsaw Pact).

CHENNAI |SALEM| MADURAI | COIMBATORE DELHI | BANGALORE | THIRUVANANTHAPURAM


www.shankariasacademy.com 72
www.iasparliament.com

 It is also known as the ‘Tashkent Pact’ or o Water for sustainable development


‘Tashkent Treaty’.
o Water for Climate, Resilience and
 Signatories – Originally by Armenia, Environment
Kazakhstan, Kyrgyzstan, Russia, Tajikistan and
Uzbekistan. o Water Action Decade (2018-
2028)
 Later, it was joined by Azerbaijan, Belarus and
Georgia (1993). o Water for Cooperation

 Formation - The treaty entered into force on  Water Action Decade – It pursues 2 main
April 20, 1994 and in 2002, it became a full- goals
blown military alliance.
o To improve knowledge sharing related
 In 2004, the Organization received an Observer to water and water pollution control,
including information on the water-
status in the UN General Assembly.
related SDGs outlined in the 2030
 Headquarters – Moscow, Russia Agenda for Sustainable Development.

o To improve communication to achieve


 Members – 6 (Armenia, Belarus, Kazakhstan,
the water related SDGs.
Kyrgyzstan, Russia and Tajikistan).
18. d
 Azerbaijan, Georgia and Uzbekistan ceased its
membership. Global Education Monitoring Report
 Article 4 of CSTO – It is very similar to The 2023 edition of the Global Education Monitoring
NATO’s Article 5. Report was published
 It states “If one of the States Parties is subjected  Released by – United Nations Educational,
to aggression by any state or group of states, Scientific and Cultural Organization
then this will be considered as aggression (UNESCO).
against all States Parties to this Treaty.”
 2023 Report – Only 17% of countries cover
 Tashkent Declaration or Agreement – It is the sexual orientation, gender identity and gender
accord signed by India and Pakistan to end the expression issues.
17-day war between them in 1965.
 Only 20% countries have laws and 39% have a
17. d national policy that specifically addresses
sexuality education.
UN 2023 Water Conference
 Young people between the age 2 and 17 years
The United Nations 2023 Water Conference was held
which showed that higher screen time was
in New York in 2023, co-hosted by the Netherlands and
associated with poorer well-being; less
Tajikistan.
curiosity, self-control and emotional stability;
 It is the most important UN-hosted event on higher anxiety; and depression diagnoses.
water supply, sanitation and hygiene in some
 During the Covid-19 Pandemic, the rapid shift
50 years.
to online learning left out at least half a billion
 It is only the 2nd UN Conference dedicated students worldwide, disproportionately
to water, following the Conference in Mar del affecting the poorest and those in rural areas.
Plata, Argentina in 1977.
 Children’s data are being exposed, yet only 16%
 Aim – To promote a comprehensive of countries explicitly guarantee data privacy in
understanding and appreciation of the education by law.
importance of water, sanitation, and hygiene,
UNESCO
and to facilitate effective management of these
critical resources.  A specialized agency of the United Nations
(UN) that seeks to build peace through
 It embraces the principles of inclusion, cross-
international cooperation in Education, the
sectoral responsibility, and action orientation.
Sciences and Culture.
 5 themes
 Headquarters – Paris, France.
o Water for Health
 Membership – India is a member.

CHENNAI |SALEM| MADURAI | COIMBATORE DELHI | BANGALORE | THIRUVANANTHAPURAM


www.shankariasacademy.com 73
www.iasparliament.com

 U.S and Israel have formally quit the UNESCO. Earlier provisions Amended provisions

 It is known for its World Heritage Missiosn  Proposed to include


 In 2019, there was an
which encourages world countries to protect foreign investors in the
exemption from the
Natural and Cultural Heritage sites. ambit of angel
Angel Tax for startups
investors to startups.
 It publishes the Global Education Monitoring on fulfilment of certain
conditions.
report and Gender Parity Index.  When a start-up raises
funding from a foreign
 It also leads the Man and Biosphere  This levy is applicable
investor that too will
Programme for protecting Biosphere reserves only to domestic
now be counted as
across the world. investments.
income and be taxable.

19. a 20. c
Angel Tax National Platform for Disaster Risk Reduction
(NPDRR)
The Finance Bill, 2023 has proposed to amend Section
56(2) VII B of the Income Tax Act thereby expanding  The Government of India has constituted a
the scope of Angel tax. multi-stakeholder National Platform for
Disaster Risk Reduction (NPDRR) in 2013.
 Angel investor – Usually a high-net-worth
individual who funds start-ups at the early  It aims to bring together the whole range of
stage, often with their own money. India’s disaster risk community.
 Angel tax – A tax levied on the capital Composition
raised via the issue of shares by unlisted
companies through off market transaction.  The NPDRR is chaired by the Union Home
Minister and the Minister of State in-charge of
 It taxes on funds raised by startups if Disaster Management in the Ministry of Home
they exceed the fair market value of the Affairs.
company.
 Vice-Chairman, National Disaster Management
 The excess funds raised at prices above fair Authority is the Vice-Chairperson of the
value is treated as income, on which tax is NPDRR.
levied.
Functions
o For Example: If the fair market value
of a start-up share is Rs 10 apiece, and  To review the progress made in the field of
in a subsequent funding round they disaster management from time to time.
offer it to an investor for Rs 20, then the
difference of Rs 10 would be taxed as  To review the National Disaster Management
income. Policy.

 Statutory backing – Section 56(2) VII B of 21. b


the Income Tax Act, 1961, which was 1st
introduced in 2012. Prepaid Payment Instruments (PPI)

 Aim – To discourage laundering of National Payments Corporation of India (NPCI) has


unaccounted money via unlisted firms introduced interchange fees of up to 1.1% on merchant
disguised as capital investments. UPI transactions done using prepaid payment
instruments.
 The tax covers investment in any private
business entity and startups.  PPI – They are payment instruments that
facilitate purchase of goods and
 Exemptions – The only classes of investors services against the value stored on such
whose investments are exempted from angel instruments and can only be used in
tax are Indian rupees.

o SEBI-registered CAT I and II AIFs  Forms – Mobile wallets, physical smart


(alternate investment fund) cards, secure tokens, vouchers or any other
method that allows access to prepaid funds.
o IFSCA-registered CAT I and II AIFs
(under the IFSCA FME Regulations,  Issuing PPI – Banks and Non-Bank Finance
2022). Companies (NBFC), who comply with the

CHENNAI |SALEM| MADURAI | COIMBATORE DELHI | BANGALORE | THIRUVANANTHAPURAM


www.shankariasacademy.com 74
www.iasparliament.com

eligibility criteria, would be permitted to issue  US, UK, Australia, Japan, Korea, India, the
prepaid payment instruments. EU (represented by the European
Commission) and others.
 Classification
 It addresses 4 major critical minerals
o Closed System Payment Instruments challenges
o Semi-Closed System Payment o Diversifying and stabilizing global
Instruments supply chains
o Semi-open System Payment o Investment in those supply chains
Instruments
o Promoting high environmental, social,
o Open System Payment Instruments and governance standards in the
mining, processing, and recycling
 Only banks which have been permitted to sectors
provide Mobile Banking Transactions by the
RBI shall be permitted to launch mobile based o Increasing recycling of critical minerals
prepaid payment instruments (mobile wallets &
mobile accounts). 23. b

 Other entities would be permitted to issue only Central Bureau of Investigation (CBI)
closed system prepaid payment instruments
and semi-closed system prepaid payment The year 2023 marks the diamond jubilee celebration
instruments. of the Central Bureau of Investigation.

Interchange fee  Set up in – 1963 by a resolution of the Ministry


of Home Affairs, but now works under
 It is a fee charged by the receiver Ministry of Personnel, Pension and Public
bank/payment service provider to the Grievances.
merchant.
 Superintendence - The superintendence of
 The PPI owner will receive the applicable CBI related to investigation of offences under
interchange fee from the merchant. the Prevention of Corruption Act,
1988 lies with the Central Vigilance
 The interchange fee is usually levied to cover Commission (CVC) and in other matters
the cost of accepting, processing and with the Department of Personnel & Training
authorising the transaction. (DOPT) in the Ministry of Personnel, Pension &
Grievances of the Government of India.
 Users will not be charged for these transactions.
 It is not a statutory body and derives its
22. c power to investigate from the Delhi Special
Police Establishment Act, 1946.
Minerals Security Partnership (MSP)
 Recommended by – Santhanam Committee
 Need – To overcome the supply chain issues on Prevention of Corruption.
due to Russia-Ukraine war and reduced supply
from China due to impact of Covid-19.  Motto – Industry, Impartiality and Integrity.

 Launched by – United States  Composition – It is headed by a Director and


assisted by a special director or an additional
 Objectives – To accelerate the development of director.
diverse and sustainable critical energy
minerals supply chains for strategic projects  Appointed by – Search committee, headed by
along the value chain. the PM of India, and has Leader of Opposition
and the Chief Justice of India (CJI) or his
 To catalyse public and private representative.
investment in responsible critical
minerals supply chains globally.  It can conduct investigation in a state only with
the consent of the concerned state government.
 Thus, it is also called as Critical Minerals
Alliance.  2 kinds of consent of States - Case-specific
consent and General consent.
 Members – 14
 9 states have withdrawn their general consent.

CHENNAI |SALEM| MADURAI | COIMBATORE DELHI | BANGALORE | THIRUVANANTHAPURAM


www.shankariasacademy.com 75
www.iasparliament.com

 However, the Supreme Court and High  To fully decarbonise power sector by 2035 and
Courts can order CBI to investigate such a to eliminate inefficient fossil fuel subsidies by
crime anywhere in the country without the 2025 or sooner.
consent of the State.
 They pledged to collectively increase offshore
 Functions – It is the main investigating wind capacity by 150 gigawatts by 2030 and
agency of the Central Government. solar capacity to more than 1 terawatts.

o Plays an important role in the  They will accelerate the phase-out of unabated
prevention of corruption and fossil fuels so as to achieve net zero in energy
maintaining integrity in systems by 2050.
administration.
 Principles of High-Integrity Carbon
o Investigates crime of corruption, Markets have been proposed to ensure high-
economic offences and serious and integrity carbon markets.
organized crime other than terrorism.

o Provides assistance to the Central


Vigilance Commission and Lokpal.

o Nodal police agency in India, which


coordinates investigation on behalf of
Interpol Member countries

 CBI academy - Located at Ghaziabad, Uttar


Pradesh.

24. c

Group of seven (G7)

G7 Climate and Environment Ministers’ meeting


25. b
concluded in Sapporo, Japan.
Lab grown meat
 An informal forum of leading industrialised
nations. Recently, an Australian startup unveiled lab-grown
cultured meat using the genetic sequence from the
 Purpose – It meets annually to discuss issues long-extinct woolly mammoth.
such as global economic governance,
internation security and energy policy among  Mark Post, the Dutch pharmacologist was the
many other topics. 1st to present a proof of concept for cultured
meat in 2013.
 Members – US, Britain, Canada, France,
Germany, Italy and Japan.  Cultured meat – It is also called cell-based
meat or cultivated meat is genuine animal
 From 1998 to 2014, Russia was also a member meat that is produced by cultivating animal
of this group, it was then known as G8 cells directly.
 Importance – They represent 46% of the  The livestock does not need to be killed in
global GDP and 10% of the world’s population. order to produce it.
 Initiative - Build Back Better World  But it is not technically considered
(BBBW) in 2021 to provide an alternative vegan because it contains cells, fat and muscle
to China’s Belt and Road initiative. tissue derived from real animals.
G7 Summit
 It uses animal DNA to recreate in a lab the taste
and texture of meat.
 Key highlights – G7 countries set the urgent
need to reduce global GHG emissions by
 Currently lab-grown meat from cells including
around 43% by 2030 and 60% by 2035.
chicken, pork, lamb, fish and beef is underway.
 Pledge to end new plastic pollution by
 Manufacturing – It uses Stem cells that
2040, bringing the target forward by a decade. are harvested through biopsy from fresh piece
of meat.

CHENNAI |SALEM| MADURAI | COIMBATORE DELHI | BANGALORE | THIRUVANANTHAPURAM


www.shankariasacademy.com 76
www.iasparliament.com

 Isolate different types of cells (for instance o preventing sugar crystallization


muscle and fat cells).
o acting as an emulsifier in chocolate
 The cells are put in bioreactor based on cell
cultivation and tissue engineering for growth o forming a coating for glazed and filled
and reproduction. sweets and cereals.

 The cells are distributed onto moulds to o to preserve the flavour of cola and
stimulate them to differentiate into connective citrus beverages, to prevent pulp from
tissues, muscles and fats giving the final sinking to the bottom of fruit drinks
product.
o to stabilize foam in beer
 Legality – In 2023, the US cleared the sale of
o encapsulation agent in medications
lab grown chicken meat in the country.
o texturizer for oral medications
 Previously Singapore have approved cell-based
meat for consumption. o fixative in textiles, inks, tempera
paints, watercolours, and gilding

o photography, pyrotechnics, shoe


polish, and ceramic glazing.

 About 70% of the world’s supply comes from


the acacia trees in the Sahel region that runs
through.

27. c

The Coalition Linking Energy and Nature for


action (CLEANaction)

Clean energy coalition of WWF says all forms of


renewable power are better than fossil fuels.

 CLEANaction – It is a partnership to protect


nature during the energy transition.

 Aim - To ensure that the impact on nature by


clean energy systems is fully accounted for prior
26. d to any installation.
Gum Arabic
 Launched in - CoP-26 of UNFCCC.
Sudan’s eruption into conflict has left international
 Founder Partners –
consumer goods makers racing to shore up supplies of
gum Arabic. o TheAlliance for Rural Electrification
(ARE)
 It is a dried water-soluble exudate that comes
primarily from two species of acacia in sub- o Birdlife International
Saharan Africa: Acacia senegal and A. seyal.
o ICLEI-Cities Biodiversity Center
 The tasteless and nontoxic properties of gum (ICLEI Africa Secretariat)
arabic make it especially useful in the food
industry. o Carbonn Climate Center (ICLEI World
Secretariat)
Applications
o The International Renewable
 The tasteless and nontoxic properties of gum Energy Agency (IRENA)
arabic make it especially useful in the food
industry, where it is used as an emulsifier, o The Nature Conservancy (TNC) and
binding or coating agent, and stabilizer.
o The Worldwide Fund for Nature
 Some of its applications include: (WWF).

o providing texture for gum candies  CLEANaction is open to all relevant Partners
that support an energy transition that

CHENNAI |SALEM| MADURAI | COIMBATORE DELHI | BANGALORE | THIRUVANANTHAPURAM


www.shankariasacademy.com 77
www.iasparliament.com

minimizes any negative impacts of new  Threat - Human beings with a compromised
renewable energy developments on people and immune system are most vulnerable to fungal
nature. infection.

28. b  Fungal infections are expected to pose a greater


threat to humans in future due to rising
Chondrostereum purpureum temperatures, caused by climate change.
The 1st case of a plant fungus called Chondrostereum 29. c
purpureum infecting human beings has been reported
from Kolkata. Census of Waterbodies in India

 Chondrostereum purpureum – It is The first ever census of waterbodies in India was


a plant fungus that causes silver leaf disease released.
in plants, particularly those in the rose family.
 Released by – Ministry of Jal Shakti
 Silver leaf disease infects their leaves and
branches, the fungus can be fatal for the plant  It was in convergence with the 6th minor
if not quickly treated. irrigation census (reference year 2017-18),
under the centrally sponsored scheme:
 However, there were no reported instances of “Irrigation Census”.
this fungus infecting human beings from any
part of the world.  The census defines a waterbody as “all natural
or man-made units bounded on all sides with
 Fungal Disease – They aren’t exactly some or no masonry work used for storing
uncommon in humans, of the millions of known water for irrigation or other purposes.
fungal species, only a few cause infections in
humans.  Waterbodies are usually of various types known
by different names like tank, reservoirs, ponds
 Ringworm, athlete's foot, and thrush commonly and bundhies etc.
are found in damp areas of our skin, much to
our irritation.  The waterbodies census was conducted along
with the 6th Minor Irrigation Census for 2017-
 This is the 1st instance of a plant fungus causing 18.
disease in a human.
Findings
 This may be the start of a new phenomenon
when plant fungus is adapting to invade human  There are 2,424,540 waterbodies in the
cells. country, of which 97 per cent are in rural
areas.
 Process - The plant fungus is infecting
human cells by evading the process of  Of the total waterbodies, 59.5 per cent are
‘phagocytosis’ which means ‘cell eating’. ponds, 15.7 per cent are tanks, 12.1 per cent are
reservoirs, and the remaining 12.7 per cent are
 The process happens when a cell uses its plasma structures created under water conservation
membrane to engulf a large particle, giving rise schemes.
to an internal compartment called the
‘phagosome’.  The share of privately owned waterbodies
is 55.2% while the rest are government-
 Organisms clean and defend themselves by this owned.
process.
 78% water bodies are man-made water
 Reason - Rising temperature due to global bodies whereas 22% are natural water bodies.
warming is one of the major reasons for
this new threat to human beings.  The greatest number of waterbodies are ponds,
followed by tanks, reservoirs, water
 Most fungi thrive in the range of 12°C to 30°C conservation schemes/percolation tanks/check
and many species are thermos-tolerant. dams, lakes, and others.

 Those fungal species that are able to grow at  West Bengal has the highest number of
35–37 °C can become a human pathogen or ponds and reservoirs, whereas Andhra
commensal flora. Pradesh has highest number of tanks.

CHENNAI |SALEM| MADURAI | COIMBATORE DELHI | BANGALORE | THIRUVANANTHAPURAM


www.shankariasacademy.com 78
www.iasparliament.com

 Tamil Nadu has the highest number of lakes  It is a Centralized Online System for seed
and Maharashtra is the leading state with traceability, authentication.
water conservation schemes.
 It is also an inventory designed to deal with the
 No encroachment on waterbodies was reported challenges of seed production, quality seed
from West Bengal, Sikkim, Arunachal Pradesh identification and seed certification.
and Chandigarh.
 There will be a QR code under this system,
30. a through which the seeds can be traced.

Abhilekh Patal  Training should be imparted through Indian


Council of Agricultural Research (ICAR), Krishi
The Prime Minister praised Abhilekh patal, a portal Vigyan Kendras and State Governments.
with over 1 Crore Pages of Historical Records of the
National Archives. 31. c

 It is a digital repository of historical UN Statistical Commission


records.
India has been elected to the highest statistical body of
 Curated by – National Archives of India (NAI). the United Nations for a 4-year term beginning
January 1, 2024.
 Abhilekh – A Sanskrit term for records since
ancient times.  Established in – 1947, by UN Economic
and Social Council as its Functional
 Patal – A Sanskrit term for a board, platform, Commission.
or a surface.
 It is the highest body of the global statistical
 It has been used as an acronym for Portal for system.
Access to Archives and Learning.
 It is the highest decision-making body for
 This platform serves as a gateway to India's international statistical activities.
historical legacy, offering a treasure trove of
meticulously preserved documents and  Functions – It sets statistical standards and
records. develop concepts and methods, including their
implementation at the national and
Yuva Portal international level.

Union Minister of Science & Technology launched the  It oversees the work of the United Nations
Yuva Portal. Statistics Division (UNSD).

 Objectives – To help in connecting and  Memberships – 24 member countries of the


identifying potential young Start-Ups. United Nations.
SATHI Portal  Term of office – 4 years
Union Agriculture and Farmers Welfare Minister had  The members are elected by the United Nations
launched the SATHI Portal and Mobile App. Economic and Social Council on the basis of an
equitable geographical distribution.
 SATHI (Seed Traceability, Authentication and
Holistic Inventory) Portal  India has been elected from the Asia-Pacific
States group for a 4-year term.
 Ministry – Ministry of Agriculture and
Farmers Welfare 32. b
 Aim –It aims to ensure quality assurance Operation Kaveri
system, identify the source of seed in the seed
production chain. India has started ‘Operation Kaveri’ to evacuate its
nationals from conflict-torn Sudan.
 Developed by – National Informatics Centre
(NIC) in collaboration with the Union Ministry  Aim – To bring back Indian citizens
of Agriculture and Farmers Welfare. stranded in Sudan.

 Theme - Uttam Beej – Samriddh Kisan (Choice  Around 3,000 Indians are stuck in various
of seeds in prosperous Agriculture). parts of Sudan, including
capital Khartoum and in distant provinces
like Darfur.

CHENNAI |SALEM| MADURAI | COIMBATORE DELHI | BANGALORE | THIRUVANANTHAPURAM


www.shankariasacademy.com 79
www.iasparliament.com

 'Operation Karuna' – An operation by State of world's Children Report 2023


India to assist Myanmar which has
been devastated by Cyclone Mocha.  Released by – UNICEF India

33. a  It 1st went into print in 1980, 10% of the


children born that year died from preventable
Rule of law index causes and each State of the World's Children
(SOWC) examines a key issue affecting
 It is released annually by the World Justice children.
Project.
 Findings – It warns of the growing threat of
 It measures people’s perceptions and vaccine hesitancy which comes amid the largest
experiences of the rule of law in 140 countries sustained backslide in childhood immunization
and jurisdictions. in 30 years, fuelled by the COVID-19 pandemic.

 It looks at 8 factors, sub-divided into 44 sub-  India’s achievement – It was able to bring down
factors. the number of zero-dose (unreached or missed
out) children to 2.7 million.
 Range – Between 0 and 1, 1 being the score for
complete adherence to the rule of law.  This represents a smaller proportion of India’s
under-5 child population given its size and the
 Status of India – India’s overall score in the world’s largest birth cohort.
2022 report is 0.5 and its global rank is 77 out
of 140 countries. 34. c

 Within the South Asia region, India is placed Idu Mishmis


3rd behind Nepal and Sri Lanka, out of six
countries.  The Idu Mishmi is a sub-tribe of the larger
Mishmi group in Arunachal Pradesh and
 Countries such as Senegal, Ghana, Indonesia neighboring Tibet.
and Ukraine were ahead of India as of the 2022
report.  The other two Mishmi groups are Digaru and
Miju.
 It does not provide sun-national ranks.
 They live in Mishmi Hills, bordering Tibet.
 However, the India Justice Report (IJR), a
collaborative effort between various  Their ancestral homelands are spread over the
organisations ranks UP at 18 among the 18 districts of Dibang Valley and Lower Dibang
large and mid-sized states of India. Valley as well as parts of Upper Siang and Lohit.

State of Global Climate Report  They speak the Tibeto-Burman language and
their language (also called Idu Mishmi) is
 It is an annual publication. considered endangered by UNESCO.

 Released by – UN’s World Meteorological  Their occupation is Weaving and


Organization. craftsmanship.

 Findings – Sea levels are rising at an annual  Animals such as the hoolock gibbons and tigers
average of 4.62 millimetres in the past decade. have deep cultural relations with the Idu
Mishmi.
 The La Nina cooling weather phenomenon
failed to ease the temperature hikes, with the  The major festivals of the Idus are ‘Reh’
past eight years considered the highest mean and ‘Ke-meh-ha’.
temperatures on record.
 According to Idu mythology, they were born to
 58% of ocean surfaces experienced a marine the same mother of tigers, so tigers are their
heatwave. “elder brothers”.

 India – Heatwaves in 2022 brought down the  They follow a strict belief system iyu-ena that
yield of wheat crops. restrict them from hunting many animals,
including a complete prohibition on killing
 The heatwaves have become 30 times more tigers.
likely due to climate change.

CHENNAI |SALEM| MADURAI | COIMBATORE DELHI | BANGALORE | THIRUVANANTHAPURAM


www.shankariasacademy.com 80
www.iasparliament.com

35. a  It depicts religious and mythological stories,


such as gods, goddesses, saints, and scenes
PM MUDRA Yojana from epics like Ramayana and Mahabharata.

 MUDRA – Micro Units Development  It is the first independent GI tagged product


& Refinance Agency. from Jammu region.

 Pradhan Mantri Mudra Yojana (PMMY) is a  Manamadurai Pottery – Manamadurai in


flagship scheme launched by the Prime the Sivagangai district of Tamil Nadu is
Minister Shri Narendra Modi on April 2015. known for pottery making.

 MUDRA is registered as a Company under the  The Vaigai River enriches the clay used for the
Companies Act 2013 and as a Non Banking Manamadurai pottery.
Finance Institution with the RBI.
 The circumference of the pot and the neck
 The scheme facilitates micro credit or should be proportionate so that the pot sits flat
Loan up to Rs. 10 lakhs to income on the ground.
generating micro enterprises engaged in the
non-farm sector in manufacturing, trading or  Mircha Rice – It is also known as Marcha rice.
service sectors including activities allied to
agriculture such as poultry, dairy, beekeeping,  It is a short-grain rice variety native to the West
etc. Champaran region of Bihar.

 These loans are given by Commercial Banks,  The grains are shaped like black pepper, which
RRBs, Small Finance Banks, MFIs and NBFCs. is why the rice is named "Mircha" (pepper in
Hindi).
 The loans under Pradhan Mantri Mudra Yojana
can be availed through eligible Member 37. c
Lending Institutions (MLIs), which include:
 Kuttamperoor – It is a west-
o Public Sector Banks, Private Sector flowing River originating in the Western
Banks and State operated cooperative Ghats of Kerala.
banks,
 It is a tributary of both the Pamba and the
o Rural banks from regional sector and Achankovil rivers in Kerala and it forms a link
Micro Finance Institution (MFI), between the two rivers.
o Non-Banking Finance Company  Hakki Pikki – A tribe that lives in several
(NBFC) and Small Finance Banks states in west and south India, especially near
(SFBs) and forest areas
o Other financial intermediary approved  They live majorly in Davangere, Mysuru, Kolar,
by Mudra Ltd. as member financial Hassan and Shivmogga districts
institutions. of Karnataka.
o The loans under Pradhan Mantri  In different regions, they are known by different
Mudra Yojana can be availed through names, such as Mel-Shikari in northern
eligible Member Lending Institutions Karnataka and Maharashtra.
(MLIs).
 Uttaramerur – Prime Minister Narendra
 The loans under Pradhan Mantri Mudra Yojana Modi referred to the Uttaramerur inscription
can be availed through eligible Member in Kanchipuram, Tamil Nadu, while
Lending Institutions (MLIs) does not include discussing India’s democratic history.
micro entrepreneurs.
 The inscription gives details of the functioning
 MUDRA loans are available for purchase of of the local Sabha, i.e. the village assembly.
CNG Tempo/Taxi, in case the applicant intends
to use the vehicle for commercial purposes. 38. c
36. a National Party Status Criteria
Geographical Indications (GI) Products  Election Symbols (Reservation and Allotment)
Order, 1968, stipulates the criteria for
 Basholi Painting – It is a miniature art style recognition as a national or state party.
from Kathua district of Jammu and
Kashmir.

CHENNAI |SALEM| MADURAI | COIMBATORE DELHI | BANGALORE | THIRUVANANTHAPURAM


www.shankariasacademy.com 81
www.iasparliament.com

 A political party can be recognized as a national  ‘The Guardian Ring’ is a unique and
party by attaining any one of the following innovative programme envisaged for
criteria. International Yoga Day, 2022.

o It must get at least 6% vote share in 4  Theme for 2022 – Yoga for humanity
or more states in the last Lok Sabha or
Assembly elections  Implemented by – Ministry of Ayush and
Ministry of External Affairs.
o It must have at least four MPs or
 Participants – It is a collaborative
o It should secure at least 2% of the seats exercise between 79 countries and United
in the Lok Sabha, with its candidates Nations organisations along with Indian
having been elected from at least 3 Missions abroad.
states.
 The programme celebrates the movement of the
o It should be recognized as a state party Sun, underlining the concept of ‘One Sun,
in 4 states. One Earth’.
 The criteria of having representation of at  Program - From East to West, people from all
least one Member of Parliament for every 25 countries welcomed the Sun with Yoga with
members or any fraction allotted to the state in Surya Namaskar or sun-salutation.
the Lok Sabha” applies to the state party
status.  Starting from Fiji in the east it moves westward
and end in San Francisco.
 There are now only 6 national parties in the
country.  People performing yoga in 16 different time
zones along with the rise of the sun was
 List of 6 national parties are BJP, Congress, exclusively live-streamed on DD-India.
Bahujan Samaj Party (BSP), CPI (M), National
People’s Party (NPP) and AAP. 41. a
39. b Stockholm Agreement
UN Convention against Torture (UNCAT) Hundreds of war prisoners were released as agreed by
Stockholm agreement signifying peace in the war-
 It is an international human rights treaty ravaged Yemen.
adopted by the United Nations General
Assembly on 1984.  It is an UN brokered agreement, signed
by Yemen in 2018 to free conflict-related
 It aims to prevent torture and Other Cruel, detainees.
Inhuman or Degrading Treatment or
Punishment to humans both physically and  3 main components – The Hudayah
mentally. agreement, the prisoner exchange agreement
and the Taiz agreement.
 The Convention requires member states to take
effective measures to prevent torture in any o Hudayah agreement – A ceasefire
territory under their jurisdiction. in the city of Hodeidah.

 It also forbids member states to transport o Prisoner exchange agreement –


people to any country where there is reason to Release of prisoners by both sides.
believe they will be tortured.
o Taiz agreement – It includes the
 India has signed the United Nations formation of a joint committee with
Convention against Torture (UNCAT) on participation from civil society and the
October 1997 but India has yet to ratify the UN.
convention.
 Yemen crisis – The conflict in Yemen began
40. b in 2011 as part of the Arab Spring protests.

Guardian Ring Program  UN – Civil war in Yemen is now the largest


humanitarian crisis in the world.
The International Yoga Day is being observed today on
21 June across the world. Other Agreements

 Good Friday Agreement – It was signed in


1998, between factions of Northern Ireland,

CHENNAI |SALEM| MADURAI | COIMBATORE DELHI | BANGALORE | THIRUVANANTHAPURAM


www.shankariasacademy.com 82
www.iasparliament.com

and governments of Britain and Ireland, to end  Bigger hotter stars have their Goldilocks Zones
30 years of violence known as ‘The Troubles’. further out, while smaller cooler stars have
habitable zones much closer in.
 Investment Facilitation Agreement – It is
trade agreement proposed by WTO to create a  Obviously, our Earth is in the Sun’s Goldilocks
legally binding provisions by facilitating zone.
investment flows.
43. d
 Investor-state dispute settlement
(ISDS) – A free trade agreement (FTA) that SaMD and SiMD
provides foreign investors with the right to
access an international tribunal to resolve Medical technology devices usually connected to the
investment disputes. internet, mobile phones, servers, and the cloud are
under threat of cyber-attacks.
42. c
 SaMD – Software AS a medical device
Star-Planet Pair & Goldilocks Zone (SaMD) means that the software itself is the
device.
Astronomers find a star-planet pair that mimics the
relationship between our sun and earth.  They are software solutions that do not reside
in a medical device but are used for medical
 YZ Ceti b – It is a rocky, earth-sized exoplanet purposes.
(planet orbiting a star other than our sun)
rotating around a small red dwarf star and is 12  Requisite – It needs to function
light-years from Earth. completely independent of existing
medical devices.
 Astronomers have detected a repeating radio
signal from this exoplanet that suggests the  Examples - Software that processes images,
presence of a magnetic field around it. X-rays and scans to detect diseases.

 Magnetic field is one of the prerequisites  SiMD - Software IN a medical device


for a habitable planet. (SiMD) means that the software is part of
another medical device which helps it function
 The astronomers determined that the planet in some way.
takes just a couple of earth days to circle its star.
 SiMDs are software that powers the mechanics
 To have an atmosphere and sustain water, a of a medical device or processes the
planet has to be at a certain distance from its information that is produced by a medical
star (Goldilocks zone), or it will get burnt. device is obviously considered.

 Goldilocks zone – It is also called as  Software that controls the device


habitable zone. remotely is also a SiMD.

 It is the range of orbital distances from a  Examples - Oximeters, hearing aids,


star at which liquid water can exist on the glucometers
surface of a planet, without water boils away
or freezes.  Regulation - Both SiMD and SaMD are
subject to the medical device regulations of the
 This range of distances changes depending on country where they are used.
the size and temperature of the star.
 In India - The Central Drugs Standard Control
Organization (CDCSO) under the control of
Drug Controller General of India
(DCGI) classifies, regulates and registers
medical devices.

 Medical Devices Rules, 2017 have been framed


in conformity with Global Harmonisation Task
Force (GHTF) framework.

 Currently, there are no guidelines on the


regulation of SaMD and SiMD, they are not
even covered under this rule.

CHENNAI |SALEM| MADURAI | COIMBATORE DELHI | BANGALORE | THIRUVANANTHAPURAM


www.shankariasacademy.com 83
www.iasparliament.com

44. c  It is India’s first privately developed


fully Cryogenic rocket engine ‘Dhawan-I’
International Big Cat Alliance (IBCA) which will power Vikram-I.
Prime Minister of India launched the International Big  Developed by – Skyroot Aerospace for its
Cats Alliance (IBCA). heavier vehicle, Vikram II.
 It is a multi-country, multi-agency coalition.  It will be used as the upper stage of the updated
version Vikram II.
 Launched in – 2023, by India in
commemoration of 50th anniversary of Project  Fully cryogenic engines are ideal for the upper
Tiger (1973). stages of a rocket due to their higher specific
impulse that enhances payload-carrying
 Objective – To ensure cooperation for capabilities.
the conservation of 7 big cats by arresting
decline in population and reverse the trend.  Liquid Natural Gas (LNG) and Liquid Oxygen
(LoX) are the 2 cryogenic rocket fuels which
 To establish a central repository for knowledge require cryogenic emperatures (below -150°
sharing, capacity building, networking, Celsius) for storage and operation.
advocacy, finance and support research.
JUICE
 Partnership – 96 big cat range countries,
also include interested non-range countries,  JUICE – Jupiter Icy Moons Explorer
conservation partners and scientific
organisations working in the field of big cat  Agency – European Space Agency (ESA)
conservation.
 It is planned to reach Jupiter in 2031 and will
 Administration – An assembly of members, spend at least 3 years on Jupiter’s moons after
a standing committee as well as a secretariat it arrives.
based in India.
 Aim – To make detailed observations of
 Framework of Agreement (statute) – It Jupiter and its 3 large ocean-bearing
has been drafted largely on the pattern of the moons i.e. Ganymede, Callisto and Europa.
International Solar Alliance (2015)
 Moons of Jupiter - Io, Europa, Ganymede
 7 big cats – Cougar (Puma), Jaguar, and Callisto are 4 of Jupiter’s moon which are
Leopard, Snow Leopard, Cheetah, Lion relatively large, spherical complex worlds.
and Tiger.
 Europa, Ganymede and Callisto – are home to
 Clouded leopard is not included in this large, underground oceans of liquid water that
alliance. could support life.

 Ganymede is the largest moon in the Solar


System and the only one to have its own
magnetic field.

PSLV Orbital Experimental Module (POEM)

 POEM is the spent in 4th stage (PS4) of


the launch vehicle that would be used as an
orbital platform to carry out scientific
experiments through non-separating payloads.

 While the 1st 3 stages are jettisoned into the


ocean after they push the mission to desired
orbit, the 4th stage remains in orbit and
becomes space junk.

Hakuto R Mission
45. c A private mission by Japanese Hakuto R, operated by
I Space to the moon failed to communicate after a lunar
Dhawan II Engine touchdown
Skyroot Aerospace successfully test-fired 3D-printed
engine ‘Dhawan-II’ for a record 200 seconds.

CHENNAI |SALEM| MADURAI | COIMBATORE DELHI | BANGALORE | THIRUVANANTHAPURAM


www.shankariasacademy.com 84
www.iasparliament.com

 It is a lunar mission operated by a  Report in 2 categories


private company, I Space, Japan.
o 18 large and medium sized states with
 If the mission succeeded, it would have placed population of over 1 crore
Japan and the UAE in the global lunar club with
the US, Russia and China. o 7 small states with population less than
1 crore.
 Aim – To put a small NASA satellite into lunar
orbit to search for water deposits before  Funding – Except for 2 union territories,
touching down in the Atlas Crater. Delhi and Chandigarh, no state spends more
than 1% of its total annual expenditure on the
46. d judiciary.

World Development Report 2023  Vacancy – 22% among the sanctioned posts in
judiciary (30% vacancy of judges in HC).
 Released by – World Bank
 Gender disparity – Only 13% of High Court
 It is an annual report that provides a judges and 35% of Subordinate Court judges are
comprehensive analysis of international women.
migration and its potential to serve as a force
for growth and shared prosperity in all  Pendency - 1 in every 4 high court cases is
countries. pending for more than 5 years.

 It proposes an integrated framework to Large and Mid-sized


Rank Small States
maximize the development impacts of cross- States
border movements on both destination and
origin countries and on migrants and refugees 1 Karnataka Sikkim
themselves.
2 Tamil Nadu Arunachal Pradesh
 The framework rests on a Match and Motive
matrix.
3 Telangana Tripura
 Key Findings – A large share of migrants and
refugees live in low- and middle-income Last Uttar Pradesh (18th) Goa (7th)
countries.
47. b
 120% income gain for Indians going to work
overseas compared to a 40% rise in case of  Pattanam Excavation Site – Ancient DNA
internal migration. study confirms West Eurasian genetic imprints
in Kerala’s Pattanam.
 Top Migration Corridors - India-US, India-
GCC and Bangladesh-India, Mexico-US, China-  The excavations suggest that the site was first
US, Philippines-US and Kazakhstan-Russia. occupied by the indigenous and ‘Megalithic’
(Iron Age) people, followed by the Roman
 Mostly developing and poor countries like India contact in the early historic period.
are witnessing a rise in young population, while
the developed countries have crossed that  It appears that the site was continuously
phase as depopulation has set in. occupied at least from the 2nd century BC to the
10th century AD.
India Justice Report
 Most excavated skeletal remains from the
 Initiated by – Tata Trusts in 2019, 2022 Pattanam site were in a very fragile state due to
report is its 3rd edition. the tropical, humid, and acidic soil conditions.

 Aim – To track the performance of States in  Siju Cave (Amolops siju) – Scientists of the
capacitating their justice delivery structures to Zoological Survey of India (ZSI) have named
effectively deliver mandated services for 24 the new species, Amolops siju after the Siju
hours. cave system of Meghalaya.

 It separately assesses the capacity of the 25  The cave is a natural limestone cave located in
State Human Rights Commissions in the the South Garo Hills District of Meghalaya,
country. Northeast India.

 4 pillars of justice – Police, Judiciary,  The cave contains the twilight zone, an area
Prisons, and Legal Aid. with limited light.

CHENNAI |SALEM| MADURAI | COIMBATORE DELHI | BANGALORE | THIRUVANANTHAPURAM


www.shankariasacademy.com 85
www.iasparliament.com

 This 'dark zone' has consistent 'temperature  Po River - This is the longest river in Italy.
and humidity' all year long.
 It starts as a spring in the Val Po and ends at
 Paigah tombs – Located at the Pisal Banda the Adriatic Sea.
suburbs in Hyderabad, Telangana enclosed
in facades of pierced marble.  The valley surrounding the river is where 35%
of Italy’s agricultural products come from.
 The tombs are example of Indo-Islamic
architecture, which is a blend of both features  It is also the main area for industrial activities
of Asaf Jahi and Rajputani style. like manufacturing and utilities.

 The tombs are made up of limestone mortar, 49. b


marble and pietra dura.
 Cumbum Grapes – Located at Cumbum
 It represents the tombs of Paiga nobles, the valley located of the Western Ghats in Tamil
highest order of nobility under the Nizam of Nadu.
Hyderabad.
 Cumbum valley is known as the Grapes city of
48. b South India and cultivates the Panneer
Thratchai.
Drying rivers in Europe
 This variety is also known as Muscat Hamburg.
Europe’s 4 Major Rivers Are Drying up
 The grapes grown here are suitable for making
 Danube River – Second longest river wine, spirit, jams, canned grape juice and
in Europe. raisins.

 This river passes through several countries  Aranmula Mirror – Is an indigenous


including Austria, Bulgaria, Croatia, Germany, product of Aranmula, Kerala, a heritage
Hungary, Moldova, Romania, Serbia, Slovakia, village on the banks of the Pamba River.
and Ukraine. It ends in the Black Sea.
 The uniqueness of the Aranmula kannadi is that
 Rhine River – Starts in it is front reflecting, unlike plane glass mirrors
Switzerland, Europe. where reflection takes place on the back surface
of the glass, where the reflective coating is
 It ends in the North Sea in the Atlantic Ocean. applied.

 It passes through Switzerland, Liechtenstein,  In the case of the Aranmula kannadi, the light
Austria, Germany, France and the Netherlands. does not penetrate any refractive medium like
glass. The reflection is on the top metal surface.
 Loire River – It is a river in France, Europe.
 Gond Painting – Of Madhya Pradesh has
 This river starts in the Massif Central received the prestigious Geographical
Mountains and ends in Saint-Nazaire where it Indication (GI) tag.
empties into the Bay of Biscay in the Atlantic
Ocean.  They tell us about nature, trees, plants, animals,
moon, sun, river, drains, God and Goddesses.

 The main source of Gond painting has been


Dindori, it has been expanded from place to
place in Dindori.

50. a

Great Pacific Garbage Patch.Pacific Trash


Vortex

 Great Pacific Garbage Patch is also called


as Pacific Trash Vortex.

 It is the largest collection of marine debris in


the North Pacific Ocean.

CHENNAI |SALEM| MADURAI | COIMBATORE DELHI | BANGALORE | THIRUVANANTHAPURAM


www.shankariasacademy.com 86
www.iasparliament.com

 The garbage patch is actually two distinct 52. a


collections of debris bounded by the
massive North Pacific Subtropical Gyre. LockBit

 Gyre is a large system of swirling ocean  It is also known as the “abcd” virus and
currents. designed to infiltrate victim’s systems and
encrypt important files.
 The area in the centre of a gyre tends to be very
calm and stable.  It works as a self-spreading malware that does
not requires additional instructions.
 It is located between California and Japan, and
formed due to converging ocean currents.  It hides executable encryption files by
disguising them in the .PNG format to avoid
 The patch is actually comprised of the Western detection by system defences.
Garbage Patch, located near Japan, and the
Eastern Garbage Patch, located between the  Crypto virus - The virus is categorised as a
U.S. states of Hawai'i and California. “crypto virus” due to its requests for payment
in cryptocurrency to decrypt the files on the
 Great Pacific Garbage patches are almost victim’s device.
entirely made up of tiny bits of plastic, called
microplastics.  Historically, ransomware has targeted
Windows, Linux, and VMware ESXi servers.
 Since the Great Pacific Garbage Patch is so far
from any country’s coastline, no nation  However, the LockBit is now working to create
will take responsibility or provide the funding encryptors targeting MacOS for the first
to clean it up. time.

 Cleaning up the garbage patch would bankrupt 53. a


any country.
Wagner Group

 It is a private Russian paramilitary


unit of armed force owned and financed by a
Russian businessman.

 Role – They provide targeting intelligence,


training, logistical support, infrastructure
protection, and backstop proxy militias and
paramilitary groups in key hotspots around the
world.

 They 1st surfaced in 2014, during Russia’s


annexation of Crimea.

 Spread – They have also been deployed in


Syria, Libya, Central African Republic, Mali,
51. c Sudan and Mozambique.
C+C5 Summit  Human Rights Watch reported that Russian
and Wagner Group forces appear to have
 C+C5 denotes China and the five Central Asian summarily executed, tortured, and beaten
countries. civilians since 2019 in Central African Republic.
 The five Central Asian countries Wassenaar Arrangement
include Kazakhstan, Kyrgyzstan,
Tajikistan, Turkmenistan, and  It is a multilateral export control regime on
Uzbekistan. export controls for conventional arms and dual-
use goods and technologies.
 The C+C5 summit was held at Shaanxi
Province, China in 2023.  It came into being in 1996 to succeed the Cold
War-era Coordinating Committee for
 The C+C5 is supported by the Shanghai Multilateral Export Controls.
Cooperation Organisation in terms of security
and trade.

CHENNAI |SALEM| MADURAI | COIMBATORE DELHI | BANGALORE | THIRUVANANTHAPURAM


www.shankariasacademy.com 87
www.iasparliament.com

 With the only exception of China, all other  Group 1 Diseases - Disorders amenable to
permanent members of the U.N. Security one-time curative treatment.
Council are signatories of the WA.
 Financial support upto Rs. 20 lakh under the
 India became a member of the Arrangement Umbrella Scheme of Rashtriya Arogaya
in 2017. Nidhi shall be provided by the Central
Government for treatment, of those rare
54. b diseases that require a one-time treatment
(diseases listed under Group 1).
Logistic Performance Index (LPI)
 Beneficiaries for such financial assistance
2023 edition of Logistics Performance Index (LPI) was would not be limited to BPL families, but
released. extended to about 40% of the population, who
are eligible as per norms of Pradhan Mantri Jan
 Released by – World Bank Arogya Yojana, for their treatment
in Government tertiary hospitals only.
 It is an interactive benchmarking tool that helps
countries to identify the challenges and  Group 2 Diseases - Diseases requiring long
opportunities they face in their performance on term/lifelong treatment having relatively lower
trade logistics. cost of treatment and benefit has been
documented in literature and annual or more
 6 parameters – Customs performance, frequent surveillance is required.
Infrastructure quality, Ease of arranging
shipments, Logistics services quality,  State Governments can consider supporting
Consignment tracking and tracing, Timeliness patients of such rare diseases that can be
of shipments managed with special diets or hormonal
supplements or other relatively low-cost
 It was reported by the World Bank every 2 interventions.
years from 2010 to 2018 with a break in
2020 due to the COVID-19 pandemic and a  Group 3 Diseases - Diseases for
restructuring of the index methodology, which definitive treatment is available but
eventually came out in 2023. challenges are to make optimal patient
selection for benefit, very high
 LPI 2023 – It allows for comparisons across cost and lifelong therapy.
139 countries and measures the speed of trade
with indicators derived from big datasets  The new policy has absolutely no consideration
tracking shipments. for Group 3 patients, who require lifelong
treatment support.
 For the 1st time, it measures the speed of
trade with indicators derived from big datasets  The policy states that, keeping in view the
tracking shipments. resource constraint and competing health
priorities, it will be difficult for the Government
 Earlier, Ministry of Commerce and to fully finance treatment of high-cost rare
Industry released the Logistics Ease Across diseases.
Different States (LEADS) Report 2022.
 The gap can however be filled by creating a
55. b
digital platform for bringing together notified
National Policy for Rare Diseases, 2021 hospitals where such patients are receiving
treatment or come for treatment, on the one
 Launched by – Ministry of Health and hand, and prospective individual or corporate
family Welfare donors willing to support treatment of such
patients.
 Objectives – To lower the incidence and
prevalence of rare diseases based on 56. d
comprehensive preventive strategy.
Mera Gaon Meri Dharohar Programme
 WHO defines rare disease as often debilitating
 It is a pan-India initiative of Ministry of
lifelong disease or disorder with a prevalence of
Culture under National Mission on Cultural
1 or less, per 1000 population
Mapping.
 Classification - Accordingly, a ‘rare disease’
 Launched in – 2023
has been categorised into three groups.
 Objectives – To culturally map India's 6.5
lakh villages, spanning 29 States and 7 Union

CHENNAI |SALEM| MADURAI | COIMBATORE DELHI | BANGALORE | THIRUVANANTHAPURAM


www.shankariasacademy.com 88
www.iasparliament.com

Territories, on a comprehensive virtual  HydroSOS – Global Hydrological Status and


platform. Outlook System.

 To enable people to get an opportunity to  Launched by – World Meteorological


immerse themselves in the diverse and vibrant Organisation
cultural heritage of India.
 Aim – To provide an operational system
 To encourage appreciation for India's culture capable of assessing the current hydrological
and traditions, paving way for economic status and its likely near-future outlook for all
growth, social harmony, and artistic areas of the globe.
development in rural communities.
 It is the backbone supporting the Early
 The cultural mapping of India's 6.5 lakh villages Warnings for All initiative for hydrological
is being made available on a virtual platform disasters and thus supports local resilience and
preparedness.
 7 broad categories
 It is a global operational mechanism for
o Arts and Crafts Village hydrological data.
o Ecologically Oriented Village  It integrates hydrological status assessments
and outlooks from, and for National
o Scholastic Village linked with Textual Hydrological Services (NHS).
and Scriptural Traditions of India
 The system offers simple, accessible
o Epic Village linked with Ramayana,
hydrological information to users such as
Mahabharata and/or Puranic legends
government bodies, UN bodies, aid agencies,
and oral epics
and the general public.
o Historical Village linked with Local and
 It provides hydrological products to National
National History
Meteorological and Hydrological Services
o Architectural Heritage Village (NMHSs) for its informed decision-making.

o Any other characteristic that may need 58. c


highlighting such as fishing village,
National Commission for Protection of Child
horticulture village, shepherding
Rights (NCPCR)
village etc.
 It emphasizes the principle of universality and
 Villages under this programme
inviolability of child rights and recognizes the
Villages States Famous for tone of urgency in all the child related policies
of the country.
Bishnoi Rajasthan Living in harmony
 For the Commission, protection of all children
Raini village Uttarakhand Chipko movement in the 0 to 18 years age group is of equal
importance.
Gujarat 1st solar powered
Modhera  It has been constituted under
village the Commission for Protection of Child
Rights (CPCR) Act, 2005.
Madhya
Kandel Jal Satyagraha’  The commission consists of a
Pradesh
Chairperson who, is a person of eminence
Uttarakhand and has done outstanding work for promoting
linked to the the welfare of children and 6 members, out of
Hanol
Mahabharata which at least two are woman.

linked to the  The commission can inquire into complaints


Vidurashwathar Karnataka
Mahabharata and take suo motu notice of matter relating
to:
Himachal Asia’s oldest fossil
Suketi Deprivation and violation of child
Pradesh park o
rights,
57. d

HydroSOS

CHENNAI |SALEM| MADURAI | COIMBATORE DELHI | BANGALORE | THIRUVANANTHAPURAM


www.shankariasacademy.com 89
www.iasparliament.com

o Non implementation of laws providing  India is the world’s second-largest fish-


for protection and development of producing country.
children,
 One of the key challenges faced by the global
o Non-compliance of policy decisions, marine ecosystem is abandoned, lost
guidelines or instructions aimed at or discarded fishing gear (ALDFG).
mitigating hardships to and ensuring
welfare of the children and provide  A large part of ALDFG is lost in deep seas,
relief to such children, making it difficult to recover.

o Take up the issues arising out of such  Another significant source of marine litter
matters with appropriate authorities. is tourism.

 Under the Right to Education (RTE) Act, 2009,  These waste products are either not collected or
the commission can are mismanaged.
o Examine and review the safeguards for  They eventually leak into the oceans through
rights provided by or under this act and the stormwater drainage system, canals and
recommend measure for their effective small and big rivers.
implementation and
 The other contributors to marine litter
o Inquire into complaints relating include flood waters, discharge of
to child's right to free and untreated municipal
compulsory education. sewage, automobile and industrial
waste generated at the coasts and waste
 Under the Protection of Children from Sexual from shipbreaking yards.
Offences Act (POCSO), 2012
60. a
o It can monitor the designation of
Special Courts by State Governments, UN Permanent Forum on Indigenous Issues
(UNPFII)
o It can monitor the appointment of
Public Prosecutors by State 22nd session of the United Nations Permanent Forum
Governments and on Indigenous Issues (UNPFII) held at the UN
headquarters in New York
o It can call for a report on any specific
case of child sexual abuse falling within  It is a Permanent Forum under United Nations
the jurisdiction of a CWC. (UN).
59. d  It is an advisory body to the Economic and
Social Council established by resolution, July
Coastal Cities to Combat Marine Litter
2000.
Delhi-based non-profit Centre for Science and
Environment (CSE) has launched a coalition of coastal  Aim - It mandates to discuss indigenous issues
cities. related to economic and social development,
culture, the environment, education, health and
 Aim – It aims to fight marine litter pollution human rights.
across India
 It holds annual two-week sessions.
 About 80 % of marine litter comes from land-
based mismanagement of solid waste that  The Permanent Forum is one of 3 UN
reaches the ocean bed through various land-to- bodies that is mandated to deal
sea pathways. specifically with indigenous people’s
issues.
 The remaining 20 % is contributed by coastal
settlements.  The others are the Expert Mechanism on the
Rights of Indigenous Peoples and the Special
 Plastic accounts for 90% of all the waste that Rapporteur Rights of Indigenous Peoples.
ends up in the marine ecosystem.
 The experts nominated by governments are
 Tributaries of major Indian rivers carry around elected by ECOSOC based on the five regional
15-20 % of plastic waste into the marine groupings (Africa; Asia; Eastern Europe; Latin
environment. America and the Caribbean; and Western
Europe and Other States).

CHENNAI |SALEM| MADURAI | COIMBATORE DELHI | BANGALORE | THIRUVANANTHAPURAM


www.shankariasacademy.com 90
www.iasparliament.com

 The Permanent Forum’s engagement and role  Objective – To focus on terrestrial disasters,
in promoting indigenous peoples rights are tropical cyclone analysis, and cyclone
made possible through the Trust Fund on modelling and data assimilation and
Indigenous Issues. understand tropical cyclones and tropical
dynamics.
61. c
 It also provides real-time data on hurricanes
69th Constitutional Amendment Act, 1991 and cyclones which enables timely evacuation
of people in coastal areas.
 In accordance with the recommendation of
the Balakrishnan Committee (1987), the  It will provide rapid microwave
parliament passed the Constitution 69th measurements over the tropic regions to
Amendment Act, 1991. observe the thermodynamics of the
troposphere.
 The amendment inserted the new
Articles 239AA and 239AB in the Constitution  It consists of 4 Cubesats in 3 low earth orbital
providing, inter alia, for a Legislative Assembly planes.
for Delhi.
 These Cubesats are different from other
 Article 239AA (1) - UT of Delhi shall be called weather tracking satellites in frequency of
the National Capital Territory of Delhi observations, as they collect data more
(NCTD) and the administrator appointed shall frequently for scientists involved in weather
be designated as Lieutenant Governor (L-G). forecasts.

 Article 239AA (2) – A Legislative Assembly XPoSat


for the NCT and the seats shall be filled by
members chosen by direct election.  XPoSat – X-Ray Polarimeter Satellite
(XPoSat)
 Article 239AA (3) – Legislative assembly
shall have power to make laws for the whole or  Launched by – ISRO.
any part of the NCTD in any matters
enumerated in the State list or in the  It is being built in collaboration with the Raman
Concurrent list except on the subjects of police, Research Institute (RRI), Bengaluru.
public order, and land.
 Aim – To study various dynamics of bright
 Article 239AA (4) – There shall be a Council astronomical X-ray sources in extreme
of Ministers consisting of not more than conditions.
10% of the total number of members in the
Legislative Assembly, with the Chief Minister  It is India’s 1st and only the world’s 2nd
(CM) at the head to aid and advise the polarimetry mission.
Lieutenant Governor.
 IXPE - The first polarimetry mission is NASA’s
 In the case of difference of opinion between the Imaging X-ray Polarimetry Explorer (IXPE)
L-G and his Ministers on any matter, the L-G that was launched in 2021.
shall refer it to the President for decision and
act accordingly.  2 scientific payloads in a low earth orbit.

 Article 239AA (5) – CM shall be appointed  POLIX – Polarimeter Instrument in X-rays,


by the President and other Ministers shall be will measure the polarimetry parameters
appointed by the President on the advice of the (degree and angle of polarisation).
CM.
 XSPECT – X-ray Spectroscopy and Timing
 The Ministers shall hold office during payload will give spectroscopic information.
the pleasure of the President.
 It would observe several types of sources, such
62. d as X-ray pulsars, blackhole binaries, low-
magnetic field neutron star, etc.
TROPICS Mission
Luna-25
 Launched by – NASA
 It is Russia’s lander mission aims to land on the
 TROPICS – It is a constellation of observing Moon's lunar South Pole.
platforms that will measure temperature,
humidity and precipitation with spatial
resolution on hourly basis.

CHENNAI |SALEM| MADURAI | COIMBATORE DELHI | BANGALORE | THIRUVANANTHAPURAM


www.shankariasacademy.com 91
www.iasparliament.com

 Luna-25 aims to soft-land, analyse soil samples Benefits


and conduct long-term scientific research on
the Moon’s surface.  Mitigate exchange rate risk - It can lower
transaction costs of cross-border trade and
63. b investment operations by mitigating exchange
rate risk.
India Meteorological Department (IMD)
 Eliminates the risk of exposure to currency
 It is the national meteorological agency volatility faced by Indian businesses.
of India.
 Competitive exports - Reduced currency
 Established in – 1875, as the Meteorological risk can reduce the cost of doing business and
Office of India. thus helps in making exports more competitive
in the global market.
 It was renamed the India Meteorological
Department in 1901.  Integrate with global system - It will
help to integrate the Indian financial
 It is a member of the World Meteorological system with the global financial system.
Organization (WMO)
 Lead to increased investment and economic
 It is a scientific department under the Ministry growth.
of Earth Sciences of the Government of India.
 Reduces the demand for forex reserves -
 Function – It is responsible for Reduces the need to maintain foreign exchange
providing weather forecasts, warnings, reserves if a sizeable share of India’s trade can
and other meteorological services to the be settled in terms of the domestic currency.
Indian public.
65. a
Regional Specialised Meteorological Centres
(RSMCs) Carbon Border Tax Mechanism (CBAM)

 IMD is one of the six Regional Specialised Carbon Border Tax Mechanism was introduced by the
Meteorological Centres (RSMCs) of the World European Union.
Meteorological Organisation (WMO).
 Aim - It is a tool to put fair price on the carbon
 It has the responsibility for forecasting, naming emitted during the production of carbon
and distribution of warnings for tropical intensive goods that are entering the EU,
cyclones in the Northern Indian Ocean region. and to encourage cleaner industrial
production in non-EU countries.
64. c
 In other words, CBAM imposes a levy on
Internationalization of the Rupee imported carbon-intensive goods from non-
EU countries where climate rules are less
The Reserve Bank of India (RBI) has put in place the strict.
mechanism for rupee trade settlement with as many as
18 countries.  The CBAM is designed to be compatible with
WTO-rules.
 Internationalization of the Rupee – It is the
process of increased cross-border transactions
involving the Indian currency.

 It corresponds to trade especially in import-


export, current account transactions, and
capital account transactions.

 It enables the international settlement of


trade in Indian rupees in foreign trades, as
opposed to other currency including US dollars.  Impacts on India – Indian firms risk losing
market share to EU-based producers or those in
 Goal – To make it a more widely accepted other more carbon-efficient nations.
currency in international trade and
investment.  It will affect substantial exports as the EU is an
important trade partner for India.

CHENNAI |SALEM| MADURAI | COIMBATORE DELHI | BANGALORE | THIRUVANANTHAPURAM


www.shankariasacademy.com 92
www.iasparliament.com

 While the product from India may be cheaper Thailand, but it differs in a number of
than an American product, product plus tax vegetative and reproductive characteristics.
price will make Indian products more
expensive.  It is the largest species of the genus described
so far in terms of tree height and girth of the
 It will lead to sharper trade diversion and more tree trunk.
trade among developed countries.
68. d
66. b
Global Greenhouse Gas Watch (G3W)
ANZUS
A new greenhouse gas (GHG) monitoring initiative
 Members – Australia, New Zealand, US was announced.
(ANZUS)
 Coordinated by - World Meteorological
 It was a trilateral defence agreement. Organisation (WMO).

 Signed in – 1951  Aim – To establish


internationally coordinated monitoring of
 Aim – To protect the security of the Pacific greenhouse gas fluxes (top-down) to support
region. the provision of the UNFCCC parties and other
stakeholders.
 It was an arrangement made after the Second
World War and the early stages of the Cold War.  It will fill critical information gaps and provide
an integrated and operational framework.
 It was further ratified by the United States and
entered into force in 1952.  Framework – It will bring all space-based
and surface-based observing systems, as well
 The members met annually to discuss their as modelling and data assimilation capabilities,
shared interests and concerns. under one roof.

 In 1984, New Zealand declared its country a  4 main components


nuclear-free zone and refused to allow U.S.
nuclear-powered submarines to visit its ports. o Surface-based and satellite- based
observations.
 On 1986, the United States suspended its
treaty obligations toward New Zealand and o Prior observations.
reduced the two countries military ties.
o Near-real time international exchange
 The 3 nations remained formal parties to the of input and output data.
treaty, but ANZUS was inoperative.
o Global high-resolution modelling/data
67. a assimilation.

Meiogyne Arunachalensis  Output – It will provide sustained delivery of


monthly global estimates of net GHG fluxes into
Researchers discover new species of tree in Arunachal and out of the atmosphere at a 100 by 100 km
Pradesh. resolution.

 This new species of tree is the 1st record of  Benefits – Improved evaluation of sources
the genus from Northeast India and the eastern and sinks of greenhouse gases and also indicate
Himalayan biodiversity hotspot. their association with the biosphere, the ocean
and the permafrost areas.
 Genus – Meiogyne
69. b
 Distribution – South and Southeast Asia and
includes approximately 33 described taxa. Global Buddhist Summit, 2023

 It is also found in intervening districts of Lower The 1st Global Buddhist summit could leverage India’s
Dibang Valley and Lohit, and also in northern soft power to connect the Buddhist population around
parts of Myanmar in areas neighbouring the the world.
Namdapha National Park.
 Organised by – International Buddhist
 The species has morphological similarity with Confederation and the Ministry of
Meiogyne maxiflora, a species distributed in Culture

CHENNAI |SALEM| MADURAI | COIMBATORE DELHI | BANGALORE | THIRUVANANTHAPURAM


www.shankariasacademy.com 93
www.iasparliament.com

 Theme – ‘Responses to Contemporary  This qualitative analysis is combined with


challenges: Philosophy to Praxis’. quantitative data on abuses and acts of violence
against journalists during the period evaluated.
 Prime vision – To look into the teachings of
the Shakyamuni Buddha.  Qualitative parameter – Based on the
responses of “press freedom specialists to an
 4 themes RSF questionnaire.

o Buddha Dhamma and Peace  Quantitative parameter – A count of abuses


against journalists and media outlets.
o Buddha Dhamma: Environmental
Crisis, Health and Sustainability  Criteria – Pluralism, Media Independence,
Media Environment and Self-censorship,
o Preservation of Nalanda Buddhist Legislative framework, Transparency, and the
Tradition quality of the infrastructure that supports the
production of news and information.
o Buddha Dhamma Pilgrimage, Living
heritage and Buddha Relics.  2023 Rankings – Top 3 positions were by
Norway, Ireland and Denmark, while the
 Significance – It could be a good push to the bottom 3 positions were by Vietnam, China and
foreign soft power diplomacy. North Korea.
 It demonstrates India’s commitment to  Pakistan and Sri Lanka fared better
preserving and promoting Buddhist culture and
than India.
heritage.
71. a

Kaladan Multi Modal Transit Transport Project

 It is a joint initiative of India and Myanmar.

 It connects the Kolkata Port of India with


the Sittwe Port of Myanmar by sea, Sittwe to
Paletwa via River Kaladan, Paletwa to the
border of India, and Myanmar via road and
further ahead to Lawngtlai, Mizoram by road.

 It is named a multimodal project as it uses a


wide range of infrastructures like roads,
bridges, and floating barrages.

 Kaladan – It is a river that flows


in Mizoram of India, and Chin and
Rakhine state of Myanmar.

 The Kaladan River is called the Chhimtuipui


70. d River in India.
World Press Freedom Index

India slips in World Press Freedom Index, ranks 161


out of 180 countries in 2023.

 It is an annual ranking.

 Published by – Reporters without


Borders (RSF), an international NGO whose
self-proclaimed aim is to defend and promote
media freedom.

 Aim – To compare the level of press freedom


enjoyed by journalists and media in 180
countries and territories.

CHENNAI |SALEM| MADURAI | COIMBATORE DELHI | BANGALORE | THIRUVANANTHAPURAM


www.shankariasacademy.com 94
www.iasparliament.com

Great Observatories Program

 It is a family of 4 space-based observatories


each observing the Universe in a different
kind of light. They are:

1. Visible light Hubble Space Telescope

2. Compton Gamma-Ray Observatory

3. Chandra X-Ray Observatory

4. Infrared Spitzer Space Telescope


72. b
 NASA's James Webb Space Telescope is the
Trachoma successor to the famous Hubble telescope.
 It is a disease of the eye caused by infection with
the bacterium Chlamydia trachomatis.

 In its early stages, trachoma causes


conjunctivitis (pink eye).

 Blindness from trachoma is irreversible.

 Trachoma infection is transmitted by direct or


indirect transfer of eye and nose discharges of
infected people.

 It particularly affects young children, who


harbour the principal reservoir of infection.
74. c
 According to WHO, the prevalence of trachoma
should be less than 5% to mark it as eliminated. Drying Lakes across Asia

 WHO is yet to declare India trachoma  Aral Sea – It is a saltwater lake located
free. in Central Asia.

73. a  It shares the boundary between Kazakhstan to


the north and Uzbekistan to the south.
Spitzer Telescope
 It has shrinked in its area and volume that
 Spitzer was designed to detect infrared began in the second half of the 20th century.
radiation, which is primarily heat radiation.
 Dead Sea – It is a Salt Lake between Israel and
 It allows to peer into regions of space that are Jordan in Southwestern Asia.
hidden from optical telescopes.
 Jordan River is the main source of water for this
 It was launched in 2003 by NASA, U.S. lake.
 It was comprised of two major components:  It diverted its flow from mid-2010 so the lake is
getting drier.
1. The Cryogenic Telescope Assembly - contained
the 85-centimeter telescope and Spitzer's three  Xingkai Lake – also known as Lake Khanka,
scientific instruments. it is located in Eastern Asia.
2. The Spacecraft - controlled the telescope,  It is a freshwater lake is located in northern
provided power to the instruments, handled the Inner Mongolia near the Russian border.
scientific data and communicated with Earth.
 Toshka Lakes – It is located in north–
 It is the final mission in NASA's Great eastern corner of Africa, a hyper arid area
Observatories Program. in the Western Egyptian Desert.

 This lake is diminishing with low Nile


discharges and appeared again with the Lake
Nasser high flood.

CHENNAI |SALEM| MADURAI | COIMBATORE DELHI | BANGALORE | THIRUVANANTHAPURAM


www.shankariasacademy.com 95
www.iasparliament.com

76. d

Purana Qila or the Old Fort

 The Purana Qila is a timeless monument that


stands on the South Eastern part of the
present city of New Delhi.

 The Purana Qila, built by Sher Shah


Suri and Mughal emperor Humayun, is
believed by many to be the site of Indraprastha,
as mentioned in the Mahabharata.

 Humayun built it as a part of his new city


75. d of Dinpanah in the 16th century.
Porunai Civilization
 A strong local tradition believes that the area in
which the Purana Qila stands today is the site of
 Tamil Nadu’s Porunai Civilization dates back to
Indraprastha, the capital of the Pandavas of the
3200 years old.
great epic Mahabharata.
 Aditchanallur and Korkai (Pandya kingdom’s
 It is for the same reason that the Purana Qila is
port town) and Sivakalai, in Thoothukudi
often called the Pandavon ka Qila.
district are few among the other Porunai sites.
77. c
 The entire Tamirabharani watercourse is
known as ‘Cradle of Civilization of South India’. Forum for India-Pacific Islands Cooperation
(FIPIC)
 Large quantities of ornaments made of gold and
tin-mixed bronze, reflect the life style of people.  The Forum for India–Pacific Islands
Cooperation (FIPIC) was launched during
Prime Minister, Mr. Narendra Modi's visit to
Fiji in November 2014.

 It is a multinational grouping developed in


2014 for cooperation between India and 14
Pacific Islands Countries.

 FIPIC includes 14 of the island countries such


as Cook Islands, Fiji, Kiribati, Marshall
Islands, Micronesia, Nauru, Niue,
Palau, Papua New Guinea, Samoa, Solomon
Islands, Tonga, Tuvalu, and Vanuatu.

 FIPIC member nations are relatively small in


land area and distant from India, many have
large exclusive economic zones (EEZs), and
Thamirabarani River
offer promising possibilities for fruitful
 It is the only major perennial river in Tamil cooperation.
Nadu.
 FIPIC member nations are located in Oceania.
 It is the state’s shortest river that originates
 Oceania is a region made up of thousands of
from the Agastyarkoodam peak of Pothigai hills
islands throughout the Central and South
of the Western Ghats, Tamil Nadu.
Pacific Ocean.
 It empties into the sea at the Gulf of Mannar
78. a
after passing through Tirunelveli and
Thoothukudi districts. Indian Council for Cultural Relations (ICCR)
 Naming - Thamirabarani River was  The Indian Council for Cultural Relations
historically called Porunai, Than Porunai, (ICCR) is an autonomous body under the
Porunal and Poruntham in Tami Literature. administrative control of the Ministry of
External Affairs.
 The word ‘Than Porunai’ evolved into ‘Tamira
Porunai’ before becoming ‘Thamirabarani’

CHENNAI |SALEM| MADURAI | COIMBATORE DELHI | BANGALORE | THIRUVANANTHAPURAM


www.shankariasacademy.com 96
www.iasparliament.com

 ICCR was founded in 1950 by Maulana Abul  The major four species of primates are also
Kalam Azad, independent India’s first found at Periyar - the rare lion-tailed macaque,
Education Minister. the Nilgiri Langur, Gee's Golden Langur,
Common Langur and Bonnet Macaque.
 Maulana Abul Kalam Azad is the first
president of ICCR.  Tribes - Paliyans, Mannans, Malayarayans,
Mala Pandarams, Uralis and Ulladans.
 Right from its establishment until 1958, the
ICCR was under the administrative jurisdiction 80. c
of the Education Ministry.
Ministry of MSME launched an outreach programme
 The jurisdiction of the Council was transferred under the Self Reliant India (SRI) Fund at Dimapur,
to the Ministry of External Affairs following a Nagaland.
decision of the Cabinet Committee on Foreign
Affairs. SRI (Self-Reliant India) Fund

 The Ministry of External Affairs assumed  It is a SEBI registered Category II Alternative


administrative and operational control of the Investment Fund.
Council in 1970-71 with a view to making the
Council an effective instrument of India's  Aim - It aims to make equity investments in
foreign policy. micro, small and medium enterprises (MSMEs)
to address the current capital gap and catalyze
 In 1978, following the suggestions of the flow of capital in the sector.
the Asoka Mehta Committee, the Council
took over management of all cultural  The anchor investor of the Fund is the
delegation from the Department of Culture. Government of India, through the Ministry of
Micro, Small and Medium Enterprises.
 ICCR promotes cultural exchanges with other
countries and people, and to develop relations  SRI Fund has been sponsored by The
with nations. National Small Industries Corporation
Ltd. (a mini-Ratna Corporation of the
79. b Government of India under the Ministry of
Micro, Small and Medium Enterprises).
Periyar Tiger Reserve
 It is setup under its wholly owned
 The Periyar Tiger Reserve (PTR) in Thekkady subsidiary, NSIC Venture Capital Fund
located in the southern region of the Western Limited (NVFCL).
Ghats in Tamil Nadu and Kerala.
 NVCFL has appointed SBICAP Ventures Ltd
 It gets its name from the River Periyar which (SVL), an Asset Management Company, as the
has its origin deep inside the reserve. Investment Manager of the Fund.

 It was declared as a Sanctuary in 1950 and Objectives


declared as Tiger Reserve in 1978.
 Helping MSME businesses grow faster so as to
 Habitat - The terrain is Hilly and undulating. energize the economy and create employment
opportunities;
 Rivers - Two major rivers namely Periyar and
Pamba drain the area.  Supporting enterprises which have the
potential to graduate beyond the MSME
 Mullai Periyar Dam is located within the PTR. bracket and become National / International
champions; and
 The vegetation comprises of Tropical evergreen
forests, semi- evergreen forests, Moist  Supporting MSMEs which help making India
deciduous Forests, Transitional fringe ever self-reliant by producing relevant technologies,
green forests, grass lands and eucalyptus goods and services.
plantations.
81. b
 Fauna - In addition to Tiger, Periyar is an
important elephant habitat. Madden-Julian Oscillation (MJO)

 Gaur, sambar deer, leopard, sloth bear, wild  It is an oceanic-atmospheric phenomenon


dog, Lion tailed monkey, Nilgiri langur, barking which affects weather activities across the
deer and Nilgiri tahr are found here. globe.

CHENNAI |SALEM| MADURAI | COIMBATORE DELHI | BANGALORE | THIRUVANANTHAPURAM


www.shankariasacademy.com 97
www.iasparliament.com

 It brings major fluctuation in tropical weather the Modified Mumbai Interbank Forward
on weekly to monthly timescales. Outright Rate (MMIFOR), replacing MIFOR.

 Definition – It is an eastward moving  LIBOR – London Interbank Offered Rate, a


system of wind, cloud and pressure that global benchmark interest rate.
brings rain as it circles around the
equator.  It is calculated daily based on the estimated
interest rates at which banks can borrow
 It’s a traversing phenomenon and is most unsecured funds from other banks in the
prominent over the Indian and Pacific Oceans. London wholesale or interbank market.

 It goes around the globe in 30-60 days on  It is used as a benchmark to settle trades in
average. Sometimes, it can take 90 days. futures, options, swaps and other derivative
financial instruments in over-the-counter
 In active phase – It results in more than markets (participants engaging directly without
average rainfall for that time of the year. using an exchange) and on exchanges globally.

 In suppressed phase – The area receives less  MIFOR – Mumbai Interbank Forward Offer
than average rainfall. Rate is the rate that Indian banks use as a
benchmark for setting prices on forward-rate
 The Indian Ocean Dipole (IOD), El Nino and agreements and derivatives.
MJO are all oceanic and atmospheric
phenomena, which affect weather on a large  MIFOR is a mix of the London Interbank
scale. Offered Rate and a forward premium derived
from Indian forex markets.
 IOD only pertains to the Indian Ocean, but the
other 2 affect weather on a global scale-up to 83. b
the mid-latitudes.
Civil Liability for Nuclear Damage Act (CLNDA)
 IOD and El Nino remain over their respective
positions, while MJO is a traversing A French private company and Nuclear Power
phenomenon. Corporation of India (NPCIL) are resolving the issues
in building the six nuclear power reactors in
Maharashtra’s Jaitapur.

 Launched in – 2010

 Aim – To enable a speedy compensation


mechanism for victims of a nuclear accident.

 It is enacted to keep India in line


with international convention on civil liability
on nuclear damage.

o Vienna Convention on Civil


Liability for Nuclear Damage

o Convention on Supplementary
82. c Compensation for Nuclear Damage
(CSC).
Secured Overnight Financing Rate (SOFR)
 Convention on Supplementary
RBI announced that the LIBOR and MIFOR would Compensation (CSC) – It was adopted in
cease to be a representative benchmark from June 30, 1997 with the aim of establishing a minimum
2023. national compensation amount.
 Secured Overnight Financing Rate  India was a signatory to CSC but Parliament
(SOFR) – It is a benchmark overnight ratified it only in 2016.
interest rate and reference rate for dollar-
denominated derivatives and loans.  Features – It provides for strict and no-fault
liability on the operator of the nuclear plant,
 It is established as an alternative to LIBOR. where it will be held liable for damage
regardless of any fault on its part.
 Accordingly, in India, new transactions were to
be undertaken using the SOFR and

CHENNAI |SALEM| MADURAI | COIMBATORE DELHI | BANGALORE | THIRUVANANTHAPURAM


www.shankariasacademy.com 98
www.iasparliament.com

 It also specifies the amount (Rs.1,500 crore) the 85. c


operator will have to shell out in case of damage
and requires the operator to cover liability Leatherback Turtles
through insurance.
 It is the largest turtle in the world.
 In case the damage claims exceed Rs.1,500
crore, the government shall step in.  It is the only species of sea turtle that lack
scales and a hard shell.
 It has limited the government liability amount
to the rupee equivalent of 300 million Special  Name – For their tough rubbery skin and have
Drawing Rights (SDRs) or about Rs.2,100 to Rs. existed in their current form since the age of the
2,300 crore. dinosaurs.

 It lays down 2 conditions under which the  Distribution – It is found in every ocean
national law of a country may provide the except the Arctic and Antarctic.
operator with the “right of recourse”, where
they can extract liability from the supplier.  They have the widest global distribution of any
reptile, with nesting mainly on tropical or
o If it is expressly agreed upon in the subtropical beaches.
contract
 Highly migratory – It can swim over 10,000
o If the nuclear incident results from an miles a year between nesting and foraging
act or omission done with intent to grounds.
cause damage
 Pacific leatherbacks migrate from nesting
 However, India went beyond these 2 conditions beaches in the Coral Triangle all the way to the
and introduced the concept of supplier liability California coast to feed on the abundant
over and above that of the operator’s in the jellyfish.
CLNDA.
 Accomplished divers – The deepest
84. a recorded dive reaching nearly 4,000 feet deeper
than most marine mammals.
Critical and Emerging Technology (iCET)
 It has a unique thermoregulatory adaptation
INDUS-X was launched to promote partnerships allows them to maintain core body
between India and U.S. temperatures at extremely cold depths.

 INDUS-X – It will be launched under the  Conservation status


Initiative on Critical and Emerging
Technologies (iCET) to promote partnerships o IUCN – Vulnerable, but many
between two countries defence innovation subpopulations (such as in the Pacific
ecosystem. and Southwest Atlantic) are Critically
Endangered.
 iCET was launched in – 2022, an U.S.-India
initiative o CITES – Appendix I

 Objective – To elevate and expand strategic


technology partnership and defence industrial
cooperation between the governments,
businesses, and academic institutions of 2
countries.

 Led by – Nationals Security Advisor’s


(NSA) of both the countries.

 6 priority sectors – defence, space,


quantum, artificial intelligence,
semiconductors and telecommunications.

 It is expected to remove barriers in technology


transfers and co-production in the areas of
critical and emerging technologies.

CHENNAI |SALEM| MADURAI | COIMBATORE DELHI | BANGALORE | THIRUVANANTHAPURAM


www.shankariasacademy.com 99
www.iasparliament.com

86. b  So, to avoid classifying a loan as an NPA, banks


adopt the ever-greening of loans.
AIRAWAT (AI Research, Analytics and
knowledge Assimilation)  In the past, many banks had indulged in
dressing up bad loans and given additional
AI supercomputer 'AIRAWAT' has secured the 75th funds to companies who didn’t have the
position in the prestigious Top 500 Global capacity to repay.
Supercomputing List.
 Impact – An increase in lending to
 It is an AI-based cloud computing unproductive firms, popularly referred to as
infrastructure (supercomputer). zombies will inflates credit growth and the
resultant loan defaults haunt financial
 This platform will assist in the R&D of new institutions at a later stage.
technologies that help in solving business and
governance use cases. 88. d
 Recommended by – National Strategy Laundromat countries
for Artificial Intelligence (NSAI).
A report by Finland based group cited that India is
 Installed at – C-DAC Pune. leader of five countries named as the “Laundromat”
countries.
 Nodal Ministry – Ministry of Electronics
and Information Technology (MeitY)  Laundromat countries – Countries which
buy Russian oil and sell processed products to
 The AIRAWAT PoC of 200 AI Petaflops European countries, sidestepping European
integrated with PARAM Siddhi, AI of 210 AI sanctions against Russia.
Petaflops gives a total peak compute of 410 AI
Petaflops Mixed Precision.  5 Countries – India, China, United Arab
Emirates, Singapore and Turkey
 It has sustained compute capacity of 8.5
Petaflops (Rmax) Double Precision.  These countries are responsible for 70% of
Russia’s crude oil exports.
 Peak compute capacity (Double Precision,
Rpeak) – 13 Petaflops.  Price cap coalition countries have increased
imports of refined oil products from countries
 Supercomputers aggregate the computational that have become the largest importers of
power of multiple servers or compute devices Russian crude.
for processing huge amounts of data and
performing complex calculations.  The Price Cap Coalition – It comprises
Australia, Canada, the European Union, Japan,
 Peta flops – A unit of computing speed equal the United Kingdom and the United States.
to 1000 million (1015) floating-point operations
per second.  India emerged as the leading exporter of
refined oil products at 3.7 million tonnes to
87. b Price Cap Coalition countries in 2022.
Zombie Lending  Among laundromat countries, India, which in
April remained the highest global consumer of
RBI Governor Shaktikanta Das warned about the seaborne Russian crude.
banks adopting innovative methods for evergreening
of loans.  The EU was the largest importer of oil products
from these laundromat countries worth €17.7
 Definition – It refers to the practice of billion, despite partially banning crude oil
providing credit to entities that do not imports from Russia last year.
have the capability to repay.
89. b
 Evergreening – If an account turns into a
non-performing asset (NPA), banks are Monkey Pox (mpox)
required to make higher provisions which will
impact their profitability.  Mpox (formerly known as monkeypox) is
a viral disease caused by infection with the
 Non-performing asset (NPA) – A loan monkeypox virus.
turns into an NPA, if the interest or instalment
remains unpaid for more than 90 days.  Mpox virus is part of the same family of viruses
as variola virus, the virus that causes smallpox.

CHENNAI |SALEM| MADURAI | COIMBATORE DELHI | BANGALORE | THIRUVANANTHAPURAM


www.shankariasacademy.com 100
www.iasparliament.com

 Mpox is rarely fatal and its symptoms are  According to the recent RBI Framework
similar to smallpox, but milder. for green deposits Sectors eligible for
green deposits are:
 Mpox is not related to chickenpox.
o Green Buildings
 Varicella (chickenpox) is an acute infectious
disease caused by varicella-zoster virus (VZV). o Energy efficiency

 Mpox spreads from contact with infected in the o Renewable Energy


following ways:
o Clean Transportation
o Persons, through touch, kissing, or sex
o Climate Change Adaptation
o Animals, when hunting, skinning, or
cooking them o Pollution Prevention and Control

o Materials, such as contaminated o Wastewater Management, and others


sheets, clothes or needles
o Sustainable Water, and waste
o Pregnant persons, who may pass the management
virus on to their unborn baby.
o Terrestrial and Aquatic Biodiversity
 Skin lesion material by Polymerase chain Conservation
reaction (PCR) is the laboratory confirmation
o Sustainable Management of Living Natural
test for it.
Resources and Land Use
 Common symptoms are a skin rash or mucosal Exclusions
lesions which can last 2–4 weeks accompanied
by fever, headache, muscle aches, back pain,  Projects involving new or existing extraction,
low energy, and swollen lymph nodes. production and distribution of fossil fuels,
including improvements and upgrades; or
 Vaccines and therapeutics developed for where the core energy source is fossil-fuel
smallpox and approved for use in some based.
countries can be used for mpox in some
circumstances.  Nuclear power generation.
 According to the WHO, two distinct clades  Direct waste incineration.
(clade I and clade II) are:
 Alcohol, weapons, tobacco, gaming, or palm oil
1. The West African clade industries.
2. The Congo Basin clade (the Central
African clade)  Renewable energy projects generating energy
from biomass using feedstock originating from
 In 2022–2023 a global outbreak of mpox was protected areas.
caused by a strain known as clade IIb.
 Landfill projects.
90. a
 Hydropower plants larger than 25 MW.
Green Deposits
91. a
 A green deposit is a fixed-term deposit for
investors looking to invest in environmentally Bihan Mela
friendly projects.
 It is a seed festival aims to help
 Green deposits vary from the regular deposits tribal Kondh farmers in Odisha return to
only on that banks promise to earmark the their agricultural traditions.
money to be used towards environment-
friendly projects.  Preparations begin as soon as farmers have
harvested kharif crops, which includes both
 Banks will have to come up with a set of rules or hybrid and indigenous varieties of paddy,
policies approved by their respective Boards millets, maize and sorghum.
that need to be followed while investing green
deposits from customers.  Women, who are at the helm of this festival,
carefully collect seeds of the indigenous
varieties and store them in earthen pots.

CHENNAI |SALEM| MADURAI | COIMBATORE DELHI | BANGALORE | THIRUVANANTHAPURAM


www.shankariasacademy.com 101
www.iasparliament.com

 Then, on a designated day in December, they  It flows through the north side of Vapi city in
decorate the pots with red and white motifs, Gujarat.
place them in a bamboo basket and carry it on
head to the village where the fair is being  Kolak village on the bank of River Kolak in
organised. Gujarat’s Valsad district.

 The fair mimics a traditional market where Channapattana Toys


farmers used to exchange seeds.
 Channapattana is a place
 Even in dongars or hilltops, where families used in Ramanagara district
to practice mixed cropping until recently, have in Karnataka famous for handmade lacquer
shifted to monoculture cash crops like cashew. wooden toys.

 This has not only affected their food and  The origin of the wooden toys’ dates back to
nutritional security, but also degraded the soil the period of Tipu Sultan.
and made the farmers more vulnerable to crop
loss.  Tipu Sultan, the ruler of Mysore, was a great
admirer of wooden toys.
 The seed festival was thus introduced to help
farmers return to their traditional ways of  He invited artisans from Persia and trained the
farming like mixed-cropping. local artists to earn the process of making of the
wooden toys.
92. d
 The artisans still follow the traditional method
Kolak River of making the toys, using only ivory wood for
making the toys.
 Kolak River is a river in Gujarat in western
India.  Very rarely rosewood and sandalwood are used.

 The west flowing river originates in Saputara  Now the artisans have also started to use other
hills near Valveri and flows into the Arabian Sea woods like teak, pine, rubber and cedar wood.
near Udwada.
 The colours used on these toys are vegetable
 Its basin has a maximum length of 50 km and dyes, non-toxic and safe for children.
also connected to Madhuban reservoir of
Damanganga River.  Channapattana toys hold GI tag under
Handicrafts category.

93. c

Sanchar Saathi Portal

 It is an initiative of Department of
Telecommunications to empower mobile
subscribers and strengthen their security.

 It empowers citizens by allowing them to:

o Check the connections registered on their


names

o Report fraudulent or unrequired connections

o Block the mobile phones which are stolen/lost

o Check IMEI genuineness before buying a


mobile phone

 The Sanchar Saathi facilities have been


developed by Centre for Development of
Telematics (C-DoT).

 Sanchar Saathi contains various modules like


CEIR, TAFCOP, etc.

CHENNAI |SALEM| MADURAI | COIMBATORE DELHI | BANGALORE | THIRUVANANTHAPURAM


www.shankariasacademy.com 102
www.iasparliament.com

o CEIR module facilitates tracing of the  Panchayat Empowerment and


lost/stolen mobile devices. Accountability Incentive Scheme is
implemented by the Ministry of
o TAFCOP module facilitates a mobile Panchayati Raj.
subscriber to check the number of
mobile connections taken in his/her 95. a
name.
Arab League
o Know Your Mobile facilitates the
citizens to check the genuineness of  It is an intergovernmental organisation
IMEI of their mobile device. encompassing all Arab states in the Middle
East and North Africa.
o ASTR tool to identify SIMs issued using
fraudulent/forged documents.  Aim - To improve ties among its member
countries, coordinate their policies, and
94. b promote cooperation while respecting the
independence and sovereignty of each member.
Khadi and Village Industries Commission
 The Arab League, formally known as the
 The Khadi and Village Industries Commission League of Arab States, was established in 1945
(KVIC) is a statutory body established by Khadi with initially just 6 nations: Egypt, Iraq,
and Village Industries Commission Act of Jordan, Lebanon, Saudi Arabia, and Syria.
1956 in 1957.
 Currently, it has 22 member states, who have
 KVIC took over the work of former All India pledged to cooperate on economic and military
Khadi and Village Industries Board since then. affairs, among other issues.
 It was established with the object to plan,  It is headquartered at Cairo, Egypt.
support and promote Khadi and Village
Industries.  Syria – Syria was ousted from the Arab
League in 2011 following President Bashar al-
 KVIC functions under the Ministry of Micro, Assad’s brutal crackdown on pro-democracy
Small and Medium Enterprises (MoMSME). protests, which led to the ongoing civil war in
the country.
 It is an apex body with regard to khadi and
village industries within India.  Membership in the Gulf Cooperation
Council is not a prerequisite for joining
 It is charged with the planning, promotion, this league.
organisation and implementation of programs
for the development of Khadi and other village
industries in the rural areas.

 It acts in coordination with other agencies


engaged in rural development wherever
necessary.

 Some of the schemes and projects implemented


by KVIC are:

o PM’s Employment Generation


Programme (PMEGP)

o National Beekeeping & Honey 96. a


Mission (NBHM)
World Health Organization (WHO)
o Scheme of Fund for Regeneration
of Traditional Industries  World Health Organization (WHO), founded in
(SFURTI) 1948 connects nations, partners and people to
promote health, keep the world safe and serve
o Project RE-HAB
the vulnerable so everyone, everywhere can
o Project BOLD attain the highest level of health.

 In order to effectively implement the schemes  The WHO is a specialized agency of the
of KVIC, the state level Khadi and Village United Nations (UN) that is responsible for
Industries Boards (KVIBs) were also set up. international public health.

CHENNAI |SALEM| MADURAI | COIMBATORE DELHI | BANGALORE | THIRUVANANTHAPURAM


www.shankariasacademy.com 103
www.iasparliament.com

 The WHO's mission is to achieve health for all which led to the creation of Chapters of the
people. Group of 77.

 The World Health Assembly is the decision-  The Group of 77 is the largest
making body of WHO. intergovernmental organization of developing
countries in the United Nations.
 Headquarters - Geneva, Switzerland with 6
regional and 150 country offices across the  The South Summit is the supreme decision-
world. making body of the Group of 77.

 WHO receives funding through membership  The Annual Meeting of the Ministers for
dues paid by Member States and voluntary Foreign Affairs of the Group of 77 is convened
contributions from Member States and other at the beginning of the regular session of the
partners. General Assembly of the United Nations in New
York.
 The State of the World’s Children Report is
published by UNICEF.  For the year 2024, the Republic of Uganda
holds the Chairmanship of the G-77.
97. a
 The activities of the Group of 77 are financed
Animal Welfare Board of India (AWBI) through contributions by Member States in
accordance with the relevant decisions of the
 The Animal Welfare Board of India is a First South Summit.
statutory advisory body on Animal
Welfare Laws and promotes animal welfare in 99. c
the country.
UN Economic and Social Commission for Asia
 It is established in 1962 under Section 4 of the and the Pacific (ESCAP)
Prevention of Cruelty to Animals Act,
1960. A new study says most countries in Asia and the Pacific
are not prepared to manage extreme weather.
 Headquarters - Faridabad, Haryana.
 Aim - It aims to increase economic activity in
 Ministry - Ministry of Fisheries, Animal Asia and the Far East.
Husbandry and Dairying.
 It is the most inclusive intergovernmental
 It was started under the stewardship of Late platform in the Asia-Pacific region.
Smt. Rukmini Devi Arundale, well known
humanitarian.  The Commission promotes cooperation among
its 53 member States and 9 associate members
 Board ensures that animal welfare laws in the in pursuit of solutions to sustainable
country are diligently followed, provides grants development challenges.
to Animal Welfare Organizations and advises
the Government of India on animal welfare  ESCAP is one of the 5 regional
issues. commissions of the United Nations under the
jurisdiction of the UN Economic and Social
 The Board consists of 28 Members including 6 Council (ECOSOC).
Members of Parliament (2 Members of
Parliament from Rajya Sabha and 4 Members  ESCAP also provides support to partners at the
of Parliament from Lok Sabha). national level.

 The term of office of Members is for a period  The commission is composed of 53 Member
of 3 years. States (including India) and nine Associate
members, mostly from the Asia and Pacific
98. b regions

Group of 77  In addition to countries in Asia and the Pacific,


the commission's members includes France,
 The Group of 77 (G-77) was established on 15 the Netherlands, the United Kingdom and the
June 1964 by seventy-seven developing United States.
countries signatories of the “Joint Declaration
of the Seventy-Seven Developing Countries”.

 Charter of Algiers is a permanent


institutional structure gradually developed

CHENNAI |SALEM| MADURAI | COIMBATORE DELHI | BANGALORE | THIRUVANANTHAPURAM


www.shankariasacademy.com 104
www.iasparliament.com

100. a

UN report says that India tops the list of 10 countries,


bear 60% of global maternal deaths, stillbirths &
newborn deaths burden.

Neonatal Mortality Rate (NMR)

 NMR refers to the number of deaths of infants


aged 0–27 days per 1,000 live births.

 Infant mortality rates refer to the number of


deaths under age 1 year per 1,000 live births.

 The NMR and IMR are commonly accepted as


a measure of the general health and wellbeing
of a population.

 Neonate - Infants 0 to 27 days of life, under 28


days of life.

 Early neonate - Infants 0 to 6 days (under 7


days) of age.

 Late neonate - Infants 7 to 27 days (under 28


days) of age.

 Neonatal mortality rate - [annual number


of death of infants 0 to 27 days of life / the
annual number of live births] × 1,000.

 It is declined in India from 2013 but the rate of


decline has been slow and lags behind that of
infant and under-five child mortality rates.

Maternal Mortality Rate

 MMR is the number of maternal deaths during


a given time period per 100,000 live
births during the same time period.

 Maternal mortality in a region is a measure of


the reproductive health of women in the area.

 Many women in reproductive age-span die due


to complications during and following
pregnancy and childbirth or abortion.

 A maternal death occurred almost every two


minutes in 2020.

 Between 2000 and 2020, the maternal


mortality ratio (MMR, number of maternal
deaths per 100 000 live births) dropped by
about 34% worldwide.

 Almost 95% of all maternal deaths occurred in


low and lower middle-income countries in
2020.

 Still birth - According to WHO, a baby who


dies after 28 weeks of pregnancy, but before
or during birth, is classified as a stillbirth.

CHENNAI |SALEM| MADURAI | COIMBATORE DELHI | BANGALORE | THIRUVANANTHAPURAM


www.shankariasacademy.com 105
www.iasparliament.com

TEST – III
1. Consider the following statements about KAVACH 4. Consider the following statements with respect
System QUAD grouping

1. It activates the train’s braking system 1. It was established in 2017 as a Quadrilateral


automatically if the driver fails to control the Group of India, Australia, China and USA.
train as per speed restrictions.
2. It aims to ensure and support a free, open and
2. It uses Radio Frequency Identification prosperous Indo-Pacific region.
(RFID) technology to read information from a
3. Clean Energy Supply Chains Initiative was
distant wireless device without physical contact
launched during the QUAD Leaders’ Summit,
or a line of sight.
2023.
3. It was developed by the Research Design and
Which of the statement(s) given above is/are correct?
Standards Organisation (RDSO) of Defence
Research & Development Organisation a. Only one
(DRDO).
b. Only two
How many of the above statements is/are correct?
c. All three
a. Only one
d. None
b. Only two

c. All three
5. Consider the following statements regarding
d. None Navigation with Indian Constellation (NavIC)

1. It enables users to determine their precise


geographic location anywhere in India and
2. Which of the following Asian river systems originate
5000 kilometres beyond India’s territorial
from the Hindu Kush Himalaya (HKH) Region?
boundary.
River Syr Darya
2. NavIC offers a positional accuracy of 15-20
River Tarim meters compared to 5 meters of Global
Positioning System (GPS).
Yellow river
3. All smartphones and navigational gadgets are
River Yangtze compatible with NavIC.

River Mekong Which of the above statement(s) is/are correct?

Select the correct answer using the codes given below: a. Only one

a. All except 3 b. Only two

b. All except 5 c. All three

c. All except 1 & 2 d. None

d. All five
6. Consider the following statements regarding Artemis
Accords
3. Mekedatu Project is a multipurpose balancing
reservoir project at the confluence of Rivers Cauvery 1. It establishes a set of principles to guide space
and _____? exploration cooperation among member
nations.
a. Kabini
2. It is non-binding on the signatory countries.
b. Arkavathi
3. India is a signatory to the Artemis Accords.
c. Hemavathi
How many of the statement(s) given above is/are
d. Survanavathi correct?

CHENNAI |SALEM| MADURAI | COIMBATORE DELHI | BANGALORE | THIRUVANANTHAPURAM


www.shankariasacademy.com 106
www.iasparliament.com

a. Only one a. Only one

b. Only two b. Only two

c. All three c. All three

d. None d. None

7. Consider the following statements with respect to 10. Consider the following statements with respect to
Gender Social Norms Index (GSNI) City Investments to Innovate, Integrate and Sustain 2.0
(CITIIS 2.0)
1. It quantifies biases against women in 4 key
dimensions like political, educational, 1. It supports projects that promotes circular
economic and physical integrity. economy with focus on integrated waste
management.
2. It is released by the UN Women.
2. It is a part of Swachh Bharat Mission and
Which of the above statement(s) is/are correct? implemented by the Ministry of Housing and
Urban Affairs.
a. 1 only
3. Haora Riverfront Development Project of
b. 2 only Mizoram is undertaken under this program.
c. Both 1 and 2 How many of the above statement(s) is/are correct?
d. Neither 1 nor 2 a. Only one

b. Only two
8. Consider the following with respect to Indian Ocean c. Only three
Dipole (IOD)
d. None
1. It is the difference in sea surface temperature
between the eastern and western sides of the
Indian Ocean.
11. Consider the following statements:
2. Positive Indian Ocean Dipole (IOD) brings
more rainfall to India. Statement-I: The Mission on Advanced and High
Impact Research (MAHIR) aims to facilitate indigenous
Which of the above statement(s) is/are correct? research and development of the emerging technologies
in the power sector.
a. 1 only
Statement-II: MAHIR is jointly launched by the
b. 2 only Ministry of Power and the Ministry of Science &
Technology.
c. Both 1 and 2
Select the correct answer using the codes given below:
d. Neither 1 nor 2
a. Both Statement-I and Statement-II are correct
and Statement-II is the correct explanation for
Statement-I
9. Consider the following statements with respect to
mRNA vaccines b. Both Statement-I and Statement-II are correct
and Statement-II is not the correct explanation for
1. It works by giving a person an inactivated or
Statement-I
weakened version of a virus that triggers an
immune response. c. Statement-I is correct but Statement-II is
incorrect
2. mRNA from vaccines does not enter the nucleus
and does not alter DNA. d. Statement-I is incorrect but Statement-II is
correct
3. COVAXIN is India’s first indigenous mRNA
vaccine.

How many of the above statement(s) is/are correct?

CHENNAI |SALEM| MADURAI | COIMBATORE DELHI | BANGALORE | THIRUVANANTHAPURAM


www.shankariasacademy.com 107
www.iasparliament.com

12. Consider the following statements regarding d. All four


International Covenant on Civil and Political Rights
(ICCPR)

1. The ICCPR is also known as the International 15. Consider the following pairs with respect to the
Bill of Human Rights. Schools of Indian Philosophy:

2. India is a signatory to this treaty but not ratified Philosophies Significance


yet.
1. Vaishesika – Reality
Which of the statement(s) given above is/are correct?
2. Samkhya – Naturalism
a. 1 only
3. Purva Mimansa – Power of Yajnas and
b. 2 only Mantras

c. Both 1 and 2 4. Uttara Mimansa – World is unreal and


Brahman is the only real
d. Neither 1 nor 2
How many of the pair(s) given above is/are correctly
matched?

13. Consider the following statements with respect to a. Only one


Environmental Information, Awareness, Capacity
Building and Livelihood Programme (EIACP) b. Only two

1. EIACP is a network of centres aims to facilitate c. Only three


collection, analysis and dissemination of
information on various facets of environment. d. All of the above

2. It is a project funded by the Ministry of


Environment, Forests and Climate Change
(MoEF&CC). 16. Consider the following statements with respect to
semiconductors.
3. INFOTERRA is the global information network
of the United Nations Environment Programme Statement-I: India produces over 60% of the world's
(UNEP). semiconductors.

How many of the statement(s) given above is/are Statement-II: India Semiconductor Mission (ISM)
correct? was launched by Ministry of Electronics and
Information Technology (MeitY) to build a vibrant
a. Only one semiconductor and display ecosystem.

b. Only two Select the correct answer using the co des given below:

c. All three a. Both Statement-I and Statement-II are correct


and Statement-II is the correct explanation for
d. None Statement-I

b. Both Statement-I and Statement-II are correct


and Statement-II is not the correct explanation for
14. In which of the following, graphene is used? Statement-I

1. Touchscreens c. Statement-I is correct but Statement-II is


incorrect
2. Conductive inks
d. Statement-I is incorrect but Statement-II is
3. Super capacitors correct
4. High performance batteries

Select the correct answer using the codes given below: 17. Consider the following statements:
a. All expect 1 1. A "hawkish pause" denotes a scenario where a
central bank opts to increase interest rates, but
b. All expect 2 still committed to fight inflation.
c. All expect 3

CHENNAI |SALEM| MADURAI | COIMBATORE DELHI | BANGALORE | THIRUVANANTHAPURAM


www.shankariasacademy.com 108
www.iasparliament.com

2. A "dovish pause" refers to a circumstance where 3. Chang’e 5 - China


a central bank halts interest rate hikes due to
robust economic growth potentially leading to How many of the above pair(s) is/are correctly
hyperinflation. matched?

How many of the statements given above are correct? a. Only one

a. 1 only b. Only two

b. 2 only c. All three

c. Both 1 and 2 d. None

d. Neither 1 nor 2
21. Consider the following statements with respect to
Commission of Railway Safety.
18. C+1 Strategy, sometimes seen in the news recently,
is associated with? 1. It works under the administrative control of
Ministry of Railways.
a. An outcome of the China-Central Asia Summit to
boost diplomatic relations 2. It is entrusted with the responsibility of
investigating serious train accidents.
b. A business strategy that encourages companies
to diversify their supply chains and manufacturing Which of the above statement(s) is/are incorrect?
activities outside of China
a. 1 only
c. A strategy that involves capturing carbon and
storing it in places where it's unlikely to be released b. 2 only
back into the atmosphere
c. Both 1 and 2
d. A strategy by China to secure a reliable supply of
critical minerals for a decade d. Neither 1 nor 2

19. Consider the following statements with respect to 22. The International Centre for Integrated Mountain
naming of a species. Development (ICIMOD) aims to make Hindu Kush
Himalaya region greener, more inclusive and climate
1. No two species of animals may have the same resilient. Which of the following countries are regional
species name. member countries of ICIMOD?

2. No one species of animal may have more than 1. Nepal


one valid species name.
2. China
3. A species is always named after the people who
discover them. 3. Pakistan

How many of the above statement(s) is/are correct? 4. Bangladesh

a. Only one 5. Thailand

b. Only two 6. Myanmar

c. All three Select the answer using the code given below:

d. None a. All except 2

b. All except 4

20. Consider the following pairs: c. All except 5

Lunar missions Country d. All except 6

1. Hakuto-R - Japan

2. Danuri - South Korea

CHENNAI |SALEM| MADURAI | COIMBATORE DELHI | BANGALORE | THIRUVANANTHAPURAM


www.shankariasacademy.com 109
www.iasparliament.com

23. Consider the following statements with respect to humans. Aspartame is used in how many of the
United Nations Educational, Scientific and Cultural following substances?
Organization (UNESCO).
1. Diet drinks
Statement-I: Palestine is a member of UNESCO.
2. Chewing Gum
Statement-II: States that are not members of United
Nations cannot be admitted to UNESCO. 3. Dairy products

Select the correct answer using the codes given below: 4. Toothpaste

a. Both Statement-I and Statement-II are correct 5. Cough drops


and Statement-II is the correct explanation for
Statement-I Select the answer using the code given below:

b. Both Statement-I and Statement-II are correct a. Only 2


and Statement-II is not the correct explanation for
Statement-I b. Only 3

c. Statement-I is correct but Statement-II is c. Only 4


incorrect
d. All 5
d. Statement-I is incorrect but Statement-II is
correct
27. Which of the following releases the Global Slavery
Index?
24. In light of recent Kosovo-Serbia conflict, which one
a. World Bank
of the following state borders both Kosovo and Serbia?
b. Walk Free Foundation
a. Albania
c. Amnesty International
b. Romania
d. Human Rights Foundation
c. Bulgaria

d. Montenegro
28. Which of the following statements regarding
underwater vehicles is/are incorrect?
25. Helmand River is a vital source of water in which of
1. A Submersible is a watercraft capable of
the following countries?
independent operation underwater.
1. Iran
2. A submarine can function as an autonomous
2. Afghanistan craft and does not need a mother ship to launch
and recover it.
3. Pakistan
Select the answer using the code given below:
4. Iraq
a. 1 only
Select the answer using the code given below:
b. 2 only
a. 1 and 2 only
c. Both 1 and 2
b. 2 and 3 only
d. Neither 1 nor 2
c. 1 and 4 only

d. 1, 2 and 3 only
29. NANDI Portal, sometimes seen in the news, refers
to

26. The International Agency for Research on Cancer a. A portal to expedite regulatory process for
(IARC) of the World Health Organisation (WHO) has approval of animal drugs and vaccines.
classified aspartame as possibly carcinogenic to
b. A portal for trading animal and animal based
products across the country.

CHENNAI |SALEM| MADURAI | COIMBATORE DELHI | BANGALORE | THIRUVANANTHAPURAM


www.shankariasacademy.com 110
www.iasparliament.com

c. A portal that monitors the implementation of b. Only two


Centrally Sponsored Schemes of Ministry of
Fisheries, Animal Husbandry and Dairying. c. All three

d. A portal that connect breeders and farmers d. None


regarding availability of bovine germplasm.

33. Consider the following statements with respect to


30. Consider the following statements with respect to Global Partnership on Artificial Intelligence (GPAI).
Global Forest Watch (GFW).
Statement-I: It aims to bridge the gap between theory
1. It is an open source web application to monitor and practice on AI by supporting cutting-edge research
global forests in near real time. and applied activities on AI-related priorities.

2. It works under the aegis of the United Nations Statement-II: Bletchley declaration is the major
Environmental Programme (UNEP). outcome of GPAI summit 2023.

3. Brazil and the Democratic Republic of Congo Select the correct answer using the codes given below:
are the 2 countries with the most tropical forest
cover in the world. a. Both Statement-I and Statement-II are correct
and Statement-II is the correct explanation for
How many of the statements given above are correct? Statement-I

a. Only one b. Both Statement-I and Statement-II are correct


and Statement-II is not the correct explanation for
b. Only two Statement-I

c. All Three c. Statement-I is correct but Statement-II is


incorrect
d. None
d. Statement-I is incorrect but Statement-II is
correct

31. Cholederm, often seen in the news, is associated with


which of the following?
34. Which of the following statement is correct
a. An extracellular matrix derived from the bark of regarding biochar?
the plant, used in medical surgeries.
1. Biochar is a carbonaceous material obtained by
b. A wound healing material used to heal different thermal degradation of biomasses in oxygen
types of skin wounds. rich conditions.
c. A cosmetic material, which was classified as 2. It is a carbon positive process which finds its
carcinogenic by World Health Organisation. major application in carbon sequestration.
d. A new algae that was discovered to be in Which of the above statement(s) is/are correct?
symbiotic association with fungus.
a. 1 only

b. 2 only
32. Consider the following statements about Nyaya
Vikas Portal c. Both 1 and 2

1. It is an initiative of Ministry of Law and Justice. d. Neither 1 nor 2

2. The primary responsibility of the portal is to


check the status of legal case.
35. Consider the following statements with respect to
3. Technical assistance is provided by Centre for UN Peace Keeping.
Development of Advanced Computing (C-
DAC). 1. In peacekeeping missions, women are barred in
uniformed military, police, and corrections
How many of the above statement(s) is/are correct? personnel.

a. Only one

CHENNAI |SALEM| MADURAI | COIMBATORE DELHI | BANGALORE | THIRUVANANTHAPURAM


www.shankariasacademy.com 111
www.iasparliament.com

2. Peacekeeping Operation was deployed only 1. Amroha Dholak


once in India to supervise the ceasefire along
the India-Pakistan border. 2. Kalpi Handmade Paper

Which of the above statement(s) is/are correct? 3. Sambhal Horn Craft

a. 1 only 4. Mainpuri Tarkash

b. 2 only 5. Baghpat Home Furnishings

c. Both 1 and 2 Select the correct answer using the code given below:

d. Neither 1 nor 2 a. All expect 4

b. All expect 3

36. Consider the following statements with respect to c. All expect 2


SANKALP Programme
d. All of the above
1. It is a certification scheme for aquaculture
products developed by the Marine Products
Exports Development Authority (MPEDA).
39. Consider the following statements regarding North
2. The programme is supported by the Asian Atlantic Treaty Organization (NATO).
Development Bank.
1. It is a political and military alliance of 31
3. It is being implemented by the Ministry of members with Finland being the latest
Commerce And Industry. member.

How many of the statements given above are correct? 2. The membership is open to any non-European
States.
a. Only one
Which of the above statement(s) is/are correct?
b. Only two
a. 1 only
c. All Three
b. 2 only
d. None
c. Both 1 and 2

d. Neither 1 nor 2
37. Consider the following statements with respect to
Yakshagana

1. It is a traditional dance-drama performance 40. Consider the following pairs


similar to Terukkuttu of Tamil Nadu.
Sites in news Countries
2. It is performed in Karnataka and the Kasaragod
district of Kerala. 1. Mayon Volcano – Philippines

3. It has been inscribed in the UNESCO's list of 2. Kakhovka Dam – Russia


intangible cultural heritage.
3. Zaporizhzhia Power Plant –Ukraine
How many of the statements given above are correct?
How many of the pair(s) given above is/are correctly
a. Only one matched?

b. Only two a. Only one

c. All three b. Only two

d. None c. All three

d. None of the above

38. Which of the following Geographical Indication (GI)


products are associated with Uttar Pradesh?

CHENNAI |SALEM| MADURAI | COIMBATORE DELHI | BANGALORE | THIRUVANANTHAPURAM


www.shankariasacademy.com 112
www.iasparliament.com

41. Consider the following statements with respect to 4. Lakdawala - Calorie based consumption
Goods and Services Tax (GST) expenditure

1. GST is a destination-based tax, levied at each How many of the above pairs are correctly matched?
stage of the supply chain, from the
manufacturer to the consumer. a. Only one

2. Import of goods or services would be treated as b. Only two


inter-state supplies and would be subject to
Integrated GST (IGST). c. Only three

3. Every decision of the GST Council is taken at its d. All four


meeting by a majority of not less than 2/3rd of
the weighted votes of the Members present and
voting.
44. Consider the following statements regarding
Which of the above statement(s) is/are correct? National Multidimensional Poverty Index

a. Only one 1. It is published by Ministry of Statistics and


Programme Implementation.
b. Only two
2. For 2023 index, data inputs are taken from
c. All three National Family Health Survey (NFHS-5).

d. None 3. India’s national MPI has 4 equally weighted


dimensions Health, Education, Income and
Standard of living.

42. Consider the following statements with respect to How many of the above statements is/are correct?
Heat Waves
a. Only one
1. A heat wave occurs when the maximum
temperature of a station reaches at least 50 b. Only two
degrees Celsius or more for plains and at least
40 degrees Celsius or more for hilly regions. c. All three

2. A marine heat wave occurs when the surface d. None


temperature of a particular region of the sea
rises to 3 or 4 degrees Celsius above the average
temperature for at least five days. 45. With reference to Adjournment Motion, consider
the following statements:
3. Wet bulb is a condition when high temperatures
combine with high humidity. 1. It involves an element of censure against the
government and Rajya Sabha is not permitted
Which of the statements given above are correct?
to use the motion.
a. 1 and 2 only
2. In India, it was established through
b. 1 and 3 only Government of India Act, 1935.

c. 2 and 3 only 3. It needs the support of 10 members to be


admitted in the house.
d. 1, 2 and 3
How many of the statement(s) given above is/are
correct?

43. Consider the following pairs a. Only one

Committee Recommendations b. Only two

1. Tendulkar – Mixed Reference Period c. All three

2. Rangarajan – Uniform poverty line basket d. None

3. Alagh – State specific poverty lines

CHENNAI |SALEM| MADURAI | COIMBATORE DELHI | BANGALORE | THIRUVANANTHAPURAM


www.shankariasacademy.com 113
www.iasparliament.com

46. Consider the following statements regarding Project a. Both Statement-I and Statement-II are correct
Cheetah and Statement-II is the correct explanation for
Statement-I
1. The project involves the translocation of Asiatic
cheetahs from South Africa and Namibia to b. Both Statement-I and Statement-II are correct
Kuno National Park in Madhya Pradesh. and Statement-II is not the correct explanation for
Statement-I
2. This is the first intercontinental reintroduction
of a wild, large carnivore species. c. Statement-I is correct but Statement-II is
incorrect
3. National Tiger Conservation Authority (NTCA)
is the apex body entrusted with the d. Statement-I is incorrect but Statement-II is
implementation of Project Cheetah. correct

Which of the statement(s) given above is/are correct?

a. 1 and 2 only 49. Consider the following statements with respect to


Forest (Conservation) Amendment Act, 2023
b. 1 and 3 only
1. According to the act, silvicultural operations,
c. 2 and 3 only safaris, and eco-tourism facilities can be
allowed in forests without the prior approval of
d. 1, 2 and 3 the Central government.

2. Forest lands situated within 100 km along the


international borders used for construction of
47. Consider the following statements with respect to strategic linear project are exempted from the
No confidence Motion provisions of this Act.
1. The Constitution of India does not mention Which of the statement(s) given above is/are correct?
about either a Confidence or a No Confidence
Motion. a. 1 only
2. Article 75(3) of Indian Constitution specifies b. 2 only
that the Council of Ministers are collectively
responsible to the Parliament. c. Both 1 and 2

3. More than 50 no-confidence motions have been d. Neither 1 nor 2


moved in the Lok Sabha since India’s
independence.

How many of the statement(s) given above is/are 50. Consider the following with respect to National
correct? Institutional Ranking Framework (NIRF)

a. Only one 1. It is published by the Union Ministry of


Education.
b. Only two
2. It is released annually since its establishment in
c. All three 2016.

d. None 3. It evaluates the quality of educational programs


offered by higher education institutions in
colleges and universities in India.
48. Consider the following statements: How many of the above statements are correct?
Statement-I: In India, the post of Leader of a. Only one
Opposition in Lok Sabha is vacant since 2019.
b. Only two
Statement-II: According to Indian Constitution, the
leader of the largest opposition party which is having at c. All three
least 1/1oth seats in Lok Sabha is recognised as Leader
of Opposition. d. None

Select the correct answer using the codes given below:

CHENNAI |SALEM| MADURAI | COIMBATORE DELHI | BANGALORE | THIRUVANANTHAPURAM


www.shankariasacademy.com 114
www.iasparliament.com

51. Consider the following statements with respect to b. 2 only


Permanent Court of Arbitration (PCA)
c. Both 1 and 2
1. It was established by the Convention for the
Pacific Settlement of International Disputes, d. Neither 1 nor 2
concluded at The Hague in 1899.

2. The rulings of PCA are binding but the tribunal


has no powers for enforcement. 54. Silver Cockscomb, sometimes seen in the news, is
related to?
3. India and USA are not members of this
Arbitration Court. a. A short-lived weed renowned for its nutritional
value.
How many of the statement(s) given above is/are
correct? b. A recently discovered mineral possibly serving as
a substitute for silver.
a. Only one
c. One of the world’s 100 most invasive alien
b. Only two species considered by IUCN.

c. All three d. A newly identified species of Silverline butterfly


discovered in the Western Ghats.
d. None

55. Ambergris, sometimes seen in the news, refers to?


52. NATO Plus is a security arrangement that brings
together NATO and five aligned nations. Which of the a. Nearest nebula to the Milky Way galaxy
following countries are part of it?
b. Waxy substance in the digestive system of sperm
1. Australia whales

2. India c. Ancient fossils of ammonites celebrated by


Hindus and Buddhists
3. Japan
d. Fossilized tree resin that is valued for its grey
4. Israel color and natural beauty

5. Sweden

Select the correct answer using the codes given below: 56. Consider the following statements with respect to
Protection of Plant Varieties and Farmers Rights Act,
a. Only two 2001.

b. Only three 1. The Act aims to prioritise the farmers’ rights


over breeders’ rights.
c. Only four
2. The farmers are entitled to sell branded seed of
d. All five a variety protected under this Act.

Which of the above statement(s) is/are correct?


53. Consider the following statements about Zero FIR a. 1 only
1. It is an FIR that can be registered by any police b. 2 only
station, irrespective of jurisdiction, when it
receives a complaint regarding a cognisable c. Both 1 and 2
offence.
d. Neither 1 nor 2
2. The provision of Zero FIR came up after the
recommendation of the Justice Verma
Committee, constituted after the 2012
Nirbhaya case. 57. With reference to Indo-US foundational defence
agreements, consider the following pairs:
Which of the statement(s) given above is/are correct?

a. 1 only

CHENNAI |SALEM| MADURAI | COIMBATORE DELHI | BANGALORE | THIRUVANANTHAPURAM


www.shankariasacademy.com 115
www.iasparliament.com

Agreements Purposes 60. How many of the following countries are part of the
Comprehensive and Progressive Agreement for Trans-
1. GSOMIA - Geo-spatial cooperation Pacific Partnership (CPTPP)?

2. LEMOA - Logistics Exchange 1. USA

3. COMCASA - Communications interoperability 2. Russia

4. BECA - Intelligence sharing 3. United Kingdom

How many of the above pairs are correctly matched? 4. India

a. Only one Select the correct answer using the codes given below:

b. Only two a. Only 1

c. All three b. Only 2

d. None c. Only 3

d. All 4

58. Which of the following characterises the Indian


Ocean Geoid Low (IOGL)?
61. Consider the following statements with respect to
1. Weaker Gravitational Pull Ethanol Blending in India

2. Lower Mass 1. In India, Department of Food and Public


distribution is the nodal agency for promotion
3. Dip in sea level of fuel grade ethanol producing distilleries.
Select the correct answer using the codes given below: 2. India has set a target of 10% ethanol blending
by 2022 and 20% blending by 2030.
a. 1 and 2
Which of the above statement(s) is/are correct?
b. 1 and 3
a. 1 only
c. 2 and 3
b. 2 only
d. 1, 2 and 3
c. Both 1 and 2

d. Neither 1 nor 2
59. Consider the following pairs

Terms Regions
62. Consider the following statements:
1. Khazan lands – Goa
1. Bt Cotton is the only GM crop that is allowed to
2. Pakkasuran Mala – Tamil Nadu be cultivated.
3. Kaas Plateau – Meghalaya 2. GM Mustard is the first GM food crop that India
has permitted for commercial release.
How many of the pair(s) given above is/are correctly
matched? Which of the statement(s) given above is/are correct?
a. Only one a. 1 only
b. Only two b. 2 only
c. All three c. Both 1 and 2
d. None d. Neither 1 nor 2

CHENNAI |SALEM| MADURAI | COIMBATORE DELHI | BANGALORE | THIRUVANANTHAPURAM


www.shankariasacademy.com 116
www.iasparliament.com

63. The “Clarion-Clipperton Fracture Zone” is the most a. Phanerozoic Era – Cenozoic Eon – Quaternary
focused area of exploration for Polymetallic nodules. Epoch – Holocene Period
The zone is located in?
b. Proterozoic Eon – Mesozoic Era – Neogene
a. Arctic Ocean Period – Pleistocene Epoch

b. Indian Ocean c. Proterozoic Era – Mesozoic Eon – Neogene


Epoch – Pleistocene Period
c. Pacific Ocean
d. Phanerozoic Eon – Cenozoic Era – Quaternary
d. Atlantic Ocean Period – Holocene Epoch

64. Consider the following statements: 67. Consider the following pairs:
Statement-I: Gambusia is an insectivorous fish that is Terms Countries
found mostly in fresh and brackish water and has a high
breeding capacity. 1. Crawford Lake – Russia

Statement-II: Recently, World Health Organisation 2. Batagaika Crater – South Africa


has recommended Gambusia as a mosquito control
agent. 3. Parkachik Glacier – China

Select the correct answer using the codes given below: How many of the pair(s) given above is/are correctly
matched?
a. Both Statement-I and Statement-II are correct
and Statement-II is the correct explanation for a. Only one
Statement-I
b. Only two
b. Both Statement-I and Statement-II are correct
and Statement-II is not the correct explanation for c. All three
Statement-I
d. None
c. Statement-I is correct but Statement-II is
incorrect

d. Statement-I is incorrect but Statement-II is 68. Arrange the following straits from South to north.
correct
1. Bass Strait

2. Dover Strait
65. Consider the following statements regarding Light
3. Sunda Strait
Fidelity (Li-Fi)
4. Bab-el-Mandeb
1. Li-Fi is faster than Wi-Fi.
Select the correct answer using the code given below.
2. Li-Fi covers a broader coverage area than Wi-
Fi. a. 2-3-4-1
3. Li-Fi will deliver data through radio b. 1-4-3-2
frequencies.
c. 2-4-3-1
Which of the above statement(s) is/are incorrect?
d. 1-3-4-2
a. 1 only

b. 1 and 2 only
69. With reference to National Adaptation Fund for
c. 1 and 3 only Climate Change (NAFCC), consider the following
statements:
d. 2 and 3 only
1. It is a Central Sector Scheme that aims to
support adaptation activities in the vulnerable
66. We are currently living in which of the following States and UTs in India
geological time scale?

CHENNAI |SALEM| MADURAI | COIMBATORE DELHI | BANGALORE | THIRUVANANTHAPURAM


www.shankariasacademy.com 117
www.iasparliament.com

2. In India, NABARD is the National 72. Consider the following statements with respect to
Implementing Entity (NIE) for the fund. Paris Club

3. Agriculture, Animal husbandry and Tourism 1. It is an informal group that assists debtor
sectors are eligible for funding under NAFCC. nations in addressing their debt issues.

How many of the statements given above are correct? 2. India is one of the 22 members of Paris Club.

a. Only one 3. The Paris Club only deals with debt between
governments and does not handle any private
b. Only two or individual loan repayment.

c. All three Which of the above statement(s) is/are correct?


d. None a. Only one

b. Only two

70. Which of the following are the indicators used in the c. All three
UN’s Multidimensional Poverty Index?
d. None
1. Poverty

2. Housing
73. The terms ‘Yago’ and ‘Zoe’, sometimes seen in the
3. Electricity news, is associated with?

4. Sanitation a. Local winds

5. Cooking fuel b. Active volcanoes


Select the correct answer using the codes given below: c. Heatwaves
a. Only two d. Littoral Islands

b. Only three

c. Only four 74. Which of the following bacteria are classified as


ESKAPE pathogens, known for their high resistance to
d. All five antibiotics?

1. E. coli bacteria
71. Consider the following statements with respect to 2. Pseudomonas aeruginosa
the World Drug Report, 2023
3. Acinetobacter baumannii
1. It was released by the UN Office on Drugs and
Crime. 4. Staphylococcus aureus
2. Opioids are identified as the most widely 5. Salmonella
consumed drug globally.
Select the correct answer using the codes given below:
3. India ranks highest in terms of opioid usage
prevalence worldwide. a. Only two

How many of the statement(s) given above is/are b. Only three


correct?
c. Only four
a. Only one
d. All five
b. Only two

c. All three
75. The Performance Grading Index is released in India
d. None by?

a. Ministry of Coal

CHENNAI |SALEM| MADURAI | COIMBATORE DELHI | BANGALORE | THIRUVANANTHAPURAM


www.shankariasacademy.com 118
www.iasparliament.com

b. Ministry of Education 4. Fugitive Economic Offenders Act (FEOA), 2018

c. Ministry of Commerce and Industry In India, the Enforcement directorate (ED) currently
draws its statutory powers from how many of the above
d. Ministry of Environment, Forest and Climate acts?
Change
a. Only one

b. Only two
76. Which of the following sectors are part of the Index
of Eight Core Industries (ICI)? c. Only three

1. Coal d. All four

2. Automobile

3. Electricity 79. Horizon 2047, sometimes seen in the news recently,


refers to?
4. Textiles
a. It is NASA's 1st venture in the New Frontiers
5. Fertilizers programme that aims to explore distant celestial
objects.
Select the correct code.
b. It is a centenary celebration of free trade
a. All except 1 and 3 agreement between 11 Pacific Rim countries.
b. All except 2 and 4 c. It refers to Rupee-Rouble trade arrangement, an
alternative payment mechanism to settle dues in
c. All except 2 and 5 rupees instead of dollars or euros.
d. All except 3 and 5 d. It is relevant to recognition of long-standing
diplomatic relationship between Indian and France.

77. Consider the following statements with respect to


Anti-defection in India 80. Which of the following countries are members of
Shanghai Cooperation Organization (SCO)?
1. It was added by the 52nd Constitutional
Amendment Act, 1985. 1. Afghanistan
2. The Speaker is empowered to make rules to give 2. Tajikistan
effect to the provisions of 10th schedule in
Parliament. 3. Iran
3. Provisions of the Anti-defection law does not 4. Turkmenistan
apply to the Presidential Elections in India.
5. Uzbekistan
How many of the above statements are correct?
Select the correct answer using the codes given below:
a. Only one
a. Only two
b. Only two
b. Only three
c. All three
c. Only four
d. None
d. All five

78. Consider the following:


81. Which of the following countries are members of
1. Foreign Exchange Regulation Act (FERA), 1973 BRICS?
2. Foreign Exchange Management Act (FEMA), 1. Iran
1999
2. Egypt
3. Prevention of Money Laundering Act (PMLA),
2002 3. Ethiopia

CHENNAI |SALEM| MADURAI | COIMBATORE DELHI | BANGALORE | THIRUVANANTHAPURAM


www.shankariasacademy.com 119
www.iasparliament.com

4. Argentina d. Statement-I is incorrect but Statement-II is


correct
5. United Arab Emirates

Select the answer using the codes given below:


84. Consider the following pairs:
a. 1, 4 and 5 only
Committee Role
b. 1, 2, 3 and 5 only
1. Rohini commission – Subcategorization of OBCs
c. 2, 3, 4 and 5 only
2. Amitabh Kant committee – Manipur Violence
d. 1, 2, 3, 4 and 5
3. Gita Mittal Committee – Real Estate Projects

How many of the above pairs are correct?


82. The Global Environment Facility (GEF) serves as a
financial mechanism to which of the following a. Only one
conventions?
b. Only two
1. Minamata Convention on Mercury
c. All three
2. Convention on Biological Diversity (CBD)
d. None
3. UN Convention to Combat Desertification
(UNCCD)

4. Stockholm Convention on Persistent Organic 85. Consider the following statements:


Pollutants (POPs)
1. As per Indian Constitution, the state
5. United Nations Framework Convention on governments, as the owners of minerals can
Climate Change (UNFCCC) grant mineral concessions for exploration and
mining.
Select the correct answer using the codes given below:
2. 100% FDI is allowed in mining sector.
a. All except 1
3. Revenue generated from auctions of mines will
b. All except 1 and 4 go to respective state governments.

c. All except 1, 3 and 4 How many of the above statements is/are correct?

d. All five a. Only one

b. Only two

83. Consider the following statements: c. All three

Statement-I: Finance Bill-I can be introduced only in d. None


Lok Sabha and only after prior recommendations of the
President of India.

Statement-II: Finance Bill-I deals with general legal 86. Consider the following statements with respect to
issues & provisions related to Article 110 of Constitution Katchatheevu Island
of India.
1. It is of volcanic origin.
Select the correct answer using the codes given below:
2. It is an uninhabited off-shore island in the Palk
a. Both Statement-I and Statement-II are correct Strait.
and Statement-II is the correct explanation for
Statement-I 3. 1976 Agreement marked the international
maritime boundary of India and Sri Lanka.
b. Both Statement-I and Statement-II are correct
and Statement-II is not the correct explanation for How many of the statement(s) given above
Statement-I is/are incorrect?

c. Statement-I is correct but Statement-II is a. Only one


incorrect
b. Only two

CHENNAI |SALEM| MADURAI | COIMBATORE DELHI | BANGALORE | THIRUVANANTHAPURAM


www.shankariasacademy.com 120
www.iasparliament.com

c. All three c. All three

d. None d. None

87. Consider the following statements: 90. Consider the following statements:

1. In nuclear fusion reaction, alpha, beta and 1. Article 2 relates to the formation of or changes
gamma radiations were emitted. in the existing states of the Union of India.

2. Atomic bomb is based on the principle of 2. Article 3 of Indian Constitution grants


nuclear fission reaction whereas hydrogen Parliament, the power to admit new states into
bomb is based on nuclear fusion reaction. the Union of India and the power to establish
new states.
3. In nuclear fission reaction, atleast one thermal
neutron from outside is compulsory. 3. Amendments to Article 2 and Article 3 are not
considered as amendments under Article 368
How many of the above statements is/are correct? and thus can be passed by a simple majority.

a. Only one How many of the statement(s) given above


is/are incorrect?
b. Only two
a. Only one
c. All three
b. Only two
d. None
c. All three

d. None
88. Consider the following statements with respect to
3D printing

1. It works based on the subtractive 91. Which of the following is incorrect with respect to
manufacturing process. cfDNA?

2. Fracking is one of the methods of 3D printing. a. It can be generated by the death of body tissue.
Which of the statement(s) given above is/are correct? b. It refers to large DNA fragments found inside the
human hair.
a. 1 only
c. It aids in assessing the risk of chromosomal
b. 2 only abnormalities in the foetus.
c. Both 1 and 2 d. The presence of cfDNA is an indication of
various processes that include physical injury,
d. Neither 1 nor 2 inflammation, and cancer.

89. Consider the following with respect to International 92. Consider the following statements with respect to
Atomic Energy Agency (IAEA) Convention on Cluster Munition (CCM).
1. It was created in response to the 1973-74 oil 1. It bans the use, production, stockpiling, and
crisis when an oil embargo by major producers transfer of cluster bombs.
pushed prices to historic levels.
2. The member countries are legally bound to
2. It is the only organization within the UN system provide support and rehabilitation to the
with expertise in nuclear technologies. cluster bomb victims in their jurisdiction.
3. India is a founding member of IAEA. 3. India is a signatory member to CCM.
How many of the above statements is/are correct? Which of the statements given above are correct?
a. Only one a. 1 and 2 only
b. Only two b. 1 and 3 only

CHENNAI |SALEM| MADURAI | COIMBATORE DELHI | BANGALORE | THIRUVANANTHAPURAM


www.shankariasacademy.com 121
www.iasparliament.com

c. 2 and 3 only The above-mentioned features correspond to which one


of the following?
d. 1, 2 and 3
a. Ek Bharat Shreshtha Bharat

b. National Mission for Manuscripts


93. Consider the following pairs:
c. Heritage City Development and Augmentation
CAPF Mandate to guard Yojana (HRIDAY)

1. Assam Rifles – Indo-Nepal and Indo-Bhutan d. Pilgrimage Rejuvenation and Spiritual


borders Augmentation Drive (PRASAD)

2. Border Security Force – Indo-Myanmar border

3. Indo Tibetan Border Police – Indo-China border 96. Which of the following best describes the
term “Mrityu Bahini”?
4. Sashastra Seema Bal – Indo-Pakistan and Indo-
Bangladesh borders a. A digital database to store and document
endangered languages in India.
How many of the pair(s) given above
is/are incorrectly matched? b. A tribal book written to preserve the culture and
heritage of the community.
a. Only one
c. A sub-unit of Indian national Army in India to
b. Only two carry out revolutionary attacks against British.
c. Only three d. A suicide squad prepared to sacrifice their lives
while attempting to hoist the national flag atop
d. All four police stations.

94. Consider the following pairs 97. Consider the following sites:
Places in News Reasons 1. Sivagalai
1. Maui Island – Forest fire 2. Adichanallur
2. Novorossiysk Port – Oil leak 3. Kodumanal
3. Kuril Islands – China-Japanese conflict 4. Porpanaikottai
4. Sulina Channel – Ukraine’s trade route 5. Tirunavaya
How many of the above pair(s) is/are correctly How many of the above sites belongs to Sangam age?
matched?
a. Only two
a. Only one
b. Only three
b. Only two
c. Only four
c. Only three
d. All five
d. All four

98. Consider the following statements with respect to


95. Consider the following statements: Interpol Global Academy Network (IGAN)
1. It is an autonomous body, working under the 1. It aims to support Interpol in leading a global
ambit of Ministry of Culture. approach to law enforcement training.
2. Its motto is ‘Conserving the past for the future’. 2. Enforcement Directorate (ED) of India has
recently joined it as a member.
3. It has emerged as the most popular and
effective among all the heritage conservation Which of the above statement(s) is/are correct?
initiatives in the country.

CHENNAI |SALEM| MADURAI | COIMBATORE DELHI | BANGALORE | THIRUVANANTHAPURAM


www.shankariasacademy.com 122
www.iasparliament.com

a. 1 only

b. 2 only

c. Both 1 and 2

d. Neither 1 nor 2

99. Consider the following statements with respect to


PM Vishwakarma Scheme

1. It is a centrally sponsored scheme that aims to


provide end-to-end support to artisans and
craftspeople who work with their hands and
tools.

2. Ministry of Skill Development and


Entrepreneurship is the nodal ministry of this
scheme.

3. The artisans and crafts persons will be provided


with collateral free enterprise development
loans of up to Rs. 5 lakh.

How many of the above statements is/are correct?

a. Only one

b. Only two

c. All three

d. None

100. Consider the following statements:

1. This scheme is aimed at saving the soil and


promote sustainable, balanced use of fertilizers.

2. It has no separate budget, but a 50% subsidy


savings will be provided to States/UTs.

3. It has been initiated by the Union Ministry of


Chemicals and Fertilizers.

Which one of the following initiatives of Government of


India corresponds to the above given features?

a. PM PRANAM Scheme

b. Paramparagat Krishi Vikas Yojana

c. Integrated Fertiliser Management Scheme

d. National Programme for Organic Production


(NPOP)

CHENNAI |SALEM| MADURAI | COIMBATORE DELHI | BANGALORE | THIRUVANANTHAPURAM


www.shankariasacademy.com 123
www.iasparliament.com

Answer Key – Test - III


1 2 3 4 5 6 7 8 9 10

B D B B D C A C A B

11 12 13 14 15 16 17 18 19 20

C D C D B D D B B C

21 22 23 24 25 26 27 28 29 30

A C C D A D B A A B

31 32 33 34 35 36 37 38 39 40

B A C D B D B D B B

41 42 43 44 45 46 47 48 49 50

B C B A A C A C C C

51 52 53 54 55 56 57 58 59 60

B B C A B D B D B A

61 62 63 64 65 66 67 68 69 70

A C C C A D D D C C

71 72 73 74 75 76 77 78 79 80

A B C B B B B C D B

81 82 83 84 85 86 87 88 89 90

B D A A C D B D B B

91 92 93 94 95 96 97 98 99 100

B A C B B D C A D A

EXPLANATION
1. b

KAVACH – Traffic Collision Avoidance Systems


(TCAS)

The triple-train collision in Odisha’s Balasore, one of


the deadliest in India, raises questions about safety in
rail travel.

 KAVACH is an indigenously developed


Automatic Train Protection (ATP) system.

 It is a cab signalling train control system


designed to prevent collision and overspeeding.

 Developed by – Research Design and


Standards Organisation (RDSO) under
Indian Railways in collaboration with 3
Indian industry.

 It costs Rs. 50 lakh per kilometre.

CHENNAI |SALEM| MADURAI | COIMBATORE DELHI | BANGALORE | THIRUVANANTHAPURAM


www.shankariasacademy.com 124
www.iasparliament.com

 Objective – To provide protection by


preventing trains to pass the signal at Red and Hindu Kush Himalaya (HKH) region - Water
avoid collision. tower of Asia

 It activates the train’s braking system  They are the source of major Asian river system,
automatically if the driver fails to control the provides clean water for a third of the world’s
train as per speed restrictions. population.

 There are 3 components along the railway  At least 12 rivers fan out across the Asian continent
route. from it.

o RFID technology in tracks – It uses radio  The Indus, Ganga and Brahmaputra towards the
waves to automatically identify and read Arabian Sea and Bay of Bengal
information from a distant wireless device
without physical contact or a line of sight.  Syr Darya and Amu Darya towards the now-
dead Aral Sea
o Locomotive components – The driver’s
cabin is provided with RFID readers, computer,  The Tarim toward the Taklamakan
and brake interface equipment.
 The Yellow River towards the Gulf of Bohai
o Radio infrastructure – These are the towers
and modems which are installed at railway  The Yangtze towards the East China Sea
stations.
 The Mekong towards the South China Sea
 Significance – It is a state-of-the-art
electronic system with Safety Integrity  The Chindwin, Salween and Irrawaddy towards
Level-4 (SIL-4) standards. the Andaman Sea

 SIL Standards – Safety Integrity Level (SIL)


is a measure of the reliability of the safety  Importance – It comprises 4 biodiversity
function performing to specification. hotspots out of the world’s 36 global
biodiversity hotspots, 2 of the global 200 eco
 SIL 4 the most dependable and SIL 1 the least. regions, 575 Protected Areas, 335 important
bird areas.
2. d
 Still 85% of mountain communities remain
Hindu Kush Himalaya (HKH) region dependent on this biodiversity, for food, water,
flood control and cultural identity.
 Geography – It stretches 3,500 kms and
spans 8 countries.  High Mountain Asia – It comprises of
mountain ranges such as the Tian Shan,
 Range Countries – Afghanistan, Pakistan, Kunlun Shan, Pamir, Karakoram, Hindu Kush,
India, Nepal, Bhutan, China, Bangladesh and the Tibetan Plateau and the Himalayas.
Myanmar.
 Mountains cover 22% of Earth’s land surface
 It harbours the highest mountain ranges in the but hold 50% of the world’s global biodiversity
world. hotspots.
 Third pole – It is called so as it contains the 3. b
largest volume of ice on earth outside of the
polar areas (Arctic and Antarctica). Mekedatu project

The governments of Karnataka and Tamil Nadu


clashed over Mekedatu project, which supplies
drinking water to Bengaluru.

 It is a multipurpose balancing reservoir project


at the confluence of the River Cauvery and
its tributary River Arkavathi.

 Located in – Ramanagaram district (about


100 km south of Bengaluru) close to where the
Cauvery enters Tamil Nadu.

CHENNAI |SALEM| MADURAI | COIMBATORE DELHI | BANGALORE | THIRUVANANTHAPURAM


www.shankariasacademy.com 125
www.iasparliament.com

 The dam has a proposed capacity of 48 TMC  Formed in – 2017, as a Quadrilateral Group of
(thousand million cubic) feet and an estimated 4 countries.
cost of Rs 6,000 crore.
 Members – India, Australia, Japan and
 Aim – To supply drinking water to the U.S.
Bengaluru and replenish the regional
groundwater table.  Proposed by – The former Japan’s Prime
Minister, Shinzo Abe in 2007 to check China’s
 It will be larger than the Krishnaraja Sagar growing influence and assertiveness in the
project on the Cauvery. Indo-Pacific region.

 River Cauvery – It originates at Talakaveri in  Objectives – To strengthen the defence and


Coorg, often called the Dakshina Ganga (the security cooperation amongst the 4 countries.
Ganges of the South) and considered one of the
sacred rivers of India.  To ensure and support a “free, open and
prosperous” Indo-Pacific region.
 Tributaries – Rivers Kakkabe, the Kadanur
and the Kummahole in Coorg, Karnataka.  2023 Summit – It committed to the common
vision of Free, Open, and Inclusive Indo-Pacific
 Rivers Harangi, the Hemavathi, the Shimsha (FOIP) and an international order based on the
and the Arkavathi in Mysore, Karnataka (joins rule of law.
to the left).
 Announced 6 initiatives to strengthen the
 Lakshmanathirtha, the Kabbani (origin in resilience and prosperity of the Indo-Pacific
Kerala), and the Survanavathi in Mysore, region.
Karnataka (joins to the right).
o Clean Energy Supply Chains
 Rivers Bhavani, the Noyil, and the Amravathi in Initiative
Tamil Nadu.
o Quad Principles of Clean Energy
Supply Chains

o Quad Infrastructure Fellowships


Programme

o Partnership for Cable Connectivity and


Resilience

o QUAD support for a small-scale ORAN


deployment in Palau, the first in the
Pacific region.

o QUAD Investors’ Network (QUIN)

5. d

NavIC satellite

ISRO had launched the first of the second-generation


satellites for its navigation constellation successfully.

 NavIC - It is a satellite based navigational


system, developed by the Indian Space
Research Organization (ISRO).

 It is erstwhile known as the Indian Regional


Navigation Satellite System (IRNSS).
4. b  NavIC enables users to determine their precise
geographic location and track their movements
QUAD
anywhere in India and 1500 kilometer
Prime Minister of India participated in the 3rd in- beyond India’s territorial boundary.
person Quad Leaders’ Summit in Hiroshima, Japan
 Constellation of 7 satellites –
held in 2023.

CHENNAI |SALEM| MADURAI | COIMBATORE DELHI | BANGALORE | THIRUVANANTHAPURAM


www.shankariasacademy.com 126
www.iasparliament.com

o 3 Satellites of the constellation are  Moon Agreement of 1979 – It is directed


placed in Geostationary orbit and towards the use of the Moon and other celestial
bodies.
o Other 4 are placed in
inclined Geosynchronous orbit.  It declares that these regions of space must only
be for peaceful purposes, with no state allowed
 NavIC is like GPS in its functioning and to establish military bases or place weapons on
operational principles. celestial bodies.
 The only difference being GPS is owned and  India is a signatory to the Moon Agreement,
operated by United States government while but is yet to ratify it.
NavIC is owned and operated by India.
 Significance for India – It will gain greatly
 NavIC covers all of Indian subcontinent and from the exchange of information and gain
offers a positional accuracy of 5 mtrs access to NASA’s Artemis programme which
compared to 15-20 mtrs of GPS. would greatly help India’s own Gaganyaan
mission.
 Not all smartphones and navigational gadgets
(or navigators) are compatible with NavIC.  NASA and ISRO have agreed to a joint mission
to the International Space Station (ISS) in
 To use and decode NavIC signals, a navigating 2024.
gadget should have a NavIC compatible
chipset or microchip incorporated inside it.  Outer Space Treaty 1967 – It governs the
Activities of States in the exploration and use of
 All the first-generation satellites were launched Outer Space, including the Moon and Other
by PSLV rockets. Celestial Bodies.
 2nd generation satellites – They are  Artemis mirrors – A Chinese-Russian plan
replacement satellite for NavIC, will replace for an ‘International Lunar Research Station’
satellites located at a geosynchronous orbit with (ILRS).
additional features.
7. a
 NVS-01, the first one was launched by a GSLV
rocket. Gender Social Norms Index (GSNI), 2023

6. c Gender Social Norms Index 2023 says, Prejudices


against women didn’t reduce in last decade, 90%
Artemis Accord people still hold gender bias.
India became the 27th signatory to the American-led  Released by – UN Development
Artemis Accords. Programme (UNDP).
 Established in – 2020, by US together with 7  It quantifies biases against women, in 4
other founding members. key dimensions like political, educational,
economic and physical integrity.
 Aim – To return humans to the moon by 2025
and to reinforce and implement key obligations  Findings – Covering 85% of the global
in the 1967 Outer Space Treaty. population, reveals that close to 9 out of 10 men
and women hold fundamental biases against
 These Accords are a non-binding set of women.
principles.
 Women are underrepresented in leadership in
 It currently has 27 signatories including conflict-affected countries, mainly at the
India. negotiation tables in the recent conflicts in
Ukraine (0%), Yemen (4%) and Afghanistan
 Need – It was created by the U.S to act as (10%).
a soft-law alternative to the Moon Agreement,
which limits resource extraction in celestial  Countries with no bias - Germany,
bodies. Uruguay, New Zealand, Singapore and Japan.
 A key provision in the Artemis Accords allows  25 % of people believe it is justified for a man to
for actors to extract and utilise space resources. beat his wife.

 UN Women - It is the United Nations entity


dedicated to gender equality and the

CHENNAI |SALEM| MADURAI | COIMBATORE DELHI | BANGALORE | THIRUVANANTHAPURAM


www.shankariasacademy.com 127
www.iasparliament.com

empowerment of women. A global champion


for women and girls, UN Women was
established to accelerate progress on meeting
their needs worldwide.

8. c

Indian Ocean Dipole (IOD)

In 2023, while El Nino was firmly established in the


Pacific Ocean, a positive IOD development was also
observed to be present.

 It is a coupled ocean atmosphere


phenomenon in the Indian Ocean.

 Definition – It is the difference in sea surface


temperature between the eastern and western
sides of the Indian Ocean.

 It is sometimes called India’s El-Nino.

 Impacts – It can be seen in weather and


climate patterns in India and as far as
Indonesia, Australia, New Zealand and
Africa.

 3 phases - Positive IOD, Neutral Phase,


Negative IOD

 Positive IOD – Warmer Sea surface 9. a


temperatures in the western Indian Ocean
mRNA Vaccine
relative to the east, brings more rainfall to
India. India’s 1st indigenous mRNA vaccine (GEMCOVAC-
OM) for the Omicron variant, was approved under
 Neutral IOD – Water flows from the Pacific emergency use guidelines by the Drug Controller
between Indonesia’s islands, keeping seas General of India.
warm to the northwest of Australia.
 Developed by – Pune-based Gennova
 Air rises above this area and falls across the Biopharmaceuticals Ltd.
western half of Indian Ocean basin, blowing
westerly winds along the equator.  Features – It uses spike protein of the
omicron variant (BA.1) of the SARS- CoV-2 as
 This has very less impact of Indian an antigen.
monsoon.
 Messenger RNA – A type of RNA that is
 Negative IOD - Cooler Sea surface necessary for protein production.
temperatures in the western Indian Ocean
relative to the east, brings less rainfall to India.  Once cells finish making a protein, they quickly
break down the mRNA.

 Working of mRNA vaccine – mRNA is a


type of vaccine that uses a copy of mRNA to
produce an immune response.

 Spike protein- mRNA vaccines only


introduce a piece of the genetic material that
corresponds to a viral protein.

 This is usually a protein found on the


membrane of the virus called spike protein.

 Therefore, the mRNA vaccine does not expose


individuals to the virus itself.

CHENNAI |SALEM| MADURAI | COIMBATORE DELHI | BANGALORE | THIRUVANANTHAPURAM


www.shankariasacademy.com 128
www.iasparliament.com

Traditional Vaccines vs mRNA Vaccines  Ministry- Ministry of Housing and


Urban Affairs (MoHUA).
 Traditional vaccines work by giving a
person either viral proteins or an inactivated or  Tenure- 4 years, i.e., from 2023 till 2027.
weakened version of a virus that triggers an
immune response.  Loan- AFD and KfW

 Viral vector vaccines like Covishield, carry DNA  Technical assistance grant- European Union
wrapped in another virus, but mRNA is only a
sheet of instructions to make spike proteins  Program structure-
wrapped in a lipid (or a fat molecule) to keep it
stable. o Cities- Projects promoting circular
economy
 There is no risk of an mRNA vaccine changing
DNA because mRNA from vaccines does o State- Climate action
not enter the nucleus and does not alter
DNA, rather just stimulates the immune o National- Intervention for climate
response of the body. governance

 mRNA based vaccines for COVID  Haora Riverfront Development Project is


undertaken under this program in Tripura.
o GEMCOVAC – India
 CITIES 1.0- It was launched in 2018 by
o Pfizer-BioNTech – USA-Germany MoHUA, AFD, EU and NIUA.

o Moderna – USA 11. c

Mission on Advanced and High-Impact


Research (MAHIR)

 The Ministry of Power and the Ministry


of New and Renewable Energy have
jointly launched the Mission on Advanced and
High Impact Research.

 Aim - To facilitate indigenous research,


development and demonstration of the latest
and emerging technologies in the power sector

 Fund - Funds will be given by Ministry of


Power, Ministry of New and Renewable Energy
and the Central Public Sector Enterprises under
2 Ministries.

 Planned for an initial period of 5 years from


2023-24 to 2027-28.
10. b  The Mission will follow the technology life cycle
approach of Idea to Product.
City Investments to Innovate, Integrate and
Sustain 2.0 (CITIIS 2.0) 12. d
 It is a part of Smart Cities Mission. International Covenant on Civil and Political
Rights (ICCPR)
 Aim- To support competitively selected
projects promoting circular economy with focus  It is a multilateral treaty that deals with human
on integrated waste management at the city rights.
level, climate-oriented reform actions at the
State level.  The Act was adopted by the United Nations
(UN) General Assembly on 1966 and came into
 It is conceived in partnership with the French force in 1976.
Development Agency (AFD), Kreditanstalt für
Wiederaufbau (KfW), the European Union  The Covenant respects civil and political rights
(EU), and National Institute of Urban Affairs of individuals, including right to life, freedom of
(NIUA). religion, freedom of speech, freedom of

CHENNAI |SALEM| MADURAI | COIMBATORE DELHI | BANGALORE | THIRUVANANTHAPURAM


www.shankariasacademy.com 129
www.iasparliament.com

assembly, electoral rights, and rights to due information network of the United Nations
process and a fair trial. Environment Programme (UNEP).

 The ICCPR, together with the Universal  The programme is one of the Central Sector
Declaration of Human Rights and the sub-schemes being implemented in alignment
International Covenant on Economic Social with Mission LiFE.
and Cultural Rights, are considered
the International Bill of Human Rights.  Funded by – Ministry of Environment,
Forest and Climate Change.
 Monitoring Authority - The UN Human
Rights Committee reviews regular reports by  EIACP Hub on Status of Environment
state parties on how human rights are being Related Issues- It is hosted by the Indian
implemented. State Level Basic Environmental Information
Database (ISBEID).
 Today, the covenant has 173 parties and 6 more
signatories without ratification.  The ISBEID is a centralised database
maintained by the Ministry of Environment,
 India ratified this treaty in 1979. Forest and Climate Change, Government of
India for all the States.
 Section 3 of the ICCPR Act- It prohibits the
propagation of war or advocacy of national,  EIACP conducts Environmental Awareness
racial or religious hatred that constitutes Programs to School Children, Colleges and
incitement to discrimination, hostility or Universities on Environmental Important Days
violence. like, World Environment Day, World Earth
Day, etc.
 Offences under this section are cognizable as
well as non-bailable. 14. d

 Afghanistan and South Sudan have ratified this Graphene


treaty.
 It is an allotrope of carbon consisting of a single
13. c layer of atoms arranged in a hexagonal lattice
nanostructure.
Environmental Information, Awareness,
Capacity Building and Livelihood Programme  Graphene was discovered in 2004 for which the
(EIACP) scientists received the Nobel Prize in Physics in
2010.
 The Environmental Information System
(ENVIS) came into existence as a plan  It is the world’s thinnest, strongest, and most
programme in 1983. conductive material of both electricity and
heat.
 It has been subsumed within the revamped
scheme of Environment Education, Awareness, Properties
Research and Skill Development.
 It is 200 times stronger than steel but 6 times
 It is later renamed as EIACP (Environmental lighter.
Information, Awareness, Capacity Building and
Livelihood Programme).  It is almost perfectly transparent as it absorbs
only 2% of light.
 EIACP is a network of subject-specific
centres located in various institutions  It is impermeable to gases, even those as light
throughout India. as hydrogen and helium.

 The primary objective of all EIACP centres is to  It has the potential to absorb and dissipate
collect, collate, store, and disseminate electromagnetic waves.
environment-related information to
various user groups, including researchers,  It is highly sensitive to environmental changes.
policy planners, and decision-makers.
Applications
 The focal point of the 60 EIACP centres in India
is at the Ministry of Environment, Forest and  Graphene composites are used in aerospace,
Climate Change (MoEFCC) automotive, sports equipment and
construction.
 MoEFCC further serves as the Regional Service
Centre (RSC) for INFOTERRA, the global

CHENNAI |SALEM| MADURAI | COIMBATORE DELHI | BANGALORE | THIRUVANANTHAPURAM


www.shankariasacademy.com 130
www.iasparliament.com

 It is used for high-performance  It aims to build a vibrant semiconductor and


batteries and super- display ecosystem to enable India’s emergence
capacitors, touchscreens, as a global hub for electronics manufacturing
and conductive inks. and design.

 Graphene-based sensors are used for  The Secretary of Ministry of Electronics and
environmental monitoring, healthcare and Information Technology acts as the chairman
wearable devices. for ISM committee.

 Graphene oxide membranes are used for water  It operates under the guidance of the Ministry
purification and desalination. of Electronics and Information
Technology (MeitY) and other relevant
 Graphene has an attractive potential in defence government bodies.
and aerospace as a promising material for
armour and ballistic protection due to its  The advantage of semiconductors having
exceptional strength. a small freight-to-price ratio and a zero-custom
duty regime under the Information Technology
15. b Agreement, 1996.

Schools of  The India Semiconductor Mission focuses on:


Indian Founder Significance
Philosophy o Integrated Circuit (IC) design and
manufacturing
Vaishesika Kanada Naturalism
o Semiconductor packaging and
Logic, assembly
Methodology,
Nyaya Gautama Muni o Testing and validation of
and
Epistemology semiconductor devices

Samkhya Kapila Reality o Skill development and human resource


capacity building
Meditation for
Yoga Patanjali  Taiwan produces over 60% of the world's
achieving Mukti
semiconductors and over 90% of the most
Power of advanced ones.
Purva
Jaimini Yajnas and
Mimansa  Semicon India Programme is a conference
Mantras
organised by the India Semiconductor Mission
Vedanta (ISM).
or Uttara
Mimansa  It aims to provide attractive incentive
support to companies/consortia that are
Three major engaged in Semiconductor sector.
branches of
Vedanta 17. d
World is
unreal and Hawkish Pause and Dovish Pause in economy
1. Advaita Vyasa
Brahman is
(Sankaracharya)
the only real  The term "pause" is used to describe the
2. Visistadvaita temporary nature of the decision to stop raising
interest rates.
(Ramanujacharya)
 A hawkish pause is a situation in which a central
3. Dvaita bank decides to temporarily stop raising
(Madhavacarya) interest rates, but makes it clear that they are
still committed to fighting inflation and may
16. d resume raising rates in the future.
India Semiconductor Mission (ISM)  A dovish pause is a situation in which a central
bank decides to stop raising rates because they
 India Semiconductor Mission (ISM) is are worried about slowing economic
a specialized and independent Business growth.
Division within the Digital India Corporation.

CHENNAI |SALEM| MADURAI | COIMBATORE DELHI | BANGALORE | THIRUVANANTHAPURAM


www.shankariasacademy.com 131
www.iasparliament.com

 The hawkish pause and dovish pause are used o Based on their distinctive features,
to denote the behaviour of the hawks and dove
respectively. o After people who discover them
(not mandatory)
 The Federal Reserve and the RBI have both
implemented hawkish pauses in recent o In honour of somebody such as Hitler
months. beetle.

 Both central banks of USA and India have  No two species of animals may have the
signaled that they are willing to continue same species name.
raising rates in the future, but they have also
taken a pause to assess the impact of their  No one species of animal may have more
previous rate hikes. than one valid species name.

18. b  A new name is considered to be valid only when


it is published in an “openly distributed
C+1 Strategy publication, and it must be accompanied by a
detailed description of the specimens the
 C+1 Strategy also known as China Plus One or author claims are typical for the group.
just Plus One is a Strategy followed by
companies to avoid investing only in International bodies governing
China and diversify business into other the nomenclature codes
countries.
International
 For the past few decades western countries Commission of Governs the naming of
investing heavily in China have led to Zoological animals
overconcentration of their business interests in Nomenclature (ICZN)
China.
International Code of
 The driving factors C+1 Strategy includes Nomenclature for Governs Naming of plants
China's cost advantage diminishing in recent algae, fungi, and plants (including cyanobacteria)
years to growing geopolitical distrust between (ICNafp)
China and the West.
International Code of Governs Naming of
 China’s continuing Zero-Covid Policy led to Nomenclature of bacteria (including
industrial and supply chain disruption. Bacteria (ICNB) Archaea)

 China has emerged as India’s top source of International


imports and the share of China in India’s total Committee on
merchandise imports has steadily increased. Governs virus names
Taxonomy of Viruses
(ICTV)
 While India has signed many Free Trade
Agreements with different countries there has 20. c
been no noticeable decline in China’s market
share in India’s total imports.

19. b

Naming of species

 Every species of animal or plant has two


scientific names.

 The first name of every species denotes


the genus to which the species belongs. It is a
generic name and is always capitalised.

 The second name of every species identifies the


species within the genus and is never
capitalised.

 Both first and second names are italicised.

 A newly found organism can be named

CHENNAI |SALEM| MADURAI | COIMBATORE DELHI | BANGALORE | THIRUVANANTHAPURAM


www.shankariasacademy.com 132
www.iasparliament.com

21. a  To promotes knowledge sharing and the free


flow of ideas to accelerate mutual
Commission of Railway Safety understanding and a more perfect knowledge of
each other's lives.
 It is a government body that acts as the railway
safety authority in the country.  Headquarters – Paris, and has more than 50
offices globally.
 Headquartered in – Lucknow, Uttar
Pradesh.  Member states – 194 Members and 12
Associate Members.
 Administrative control by – Ministry of
Civil Aviation (MoCA).  States that are not members of United
Nations may be admitted to UNESCO,
 Such cross control is made to keep the CRS upon recommendation of the Executive Board,
insulated from the influence of the country’s by a two-thirds majority vote of the General
railway establishment and prevent conflicts of Conference.
interest.
 UNESCO accorded recognition to
 Functions – It deals with matters related to Palestine as its member in 2011 and it is
safety of rail travel and operations. the first U.N. agency to do so.
 It also does other statutory functions like  The member states of UNESCO which are not
inspectorial, investigatory, and advisory as laid members of the UN include Niue, the Cook
down in the Railways Act, 1989. Islands, and Palestine.
 Investigating serious train accidents is  Shared Heritage – World Heritage Sites,
one of the key responsibilities of the CRS. Intangible Heritage Elements, Biosphere
Reserves and Global Geoparks.
22. c
24. d
International Centre for Integrated Mountain
Development Kosovo-Serbia Conflict
 It is a regional intergovernmental organization.  Geography – Both Kosovo and Serbia are
countries that lie in the Balkan region of
 Established in – 1983 in Kathmandu, Nepal Europe.
 Aim – To make Hindu Kush Himalaya region  North Macedonia & Montenegro are the
greener, more inclusive and climate resilient. 2 states that borders both Kosovo and
Serbia.
 8 Members – Afghanistan, Pakistan,
Bangladesh, India, China, Nepal,  Political background – Kosovo is a former
Bhutan, Myanmar. province of Serbia that unilaterally declared
independence in 2008.
 3 core pillars – Enhance livelihoods, sustain
resources and valuing culture.

 Significance – They work to strengthen


regional cooperation for conservation and
sustainable mountain development.

23. c

UNESCO

The United States said that it was re-joining the United


Nations Educational, Scientific and Cultural
Organization (UNESCO).

 UNESCO – United Nations Educational,


Scientific and Cultural Organization

 Objectives – To ensure peace and security by


promoting international cooperation in
education, sciences, culture, communication
and information.

CHENNAI |SALEM| MADURAI | COIMBATORE DELHI | BANGALORE | THIRUVANANTHAPURAM


www.shankariasacademy.com 133
www.iasparliament.com

 It is recognised as a country by about 100 The IARC of the WHO classified aspartame as “possibly
nations including the U.S. and several EU- carcinogenic” to humans, the 3rd category among the
countries but Serbia does not recognise 4 levels of carcinogens.
Kosovo’s sovereignty and continues to consider
it as a part of it.  Aspartame – An odourless powder that is white
and one of the world’s most common artificial
 An ethnic Serb minority resides in Kosovo sweeteners.
makes up to 5.3% of its population.
 It has 2 amino acids - L-aspartic acid and L-
 In 2013, both Serbia and Kosovo reached the phenylalanine.
Brussels Agreement brokered by the EU but the
agreement was not fully implemented on the  Aspartic acid is produced naturally by your
ground. body, and phenylalanine is an essential amino
acid that is intaken from food.
 Recent issue – In 2022, violent clashes broke
out in the northern region over the issue of  Usage – Food and beverage products since the
changing Serbian number plates of vehicles 1980s, including diet drinks, chewing
while in Kosovo. gum, gelatin, ice cream, dairy
products such as yogurt, breakfast
 Serbia has strong historic and military ties with cereal, toothpaste and medications such
Russia which also supports Serbia’s claim. as cough drops and chewable vitamins.

25. a  It is less sweet than other artificial sweeteners


like advantame and neotame, but it is about
Helmand River 200 times sweeter than table sugar.

Iran and Afghanistan are locked in a long-standing  As per US Food and Drug Administration
dispute over the sharing of water from the Helmand (FDA), the acceptable daily intake (ADI) is at 50
River. mg/kg of body weight per day.

 Helmand River – It is a vital source of water  Foods containing aspartame carry the warning
for both Iran and Afghanistan. “Not for phenylketonurics”.

 It is Afghanistan’s longest river, originates near  Around 100 countries around the world,
Kabul in the western Hindu Kush mountain including India, permit the use of aspartame.
range.
 In India, FSSAI recommends the maximum
 It flows in a southwesterly direction through permitted levels of aspartame.
desert areas before emptying into Lake
Hamun, which straddles the Afghanistan-Iran  Prior to this, working overnight, consuming red
border. meat, and using mobile phones were put under
cancer-causing categories.
 Lake Hamun is the largest freshwater lake in
Iran. 27. b

Global Slavery Index 2023

 Published by – Walk Free Foundation,


Australia-based human Rights Group.

 3 key variables - Prevalence of modern slavery


in each country, vulnerability and government
responses to modern slavery

 Modern slavery – It refers to situations of


exploitation that a person cannot leave because
of threats, violence, coercion and abuse of
power or deception.

 Data sources – It uses data released by the


International Labour Organisation (ILO), Walk
26. d Free, and the International Organisation for
Migration (IOM).
Aspartame

CHENNAI |SALEM| MADURAI | COIMBATORE DELHI | BANGALORE | THIRUVANANTHAPURAM


www.shankariasacademy.com 134
www.iasparliament.com

 2023 Index – It shows the world’s 20 richest  Launched by – Ministry of Fisheries, Animal
countries account for more than half the Husbandry and Dairying.
estimated 50 million people living in modern
slavery.  Developed by – Department of Animal
Husbandry and Dairying through Centre for
 Among the G20 nations, India tops the Development of Advanced Computing in
list with 11 million people working as forced coordination with Central Drugs Standard
labourers, followed by China, Russia, Control Organization (CDSCO).
Indonesia, Turkey and the U.S.
 Veterinary Drugs & Vaccines - The
28. a regulation of the import, manufacturing and
marketing of veterinary drugs and vaccines
Submarines vs. Submersible comes under the purview of the Central Drugs
Standard Control Organization (CDSCO).
The Titan, one of the crewed submersibles operated by
OceanGate, went missing in the area of the Titanic  The Permission for the import/manufacture of
wreck in the North Atlantic. veterinary drugs, vaccines are granted in
consultation with the Ministry of Fisheries,
 Titan – It is the only crewed submersible in the Animal Husbandry and Dairying.
world that can take 5 people as deep as 4,000
meters enabling it to reach almost 50% of the  Ministry of Health and Family Welfare gives
world’s oceans. final approval of drugs and vaccines.
Submarines Submersible  The NOC for veterinary drugs is provided by
Ministry of Fisheries, Animal Husbandry and
 It is a watercraft  It is a small boat or Dairying.
capable other craft, does not
of independent function as an  Nyaya Vikas Portal – It has been created by
operation autonomous Ministry of Law and Justice for monitoring the
underwater and craft and needs a implementation of Centrally Sponsored
does not need any mother ship that can Schemes (CSS).
mother ship launch and recover it.
 e-pashuhaat portal - Aims to connect
 Working – When  Features – Top speed breeders and farmers regarding availability of
submarine is to dive, of 3 knots (5.5 kmph). bovine germplasm.
water is filled in water
tanks and it is made  It can’t stay underwater 30. b
heavier. for as long compared to
submarines. Global Forest Watch (GFW)
 Since the average
density of submarine  Since there is no GPS  An online platform to provide data and tools for
becomes greater than underwater, it is only monitoring forests.
the density of sea guided by text
water, it sinks. messages from the  Created by – World Resources Institute
surface ship. (WRI), a global research organization that
 To make the submarine works to develop practical solutions that
rise to the surface of improve people’s lives and ensure nature can
water, water tanks are thrive.
emptied.
 Highlights of the study – In 2022, Tropical
29. a areas lost 4.1 million hectares of forest cover.

NANDI Portal  Brazil and the Democratic Republic of


Congo are the 2 countries with the most
 Nandi – NOC Approval for New Drug and tropical forest cover and both registered
Inoculation System. losses of this resource in 2022.

 Aim – To expedite regulatory process for  The total global tree cover loss in 2022 declined
approval of animal drugs and vaccines. by 10%.

 For timely process applications and grant non-  Primary forests are mature, natural forests
objection certification (NOC) for veterinary seeing loss in tropical areas in 2022, 10% more
drugs and vaccines. than in 2021.

CHENNAI |SALEM| MADURAI | COIMBATORE DELHI | BANGALORE | THIRUVANANTHAPURAM


www.shankariasacademy.com 135
www.iasparliament.com

 India – Lost 43.9 thousand hectares of humid  Financial assistance is provided to State
primary forest between 2021 and 2022, Governments / UTs in the prescribed fund
accounting 17% of the country’s total tree cover sharing pattern between Centre and States.
loss.
 Technical assistance- It is provided
 The total tree cover loss between 2021 and 2022 by National Remote Sensing Centre,
was 255 thousand hectares. ISRO.

31. b Level Monitoring mechanisms

Cholederm At State level High Court Level Monitoring


Committee
Recently, Cholederm got approval from Central Drugs
Standard Control Organisation (CDSCO) as a Class D At Central level Monitoring Committee in the
medical device. Department of Justice, required to
meet every 6 months
 Cholederm – It is a wound healing
material derived from the extracellular Nyaya Vikas Online monitoring system.
matrix of de-cellularised gall bladder of
pig and tissue engineered as membrane forms  eCourts Services portal- It enables to check
of scaffold. the status of legal case, it is different from
Nyaya Vikas portal.
 Developed by – Sree Chitra Tirunal Institute
for Medical Sciences and Technology 33. c
(SCTIMST).
Global Partnership on AI (GPAI)
 Healing properties – Healed different types
of skin wounds including burn and diabetic  It is an international initiative to support
wounds in rats, rabbits or dogs. responsible and human-centric development
and use of Artificial Intelligence (AI).
 Significance – It is the 1st indigenously
developed product to meet all regulatory  Aim - To bridge the gap between theory and
requirements. practice on AI by supporting cutting-edge
research and applied activities on AI-related
 It is in compliance with Medical Devices Rules priorities.
2017.
 Launch year- 2020
 The treatment cost can be reduced from Rs
10,000 to Rs 2,000 making it more affordable  It built around a shared commitment to
to the common man. the OECD Recommendation on AI.

32. a  GPAI is a group of 25 member countries.

Nyaya Vikas Portal  The 1st 3 GPAI summits were held in Montreal,
Paris and Tokyo respectively.
 Aim - It is a digital platform to monitor
the Centrally Sponsored Scheme for  India- It joined the group as a founding
Development of Infrastructure for member in 2020.
Judiciary.
 New Delhi declaration on AI- It was
 Outlay - Centrally Sponsored Scheme (CSS) adopted on GPAI summit 2023 on advancing
safe, secure, and trustworthy AI and
 Ministry - Ministry of Law and Justice commitment to supporting the sustainability of
GPAI projects.
 Year - The scheme being implemented since
1993-94.  Bletchley Declaration- It was signed by 28
major countries including India to initiate a
 The primary responsibility of development of global action to tackle the potential risks of AI
infrastructure facilities for judiciary rests with at the World’s 1st Artificial Intelligence
the State Governments. (AI) Summit in Bletchley Park, London.
 It covers the construction of court buildings and 34. d
residential accommodations for Judicial
Officers of District and Subordinate Judiciary. Biochar

CHENNAI |SALEM| MADURAI | COIMBATORE DELHI | BANGALORE | THIRUVANANTHAPURAM


www.shankariasacademy.com 136
www.iasparliament.com

 Biochar is a carbon-rich material that is made 2. Impartiality;


from biomass through a thermochemical
conversion process known as pyrolysis. 3. Non-use of force except in self-defence
and defence of the mandate.
 Pyrolysis –Organic materials are exposed to a
very high temperature, in the absence of  There are currently 12 UN peacekeeping
oxygen. operations deployed in Africa, the Middle East,
Europe, and Asia.
 According to the UN's Intergovernmental Panel
on Climate Change (IPCC), biochar could  As of 2021, women constitute nearly 8% of
potentially be used to capture 2.6 billion of the all uniformed military, police, and corrections
40 billion tonnes of CO2 currently produced by personnel in peacekeeping missions.
humanity each year.
 U.S. Authorization – As a veto-wielding
 The energy or heat created during pyrolysis can member of the Security Council, no UN
be captured and used as a form of clean energy. Peacekeeping operation can be authorized,
deployed, or withdrawn without U.S. support.
 Physical Attribute - Biochar is black, highly
porous, lightweight, fine-grained and has a  UN peacekeeping is a unique global partnership
large surface area. that brings together the following in a
combined effort to maintain international
 Approximately 70% of its composition is carbon peace and security:
while the remaining percentage consists of
nitrogen, hydrogen and oxygen among other o The General Assembly
elements.
o The Security Council
 Biochar production is a carbon-negative
o The Secretariat
process, which means that it actually reduces
CO2 in the atmosphere. o Troop and police contributors
Advantages o The host governments
 Enhancing soil  Improving porosity  United Nations India-Pakistan
structure Observation Mission (UNIPOM) - It was
 Regulating nitrogen established in 1965 to supervise the
 Improves soil quality leaching ceasefire along the India-Pakistan
border except in the State of Jammu and
 Produces energy as a  Improving electrical Kashmir and the withdrawal of all armed
by-product conductivity personnel to the positions held by them before
5 August 1965.
 Increasing water  Improving microbial
retention and properties  After the withdrawal of the troops by India and
aggregation Pakistan had been completed, UNIPOM was
 Reducing nitrous oxide terminated.
 Decreasing acidity emissions
36. d
 Sequesters Carbon
SANKALP Programme
35. b
 Skill Acquisition and Knowledge Awareness for
UN Peace Keeping Livelihood Promotion (SANKALP) is a
programme of the Ministry of Skill
 UN Peacekeeping helps countries navigate the Development and
difficult path from conflict to peace. Entrepreneurship with loan assistance
from the World Bank.
 It is governed by the Department of Peace
Operations.  It aims to improve short term skill
training through strengthening institutions,
 It aims to address a range of mandates set by bringing in better market connectivity and
the UN Security Council and General Assembly. inclusion of marginalised sections of the
society.
 Principles – UN Peacekeeping is guided by 3
basic principles:  SANKALP was launched on January 2018 and
has a tenure till March 2023.
1. Consent of the parties;

CHENNAI |SALEM| MADURAI | COIMBATORE DELHI | BANGALORE | THIRUVANANTHAPURAM


www.shankariasacademy.com 137
www.iasparliament.com

 SANKALP has three result areas namely:  15 elements from India have been inscribed on
the Lists of Intangible Cultural Heritage
1. Institutional Strengthening at but Yakshagana is not a part of it.
National, State and District level
38. d
2. Quality Assurance of skill development
programmes Recently GI Tags
Significance
from UP
3. Inclusion of marginalized population
in skill development programmes  It is a musical instrument
Amroha Dholak made of natural wood.
 The Ministry has on boarded Indian Institute of
Management (IIM) Indore as the Independent  It is a stone craft, a very
Verification Agency (IVA) to verify the unique and soft stone with
Disbursement Linked Indicators (DLIs) wise scientific name, the ‘Pyro
achievements. Flight Stone’.
37. b  Gaura stone craft is made
of radiant white-coloured
Yakshagana
Mahoba Gaura stone that is
Patthar predominantly found in
 Yakshagana is also known as Bayalatam,
Hastashlip this region.
a traditional dance-drama
performance.
 Only cotton yarn are used
 “Yaksha” means “celestial” and “Gana” means in the handloom weaving
“music”. process.

 It is a rare combination of dance, music, song,  It is famous for weaving on


the frame loom instead of
scholarly dialogues and colourful costumes.
Baghpat Home the pit loom since
 It is performed in Karnataka and the Furnishings generations.
Kasaragod district of Kerala.
 This handloom product
 Traditionally, Yakshagana was performed in mainly consists of cotton.
the open air by all-male casts sponsored by Barabanki
various Hindu temples. Handloom  It is manufactured using
Product traditional techniques.
 Since the mid-20th century many
performances have been held on indoor stages,  Kalpi has historically been
and women began to train in the tradition in the Kalpi Handmade a centre of handmade
1970s. Paper paper manufacturing.

 Each performance typically focuses on a small  Sambhal city is associated


Sambhal Horn
sub-story (known as ‘Prasanga’) from ancient with a unique kind of horn
Craft
Hindu epics of Ramayana or Mahabharata. and bone craft items.

 The Musical instruments used are Chande  Tarkashi is a technique of


(drums), harmonium, maddale, taala (mini inlaying brass, copper,
metal clappers), flute, etc. and silver wires in wood.
Mainpuri
 The Troupes (known as Melas) inclue Tarkash  It is used for decorating
Saligrama mela, Dharmasthala Mela, jewelry boxes, name
Mandarthi Mela, Perduru Mela, etc. plaques, and other similar
items.
 The Costumes include large headgear, brightly
colored faces, elaborate costumes all over the 39. b
body, and musical beads on the legs (Gejje).
North Atlantic Treaty Organization (NATO)
 The Terukkuthu of Tamilnadu, Kootiyatam
and Chakyarkuttu of Kerala, Veedhinataka of  It is a 32-member alliance, with 30
Andhra Pradesh can be considered as sister art European nations, and two American nations
forms. (United States and Canada).

CHENNAI |SALEM| MADURAI | COIMBATORE DELHI | BANGALORE | THIRUVANANTHAPURAM


www.shankariasacademy.com 138
www.iasparliament.com

 It was set up in 1949 by the US, Canada and  It was built in 1956 as part of the Kakhovka
several Western European nations to provide hydroelectric power plant.
collective security against the Soviet Union.
Zaporizhzhia Nuclear Power Plant
 Aim - To protect peace and to guarantee the
territorial integrity, political independence and  It is situated in the Southeastern
security of the member states. Ukraine and is the largest nuclear power
plant in Europe.
 It was the US’s first peacetime military alliance
outside the western hemisphere.  It generates up to 42 billion kWh of electricity,
accounting for about 40% of the total electricity
generated by all the Ukrainian NPPs and 1/5th
of Ukraine's annual electricity production.

 Collective defence- It is laid out in Article 5


of the North Atlantic Treaty in which members
of NATO are committed to mutual defence in
response to an attack by any external party.

 Membership- It is open to any other


European state in a position to further the
principles of this Treaty and to contribute to the
security of the North Atlantic area.

 Finland is the 31st member of the alliance


and Sweden has become the 32nd
member of the alliance.

 India- It is not a part of NATO because it is 41. b


a firmly non-aligned country and stakes its
international reputation on remaining non- Goods and services Tax
aligned.
Touted as one of the biggest tax reforms, India’s Goods
40. b & Services Tax (GST) turned six years this year.

Mayon Volcano  Established by – Constitution (101st


Amendment) Act 2016, with effect from 1st July
 It is a highly active stratovolcano in the 2017.
southeastern Luzon, Philippines.
 Motto – One nation, One market, One tax.
 It is called the world’s most perfect volcanic
cone because of the symmetry of its shape.  Objectives – To bring unity and benefits for
all stakeholders, governments, taxpayers, and
 It is located on the Pacific Ring of Fire, the rim administrators alike.
of seismic faults where most of the world’s
earthquakes and volcanic eruptions occur.  Features – It completely transformed the
indirect tax structure of India.
Kakhovka Dam
 It subsumed of 17 taxes and 13 cesses levied by
 Kakhovka dam in southern Ukraine was the Central and State governments.
collapsed recently, causing extensive flooding.
 Slabs – Apart from zero-rated goods, the new
 The dam is built on Ukraine’s Dnipro structure has just four tax slabs 5%, 12%, 18 and
River which separates Ukraine into two parts — 28%.
east and west.
 Destination-based Tax - GST is a
 It flows north to south connecting the capital, destination-based tax, levied at each stage of
Kiev, to the Black Sea. the supply chain, from the manufacturer to the
consumer.

CHENNAI |SALEM| MADURAI | COIMBATORE DELHI | BANGALORE | THIRUVANANTHAPURAM


www.shankariasacademy.com 139
www.iasparliament.com

 It is applied to the value addition at each stage,


allowing for the seamless flow of credits and
reducing the tax burden on the end consumer.

 Dual Structure - GST operates under a dual


structure, comprising the Central GST (CGST)
levied by the Central Government and the State
GST (SGST) levied by the State Governments.

 In the case of Inter-state transactions,


Integrated GST (IGST) is applicable, which is
collected by the Central Government and
apportioned to the respective State.

 Import of goods or services would be treated  They typically occur between March and June,
as inter-state supplies and would be subject and in some rare cases even extend till July in
to IGST in addition to the applicable customs India.
duties.
 Wet bulb is a condition when high
 GST Council – It is a joint forum of the Centre temperatures combine with high humidity.
and the States which makes recommendations
to the Union and the States on GST as per  In such conditions, sweat from the human body
Article 279A. isn’t able to evaporate, failing to stabilise the
body temperature.
 Every decision of the GST Council is taken
at its meeting by a majority of not less than  This could ultimately cause heat stroke and
3/4th of the weighted votes of the even death.
Members present and voting.
 Marine heat wave – It occurs when
 Recent trends – Over the last 5 years, the the surface temperature of a particular
Value of Goods Transported Inter-State region of the sea rises to 3 or 4 degree
(Domestic Only) increased by 44%. Celsius above the average temperature for
at least five days.
 The Cumulative Value of Imports and Domestic
Goods increased by 34%.  87 % of MHWs are attributable to human-
induced warming.
 The taxpayer base has witnessed a
remarkable increase, surging from 63.9 lakh in 43. b
2017 to approximately 1.40 crore in
2023. Important Committees on Poverty Estimation
42. c India must raise the per capita income, estimated in
2022-23 at 2,379 USD, by almost six times and grow at
Heat waves 7 % over the next 25 years to have a higher standard of
living and eliminate poverty.
June 2023 was the Earth’s hottest June since the
record-keeping of global temperatures began 174  Poverty – A condition wherein an individual
years ago by National Oceanic and Atmospheric lacks the financial resources & essential
Administration (NOAA). conditions for meeting the minimum standard
of living.
 Definition – A period of abnormally high
temperatures, more than the normal maximum  Poverty line – It is a minimum expenditure
temperature that occurs. required to purchase a basket of goods and
services necessary to satisfy basic human needs.
 Caused by – Either formation of heat domes
or arrival of anticyclones in most cases.  In India, it is based on the consumption
expenditure and not on the income levels.
 Occurrence – Heat wave is considered if
maximum temperature of a station reaches at  It is measured based on Consumption
least 40 degree C or more for Expenditure Surveys (CES) of the National
plains and at least 30 degree C or more Sample Survey Organisation (NSSO) that are
for hilly regions. conducted every 5 years.

CHENNAI |SALEM| MADURAI | COIMBATORE DELHI | BANGALORE | THIRUVANANTHAPURAM


www.shankariasacademy.com 140
www.iasparliament.com

Poverty Committees  MPI= H*I

 Alagh Committee (1979) – It is a task force  If the deprivation score (sum of the weighted
constituted by the Planning Commission which status of all the indicators) for an individual is
constructed a poverty line for rural and urban more than 0.33, then an individual is
areas on the basis of nutritional requirements. considered multidimensionally poor.

 It is based on a minimum daily requirement  Key findings – A steep decline in the


of 2400 and 2100 calories for an adult in Rural poverty between 2015-16 and 2019-21.
and Urban area respectively.
 It registered a decline in the number of
 Lakdawala Committee (1993) - It “multidimensionally poor” individuals, from
recommended that consumption 24.85 % in 2015-16 to 14.96 % in 2019-2021.
expenditure should be calculated based on
calorie consumption as earlier.  Around 13.5 crore Indians escaped
poverty between the 5-year time period due to
 State specific poverty lines should be improvements in indicators like access to
constructed and should be updated using the cooking fuel, sanitation, drinking water etc.,
Consumer Price Index of Industrial Workers
(CPI-IW) in urban areas and CPI Agricultural  1 in 7 Indians multidimensionally poor due to
Labour (CPI-AL) in rural areas marginal improvement in indicators like
nutrition and access to education.
 Tendulkar Committee (2009) – It
recommended to shift away from calorie
consumption-based poverty estimation and to
have a uniform poverty line basket for
rural and urban India.

 It recommended using Mixed Reference


Period (MRP) based estimates, as opposed to
Uniform Reference Period (URP) based
estimates that were used in earlier methods for
estimating poverty.

 Rangarajan committee (2012) - It


estimated a monthly per capita expenditure of
Rs. 1407 in urban areas and Rs. 972 in rural
areas.

44. a

National Multidimensional Poverty Index

A new report titled “National Multidimensional


Poverty Index: A Progress Review 2023” was released. 45. a

 Published by – NITI Aayog Adjournment Motion

 Coverage – Both States and Union Territories. Recently opposition parties moved an adjournment
motion in the Lok Sabha, citing the need for urgent
 Source data – National Family Health discussions on the ethnic violence in Manipur.
Survey (NFHS-5).
 Originated in – House of Commons in the
 3 equally weighted dimensions – Health, United Kingdom.
Education, and Standard of living, which
are represented by 12 indicators.  It was established through Government of
India Act, 1919 in India under the rules of
 Sub-indices of National MPI the pre-independent bicameral legislature.

o Headcount ratio (H): How many are  Aim – To draw attention of the House to a
poor? definite matter of urgent public importance

o Intensity of poor (I): How poor are  It is introduced only in Lok Sabha.
the poor?

CHENNAI |SALEM| MADURAI | COIMBATORE DELHI | BANGALORE | THIRUVANANTHAPURAM


www.shankariasacademy.com 141
www.iasparliament.com

 It involves an element of censure against o Shahgarh Bulge, Rajasthan


the government, so Rajya Sabha is not
permitted to use the motion. o Mukundra Tiger Reserve, Rajasthan

 Support – It needs the support of 50 o Gandhi Sagar Wildlife Sanctuary,


members to be admitted. Madhya Pradesh

 The Speaker has to decide whether to allow the o Nauradehi Wildlife Sanctuary, Madhya
MP to move the motion. Pradesh

 It results in the House dropping its scheduled  Asiatic Cheetahs – They were once abundant
list of business to discuss this urgent matter. in India but the last cheetah in the country died
in 1952.
 Time - The discussion on the Adjournment
motion must be at least 2 hours and 30 minutes  Scientific name – Acinonyx jubatus
(2hrs and 3o minutes).
 Conservation status
 Coverage
o IUCN – Vulnerable
o It should not cover more than one
o CITES – Appendix I
topic.

o It should be restricted to one issue  Apex authority – National Tiger


which had a recent occurrence. Conservation Authority (NTCA).

o It should not be a topic that is already  The Cheetah Conservation Fund (CCF) is also
discussed or under discussion, in the assisting the Indian government in
same session. reintroducing cheetahs in India.

o It should not involve the matter of  NTCA – It was established in 2005 as a


privileges. statutory body under the Ministry of
Environment, Forests and Climate
o It should not contain topics that are Change.
under adjudication by the court.
 It has been constituted under Section 38 L (1)
o It should not be a matter that can be of Wildlife (Protection) Act, 1972.
raised under a distinct motion.
 It is headquartered in New Delhi.
46. c
 The Union Minister of Environment, Forests
Cheetah Reintroduction Project and Climate Change is its Chairperson.

 Project Cheetah – It is India’s cheetah


relocation programme.

 This is the 1st intercontinental


reintroduction of a wild, large
carnivore species.

 Objective – To restore the population of


cheetahs, which were declared extinct in the
country in 1952.

 It involves the translocation of African


cheetahs from Namibia and South Africa to
Kuno National Park in Madhya Pradesh.

 Asiatic Cheetah is the subspecies of Cheetah


that is found in Iran and is listed as critically
endangered (CE) by the IUCN. 47. a

No Confidence Motion (NCM)


 It brings in 5-10 animals every year, over the
next decade, until a self-sustaining population Recently, Lok Sabha Speaker accepted the Opposition’s
of cheetahs is established. no confidence motion against the Government.
 Other sites recommended are

CHENNAI |SALEM| MADURAI | COIMBATORE DELHI | BANGALORE | THIRUVANANTHAPURAM


www.shankariasacademy.com 142
www.iasparliament.com

 History – Since India’s independence, 27  Leader of opposition – It is the leader of


NCMs have been moved in the Lok Sabha. largest opposition party which is having at
least 1/1oth seats of the house.
 The 1st NCM was moved during the 3rd ok
Sabha in 1963 by Acharya J B Kripalani against  It is not mentioned in the
the government headed by Jawaharlal Nehru. Constitution but mentioned in
Parliamentary statute.
 The Constitution of India does not
mention about either a Confidence or a No  The post is recognised by the speaker.
Confidence Motion.
 Statutory recognition – Through Salaries
 Constitution – Article 75(3) specifies that and allowances of Leaders of opposition in
the Council of Ministers are collectively Parliament Act of 1977 (both RS and LS)
responsible to the Lok Sabha (Not
Parliament)  They receive salary and allowances equivalent
to Cabinet Minister.
 For testing this collective responsibility,
the rules of Lok Sabha provide  Present status (2023) - The post of Leader
the mechanism of a motion of no- of Opposition in Lok Sabha is vacant since
confidence. 2014 as none of the opposition party could
fulfil 10% quorum.
 A no confidence motion can only be moved in
the Lok Sabha.  The 1st general elections to the Lok Sabha after
independence were conducted in 1952 and the
 Procedure for passing an NCM – Any Lok Congress won the elections.
Sabha MP who can garner the support of 50
colleagues can introduce a NCM against the  None of the opposition parties could win one-
Council of Ministers in writing. tenth of the number of seats.

 Speaker will decide whether to admit the  In the first general election, CPI won 16 seats
motion for discussion and emerged as the largest opposition party.

 The date for discussion should be within 10  In December 1969, the Congress Party’s (O)
days from the date the motion was accepted in leader Ram Subhag Singh was recognised as
the House. the opposition leader for the first time.

 Government will respond to the motion, the 49. c


opposition parties will then have the
opportunity to speak on the motion. Forest (Conservation) Amendment Act, 2023

 After the debate, they will vote and it will be  Forest Conservation Act 1980 – To contain
passed if it is supported by the majority of the regulations concerning forest conservation and
members of the House. matters related to it.

 If passed – The council of ministers must  No State Government or other authority shall
resign from office. convert forest land for non-forest purpose
except with the prior approval of the Central
 Highest number of instances – During the Government.
tenure of former PM Indira Gandhi, who faced
NCMs 15 times.  Non-forest purposes – It includes use of
land for cultivating horticultural crops or for
 Governments fallen due to NCM - V P any purpose other than reafforestation.
Singh government in 1990, H D Deve Gowda
government in 1997 and Atal Bihari Vajpayee  Hence, diversion of forest land for non-forest
government in 1999. purposes requires the prior approval of the
central government according to the 1980 Act.
48. c
 The activities allowed in forests without such
Leader of opposition prior approval are related to the conservation
and management of forests and wildlife.
The 26 Opposition political parties from across India
have formed the INDIA coalition (the Indian National  For example, permitted activities include
Developmental, Inclusive Alliance) for the 2024 Lok establishing check posts and fire lines.
Sabha election.

CHENNAI |SALEM| MADURAI | COIMBATORE DELHI | BANGALORE | THIRUVANANTHAPURAM


www.shankariasacademy.com 143
www.iasparliament.com

 This Amendment Act add more activities to the offered by higher education institutions in
list that will be excluded from non-forest colleges and universities in India.
purposes without approval such as
 5 parameters
o Zoos and safaris under the Wild Life
(Protection) Act, 1972 in forest areas o Teaching, learning and resources
other than protected areas
o Graduation outcome
o Eco-tourism facilities
o Research and professional practices
o Silvicultural operations
o Outreach and inclusivity
o Any other purpose specified by the
central government; it may specify o Perception
exclusion of any survey.
 Rankings – NIRF ranks higher education
 Covers 2 types of land - Land institutions in 5 categories and 8 subject
declared/notified as a forest under the Indian domains.
Forest Act, 1927 or any law
 Ranks are assigned based on the sum of marks
 Land not covered in the first category but secured by institutions on each of these
notified as a forest on or after October 25, 1980 parameters.
in a government record
 5 Categories – Overall, universities, colleges,
 Exempted land – Forest land along a rail line research institutions and innovations.
or a public road maintained by the government
up to a maximum size of 0.10 hectare  2023 Rankings – Introduced a new subject
namely Agriculture & Allied Sectors.
o Land situated within 100 km
along the international borders,  Integration of the 'Innovation' ranking
Line of Control, or Line of Actual previously executed by the Atal Ranking of
Control Institutions on Innovation Achievements
(ARIIA) into the India Rankings.
o Land proposed to be used for
constructing security infrastructure  Expansion of scope of 'Architecture' to
and defence related projects (not 'Architecture and Planning'.
exceeding 5 hectares in a left wing
extremism affected area)  Findings – Only 12.3% of higher educational
institutions participated in the ranking process.
 Assigning Forest land – Requires prior
approval of the central government to direct the  The 12.3% of institutions which have
assigning of forest land to any organisation not participated in the ranking contribute close to
owned by the government. 90% of scholarly output in the country.

 Delegation of power - The Central 51. b


government may issue directions for the
implementation of the Act to any other Permanent Court of Arbitration
authority.
Pakistan approached the Permanent Court of
50. c Arbitration to resolve the contention associated with
construction of hydel projects by India over the Indus
NIRF India Rankings 2023 River system.

The National Institutional Ranking Framework  It is an intergovernmental organization.


(NIRF) recently released the 8th edition of India
Rankings for 2023.  It is not a United Nations agency but
has observer status in the UN General
 Publication – Annual, since Assembly.
its establishment in 2016
 Established by – The Convention for the
 Published by – Union Ministry of Pacific Settlement of International
Education Disputes, concluded at The Hague in
1899 during the 1st Hague Peace Conference.
 It is also known as the India Rankings,
evaluates the 'quality' of educational programs  Aim – To facilitate arbitration and other forms
of dispute resolution between member states,

CHENNAI |SALEM| MADURAI | COIMBATORE DELHI | BANGALORE | THIRUVANANTHAPURAM


www.shankariasacademy.com 144
www.iasparliament.com

international organizations, or private parties  It is not officially recognised within


arising out of international agreements. NATO.

 Membership – 122 Contracting  It has been used in discussions and debates


Parties including India, Pakistan, China, regarding the potential expansion of the
Russia, and the US. alliance.

 Services – Resolution of disputes involving 53. c


various combinations of states, state entities,
intergovernmental organizations, and private Zero FIR
parties.
 When a police station receives a complaint of an
 The cases involving territorial and maritime offence that has been committed in another
boundaries, sovereignty, human rights, jurisdiction, it registers an FIR and then
international investment, and international transfers it to the relevant police station for
trade. further investigation.

 3-part organizational structure  This is called a Zero FIR. No regular FIR


number is given.
o Administrative Council – Oversees its
policies and budgets  After receiving the Zero FIR, the revenant
police station registers a fresh FIR and starts
o Members of the Court – A panel of the investigation.
independent potential arbitrators
 A Zero FIR can be filed in any police
o Secretariat – Also known as station by the victim, irrespective of their
the International Bureau. residence or the place of occurrence of crime.

 It has no sitting judges, instead, parties  Purpose – To provide speedy redressal to the
themselves select the arbitrators. victim so that timely action can be taken after
the filing of the FIR.
 The rulings of PCA are binding but the
tribunal has no powers for enforcement.  Provision – The provision of Zero FIR came
up after the recommendation of the
52. b Justice Verma Committee, constituted
after the 2012 Nirbhaya gang rape case to
NATO Plus suggest Criminal Law amendments.
During Prime Minister Narendra Modi’s visit to US in
 Zero FIR is applicable only to cognisable
June 2023, there have been talks to include India in the
offence
fold of NATO.
 For non-cognisable offence- The police
 NATO – It is a transatlantic military alliance of
cannot take immediate action without a
31 countries with the majority of members from
warrant; the police do not register an FIR.
Europe.
 Instead, they document the complaint in a
 Set up in – 1949, as a collective security
“Non-Cognizable Report” (NCR) and
against Soviet Union, headquartered in
recommend that the complainant seek legal
Brussels, Belgium.
recourse through the court.
 In 2023, Finland become its 31st member. First Information Report (FIR)
 NATO plus – It refers to a security  The term First Information Report (FIR) is not
arrangement of NATO and the 5 treaty defined in the Indian Penal Code (IPC), Code of
allies of the U.S. — Australia, New Criminal Procedure (CrPC), 1973, or in any
Zealand, Japan, Israel, and South Korea as other law.
members.
 However, in police regulations or rules,
 India is not a member yet; Sweden is the information recorded under Section 154 of
latest member (32) to NATO. CrPC is known as First Information Report
(FIR).
 Goal – To enhance “global defence
cooperation” and win the “strategic  Elements of FIR-The information must
competition against China. relate to the commission of a cognizable
offence.

CHENNAI |SALEM| MADURAI | COIMBATORE DELHI | BANGALORE | THIRUVANANTHAPURAM


www.shankariasacademy.com 145
www.iasparliament.com

o It should be given in writing or orally to o They are considered the first settlers of
the head of the police station. India.

o It must be written down and signed by o Their home, Biligiri Rangan (BR) Hills,
the informant, and its key points was among the first areas to be
should be recorded in a daily diary. declared a wildlife sanctuary in India,
in 1974.
 The Apex court in Lalita Kumari v. Government
of U.P. observed that it is mandatory to register o In 2011, when the region was declared
a FIR under Section 154 if the complaint is a tiger reserve, the Soligas were the first
related to a cognizable offence. community to win resident rights in a
tiger reserve.
54. a
55. b
Silver Cockscomb
Ambergris
 Silver cockscomb is a short-lived beautiful
but troublesome weed that is 50-60 cm-tall  Ambergris, which means grey amber, is
with simple, spirally arranged leaves around a waxy substance that originates from
the stem. the digestive system of protected sperm
whales.
 In Karnataka’s Chamarajanagara district, the
silver cockscomb is referred to as anne soppu.  Ambergris is a rare substance, which
contributes to its high demand and high price
 It is also known as lagos spinach, the weed in the international market.
belongs to the Amaranthaceae family, which
includes economically important plants like  The freshly passed ambergris is a light
spinach (Spinacia oleracea), beetroot and yellowish substance and is fatty but as it ages it
quinoa. turns waxy and gets red-brownish.

 Benefits – The Soliga tribes consume the  Traditionally, it


silver cockscomb as a leafy vegetable as they is used to
are high in nutrients such as beta- produce
carotene and folic acids, and have vitamin perfumes which
E, calcium and iron. have notes of
musk.
 Most farmers use the plant as fodder for
livestock.  While there is a
ban on the
 It is frequently used in traditional Chinese and possession and trade of ambergris in countries
Indian medicine for treating eye diseases and like the USA, Australia and India.
ulcers.
 In India, sperm whales are a protected species
 Issues – If left unchecked, it can spread under Schedule 2 of the Wildlife Protection Act,
quickly and suppress the growth of other crops, 1972.
affecting their yield.
 The possession or trade of any of its by-
 It also attracts insects, caterpillars, worms and products, including Ambergris and its by-
moths that can harm crops. products, is illegal under provisions of the
Wildlife Protection Act, 1972.
 Although it is of the same family as spinach,
silver cockscomb leaves have lower levels of Shaligrams (Ammonites)
oxalic acid (0.2%) and phytic acid (0.12%).
 For more than 2,000 years, Hinduism,
 Spinach leaves on the other hand are high in Buddhism and the shamanic Himalayan
calcium, oxalates, vitamin K and potassium, religion of Bon have venerated Shaligrams.
which can lead to formation of kidney stones.
 Shaligrams are ancient fossils of ammonites, a
 Soligas – Are an indigenous tribe of class of extinct sea
Karnataka. creatures related to
modern squids.
 They are indigenous people of South India and
are credited with being the first at many things:  Originating from a
single remote
region, Kali Gandaki

CHENNAI |SALEM| MADURAI | COIMBATORE DELHI | BANGALORE | THIRUVANANTHAPURAM


www.shankariasacademy.com 146
www.iasparliament.com

River Valley of Mustang of northern It was established in 2005, functions


Nepal. under Ministry of Agriculture & Farmers
Welfare.
 Shaligram stones are viewed primarily as
manifestations of the Hindu god Vishnu. 57. b

 Climate change, faster glacial melting, and Foundational Agreements


gravel mining in the Kali Gandaki are changing
the course of the river, which means fewer  Defence relationship has emerged as a major
Shaligrams are appearing each year. pillar of India-U.S. strategic partnership with
the signing of ‘New Framework for India-U.S.
 This is mainly because the Kali Gandaki is fed Defense Relations’ in 2005.
by meltwater from the Southern Tibetan
Plateau.  In 2016, US recognised India as “Major Defence
Partner” which commits the U.S. to facilitate
 But with the glacier disappearing, the river is technology sharing with India.
becoming smaller and shifting away from the
fossil beds that contain the ammonites needed  Both have signed 4 foundational
to become Shaligrams. agreements that cover areas of military
information, logistics exchange, compatibility,
56. d and security.

Protection of Plant Varieties and Farmers Agreemen Signe


About Purposes
Rights Act, 2001 t d in

The Delhi High Court upheld an order by the Protection General Security
of Plant Varieties and Farmers’ Rights Authority of Military Intelligence
GSOMIA 2002
(PPVFRA), revoking the registration of patent for a Information sharing
potato variety (FL-2027) to PepsiCo India Holdings Agreement
Pvt. Ltd.
Logistics
Logistics
 The Act recognizes the contributions of both LEMOA Exchange 2016
Exchange
commercial plant breeders and Agreement
farmers in plant breeding activity.
Communication
 Breeders’ Rights - Breeders will have Communication s
exclusive rights to produce, sell, market, COMCASA s Security 2018 interoperability
distribute, import or export the protected Agreement & Technology
variety. enabler

 Researchers’ Rights - Researcher can use Basics Exchange


Geo-spatial
any of the registered variety under the Act for BECA Cooperation 2020
cooperation
conducting experiment or research. This Agreement
includes the use of a variety as an initial source
of variety for the purpose of developing another
variety but repeated use needs prior permission  Other agreements - 'Security of Supply' (SoS)
of the registered breeder. arrangement and a 'Reciprocal Defence
Procurement' (RDP) agreement aims to
 Farmers' Rights - A farmer who has evolved promote long-term supply chain stability and
or developed a new variety is entitled for enhance security and defense cooperation.
registration and protection in like manner as a
breeder of a variety. 58. d

 Farmers variety can also be registered as an Indian Ocean Geoid Low (IOGL)
extant variety.
 The “gravity hole” in the Indian Ocean is
 A farmer can save, use, sow, re-sow, exchange, officially called the Indian Ocean geoid low.
share or sell his farm produce including seed of
a variety protected under the PPV&FR Act,  It is a spot where Earth’s gravitational pull
2001 but he/she shall not be entitled to sell is weaker, its mass is lower than
branded seed of a variety protected under normal, and the sea level dips by over 328
this Act. feet (100 meters).

 Protection of Plant Varieties and


Farmers' Rights Authority (PPVFRA) -

CHENNAI |SALEM| MADURAI | COIMBATORE DELHI | BANGALORE | THIRUVANANTHAPURAM


www.shankariasacademy.com 147
www.iasparliament.com

 Here gravity  Its name is derived from the Kaasa tree,


is lower than botanically known as Elaeocarpus glandulosus
average, (rudraksha family).
thus making
the sea level  The Kaas Plateau is designated as a biodiversity
lower than hotspot and is included in the UNESCO World
the global Natural Heritage Site under the name “The
average at Western Ghats” in 2012.
the site,
making it  Kas Lake is a perennial source of water supply
lowest geoid for western part of Satara.
on Earth.
 It is popular for its stunning display of
 IOGL is a consequence of a mass deficit inside wildflowers (Flower Wonder) that come to life
the Earth's mantle beneath the Indian Ocean. during monsoon in Maharashtra, this unique
characteristic has earned it the endearing
 The IOGL comprises slabs from the Tethys nickname of “Maharashtra's Valley of Flowers”.
Ocean which separated the supercontinents of
Gondwana and Laurasia.  Thoseghar Waterfalls and Sajjangad Fort is
located in this plateau.
59. b
60. a
Khazans of Goa
Comprehensive and Progressive Agreement for
 Generally, any low-lying land close to a Trans-Pacific Partnership (CPTPP)
mangrove-fringed estuary reclaimed by salinity
control structures can be called as khazan land. The United Kingdom recently signed the
Comprehensive and Progressive Agreement for Trans-
 Khazans are reclaimed lands from the river or Pacific Partnership treaty (CPTPP)
the sea; Khazans in Goa are coastal wetlands.
 The CPTPP is a free-trade agreement (FTA)
 True Khazan lands are carefully designed as launched in 2018.
topo-hydro-engineered agro-aquacultural
ecosystems.  Members – Pacific Rim of 11 countries, which
includes Australia, Brunei, Canada, Chile,
 The most important aspect of the structure of Japan, Malaysia, Mexico, New
these lands is based on the principle of salinity Zealand, Peru, Singapore and Vietnam.
regulation and knowledge of the tidal clock. (UK was recently added as a member).

 Khazan lands have 3 main features - sluice gate,  The USA, Russia and India are not
poim and 2 types of bunds. members of CPTPP.

Pakkasuran Malai  The CPTPP requires countries to eliminate or


significantly cut trade barriers (reduce tariffs)
 Pakkasuran Malai is a mountain which is also and make strong commitments to opening
called as Hullikal Drug. services and investment markets.

 It is situated in Coonoor, Nilgiri Mountains  It also has rules addressing competition,


of Tamil Nadu. intellectual property rights and protections for
foreign companies.
 It is a biodiversity hotspot primarily composed
of rocks and grasslands.  CPTPP will cut tariffs for UK exports to Asia
Pacific countries.
 Droog Fort is a historic fort on the peak of
Pakkasuran Malai and it qualifies as a sacred  With UK membership, the trading bloc will
grove. have a combined GDP of 12 trillion pounds and
account for 15% of global trade.
Kaas Plateau
61. a
 Kaas Plateau is made of igneous rock, situated
in the Western Ghats of Maharastra. Ethanol Blended Petrol Program

 Locally called as 'Kas Pathar' or 'Plateau of India in ‘G20 Energy Ministers’ 2023, has rolled out
Flowers'. 20% ethanol-blended petrol in 2023 and aims to cover
the entire country by 2025.

CHENNAI |SALEM| MADURAI | COIMBATORE DELHI | BANGALORE | THIRUVANANTHAPURAM


www.shankariasacademy.com 148
www.iasparliament.com

 Launched in – 2003  It is a product of crossing 2 plants containing


alien ‘barnase’ and ‘barstar’ genes derived from
 Aim – To promote the use of renewable fuels. a soil bacterium.

 Started with 5% blending, it has set a target  Regulatory and approval authority for
of 10% ethanol blending by GM crops in India
2022 and 20% blending (E20) by 2030
which was now shifted to 2025-26. o Environment Protection Act, 1986
(EPA)
 National Policy on Bio fuels (2018) – It
prescribes as indicative target of 5% blending of o Biological Diversity Act, 2002
biodiesel in diesel by 2030.
o Plant Quarantine Order, 2003
 Goods & Service Tax (GST) on ethanol meant
for EBP Programme has been reduced from o GM policy under Foreign Trade Policy
18% to 5%.
o Food Safety and Standards Act, 2006
 Roadmap for ethanol blending in India
o Drugs and Cosmetics Rule (8th
2020-25 – It is released by Niti Aayog which
Amendment), 1988
recommends measures for ethanol blending in
India.  Genetic Engineering Appraisal
Committee (GEAC) – It is responsible for
 Ethanol – It is Ethyl alcohol (C2H5OH), a
appraisal of proposals relating to the release of
biofuel that is naturally made by fermenting
GM organisms and products into the
sugar.
environment.
 It is basically 99.9% pure alcohol that can be
 Functions under Ministry of Environment,
blended with petrol.
Forest & Climate Change under Environment
Protection Act, 1986.
 Status in India – Ethanol production
capacity increased more than 2.5 times and no.
 It applies to large scale use of hazardous
of distilleries increased by 66% in 8 years.
microorganisms and recombinants in research
and industrial production from the
 Department of Food and Public
environmental angle.
distribution is the nodal agency for
promotion of fuel grade ethanol 63. c
producing distilleries in the country.
Clarion-Clipperton Zone (CCZ)
 Ethanol procurement – From sugar cane based
raw materials, surplus rice and maize from The International Seabed Authority (ISA) is preparing
Food Corporation of India. to resume negotiations that could open the
international seabed for mining, including for
62. c materials critical for the green energy transition.
GM crops in India  The Clarion-Clipperton Fracture Zone is
located between Hawaii and Mexico in
 GM crops – A crop which has a gene the Pacific Ocean.
artificially inserted into it from another species
to give some desired properties (pest resistant,  It is the most focused area of exploration
herbicide tolerant, etc.) for Polymetallic nodules.
 Soya bean, maize, cotton, and canola are the  These rocklike deposits contain nickel,
most widely grown GM crops around the world. manganese, copper, zinc, cobalt, etc.
 In India – Bt cotton is the only GM crop that  The region is administered by the International
is allowed to be cultivated. Seabed Authority (ISA)
 It has 2 alien genes from the soil bacterium  International Seabed Authority (ISA) –
Bacillus thuringiensis (Bt) that allows the crop An autonomous UN body established in 1994
to develop a protein toxic to the pest pink under the 1982 United Nations Convention on
bollworm. the Law of the Sea (UNCLOS)
 GM Mustard is the 1st GM food crop that India  It regulates the world’s ocean floor and control
has permitted for commercial release. all mineral-resources-related activities in the
area

CHENNAI |SALEM| MADURAI | COIMBATORE DELHI | BANGALORE | THIRUVANANTHAPURAM


www.shankariasacademy.com 149
www.iasparliament.com

 In India – Gambusia was 1st introduced


during British rule in 1928 for mosquito control
and now inhabits all around India.

 Actions taken – Multiple countries, including


India have listed Gambusia as invasive species.

 The World Health Organisation (WHO)


has stopped recommending Gambusia
as a mosquito control agent in 1982.

 International Union for Conservation of Nature


(IUCN) has declared Gambusia as one of the
100 worst invasive alien species in the world.

 Guppy fish (Poecilia reticulate) is also deployed


as a mosquito predator.

65. a

Light Fidelity (Li-Fi)

The Institute of Electrical and Electronics Engineers


has formally announced 802.11bb as a standard for
light-based wireless communications, or Li-Fi.

 Li-Fi uses special LED (light-emitting diodes)


light bulbs as routers and works on optical
wireless communications (OWC) technology.

 Headquarters - Kingston, Jamaica  Li-Fi devices will deliver data through visible,
infrared, or ultraviolet light, unlike Wi-Fi
 Members – 168, includes 167 Member States routers, which use radio frequencies to transmit
and the European Union. data.

64. c  Li-Fi has a shorter range because it relies


on visible light, which does not travel far
Gambusia Fish beyond the line of sight.
 It is an insectivorous fish that is found  Wi-Fi has a broader coverage area (wide-area
mostly in fresh and brackish, and occasionally wireless connectivity) compared to Li-Fi.
marine waters and has a high breeding capacity.
 Li-Fi's speed is said to be 100 times faster than
 When released into freshwater system, they WiGig, the fastest Wi-Fi in the 60GHz
feed on mosquito larvae and thus act frequency band, which can achieve a maximum
as biocontrol agent to control mosquitoes. speed of 7GB per second.
 Commonly used Gambusia species includes  Li-Fi decreases eavesdropping and jamming
and facilitates centimeter-precision indoor
o Gambusia affinis navigation.
o Gambusia holbrooki (eastern mosquito fish)
 If a user wants to use Li-FI, they need a Li-Fi-
equipped LED bulb in the room since light
 Native to – U.S.A
cannot penetrate walls like radio waves.
 Reasons for its spread - High breeding
 Currently, developers are working to optimise
capacity, robust adaptability and high tolerance
mixed use of Wi-Fi and Li-Fi for homes and
for fluctuating environmental conditions.
offices use.
 Impact – They displace and prey on native
66. d
fauna, leading to the extinction of native fish,
amphibians, and various freshwater Anthropocene Epoch
communities.
The proposal to declare the start of the Anthropocene,
was rejected.

CHENNAI |SALEM| MADURAI | COIMBATORE DELHI | BANGALORE | THIRUVANANTHAPURAM


www.shankariasacademy.com 150
www.iasparliament.com

 It is a fundamental scale for expressing our  The present geological


Earth’s 4.6-billion-year history. time – Phanerozoic eon, Cenozoic era,
Quaternary period, Holocene epoch and
 Named & classified by – The International the Meghalayan age.
Commission on Stratigraphy (ICS) of
International Union of Geological Sciences.  Holocene began 11,700 years ago with the
most recent retreat of the great glaciers.
 International Chronostratigraphic Chart are
the basis for the units (periods, epochs and age)  Anthropocene – The Anthropocene epoch as
of the geological time scale. a term was first coined by Nobel Prize-winning
chemist Paul Crutzen and biology professor
 The International Union of Geological Eugene Stoermer in 2000.
Sciences (IUGS), founded in 1961, with 121
national members, representing over a million  It is an unofficial unit of geologic time
geoscientists, is one of the World’s largest that denotes the most recent period in
scientific organizations. which the Earth’s ecosystem has gone
through radical changes due to human
 Criteria used – It uses rigorous criteria impact, especially since the onset of the
like Earth’s rock layers, or strata, and the Industrial Revolution.
fossils found within them to decide when each
chapter started and which characteristics  There are numerous phenomena associated
defined it. with this epoch, such as global warming, sea-
level rise, ocean acidification, mass-scale soil
 5 broad categories – Eons, eras, periods, erosion, the advent of heat waves, deterioration
epochs, and ages (Larger to smaller). of the biosphere.

o Eon – The broadest category of  Anthropocene, the term has not been
geological time. formally adopted by the International
Union of Geological Sciences (IUGS), the
o Ages – The smallest category. international organization that names and
defines epochs.
 Each of these categories is further divided into
sub-categories. 67. d
o Triassic – It was the dawn of the Crawford Lake
dinosaurs.
 Crawford Lake is a small, deep lake within a
o Paleogene – It saw the rise of protected conservation area in Southern
mammals. Ontario, Canada as part of a UNESCO
Biosphere Reserve.
o Pleistocene – It included the last ice
ages.  The lake is situated in a karstic limestone
landscape and fills a sinkhole created by the
dissolution of underground rock.

 Because of the lake’s great depth (24 meters)


relative to its surface area it is known as
a meromictic lake.

 Meromictic Lake refers to a permanently


stratified body of water where the bottom layer
of water does not mix with the upper layers
because of differences in chemical composition.

 Crawford Lake and its layers of sediment


preserved the annual impact of human
activities on the Earth’s soil, atmosphere and
biology.

Batagaika Crater

 Batagaika crater is also known as the “Gateway


to the Underworld” and has the scientific name:
a mega-slump.

CHENNAI |SALEM| MADURAI | COIMBATORE DELHI | BANGALORE | THIRUVANANTHAPURAM


www.shankariasacademy.com 151
www.iasparliament.com

 It is the world’s largest permafrost crater  It connects the Aegean Sea and the Sea of
located in Russia’s Sakha Republic. Marmara.

 The crater is marked by an uneven terrain at the  It separates Asian Turkey from European
base of the depressed, which is made of Turkey and forms part of the continental
irregular surfaces and small hummocks. boundary between Asia and Europe.

 They began to form after the surrounding forest  Gibraltar Strait – It is a 36-mile-long (58
was cleared in the 1960s and the permafrost km) channel that connects the Atlantic Ocean to
underground began to melt, causing the land to the Mediterranean Sea.
sink.
 It falls beneath the Dardanelles Strait.
 It developed first as a ravine, then by thawing in
the heat of sunny days, it started to expand.  It separates Europe from Africa and is one of
the world's busiest waterways.
Parkachik Glacier
69. c
 The Parkachik glacier is one of the largest
glaciers in the Suru River valley, National Adaptation Fund for Climate Change
Kargil, Ladakh, India. (NAFCC)

 The Suru River valley is a part of the Southern  It is a Central Sector Scheme which was set up
Zanskar Ranges in the western Himalayas. in the year 2015-16.

 Reasons for melting ice are Global warming and  It was established to support adaptation
increasing temperatures in the region. activities in the States and UTs of India that are
vulnerable to the adverse effects of climate
 It is at a lower altitude than other glaciers in the change.
Zanskar region Suru River Valley, Ladakh
Himalaya.  NABARD is the National Implementing
Entity (NIE) for the fund in India.
 If the volume of water is large, there is a
possibility of glacial lake outbursts.  Projects eligible for funding – Adaptation
projects in sectors such as agriculture, animal
 The loss of a valuable water resource since husbandry, water, forestry, tourism etc.
Ladakh primarily depends on glacial melt for
water.  Eligibility – The States/UTs are to be
prepared the project proposal in consultation
68. d with NIE (NABARD) and to be approved by the
State Steering Committee on Climate Change.
Straits in News
 The grants under NAFCC released to different
 Kerch Strait – The Kerch Strait is a strait in states and Union territories have drastically
Eastern Europe that connects the Black Sea declined over the years.
and the Sea of Azov.
70. c
 It separates the Kerch Peninsula of Crimea and
Russia. UN’s Multidimensional Poverty Index

 It is located in the north part of the black  It is an international index released annually by
sea relative to the Bosphorus Strait. United Nations.

 Bosphorus Strait – Also known as the Strait  The index reflects the multiple deprivations
of Istanbul, is a 19-mile-long (30 km) waterway that poor people face in the areas of education,
that connects the Black Sea to the Sea of health, and living standards.
Marmara.
 Poverty is not an indicator in the index.
 It is located in the southern part of the
black sea relative to the Kerch Strait. Indicators Declining Percentage

 Dardanelles Strait – Also known as the Nutrition indicator From 44.3% to 11.8%
Strait of Gallipoli and the Hellespont, is a 61-
kilometer-long waterway in northwestern Child mortality From 4.5% to 1.5%
Turkey beneath the Bosphorus Strait.

CHENNAI |SALEM| MADURAI | COIMBATORE DELHI | BANGALORE | THIRUVANANTHAPURAM


www.shankariasacademy.com 152
www.iasparliament.com

Cooking fuel From 52.9% to 13.9%  Among all countries that ranked the drugs
leading to drug use disorders,
Sanitation From 50.4% to 11.3%
o The majority 46% of countries reported
Drinking water From 16.4% to 2.7% cannabis in first place

Electricity From 29% to 2.1% o 31% of countries reported opioids in


first place, mainly heroin.
Housing From 44.9% to 13.6%
o Amphetamine-type stimulants, in
particular methamphetamine, were
 Released by – UN Development reported in first place by 13% of
Programme (UNDP) and the Oxford countries.
Poverty and Human Development
Initiative (OPHI) since 2010.  As per the report, the ranking in each country
was determined mainly by 2 factors
 Report Findings – India - According to it, — prevalence of use and dependence potential.
India has pulled 415 million Indians out of
poverty between 2005-06 and 2019-21 with  The report also found that the cocaine demand
incidence falling from 55.1% to 16.4%. and supply has also risen worldwide and
methamphetamine trafficking has also
 According to the report, 1.1 billion out of 6.1 occurred mainly in Afghanistan.
billion people live in acute multidimensional
poverty across 110 countries.  India – According to the report, in 2018
alone, India had the highest prevalence of
 Sub-Saharan Africa and South Asia are home to opioid use in South Asia at 2.1%.
approximately five out of every six poor people.
 The report states that the opioid use in India is
mainly a male phenomenon with more than
95% of all opioid users being males.

 The opioid users are among the population


aged 10-75, the overall rates of opioid users
range between 0.2 and 25.2%.

 The highest prevalence of people with opioid


use disorders in India is observed in the
eastern parts of the country.

 The largest numbers of people with opioid use


disorders are found in north-western India
(Uttar Pradesh, Punjab and Haryana) as well as
in central-western States (Maharashtra and
Madhya Pradesh).

 The high level now also found in Maharashtra


appears linked to the increasing quantities of
opiates being trafficked to India from
Southwest Asia by sea.

72. b
71. a
Paris Club
World Drug Report, 2023
 The Paris Club, started in 1956 is an informal
 The World Drug Report, 2023 was released by group that helps debtor countries to
the UN Office on Drugs and Crime (ONDCs). manage their debt problems.
 Global Scenario – The number of people  Established – 1956
globally abusing drugs rose by 23% between
2011 and 2021.  Headquarters – Paris, France
 Cannabis continues to be the most used  The Paris Club only deals with debt between
drug in the world. governments and does not handle any private
or individual loan repayment.

CHENNAI |SALEM| MADURAI | COIMBATORE DELHI | BANGALORE | THIRUVANANTHAPURAM


www.shankariasacademy.com 153
www.iasparliament.com

 Members - Mostly made up of Western  ESKAPE Pathogens doesn’t include E.


nations coli bacteria and Salmonella.

 are also members of the Organisation for  ESKAPE are able to resist multiple drugs and
Economic Co-operation and Development exhibit physical characteristics that E. coli does
(OECD), a forum where 37 democracies with not, such as the ability to remove their cell wall
market-based economies collaborate and and evade certain drugs.
develop standards and policies to promote
sustainable growth.  They pose a massive global health threat
because they can quickly evolve traits that allow
 The Club has 22 permanent members. (India them to evade immune systems and available
is not a member). treatments.

 All the members are also the members of  ESKAPE are the leading cause of nosocomial
Organisation for Economic Co-operation and infections throughout the world.
Development (OECD).
 Nosocomial infections are infections acquired
73. c during the process of receiving health care that
was not present during the time of admission.
Yago and Zoe
75. b
 Yago and Zoe are recent heatwaves in
Seville, Spain of Europe. Performance Grading Index

 Seville is among the first cities in the world to  Performance Grading Index (PGI) measures
name and rank heatwaves. the performance of states/UTs in school
education.
 Seville’s system to rank heatwaves are based on
criteria such as humidity, impacts on human  It was launched in 2017-18 and so far, has been
health, and daytime and night time released up to the year 2020-21.
temperatures.
 PGI is released by the Ministry of
 India has no such naming system for Education.
heatwaves, although it does suffer massively
from heatwaves.  PGI 2.0 – To align with the National
Education Policy, 2020, and to monitor
 The countries such as Greece, Chile and United indicators relating to Goal 4 of SDG, and to
States has adopted a law to establish a heat replace existing indicators which have achieved
wave ranking system. optimal target, the PGI has been revised and
renamed as PGI 2.0.
 The World Meteorological Organization
(WMO) has recognized the interest in  The PGI 2.0 is completely aligned with Unified
developing heatwave ranking and naming District Information System for Education Plus
systems.

74. b

ESKAPE Pathogens

 ESKAPE pathogens, a group of bacteria that are


highly resistant to antibiotics.

 The ESKAPE pathogens includes

1. Enterococcus faecium

2. Staphylococcus aureus

3. Klebsiella pneumoniae

4. Acinetobacter baumannii

5. Pseudomonas aeruginosa

6. Enterobacter species

CHENNAI |SALEM| MADURAI | COIMBATORE DELHI | BANGALORE | THIRUVANANTHAPURAM


www.shankariasacademy.com 154
www.iasparliament.com

(UDISE +), National Achievement Survey  The production of coal, Petroleum Refinery
(NAS), PM POSHAN portal, PRABAND portal Products, Fertilizers, Steel and Cement
and Vidyanjali Portal data Outcomes. increased while Crude Oil, Natural Gas and
Electricity declined.
 PGI-D grades the State/UT into 10
grades – Daksh, Utkarsh, Ati-Uttam, Uttam,  Electricity has most weightage while the
Prachesta-1, Prachesta-2, Prachesta-3, Fertilizer has least weightage in the ICI.
Akanshi-1, Akanshi-2, Akanshi-3.
77. b
 Highest achievable Grade – Daksh
(Districts scoring more than 90% of the total Anti-defection law
points in that category or overall).
Ajit Pawar being sworn in as the Deputy CM of
 Lowest grade – Akanshi-3 (scores upto 10% Maharashtra after the intraparty dissent in NCP in
of the total points) Maharashtra.

 None of the States/UTs has attained the  Anti-defection law – It punishes individual
highest Grade i.e., Daksh. MPs/MLAs for leaving one party for another.

 The top-most grade attained in PGI 2.0 is  It was added by 52nd Constitutional
Prachesta – By Punjab and Chandigarh. Amendment Act as the Tenth Schedule in
1985.
76. b
 It was a response to the toppling of multiple
Index of Eight Core Industries (ICI) state governments by party-hopping MLAs
after the general elections of 1967.
 The Index of Eight Core Industries (ICI),
which measures the combined and individual  Deciding authority – The Presiding
performance of 8 key sectors in the Indian Officer of the House is empowered to make
economy. rules to give effect to the provisions of the 10th
schedule.
 ICI is released by the Office of the Economic
Adviser (OEA), Department of Industrial Policy o Speaker in Lok Sabha, Chairman in
and Promotion (DIPP), Ministry of Commerce Rajya Sabha
and Industry.
 Role of judiciary – The decision can be
challenged before the higher judiciary.

 Exemptions – It allows a group of two-third


MP/MLAs to join (merger) another political
party without inviting the penalty for defection.

 If a person is elected as the Speaker of Lok


Sabha or the Chairman of Rajya Sabha, then he
could resign from his party and re-join the party
once he demits the post.

 91st Amendment Act, 2005 – It deleted the


exemption from disqualification in case of split
by 1/3rd members of legislature party.

 Supreme Court – It laid down the ‘Three


test formula’ in 1971 for determining which
faction is to be recognised as the original
political party by the Election Commission.
 The Automobile and Textiles sectors are
not included in the Index of Eight Core o Aims and objects of the party
Industries (ICI).
o Its affairs as per the party’s constitution
 These 8 core industries accounts for 40.27 % of that reflect inner party democracy
the items included in the Index of Industrial
o Majority in the legislative and
Production (IIP).
organisation wings
 Cement production, with a weightage of 5.37 %,
saw the highest growth rate of 15.5 %.

CHENNAI |SALEM| MADURAI | COIMBATORE DELHI | BANGALORE | THIRUVANANTHAPURAM


www.shankariasacademy.com 155
www.iasparliament.com

 Provisions of the Anti-Defection law does not The joint statement 25th Anniversary of the Strategic
apply to the Presidential Elections in Partnership between France and India: towards a
India. Century of Indo-French Relations was released.

 Members of the Electoral college can vote  It sets the course for the France-India
according to their wish and are not bound by bilateral relationship in all areas till
any party whips. 2047.

 The voting is by secret ballot. Therefore, party  The year 2047 will mark 100 years of India’s
whip does not apply in this election. Independence, 100 years of diplomatic
relations between the two countries, and 50
78. c years of the Indo-French strategic partnership.

Enforcement directorate (ED)  Focus on 3 pillar - ‘security, planet and people’


guiding the bilateral ties for the next 25 years.
There is a spike in the activity of the Enforcement
Directorate (ED) in Tamil Nadu which can be seen in  It establishes a strong mechanism to strengthen
the arrest and investigation of two Ministers from the cooperation in the areas of defence, space, civil
state. nuclear, digital technology, counter-terrorism
and the blue economy.
 It is a premier financial investigation agency
and economic law enforcement agency of the  Rupee-Rouble trade arrangement – An
Government of India. alternative payment mechanism to settle dues
in rupees instead of dollars or euros.
 Headquarter – New Delhi
 It was 1st conceived in 1953 under the Indo-
 History – In 1956, an ‘Enforcement Unit’ Soviet trade agreement.
was formed in the Department of Economic
Affairs and was renamed as ‘Enforcement  New Horizons mission – It is NASA's 1st
Directorate’ in 1957. venture in the New Frontiers programme, was
launched in 2006.
 In 1960, its administrative control was
transferred from the Department of Economic  In 2015, it achieved a ground-breaking
Affairs to the Department of Revenue. milestone by capturing the first close-up image
of Pluto and its moons.
 It was regulated under Foreign Exchange
Regulation Act (FERA) of 1973 (repealed  It reshaped our understanding of distant
later). celestial objects by expanding the boundaries of
space exploration.
 It currently draws its statutory powers
from 3 different acts  Comprehensive and Progressive
Agreement for Trans-Pacific
o Foreign Exchange Management Act, Partnership – A free-trade agreement (FTA)
1999 (FEMA) launched in 2018 with 11 Pacific Rim countries
(UK is the recent member).
o Prevention of Money Laundering Act,
2002 (PMLA) 80. b
o Fugitive Economic Offenders Act, 2018 SCO Summit
(FEOA)
India hosts the 23rd SCO summit in New Delhi for the
 The director is appointed in accordance with year 2023.
the provisions of the Central Vigilance
Commission Act 2003.  It is a permanent intergovernmental
international organization.
 The Centre appoints the director on
recommendation of a high-level committee  Established in – 2001 in Shanghai.
headed by the Central Vigilance Commissioner.
 Its predecessor was the mechanism of
 Functions – It is mandated with investigation the Shanghai Five (Russia, China, Tajikistan,
of offence of money laundering and violations Kazakhstan and Kyrgyzstan)
of foreign exchange laws.
 9 Member States — India, Iran,
79. d Kazakhstan, China, Kyrgyzstan, Pakistan,
Russia, Tajikistan and Uzbekistan.
Horizon 2047

CHENNAI |SALEM| MADURAI | COIMBATORE DELHI | BANGALORE | THIRUVANANTHAPURAM


www.shankariasacademy.com 156
www.iasparliament.com

 Turkmenistan is not a member of SCO.  Launched in – 2006, as BRIC with Brazil,


Russia, India and China as founding members.
 3 Observer states – Afghanistan, Belarus and
Mongolia.  BRIC turned into BRICS in 2010, with the entry
of South Africa.
 In 2022, at the Samarkand SCO Summit, the
process of raising the status of the Republic of  Saudi Arabia, Egypt, Ethiopia, Iran,
Belarus within the Organization to the level of a Argentina and the UAE were invited to join
member state has begun. BRICS.

 Internally, the SCO adheres to the "Shanghai  New members - Egypt, Ethiopia, Iran,
spirit", namely, mutual trust, mutual benefit, and the UAE officially joined the BRICS
equality, consultation, respect for diversity of grouping
civilizations and pursuit of common
development.  Argentina declined to join as full member
and Saudi Arabia is still considering the
 Externally, it upholds non-alignment, non- BRICS membership.
targeting at other countries or regions and the
principle of openness.  Chair – Chaired by member countries in
rotational basis.
 Significance – It is the world’s largest
regional organisation consisting of 40% of the  South Africa is the chair for 2023.
world’s population and 30% of global gross
domestic product (GDP).  First summit – It was held in Yekaterinburg
in 2009.
 It has been an observer in the UN General
Assembly since 2005.  In Fortaleza declaration, New Development
Bank was created in 2015 worth 100 bn dollars.
23rd SCO Summit
 BRICS Plus – It is a cooperation mechanism
 This is the 1st time India holds the launched in Xiamen summit 2017 to enlarge
presidency of the SCO summit. BRICS beyond 5 members.

 Theme – SECURE: Security, Economic  The Cape of Good Hope – It is the joint
development, Connectivity, Unity, Respect for statement of BRICS, underscores the use of
sovereignty and territorial integrity, and local currencies in international trade and
Environmental protection. financial transactions between BRICS and its
trade partners.
 New Entrant – Iran now becomes the full
member of SCO.  BRICS pay – It is a decentralised multi-
currency digital international payments system
 New Delhi Declaration – It deals that was launched in 2018.
extensively with challenge of terrorism,
extremism and separatism and calls for country
cooperation.

 India’s new Initiatives – SCO Millet Food


Festival, Film Festival, SCO Surajkund Craft
Mela, International Conference on Shared
Buddhist Heritage etc.,

 24th SCO chair is Kazakhstan and the summit


is to take place in Astana.

81. b

BRICS

15th BRICS Summit was held in Johannesburg in 2023.

 Originally conceived as BRIC by Jim O'Neill,


the grouping consisted of Brazil, Russia,
India, and China.

CHENNAI |SALEM| MADURAI | COIMBATORE DELHI | BANGALORE | THIRUVANANTHAPURAM


www.shankariasacademy.com 157
www.iasparliament.com

 Significance  Nagoya Protocol  UN Convention to


Implementation Fund Combat
o Almost 46% of the world's population (NPIF) Desertification
o 36% of global GDP  Capacity-building  Minamata
Initiative for Convention on
o 25% of world trade, measured in terms Transparency Trust Mercury
of exports. Fund (CBIT).
82. d
 It also provides the secretarial services to
Global Environment Facility (GEF) adaptation fund which was established under
the Kyoto Protocol in 2001.
Recently, the 7th assembly of Global Environment
Facility (GEF) was conducted in Vancouver, Canada.  7th GEF Assembly – It ratified the Global
Biodiversity Framework Fund, fund for
 It is a multilateral family of funds dedicated to protecting endangered species and their
confronting biodiversity loss, climate change, ecosystems globally.
and pollution, and supporting land and ocean
health  It also stressed on youth leaders learning
exchange, indigenous and local knowledge,
 It distributes more than 1 billion dollar a year GEF partnership forum, Intergenerational
on average to address inter-related fireside chat for wildfires and women’s
environmental challenges. leadership in environmental action.

 Conceived by – France in 1989.  Inclusive GEF Assembly Challenge Program –


A new funding initiative providing up to USD
 Established in – 1992. 100,000 to each of 23 winners.

 Partnership – It is a unique partnership of 18  Net Zero Nature-positive World – To support


agencies — including United Nations agencies, countries raising ambition in climate and
multilateral development banks, national nature.
entities and international NGOs.
83. a
 It includes 186 member governments as well as
civil society, Indigenous Peoples, women, and Money Bill vs Financial Bill
youth.
The Digital Personal Data Protection (DPDP) Bill
which was earlier reported as a financial bill, is now
 GEF Trustee – World Bank
categorised as a normal bill by the Ministry of
 Funding – It is provided by participating Parliamentary Affairs.
donor countries and made available to
 Finance bill – Any Bill that relates to revenue
developing countries and countries with
or expenditure.
economies in transition to meet the objectives
of international environmental conventions
 Money bill – A specific type of financial Bill,
and agreements
deals only with matters specified in Article 110.
Coverage - 5
Family of Funds  It deals with taxes, regulation of the
conventions
government’s borrowing of money, and
expenditure or receipt of money from the
 Global Environment  Convention on
Consolidated Fund of India.
Facility Trust Fund Biological Diversity
(CBD)  Procedure for passage of money bill – Article
 Global Biodiversity
109 delineates the procedure and confers an
Framework Fund  United Nations
overriding authority on the Lok Sabha.
(GBFF) Framework
Convention on  Role of Speaker – Final authority to decide
 Least Developed Climate Change
whether a bill is money bill or not.
Countries Fund (LDCF) (UNFCCC)
 His decision cannot be challenged in any court
 Special Climate Change  Stockholm
of the country.
Fund (SCCF) Convention

CHENNAI |SALEM| MADURAI | COIMBATORE DELHI | BANGALORE | THIRUVANANTHAPURAM


www.shankariasacademy.com 158
www.iasparliament.com

Features Money Bill Financial Bill The report of Rohini commission which was set up to
examine the sub-categorisation of OBCs was submitted
Constitution Article 110 Article 117 to the President.

Either by a minister  Rohini Commission – Constituted in 2017


Only by a
Introduced by or by a private under Article 340 of the Constitution.
minister
member
 Functions – To identify the respective castes or
Requires prior communities or sub-castes in the Central
recommendation Finance bill I - List of OBCs and classifying them into their
prior respective sub-categories.
Exemption- recommendation
Role of Amendments  Amitabh Kant Committee – It was
President related to Finance bill II – constituted in 2023, by Ministry of Housing
reduction or Prior and Urban Affairs.
abolition of any recommendation
tax  Functions – To examine stalled real estate
projects and to recommend ways to complete
Financial bill I – the stalled projects.
only in Lok
Only in Lok Sabha  Real estate is a state subject under 7th
Introduced in
Sabha schedule, hence it falls under the purview of
Financial bill II – respective state governments.
Either of the houses
 Justice Gita Mittal Committee – It is
Finance Bill 1 - constituted with 3 members.
General legal
issues &  Functions – To oversee various aspects beyond
provisions the investigation, such as relief, rehabilitation,
related to Article and compensation for those affected by
As mentioned in 110. the Manipur violence.
Subject matter
Article 110 Finance Bill 2 - 85. c
Provisions on the
costs of India's The Mines and Minerals (Development and
Integrated Fund but Regulation) Amendment Act, 2023
is not included in
Article 110. President gave her assent to the Mines and Minerals
(Development and Regulation) Amendment Bill, 2023.
It cannot amend
or reject which  In India – Entry 23 of List II (State list) deals
Powers of with regulation of mines with subject to the
may be accepted It can amend it
Rajya Sabha provision of List I.
or rejected by the
Lok Sabha
 Entry 54 of List I (Union List) deals with
Detaining A maximum regulation of mines under the control of Union
A maximum period as declared by Parliament by law to be
time for Rajya period of 14 days
of 6 months. expedient in the public interest.
Sabha only.

Speaker’s  As per Indian Constitution, the state


Requires Doesn’t require
endorsement governments, as the owners of
minerals can grant mineral
Joint sitting No provision yes concessions for exploration and mining but
subject to restriction of union if any.
Either assent or
President’s withhold but Assent, return or  Amendment Act – Removal of 6 minerals
assent power cannot return withhold the bill from the list of 12 atomic minerals limited to
the bill exploration by State agencies (i.e., lithium,
beryllium, niobium, titanium, tantalum and
zirconium).
84. a
 This opens up opportunities for private sector
Committees in news involvement in their exploration and mining.

CHENNAI |SALEM| MADURAI | COIMBATORE DELHI | BANGALORE | THIRUVANANTHAPURAM


www.shankariasacademy.com 159
www.iasparliament.com

 Empowering the government to exclusively  It marked the international maritime


auction mineral concessions for critical boundary of India and Sri Lanka without
minerals, including rare earth elements, consulting the Tamil Nadu State Assembly.
graphite, cobalt, lithium, nickel, phosphate,
potash and tin.

 Revenue generated from these


auctions will go to the concerned State
Governments, accelerating the pace of
auctions and production for vital industries like
space, electronics and energy transition.

 Introduction of Exploration Licences (EL) to


attract foreign direct investment (FDI) and
engage junior mining companies in exploring
deep-seated and critical minerals like gold,
platinum, rare earth elements, etc.

 While 100% FDI is allowed in mining,


there has been limited investment in these
sectors. 87. b
86. d Nuclear Fusion Vs Nuclear Fission
Katchatheevu Island Dispute Christopher Nolan’s film “Oppenheimer” has ignited
conversation around nuclear weapons.
Prime Minister Narendra Modi mentioned the island of
Katchatheevu during No Confidence debate in the  The Manhattan Project – US with the
Parliament. support of the UK and Canada, during World
War II to produce the 1st nuclear weapons.
 Geography – An uninhabited off-shore
island in the Palk Strait, formed due to  2 types of nuclear bombs – It is based on
volcanic eruptions in the 14th century. the process of nuclear reaction.
 It is no more than 1.6 km in length and slightly  Atomic bomb – It is based on the principle
over 300 m wide at its broadest point. of nuclear fission, an uncontrolled chain
reaction in which the number of neutrons and
 It lies northeast of Rameswaram (India) the number of fission reactions multiply in a
and southwest of Jaffna (Sri Lanka). geometrical progression.
 Maritime Boundary Agreement –  Nuclear fission was 1st discovered by German
Fishermen of both countries have been fishing Scientist Otto Hahn and F.Strassman in a
in each other's waters without conflict for a very Uranium nucleus.
long time.
 This releases a huge amount of energy in a very
 1974 agreement – To fix the maritime small-time interval and leads to an explosion.
boundary in the Palk Strait.
o Example – Hiroshima and Nagasaki
 Each country shall have sovereignty and bombing in 1945.
exclusive jurisdiction and control over the
waters, the islands, the continental shelf and  J. Robert Oppenheimer is called as the father of
the subsoil, falling on its own side of the atom bomb.
aforesaid boundary.
 Homi Jehangir Bhabha was called as the father
 Only navigational rights of the vessels of both of India’s nuclear program.
Sri Lanka and India over each other’s waters
have been preserved.  Hydrogen bomb – It is based on the principle
of nuclear fusion.
 1976 Agreement – Each Party shall respect
rights of navigation through its territorial sea  It is always designed to have an inbuilt atom
and exclusive economic zone in accordance bomb which creates the high temperature and
with its laws and regulations and the rules of pressure required for fusion when it explodes.
international law.

CHENNAI |SALEM| MADURAI | COIMBATORE DELHI | BANGALORE | THIRUVANANTHAPURAM


www.shankariasacademy.com 160
www.iasparliament.com

 Then, fusion takes place in the hydrogen core  Additive manufacturing processes build
and leads to the release of a very large amount objects by adding material layer by layer,
of energy in an uncontrolled manner. while subtractive manufacturing removes
material to create parts.
Nuclear
Features Nuclear Fusion
Fission  3 process – data processing, material
processing and robotic printing.
Lighter nuclei will Heavy nucleus is
join together to divided into 2  It is a process that uses computer-created
About
produce heavy fragments along design to make 3-dimensional objects layer by
nucleus with few neutrons layer.

Very high  A software like Slicer is used to break the 3D


Temperature Room
temperature (107 model into many types of 2D layers which is
requirement temperature
kelvin) then printed.

Critical mass of  The layers of a material like plastic, composites


Conditions High density and the substances or bio-materials are built up layer by layer to
required high temperature and high-speed construct objects that range in shape, size,
neutrons rigidity and colour.

To start fission  Benefits


atleast one
Need of No need of thermal o Production speed and flexibility
neutrons external neutrons neutron from
outside is o More design freedom
compulsory
o Low-cost customization
Energy released
per unit mass is Energy released o Less dependent on supply chains
Energy high, nearly 7 per unit mass is
o Reduced storage costs
times more than less
fission o More environmentally friendly
Can be controlled  Hydraulic fracturing, commonly known
No control on
Reaction as fracking, is the process of injecting water,
fusion reaction Example- Nuclear
sand, and/or chemicals into a well to break up
reactor
underground bedrock to free up oil or gas
Alpha rays, Alpha, beta reserves.
Emissions positrons, and and gamma
neutrinos radiations

Hydrogen
Example Atomic bomb
bomb

88. d

3D Printing

Recently India inaugurated its first 3D-printed post


office in Bengaluru.
89. b
 The 3D printing process was invented in the
International Atomic Energy Agency (IAEA)
1980s and was originally known as ‘rapid
prototyping’. The nuclear ‘chain reaction’ and neighbourhood
challenges led India to acquire nuclear weapons.
 Traditional manufacturing uses a subtractive
manufacturing process, where a block of  IAEA – It is world's central intergovernmental
material is gradually removed until a part is forum for scientific and technical co-operation
formed. in the nuclear field.
 On the other hand, 3D printing uses an  It is widely known as the world’s “Atoms for
additive manufacturing process. Peace and Development”.

CHENNAI |SALEM| MADURAI | COIMBATORE DELHI | BANGALORE | THIRUVANANTHAPURAM


www.shankariasacademy.com 161
www.iasparliament.com

 Established in – 1957, as an autonomous  It empowers the Parliament to ‘admit into the


organisation under UN. Union of India, or establish, new states on such
terms and conditions as it thinks fit’.
 Headquarters – Vienna International
Centre.  Thus, Article 2 grants two powers to the
Parliament
 Members – 178 member states, India is a
founding member. o the power to admit into the Union of
India new states; and
 Significance – It is the only organization
within the UN system with expertise in o the power to establish new states.
nuclear technologies.
 Article 3 - It relates to the formation of or
 Its unique specialist laboratories changes in the existing states of the Union of
help transfer knowledge and expertise to India.
Member States in areas such as human
health, food, water, industry and the  It deals with the internal re-adjustment of the
environment. territories of the constituent states of the Union
of India.
 It also serves as the global platform for
strengthening nuclear security.  Powers of Parliament under Article 3 – It
can form a completely new State by separating
 It works for the safe, secure and peaceful the territory from any State by uniting two or
uses of nuclear science and technology, more states or parts of states or by uniting any
contributing to international peace and security territory to a part of any state.
and the United Nations' Sustainable
Development Goals. o It can increase or decrease the area of
any State.
 In 2024 – It along with the government of
Belgium organised the 1st ever Nuclear Energy o It can change/alter the name of
Summit highlighting the role of nuclear energy any State.
in addressing climate change.
o It can also make changes to the
 The International Energy Agency (IEA) boundaries of a State.
was created in response to the 1973-1974
 Article 4 - It declares that amendments made
oil crisis when an oil embargo by major
under Article 2 and Articles 3 are not to be
producers pushed prices to historic levels.
considered as amendments of the Constitution
90. b under Article 368.

The resolution moved by Kerala CM was passed  This means that such laws can be passed by
unanimously to change the name of a state to Keralam. a simple majority and by the ordinary
legislative process.
 Origin of the name – The earliest epigraphic
record mentioning Kerala was Emperor Asoka’s 91. b
Rock Edict II of 257 BC.
cfDNA
 It refers to the local ruler as Keralaputra, and
 cfDNA – Cell-free DNA, refers to small DNA
also son of Chera referring to the Chera dynasty.
fragments found outside the cell in the
 Origin of the state – It became a separate bodily fluids like blood, spinal fluid, urine,
state of India on November 1, 1956. etc.

 In Malayalam, the state was referred to as  Origin – These fragments are derived from
Keralam, while in English it was Kerala. various sources, including the breakdown of
cells, the release of DNA from damaged or
 The resolution to change the name requires the dying cells, and the shedding of DNA by normal
approval from the Union Ministry of Home cells.
Affairs.
o Apoptosis – It is a programmed cell
death.
 Article 2 - It relates to the admission or
establishment of new states that are not part of o Necrosis – It is the death of body
the Union of India. tissue.

CHENNAI |SALEM| MADURAI | COIMBATORE DELHI | BANGALORE | THIRUVANANTHAPURAM


www.shankariasacademy.com 162
www.iasparliament.com

o NETosis – A program for formation of  Parties – To date, 112 States are parties to the
neutrophil extracellular traps (NETs), Convention.
which consist of modified chromatin
decorated with bactericidal proteins  Non-signatory countries – U.S., Russia,
from granules and cytoplasm. Ukraine, China, Israel, and India.

 Features – It obligates countries to destroy


existing stockpile of cluster munitions in their
possession.

 The countries are legally bound to provide


support and rehabilitation to the cluster
bomb victims in their jurisdiction.

 Responsibilities entrusted to the Secretary-


General of the UN.

o Collection and dissemination of


transparency reports by and to the States
Parties

o Facilitation of clarification of compliance

o Convening Meetings of States Parties and


 Applications of cfDNA – Its presence in Review Conferences
bodily fluids is an indication of various
processes that include physical injury,  A cluster munition – It is a weapon
inflammation, and cancer. consisting of a container or a dispenser from
which many submunitions or bomblets
 It is used in Non-invasive prenatal test are scattered over wide areas.
(NIPT), a screening test that can be
performed during pregnancy to assess the  Dud rate – Many bomblets are
risk of chromosomal abnormalities in the unreliable and fail to explode instantly
foetus. and remain dormant for years which is also
known as the dud rate.
 Circulating tumor DNA (ctDNA) – It refers
to a small subset of the cfDNA and consists of
small DNA fragments released into the
bloodstream by cancer cells.

 ctDNA can be detected in the blood of


individuals with cancer and can provide
information about the characteristics of the
cancer.

92. a

Convention on Cluster Munition (CCM)

United States has decided to send cluster munitions to


Ukraine, as a part of a new military aid package to
bolster Ukraine’s war efforts against Russia.

 It is an international treaty which is based


on the Oslo process, a series of meetings that
concluded in a treaty.

 Adopted in – 2008; Entered into force –


2010.
93. c
 Aim – To prohibit all use, production,
transfer and stockpiling of cluster Central Armed Police Forces
munitions.
Recently, Manipur Police have registered an FIR
accusing the Assam Rifles (AR) of preventing police
from doing their duty.

CHENNAI |SALEM| MADURAI | COIMBATORE DELHI | BANGALORE | THIRUVANANTHAPURAM


www.shankariasacademy.com 163
www.iasparliament.com

 The armed Police Organisation, referred to as  The Novorossiysk commercial seaport is one of
Central Armed Police Forces (CAPFs). Russia’s largest by volume and among the
biggest in Europe.
 Administrative control by – Ministry of
Home Affairs  The port is one of the country’s main naval hubs
for the Black Sea.
 Operational control by – Ministry of
Defence  Kuril Islands – The Ukraine war has given
some hope to Japanese that the Kuril Islands
CAPF forces can be wrestled out of Russia's control.

 Assam Rifles (AR) – It is the oldest Central  It is a conflicted territory between Japan
Para Military Force in India.Called and Russia.
as Sentinels of North East. Primary role- To
guard Indo-Myanmar border.  The Kuril Islands are strategically located
islands stretching between the north of Japan's
 Border Security Force (BSF) – India’s 1st Hokkaido Island and the southern tip of
line of defence. It is the world’s largest border Russia's Kamchatka Peninsula.
guarding force. It is the only force of the nation
with a distinctly defined war-time as well as  Japan, Russia and South Korea calls them in 3
peace-time role.Mandate- To guard the Indo- different names.
Pakistan and Indo-Bangladesh border.
o Japan – Northern territories
 Indo Tibetan Border Police (ITBP) –
Deployed along Indo-Tibet border o Russia – the Kuril Islands

 Sashastra Seema Bal (SSB) – Mandated to o South Korea – Dokdo islands


guard the Indo-Nepal and Indo-
Bhutan borders.  Sulina Channel – It is the new
alternative trade route for Ukraine after
 Central Industrial Security Force Russia’s drone strikes on ports and grain
(CISF) – Security to premises, staff, property storage facilities along the Danube River in
and establishments. It is the only CAPF with Ukraine.
a dedicated firefighting wing and has
the largest percentage of women in its force.  It is situated in Romania (a NATO member).

 Central Reserve Police Force (CRPF) –  It is a riverine ‘expressway’ crucial for transport
Deployed in aid of civil matters like Crowd & of goods from Ukraine (one of the world's top
Riot control, Counter Insurgency, deal with Left grain exporters) to the Black Sea.
Wing Extremism.
 Danube River is the Europe’s second longest
 National Security Guard (NSG) – Counter river which has been historically crucial for the
anti-hijack operations, rescue operations. movement of freight.

 Features – Among all, the AR, BSF, ITBF and  Its 3 channels are Chilia, Sulina and St George.
SSB are the 4 Border Guarding Forces.
95. b
94. b
National Mission for Manuscripts
 Maui Island - Recently there has been a
India has a large collection of ancient manuscripts, a
deadly wildfire in Hawaii’s Maui Island due to
part of the country’s cultural heritage, but over the
strong winds from Hurricane Dora.
years many have been lost or lie in museums abroad.
 A wildfire is an uncontrolled fire in an area of
combustible vegetation that occurs in the  It is an autonomous body.
countryside or rural area.
 Established in – 2003, by the Ministry of
 Wildfires can burn in vegetation located both in Tourism and Culture
and above the soil.
 Aim – To unearth and preserve the vast
manuscript wealth of India.
 Novorossiysk Port – A Russian naval
ship got damaged in a Ukrainian naval
 It has the mandate of identifying, documenting,
drone attack in the Novorossiysk Port in
conserving and making accessible the
the Black Sea.
manuscript heritage of India.

CHENNAI |SALEM| MADURAI | COIMBATORE DELHI | BANGALORE | THIRUVANANTHAPURAM


www.shankariasacademy.com 164
www.iasparliament.com

 Motto – Conserving the past for the 97. c


future.
Sangam age sites
 Significance – It has emerged as a
movement, undoubtedly the most popular A portion of a circular brick structure has been
and effective among all the heritage unearthed by archaeologists at Porpanaikottai.
conservation initiatives in the country.
 Sangam Age – It is a period that is dated from
 India possesses an estimate of ten million 300 BCE to 300 CE.
manuscripts, probably the largest collection in
the world.  Previous excavation of Sangam sites

 A manuscript is a handwritten composition on o Keeladi


paper, bark, cloth, metal, palm leaf or any other
o Sivagalai
material dating back at least 75 years that has
significant scientific, historical or aesthetic o Adichanallur
value.
o Kodumanal
 Lithographs and printed volumes are not
manuscripts.  Porpanaikottai – It is a sangam age site,
situated at Pudukottai, Tamilnadu.
96. d
 Light Detection and Ranging (LIDAR), a
Mrityu Bahini
remote sensing method, indicated that a fort
On the eve of 77th Independence Day, President had existed there.
Murmu pays tribute to Matangini Hazra, Kanaklata
 Findings – Grave goods, conch bangles,
Barua.
fragments of red and black pots, of pots
engraved with Tamili, iron fragments, glass
 Mrityu Bahini – It was a death/suicide
beads and weapons made of bones.
squad prepared to sacrifice their lives
while attempting to hoist the national
 A hero stone mentioning the name of one
flag atop police stations.
Kanangkumaran who hailed from Adhavanur
 Kanaklata Barua – She is one the youngest was found at the site.
martyrs, who died at the age of 17 in the Quit
 Tirunavaya – Pipeline work leads to
India Movement.
vast megalithic site in Kerala.
 She joined the Mrityu Bahini and was made the
 Hat stones – They are popularly called
leader of the women cadres of the Bahini.
Thoppikkallu in Malayalam, are hemispherical
 She unfurled the Tricolour at Gohpur police laterite stones used as lid on burial urns during
station in 1942 and when police did not let them the megalithic period.
move forward, an altercation led to firing,
 Ashes were found in the pots recovered from
killing Barua.
the chamber as well as from under the hat
 In 2020, the Coast Guard named a Fast Patrol stones.
Vessel (FPV) after her, the ICGS Kanaklata
98. a
Barua.
Interpol Global Academy Network (IGAN)
 Matangini Hazra – A revolutionary leader in
India's struggle for independence who earned Central Bureau of Investigation (CBI) Academy joins
the name “Gandhi Buri” (the old Gandhian the Interpol Global Academy Network.
woman).
 Launched in – 2019
 She led a march during the Quit India
movement of 1942, to take over the Tamluk  Aim – To support Interpol in leading a
police station from British authorities and fell global approach to Law Enforcement
to the british bullets at the age of 73. Training.
 She was the 1st woman revolutionary to have  It is a network of trusted law enforcement
her statue erected in the Kolkata Maidan in education institutions, both regional and
1977. national in scope.

CHENNAI |SALEM| MADURAI | COIMBATORE DELHI | BANGALORE | THIRUVANANTHAPURAM


www.shankariasacademy.com 165
www.iasparliament.com

 Members support the creation and delivery of  Benefits – Vishwakarmas will be provided
select digital and face-to-face training courses, recognition through PM Vishwakarma
covering INTERPOL’s tools and services, crime certificate and ID card.
areas, and other law enforcement topics.
 Collateral free 'Enterprise Development Loans'
 CBI Academy – CBI Academy has become of upto Rs. 3 lakh in two tranches of Rs. 1
the 10th Member of the Interpol Global lakh and Rs. 2 lakh, at a concessional rate of
Academy Network. interest fixed at 5%, with Government of India
subvention to the extent of 8%.
 It is situated in district Ghaziabad, Uttar
Pradesh.  It will further provide skill upgradation, toolkit
incentive, incentive for digital transactions and
 It is the focal point of training activities within marketing support.
the organization.
 Participants will get a stipend of Rs.500 per day
 It is responsible for identification and while undergoing training.
conducting suitable training programmes,
regulation of nominations of trainees and  Beneficiaries will also receive up to Rs. 15,000
preparation of the Annual Training Calendar. to buy modern tools.

 It conducts on-line training programmes for 100. a


personnel from CBI and other central and local
law enforcement agencies. PM PRANAM Scheme

99. d  PM-PRANAM – PM Programme for


Restoration, Awareness Generation,
PM Vishwakarma Yojana Nourishment, and Amelioration of Mother
Earth.
Union Cabinet has recently approved a new Central
Sector Scheme ‘PM Vishwakarma’.  Aim – To incentivize States that actively
contribute towards balanced use of
 Launched in – 2023 chemical fertilizers and to encourage
regions to take concrete steps towards
 Aim – To provide end-to-end support to sustainable agriculture & environmental
artisans and craftspeople who work with their preservation.
hands and tools.
 Budget – It has no separate budget, but
 It is a Central Sector Scheme, fully funded a 50% subsidy savings will be provided to
by the Government of India with an initial States/UTs.
outlay of Rs 13,000 crore.
 70% of the grant can be used for asset creation
 Duration – 5 years (FY 2023-24 to FY 2027- related to technological adoption of alternate
28). fertilisers and alternate fertiliser production
units.
 Nodal ministry – Ministry of Micro,
Small and Medium Enterprises  30% grant money can be used for incentivising
farmers, panchayats, farmer producer
 Implemented by organisations, self-help groups, etc.
o Ministry of Micro, Small and Medium  It has been initiated by the Union Ministry
Enterprises of Chemicals and Fertilizers.
o Ministry of Skill Development and  iFMS (Integrated fertilisers Management
Entrepreneurship System) data available in the Fertiliser Ministry
dashboard will be used for this purpose.
o Ministry of Finance
 Significance – It will help the government
 Significance – It aims at improving the
reduce subsidy bills and fiscal deficits.
quality, as well as the reach of products and
services of Vishwakarmas and to ensure that
they are integrated with the domestic and
global value chains.

 18 traditional trades will be covered in the first


instance under PM Vishwakarma.

CHENNAI |SALEM| MADURAI | COIMBATORE DELHI | BANGALORE | THIRUVANANTHAPURAM


www.shankariasacademy.com 166
www.iasparliament.com

TEST – IV
1. Consider the following statements regarding the Place 4. Bharatiya Nagarik Suraksha Sanhita Act
of Worship Act, 1991
Select the correct answer using the codes given below.
1. The religious nature of a house of worship must
a. 3, 1, 2, 4
continue to be the same as it existed on January
26, 1950. b. 1, 2, 3, 4
2. It limits converting a house of worship from one c. 3, 2, 4, 1
religious’ group into another religious group.
d. 3, 2, 1, 4
3. It does not restrict the conversion of places of
worship to different sect within the same
religious group.
4. Consider the following statements regarding
4. Any religious structure protected under the Parliamentary Committees in India
Ancient Monuments and Archaeological Sites
and Remains Act of 1958 are exempted from 1. It is a committee appointed or elected by the
this Act. House or nominated by the Speaker.
How many of the above statements is/are correct? 2. Ad hoc Committees are temporary committees
set up for a specific purpose and are dissolved
a. Only one once that purpose has been served
b. Only two Which of the above statement(s) is/are incorrect?
c. Only three a. 1 only
d. All four b. 2 only

c. Both 1 and 2
2. Which of the following are the tributaries of River d. Neither 1 nor 2
Cauvery?

1. Kabini
5. Consider the following pairs:
2. Penganga
Boards Headquarters
3. Achankoil
1. Tea Board – Kolkata
4. Hemavati
2. Tobacco Board – Guntur
5. Arkavathy
3. Coffee Board – Bengaluru
Select the correct answer using the codes given below:
4. Spices Board – Kochi
a. Only two
5. Rubber Board – Kottayam
b. Only three
How many of the pairs given above are correctly
c. Only four matched?
d. All five a. Only two

b. Only three
3. Arrange the following events with respect to criminal c. Only four
reforms in India in chronological order:
d. All five
1. Ranbir Singh committee

2. Justice Verma panel


6. Which of the following statements regarding
3. Malimath committee Vyommitra is/are correct?

CHENNAI |SALEM| MADURAI | COIMBATORE DELHI | BANGALORE | THIRUVANANTHAPURAM


www.shankariasacademy.com 167
www.iasparliament.com

1. It is an AI enabled tool to operate the nuclear c. Facilitating the active involvement of Indigenous
enabled missile launcher. Peoples in decision-making and public policy
formulation.
2. It has been designed, developed and integrated
indigenously by the Defence Research and d. Establishing guidelines for determining death
Development Organization (DRDO). based on clinical judgment, with supplemental
diagnostic aids if required.
Select the answer using the code given below:

a. 1 only
10. Consider the following statements with respect to
b. 2 only Economic Community of West African States
(ECOWAS)
c. Both 1 and 2
1. The Pelindaba Treaty is associated with the
d. Neither 1 nor 2 establishment of ECOWAS.

2. Niger, Burkina Faso, and Mali have announced


their withdrawal from the ECOWAS grouping.
7. Namoh 108, sometimes seen in news, is related to?
3. The Alliance of Sahel States is a subgroup
a. Unified Ambulance service to all poor citizens within the ECOWAS grouping.
across India.
How many of the statement(s) given above is/are
b. Instant online services delivery mechanism for correct?
emergency situations.
a. Only one
c. Newly developed Gene-edited mustard using
CRISPR/Cas9 technology. b. Only two
d. None of the above c. All three

d. None
8. Pradhan Mantri Uchchatar Shiksha Abhiyan (PM-
USHA) Scheme, sometime seen in the news, is
associated with? 11. Consider the following statements with respect to
Unclaimed deposits
a. To improve education quality through curricular
& programme changes, teacher training, physical 1. Balances in both the savings as well as current
and digital infrastructure. accounts which are not operated for 10 years
are classified as Unclaimed Deposits.
b. To give direct admission to the deserving foreign
nationals in the National Institutes of Technology 2. Unclaimed deposits are transferred to the
and other centrally funded institutes (other than Depositor Education and Awareness (DEA)
IITs). Fund maintained by the Ministry of Finance.

c. To award scholarships to meritorious students of 3. DAKSH is a portal maintained by the Reserve


economically weaker sections to arrest their drop Bank of India to claim the unclaimed deposits.
out at class VIII and encourage them to continue
their education at secondary stage. How many of the statement(s) given above is/are
correct?
d. To nurture a spirit of inquiry and creativity, love
for Science and Mathematics and effective use of a. Only one
technology amongst children.
b. Only two

c. All three
9. Belem Declaration, sometimes seen in the news, is
related to? d. None

a. Promoting Sustainable and Inclusive Urban


Design and Architecture.
12. Maya OS is a locally built Operating System (OS)
b. Ensuring the protection of civil and religious developed by?
rights for Palestinian Arabs.
1. CERT-In

CHENNAI |SALEM| MADURAI | COIMBATORE DELHI | BANGALORE | THIRUVANANTHAPURAM


www.shankariasacademy.com 168
www.iasparliament.com

2. National Informatics Centre 15. Consider the following statements with respect to
Convention on International Liability for Damage
3. Centre for Development of Advanced Caused by Space Objects.
Computing
1. The convention applies to damage caused by
4. Defence Research and Development space objects to other space assets, and damage
Organisation caused by falling objects on earth.

Select the correct answer using the codes given below: 2. The launching country is absolutely liable to
pay compensation for any damage caused by its
a. All except 1 space object on the earth.
b. All except 4 Which of the following statement(s) is/are correct?
c. All except 2 and 3 a. 1 only
d. All four b. 2 only

c. Both 1 and 2
13. Consider the following: d. Neither 1 nor 2
1. UK

2. India 16. Consider the following countries.


3. Australia 1. Myanmar
4. Indonesia 2. Bangladesh
Which of the countries given above is/are not the 3. Bhutan
permanent partners of Exercise Malabar?
4. Thailand
a. All except 4
India has signed the Trilateral Highway Project with
b. All except 3 and 4 which of the above countries?
c. All except 1 and 4 a. 1 and 2 only
d. None b. 1 and 3 only

c. 1 and 4 only
14. Match the following: d. 2 and 3 only
GI Products Category States

1. Jalesar Dhatu p. Uttar 17. Consider the following statements with respect to
a. Metal craft
Shilp Pradesh the PM Young Achievers Scholarship Award Scheme for
Vibrant India (PM-YASASVI)
2. Jaderi
b. Clay sticks q. Tamil Nadu
namakatti 1. The scheme covers students of vulnerable
groups comprising of Scheduled Castes (SCs)
3. Koftgari Metal c. Ornamental and Scheduled Tribes (STs).
r. Rajasthan
Craft Weaponry
2. It is implemented by the Ministry of Social
Select the correct answer using the codes given below: Justice and Empowerment.

a. 1-a-p; 2-b-q; 3-c-r Which of the following statement(s) is/are correct?

b. 1-b-r; 2-a-p; 3-c-q a. 1 only

c. 1- c-p; 2-a-r; 3-b-q b. 2 only

d. 1-b-q; 2-c-r; 3- a-p c. Both 1 and 2

d. Neither 1 nor 2

CHENNAI |SALEM| MADURAI | COIMBATORE DELHI | BANGALORE | THIRUVANANTHAPURAM


www.shankariasacademy.com 169
www.iasparliament.com

18. The terms “LockBit, Akira and Luna Moth”, 2. It resulted in adoption of New Delhi
sometimes seen in the news, are related to? Declaration, reflecting India's motto of ‘Sabka
Saath, Sabka Vikas, Sabka Vishwas’.
a. Variants of ransomware
3. African Union was formally made a permanent
b. Lunar mission by various countries member of the G20, on par with the European
Union.
c. New variants of African swine fever
How many of the above statements is/are correct?
d. Lepidopteran insects discovered in Western
Ghats a. Only one

b. Only two
19. Consider the following pairs: c. All three
Places in news Countries d. None

1. Canary Islands – Greece

2. Kemer – Turkey 22. The Nari Shakti Vandan Adhiniyam 2023 that was
cleared by both houses of Parliament proposes 33%
3. Islands of Rhodes – Spain reservation for women in which of the following?
4. Latakia – Syria 1. Lok Sabha
How many of the pairs given above are correct? 2. State Legislative Assembly
a. Only one 3. Legislative Assembly of National Capital
Territory of New Delhi
b. Only two
4. Legislative Assembly of Union Territory of
c. Only three Puducherry
d. All four 5. Legislative Assembly of Union Territory of
Jammu & Kashmir

Select the answer using the code given below:


20. Consider the following pairs:
a. All except 5
Portals/Apps Purposes
b. All except 4 and 5
1. MASI Portal – To report illegal coal mining
c. All except 3, 4 & 5
2. Flood-watch – Information on flood situation
d. All
3. Khanan Prahari – To monitor child care institutions

How many of the pairs given above are correctly


matched? 23. Consider the following pairs with respect to Major
Rock Edicts of Ashoka.
a. Only one
Rock Edicts Mentions about
b. Only two
1. I – Prohibition of animal slaughter
c. All three
2. II – South Indian Kingdoms
d. None
3. VII – Religious tolerance

4. XIII – Kalinga War


21. Consider the following statement with respect to
G20 2023 Summit. How many of the above pairs are correct?
1. The theme of G20 Summit 2023, ‘One Earth a. Only one
One Family One Future’ was drawn from
Mundaka Upanishad. b. Only two

CHENNAI |SALEM| MADURAI | COIMBATORE DELHI | BANGALORE | THIRUVANANTHAPURAM


www.shankariasacademy.com 170
www.iasparliament.com

c. Only three c. Both 1 and 2

d. All four d. Neither 1 nor 2

24. Sacred Ensembles of the Hoysalas built in the 12th 27. Consider the following pairs.
to 13th centuries have been inscribed in the UNESCO's
World Heritage List. Which of the following temples Contention Area Disputed parties
form a part of this Hoysala-style temple complex?
1. Paracel Island – China & Philippines
Kesava Temple
2. Spratly island – China & Malaysia
Virupaksha temple
3. Scarborough Shoal – China & Vietnam
Hoysaleswara temple
How many of the above pairs are correct with respect
Amruteshwara temple South China Sea Dispute?

Chennakeshava temple a. Only one

Select the answer using the code given below: b. Only two

a. 1 and 2 only c. All three

b. 3, 4 and 5 only d. None

c. 1, 3 and 5 only

d. 1, 2, 3, 4 and 5 28. Which of the following is incorrect with respect to


Non fungible Tokens (NFTs)?

a. It is an irrevocable digital certificate of


25. With respect to the special session of the Parliament, ownership for a given asset, whether digital or
which one of following is/are incorrect? physical.

a. The term ‘Special session’ is mentioned in the b. It can be created through a process called
Constitution of India. minting.

b. The procedural devices like question hour would c. It is easy to verify ownership and reassign the
not be available to Member of Parliament during token to a new owner.
the session.
d. It can be exchanged with tokens of same type.
c. The President, who summons a regular
Parliamentary session will also summon this
session.
29. Air Quality Life Index, which reports about the
d. The ‘proper special sessions’ happens with impact of air pollution on life expectancy, is released by
debates or discussions, while ‘Midnight sessions’ which one of the following?
happens without any debates.
a. Central Pollution Control Board

b. Energy Policy Institute, University of Chicago


26. Consider the following with respect to Abraham
Accords. c. IQAir, a Swiss air quality technology company

1. It aims to normalise ties between the Arab d. World Health Organisation


states and Palestine.

2. Signatories include Egypt, Jordan, UAE,


Morocco, Sudan and Saudi Arabia. 30. Consider the following statements regarding India’s
Space Policy 2023.
Which of the above statement(s) is/are incorrect?
1. It laid down the regulations of privatizing space
a. 1 only missions in India.

b. 2 only 2. As per the policy, ISRO will move out of


manufacturing space systems.

CHENNAI |SALEM| MADURAI | COIMBATORE DELHI | BANGALORE | THIRUVANANTHAPURAM


www.shankariasacademy.com 171
www.iasparliament.com

3. Indian National Space Promotion and b. 2 only


Authorization Centre (IN-SPACe) will handle
manufacturing and operation of space systems. c. Both 1 and 2

4. New Space India Limited (NSIL) will handle d. Neither 1 nor 2


transfer of technology requests, in order to give
private space firms access to already
established ISRO-owned space infrastructure.
34. Consider the following.
How many of the above statements is/are correct?
1. Green Rating for Integrated Habitat
a. Only one Assessment (GRIHA)

b. Only two 2. Simple Versatile Affordable GRIHA (SVA


GRIHA)
c. Only three
3. Energy Conservation Building Code (ECBC)
d. All four
4. Leadership in Energy and Environmental
Design (LEED)

31. Consider the following pairs Which of the above building rating systems has been
developed by The Energy and Resources Institute
Statues in India Associated Personality (TERI)?

1. Statue of Equality – Dr. B. R. Ambedkar a. 1 only

2. Statue of Oneness – Sri Ramanujacharya b. 1 and 2 only

3. Statue of Unity – Sardar Vallabhai Patel c. 1, 2 and 3 only

4. Statue of Valour – Adi Shankaracharya d. 1 and 4 only

How many of the above pairs are correct?

a. Only one 35. Which of the following best describes the aim of
Beijing convention?
b. Only two
a. Gender equality and women's empowerment
c. Only three
b. Control drug trafficking among South-East
d. All four Asian countries

c. Promotion of legal certainty in judicial sale of


ships
32. Which of the following best describes the term ‘Erg
Chech 002’? d. Promotion of exploration, development and
utilization of geothermal energy
a. It is an ancient andesite meteorite.

b. It is a fast-growing exotic mangrove species.


36. Consider the following statements.
c. It is an exoplanet roughly the same size as Earth.
1. Bengaluru is called as the Coffee capital of
d. It is a red dwarf star, nearest star to the Sun. India.

2. India is a member of International Coffee


Organisation.
33. Consider the following statements.
3. The Coffee Board of India is managed by the
1. Warm surface temperatures over the Antarctic Ministry of Commerce and Industry.
2. Cold surface temperatures over Eurasia 4. India hosted the 2023 edition of World Coffee
Conference for the first time.
Which of the above statement(s) characterises the
WACE Pattern? How many of the above statement (s) is/ are incorrect?
a. 1 only a. Only two

CHENNAI |SALEM| MADURAI | COIMBATORE DELHI | BANGALORE | THIRUVANANTHAPURAM


www.shankariasacademy.com 172
www.iasparliament.com

b. Only three b. Exploration for atmospheric gases linked to life


in the Moon.
c. All four
c. Exploration to search for past life and collect
d. None samples from Mars.

d. Exploration of a metallic asteroid that is located


between Mars and Jupiter.
37. Which Article of the Indian Constitution was
adopted as a part of the Munshi-Ayyangar formula?

a. Article 343 41. Consider the following.

b. Article 348 Products Regions

c. Article 346 1. Coffee - Arakku Valley

d. Article 351 2. Bandhakala Ikkat - Odisha

3. Zighrana Attar - Uttar Pradesh


38. Consider the following statements with respect to 4. Pashmina - Jammu and Kashmir
UPAg Portal.
How many of the above pairs is/are correctly matched?
1. It is an initiative of the Ministry of Electronics
and Information Technology. a. 1 only

2. It is a crucial component of the Digital Public b. 2 only


Infrastructure for Agriculture.
c. 3 only
Which of the above statement(s) is/are incorrect?
d. All 4
a. 1 only

b. 2 only
42. Which of the following statements regarding
c. Both 1 and 2 Hoysalas is/are incorrect?

d. Neither 1 nor 2 1. Initially, they were provincial governors under


Chalukyas of Badami.

2. Vishnuvardhan and Veera Ballala are


39. Consider the following statements with respect to important rulers of this dynasty.
Mithun.
Select the answer using the code given below:
1. It is endemic to India.
a. 1 only
2. It is the State animal of Manipur.
b. 2 only
3. It is the only animal that has been bestowed
with the food animal tag by the FSSAI. c. Both 1 and 2

Which of the above statement(s) is/are incorrect? d. Neither 1 nor 2

a. 1 and 2 only

b. 2 and 3 only 43. Which of following operations are correct with


respect to India’s military operations against Khalistani
c. 1 and 3 only movement?

d. 1, 2 and 3 1. Operation Blue Star

2. Operation Shop
40. Psyche Mission, sometimes seen in the news is 3. Operation Woodrose
associated with?
4. Operation Black Thunder
a. Exploration of Jupiter and three of its icy moons.

CHENNAI |SALEM| MADURAI | COIMBATORE DELHI | BANGALORE | THIRUVANANTHAPURAM


www.shankariasacademy.com 173
www.iasparliament.com

5. Operation Polo c. Only three

Select the correct answer using the codes given below: d. All four

a. All except 4 & 5

b. All except 3 & 5 47. Consider the following statements regarding Central
Empowered Committee.
c. All except 4
1. It was constituted by the Supreme Court of
d. All except 5 India.

2. It serves as a watchdog for issues pertaining to


environmental conservation.
44. Consider the following countries.
3. The committee can have non-governmental
1. Russia members.
2. India Which of the above statements is/are correct?
3. China a. 2 only
4. Australia b. 3 only
5. United States of America c. 1 and 2 only
How many of the above countries are members of the d. 1, 2 and 3
East Asia Summit?

a. All except 3, 4 and 5


48. Consider the following countries.
b. All except 4 and 5
1. Mali
c. All except 5
2. Chad
d. All
3. Niger

4. Mauritania
45. The terms “Nuakhai”, “Soulung” and “Gudi Padwa”
seen in news are related to? 5. Burkina Faso

a. Forms of puppetry Liptako-Gourma Charter, a mutual defence pact was


signed by how many of the above countries?
b. Types of harvest festival
a. Only two
c. Endangered animals of North-East India
b. Only three
d. GI Tagged products of Arunachal Pradesh
c. Only four

d. All five
46. How many of the following are Chola era temples of
Tamil Nadu?

1. The Thillai Nataraja Temple 49. Which of the following statements best describes
“Global Security Initiative (GSI)”?
2. The Uma Maheswarar Temple
a. An Ukraine led initiative to strengthen its ties
3. The Brihadeeswara Temple with NATO.
4. The Naganathaswamy Temple b. A China-led framework to restore the stability
and security in Asia.
Select the answer using the code given below:
c. A maritime security alliance formed by Indian
a. Only one Ocean Rim Association.
b. Only two

CHENNAI |SALEM| MADURAI | COIMBATORE DELHI | BANGALORE | THIRUVANANTHAPURAM


www.shankariasacademy.com 174
www.iasparliament.com

d. An intelligence-sharing alliance formed during 3. India is not a member of the organisation.


the times of Second World War.
How many of the statements given above are correct?

a. Only one
50. Which of the following is the first Moon-landing
attempt made by Japan Aerospace Exploration Agency b. Only two
(JAXA)?
c. All Three
a. SLIM
d. None
b. Luna 25

c. CAPSTONE
54. ciTRAN, sometimes seen in the news is related to?
d. Lunar Ice Cube
a. A type of virus that affects the citrus trees.

b. An open-source AI-powered translation tool.


51. Which of the following statements regarding the
Daivaradhane tradition is true? c. A financial inclusion scheme for transgender
people.
a. It is a dance form performed during the harvest
festival of Bigu. d. A circular RNA that helps copy the genetic
material of HIV-1.
b. It is a tradition for conservation of scared groves
in Chattisgarh.

c. It refers to rituals performed by singing hymns in 55. Consider the following statements.
temples.
University Founders
d. It is an art form for worship in Tulu Nadu.
1. Nalanda - Kumaragupta

2. Vikramshila - Dharmapala
52. Consider the following statements with respect to
the State of the Rhino Report, 2023. 3. Odantapuri - Gopala

1. It is released by United Nations Environment How many of the above pairs is/are incorrect?
Programme (UNEP).
a. Only one
2. There is a decline in the population of greater
b. Only two
one-horned rhino in India.
c. All three
How many of the statements given above are correct?
d. None
a. 1 only

b. 2 only
56. Consider the following statements with respect to
c. Both 1 and 2
the Scarborough Shoal.
d. Neither 1 nor 2
1. It is a region located in the South China Sea.

2. It is a disputed area between China and Taiwan.


53. With reference to International Organisation of
3. The 38th Parallel passes through this region.
Legal Metrology (OIML), consider the following
statements. How many of the above statement(s) is/are correct?
1. It is a specialised agency under the United a. Only one
Nations to promote the global harmonisation of
legal metrology procedures. b. Only two
2. It aims to develop model regulations, standards c. All three
and related documents for use by legal
metrology authorities and industry. d. None

CHENNAI |SALEM| MADURAI | COIMBATORE DELHI | BANGALORE | THIRUVANANTHAPURAM


www.shankariasacademy.com 175
www.iasparliament.com

57. Consider the following statements with respect to c. Both 1 and 2


the Partnership for Global Infrastructure and
Investment (PGII). d. Neither 1 nor 2

1. It is an initiative of G7 to fund infrastructure


projects across the world.
61. Consider the following features:
2. The India Middle East Europe Economic
Corridor is one of the projects under PGII. 1. Use of chemical fertilizers and pesticides

Which of the above statement(s) is/are correct? 2. Application of modern farm machines

a. 1 only 3. Multiple cropping

b. 2 only 4. Improved credit facilities

c. Both 1 and 2 5. Support price policy

d. Neither 1 nor 2 6. Improved R&D and extension infrastructure

Which of the following are the main features of Green


Revolution in India?
58. Which of the following summits is associated with
the 'Gujarat declaration'? a. All except 3

a. Nuclear Energy Summit b. All except 4 & 5

b. Voice of Global South Summit c. All except 3 & 5

c. WHO Traditional Medicine Global Summit d. All six

d. Global Partnership on Artificial Intelligence


(GPAI) Summit
62. Which of the following is incorrect with respect to
Special Marriage Act, 1954?

59. The term “Perovskite” recently seen in news is a. Consented parties should not be within the
related to? degrees of prohibited relationship as prescribed by
their religion.
a. A naturally occurring mineral of calcium
titanate. b. Parties to an intended marriage should give
notice to the marriage officer of the district.
b. A new form of life lying between viruses and
viroids. c. When the Marriage officer refuses to give
permission to marriage, an appeal can be filed in the
c. A new class of antibiotics to tackle Acinetobacter family Court.
baumannii
d. The decision of the district court in allowing such
d. A coral monitoring and surveillance robot marriage is final.
developed by the National Institute of
Oceanography (NIO).
63. Consider the following with respect to United
Nations (UN).
60. Consider the following with respect to ASEAN-India
Summit. 1. The year 2020 marks the 75th anniversary of
the United Nations (UN).
1. India hosted the 20th edition of the Summit.
2. The title “United Nations” was adopted for the
2. The ASEAN-India Trade in Goods Agreement first time in 1941 to identify the countries that
(AITIGA) was signed in the 2023 Summit. allied against Axis powers.

Which of the above statement(s) is/are correct? 3. There are seven official languages in the United
Nations.
a. 1 only
4. World Health Organisation was created by
b. 2 only United Nations.

CHENNAI |SALEM| MADURAI | COIMBATORE DELHI | BANGALORE | THIRUVANANTHAPURAM


www.shankariasacademy.com 176
www.iasparliament.com

Which of the statement(s) given above is/are correct? d. eSanjeevani for audio visual consultation does
not come under this initiative.
a. 1 and 4 only

b. 1, 2 and 3 only
67. Consider the following
c. 1, 2 and 4 only
1. It has 6 economic corridors spanning Asia,
d. 1, 2, 3 and 4 Europe and Africa.

2. Italy is the only G7 country that signs this


initiative.
64. A recent cloud burst over Lhonak Lake resulted in a
flash flood in the Teesta River and led to the breach of 3. Asian Infrastructure Investment Bank (AIIB) is
the Chungthang dam. South Lhonak Lake is located in one of the financing mechanisms for this
which one of the following states? initiative.

a. Assam Which of the above statement(s) with respect to Belt


and Road Initiative is/are incorrect?
b. Sikkim
a. 1 and 3 only
c. West Bengal
b. 2 only
d. Arunachal Pradesh
c. 1, 2 and 3

d. 3 only
65. Consider the following pairs of Alternate Dispute
Resolution (ADR) mechanism and their meaning.

ADR Mechanism Meaning 68. Ms. Claudia Goldin was awarded the Sveriges
Riksbank Prize in Economic Sciences (Nobel Prize for
1. Mediation – Non-binding recommendations Economics) for 2023. Her work in which of the
following fields yields her the Economics Nobel Prize?
2. Conciliation – Facilitated negotiation
a. For research on banks and financial crises
3. Arbitration – Binding award b. For her contribution towards behavioural
economics
4. Negotiation – Voluntary agreement
c. For establishing the link between climate change
How many of the pair(s) given above is/are correctly and women poverty
matched?
d. For advancing our understanding of women’s
a. Only one labour market outcomes

b. Only two

c. Only three 69. Which of the following statements regarding


Environment Impact Assessment (EIA) in India
d. All four is/are not correct?

1. It is statutorily backed by the Wildlife


Protection Act of 1972.
66. Which of the following is correct with respect Tele-
MANAS initiative? 2. In certain cases, State government also has
powers to give Environmental Clearances.
a. National Institute of Mental Health and Neuro
Sciences (NIMHANS) provides the technical Select the answer using the code given below:
support.
a. 1 only
b. It is three tier mechanism involving many
trained counsellors and mental health specialists. b. 2 only

c. It is implemented via a National Technical c. Both 1 and 2


Advisory Group (NTAG).
d. Neither 1 nor 2

CHENNAI |SALEM| MADURAI | COIMBATORE DELHI | BANGALORE | THIRUVANANTHAPURAM


www.shankariasacademy.com 177
www.iasparliament.com

70. Amnesty International and Human Rights Watch 3. The Kazan Declaration pertains to the welfare
have accused the Israel Defense Forces of using white of the Afghan People.
phosphorus munitions in Gaza. Which of the following
is not correct regarding white phosphorus? How many of the above statement(s) is/are incorrect?

a. It is a pyrophoric substance which can be a. Only one


extinguished when exposed to oxygen.
b. Only two
b. It emits a distinct garlic-like odour and produces
thick, light smoke. c. All three

c. It acts as smokescreen and helps in hiding d. None


ground troop movements.

d. Inhalation causes respiratory damage and can


cause burns even up to bones. 74. Mytella strigata, also known Kaaka Aazhi,
sometimes seen in the news, refers to?

a. Fungus
71. “Global Hunger Index (GHI)” is a report published
by? b. Mussels

a. Concern Worldwide & Welthungerhilfe c. Water hyacinth

b. International Food Policy Research Institute d. Flowering plant

c. World Health Organisation & Concern


Worldwide
75. Consider the following pairs:
d. Food and Agriculture Organisation and World
Exercises Countries
Health Organisation
1. SIMBEX – India and Bangladesh

2. SAMPRITI – India and Singapore


72. Consider the following pairs.
3. CHAKRAVAT – India and Egypt
1. World Energy Outlook – International Energy
Agency (IEA) How many of the pair(s) given above is/are correctly
matched?
2. International Migration Outlook –
International Organization for Migration a. Only one
3. Interconnected Disaster Risks report – UN b. Only two
Office for Disaster Risk Reduction
c. All three
How many of the pair(s) given above is/are correctly
matched? d. None
a. Only one

b. Only two 76. Consider the following statements:


c. All three 1. It is administered by the Indian Council of
Forestry Research and Education (ICFRE).
d. None
2. It is a market-based mechanism that will
incentivize voluntary environmental actions
across diverse sectors by various stakeholders
73. Consider the following statements:
like individuals, communities and private
1. The Moscow format is one of the dialogue sector companies.
platforms on Afghanistan before the Taliban’s
The above statements best describes which of the
takeover of Kabul.
following scheme/initiative?
2. Both China and India are part of the Moscow
a. Ecomark Scheme
Format.
b. Project TAMARA

CHENNAI |SALEM| MADURAI | COIMBATORE DELHI | BANGALORE | THIRUVANANTHAPURAM


www.shankariasacademy.com 178
www.iasparliament.com

c. Green Credit Program b. Only two

d. National Afforestation Programme c. All three

d. None

77. Consider the following pairs.

Operations/Projects Initiative of 80. Consider the following statements with respect to


Niobium
1. Operation Kachchhap – Indian Army
1. It is a Non-metallic element used in alloys, tools
2. Operation Sarvashakti – Indian Air Force and superconductive magnets.

3. Project Udbhav - Directorate of Revenue 2. According to the Mines and Minerals


Intelligence (Development and Regulation) Amendment
Act, 2023, Niobium is categorized as an atomic
How many of the pair(s) given above is/are correctly mineral.
matched?
3. It is classified as one of the 17 rare earth
a. One pair only elements (REE).
b. Two pairs only How many of the statement(s) given above is/are
correct?
c. All three pairs
a. Only one
d. None
b. Only two

c. All three
78. Consider the following statements with respect to
Eastern Ghats. d. None
1. The Eastern Ghats are a continuous range of
mountains running parallel to the coast of the
Bay of Bengal. 81. Habitat rights meant for Particularly Vulnerable
Tribal Groups (PVTG) are granted under the Forest
2. The Eastern Ghats are lower in height than the Rights Act, 2006. Which one of the following PVTGs do
Western Ghats. not have the habitat rights?
3. Shevaroy hills and Bilirangan hills are located a. Gond tribe
in the Eastern Ghats.
b. Baiga tribe
Which of the above statement(s) is/are correct?
c. Kamar tribe
a. 1 and 2 only
d. Bharia tribe
b. 1 and 3 only

c. 2 and 3 only
82. Consider the following statements with respect to
d. 1, 2 and 3 Parliamentary Committees.

1. An allegation of corruption against a Member of


Parliament (MP) can be sent to either Ethics
79. Consider the following statements with respect to Committee or Privileges Committee.
United Nation Forum on Forests (UNFF).
2. An allegation of corruption against a non-
1. It promotes the management, conservation, Member of Parliament can be sent only to
and sustainable development of all types of Privileges Committee and not to Ethics
forests. Committee.
2. India is a founding member of UNFF. Which of the above statement(s) is/are correct?
How many of the statements given above a. 1 only
are incorrect?
b. 2 only
a. Only one

CHENNAI |SALEM| MADURAI | COIMBATORE DELHI | BANGALORE | THIRUVANANTHAPURAM


www.shankariasacademy.com 179
www.iasparliament.com

c. Both 1 and 2 2. Schedule IV of the Act deals with protection of


plant species.
d. Neither 1 nor 2
3. The Animal Welfare Board of India is a
statutory advisory body established under this
Act.
83. Which of the following statutory acts provide
mechanism for the process of ‘mediation’? How many of the above statement (s) is/
are not correct?
1. Mediation Act of 2023
a. Only one
2. Companies Act of 2013
b. Only two
3. Commercial Courts Act of 2015
c. All three
4. Legal Services Authority Act of 1987
d. None
5. Micro Small and Medium Enterprises
Development Act of 2006

Select the answer using the code given below: 86. Consider the following statements with respect to
the Quantum Dots.
a. All
1. Quantum dots are nanocrystals of a
b. All except 2 semiconducting material.
c. All except 4 2. The properties of quantum dots can be changed
by changing their size.
d. All except 5
Which of the statements given above is/are correct?

a. 1 only
84. Consider the following statements.
b. 2 only
Statement-I: Household savings has three
components such as physical assets, financial assets and c. Both 1 and 2
ornaments of gold & silver.
d. Neither 1 nor 2
Statement-II: Household savings refer to money left
after the household pays taxes and spends on the
consumption of goods and services.
87. Phonotaxis, sometimes seen in the news is related
Select the correct answer using the codes given below: to?
a. Both Statement-I and Statement-II are correct a. The taxon that includes a particularly vocal type
and Statement-II is the correct explanation for of deer.
Statement-I
b. The movement of an organism in response to a
b. Both Statement-I and Statement-II are correct sound.
and Statement-II is not the correct explanation for
Statement-I c. A way of communication between two arthropod
species.
c. Statement-I is correct but Statement-II is
incorrect d. Autonomous cars that can operate with reduced
or no noise.
d. Statement-I is incorrect but Statement-II is
correct

88. Which of the following statements with respect to


the Dancing Frogs is correct?
85. Consider the following statements with respect to
Wildlife Protection Act of 1972. a. It is believed to have co-existed with dinosaurs.

1. The Centre has the power to notify a b. Foot-flagging is a unique behaviour to it that
conservation reserve. attracts females while warning other males in the
vicinity.

CHENNAI |SALEM| MADURAI | COIMBATORE DELHI | BANGALORE | THIRUVANANTHAPURAM


www.shankariasacademy.com 180
www.iasparliament.com

c. It remains underground for almost a year except b. 2 only


for 2-3 weeks during the monsoon as it comes out
to mate. c. Both 1 and 2

d. When frightened, they inflate themselves, stand d. Neither 1 nor 2


up on their short legs and attack the potential with
an open huge mouth followed by a high-pitched
scream.
92. Consider the following pairs.

Certification Mark Certification Agencies


89. Consider the following statements regarding the
Quick Response (QR) code. 1. Eco mark – Bureau of Indian Standards (BIS)

1. QR codes can hold more data than barcodes. 2. India Organic Certification – Department of
Agriculture
2. QR codes can be read from all directions, while
barcodes can only be read from one direction. 3. AGMARK – Agricultural and Processed Food
Products Export Development Authority (APEDA)
3. Barcodes are one-dimensional (1D) whereas
QR codes are 3 dimensional (3D). How many of the pair(s) given above is/are correctly
matched?
How many of the statements given above are correct?
a. Only one
a. Only one
b. Only two
b. Only two
c. All three
c. All Three
d. None of the above
d. None of the above

93. Which of the following entities organized the Critical


90. Consider the following pairs Minerals and Clean Energy Summit 2023?

Art Region a. International Renewable Energy Agency

1. Pichwai Painting – Mysore b. International Energy Agency

2. Dogra architecture – Jammu c. International Solar Alliance

3. Vajra Mushti Kalaga – Kerala d. Clean Energy Alliance

How many of the pair(s) given above is/are correctly


matched?
94. Consider the following statements with respect to
a. Only one Cricket.

b. Only two 1. The Los Angeles Olympics, 2028 will be the first
Olympics to feature the Cricket sport.
c. All three
2. Cricket was featured for the first time in
d. None of the above Common wealth games in the year 202

Which of the above statement(s) is/are correct?

91. Consider the following statements with respect to a. 1 only


West Antarctic Ice Shelf Melt.
b. 2 only
1. It lies between the Weddell Sea and Ross Sea.
c. Both 1 and 2
2. Thwaites Glacier, the widest glacier on Earth, is
situated in this region. d. Neither 1 nor 2

Which of the statements given above are correct?

a. 1 only

CHENNAI |SALEM| MADURAI | COIMBATORE DELHI | BANGALORE | THIRUVANANTHAPURAM


www.shankariasacademy.com 181
www.iasparliament.com

95. U.N. Environment Programme's (UNEP) Cool c. Ministry of Minority Affairs


Coalition, sometimes seen in the news, is led by?
d. Ministry of Micro, Small and Medium
1. Climate Action Network Enterprises

2. Climate and Clean Air Coalition

3. Kigali Cooling Efficiency Program 99. Consider the following statements:

4. Sustainable Energy for All (SEforALL) 1. It is a tri-partite forum that brings together
representatives of government, business and
Select the correct answer using the code given below: academia.

a. Only one 2. Pakistan and Myanmar are not members of this


forum.
b. Only two
3. Its permanent secretariat is located at Ebene,
c. Only three Mauritius.
d. All four The statements given above corresponds to which of the
following groups?

a. BASIC Countries
96. Yak Churpi, Khamti Rice & Tangsa Textile are
Geographical Indicated (GI) Products belongs to? b. Indian Ocean Rim Association (IORA)
a. Assam c. Forum for India–Pacific Islands Cooperation
(FIPIC)
b. Manipur
d. Organisation of Economic Cooperation and
c. Nagaland Development (OECD)
d. Arunachal Pradesh

100. Consider the following statements with respect to


Setu Bandhan Scheme
97. Consider the following statements with respect to
Question Hour 1. The scheme is designed to enhance inter-state
connectivity, particularly in rural border areas.
1. It is the first hour of every parliamentary
sitting. 2. The scheme is an initiative of the Ministry of
Road Transport and Highways.
2. During this hour, Members of Parliament are
allowed to ask questions to the ministers as well Which of the statement(s) given above is/are correct?
as non-ministers.
a. 1 only
3. The process for conducting the Question Hour
is regulated by the Rules of Procedure and b. 2 only
Conduct of Business in Lok Sabha.
c. Both 1 and 2
How many of the statements given above are correct?
d. Neither 1 nor 2
a. Only one

b. Only two

c. All three

d. None

98. UMMEED, sometimes seen in the news, is an


initiative of?

a. Ministry of Culture

b. Ministry of Education

CHENNAI |SALEM| MADURAI | COIMBATORE DELHI | BANGALORE | THIRUVANANTHAPURAM


www.shankariasacademy.com 182
www.iasparliament.com

Answer Key - Test - IV

1 2 3 4 5 6 7 8 9 10

B B D D D D D A C A

11 12 13 14 15 16 17 18 19 20

A A C A C C B A B A

21 22 23 24 25 26 27 28 29 30

B B D C A C A D B B

31 32 33 34 35 36 37 38 39 40

A A B B C D A A D D

41 42 43 44 45 46 47 48 49 50

D A D D B D C B B A

51 52 53 54 55 56 57 58 59 60

D D A D D A C C A D

61 62 63 64 65 66 67 68 69 70

D C C B B C D D A A

71 72 73 74 75 76 77 78 79 80

A A D B D C D C D D

81 82 83 84 85 86 87 88 89 90

A C A B B C B B B A

91 92 93 94 95 96 97 98 99 100

C A B D C D C B B C

 Restricts conversion – It limits


EXPLANATION
converting a house of worship of one
1. b religious’ group into another
religious group, or even a different sect
Places of Worship Act, 1991 within the same religious group.

A 3-judge bench of the Supreme Court upheld an order  Bar on jurisdiction of court – No new suit
by the Allahabad High Court allowing the or legal process shall be instituted if there is
Archaeological Survey of India (ASI) to conduct its already a suit or legal proceeding currently
investigation of the Gyanvapi mosque in Varanasi. before any court regarding the conversion of the
religious character of any place of worship
 Aim – To protect the religious character of all existing on August 15, 1947.
houses of worship as it stood on August 15,
1947 and forbid any changes that would  However, legal proceedings can be
compromise that character. initiated with respect to the conversion of the
religious character of any place of worship after
 Places of worship – Any structure used for the commencement of the act i.e. after August
public religious worship, such as a temple. 15, 1947.

CHENNAI |SALEM| MADURAI | COIMBATORE DELHI | BANGALORE | THIRUVANANTHAPURAM


www.shankariasacademy.com 183
www.iasparliament.com

 Penalty – Up to 3 years in prison and shall also  States – Karnataka, Tamilnadu, Kerala,
be liable to fine. Puducherry

Exemptions  Basin – It can be divided into 3 parts, the


Western Ghats, the Plateau of Mysore, and the
 Ram Janmabhoomi-Babri Masjid case and Delta.
any appeal or procedure arising out of it are
exempt from its coverage.  Right tributaries – Lakshmana
Tirtha, Kabini, Bhavani, Noyyal,
 Any religious structure protected as an Amaravati, Moyar.
ancient monument or archaeological site or
remains under the Ancient Monuments  Left tributaries – Harangi, Hemavati,
and Archaeological Sites and Remains Shimsha, Arkavathy
Act of 1958.
 Island formation – In Karnataka, it
 Any action, appeal, or procedure that was bifurcates twice, forming the sacred islands of
determined, settled, or disposed of by a court, Srirangapatnam and Sivasamudram, 80 kms
tribunal, or other body prior to the effective apart.
date of this Act.
 Immediately after crossing Tiruchirappalli
 Any disagreement over related aspects that has district in Tamil Nadu, it divides into 2 parts,
been resolved amicably between the parties the Northern branch ‘The
before the start of the proceeding. Coleron’ and Southern branch remains as
Cauvery and from here the Cauvery Delta
 Any prior conversion to religious or secular use begins.
of such a location that was allowed to take place
with the owner’s approval.  After flowing for about 16 Kms, the 2 branches
join again to form “Srirangam Island”.
2. b
 Perennial character – It carries water
River Cauvery throughout the year with comparatively less
fluctuation than the other Peninsular Rivers.
The Tamil Nadu government has sought the Supreme
Court’s intervention to make Karnataka immediately  This is because the upper catchment area
release 24,000 cubic feet per second (cusecs) from its receives rainfall during the southwest monsoon
reservoirs. season (summer) and the lower part during the
northeast monsoon season (winter).
 Cauvery – It is the 3rd largest river in South
India.  Drains in – Bay of Bengal at Poompuhar in
Tamil Nadu.
 Ponni was another name for it in Tamil
literature. 3. d
 Origin – Talakaveri on Brahmagiri range in Committees for reform of criminal laws
Western ghats.
 Malimath committee – It was formed
in 2003 to bring reforms in the criminal justice
system.

 The Committee recommended that the victim


should be allowed to participate in cases
involving serious crimes and also be given
adequate compensation.

 Justice Verma panel – It was formed to


ensure quicker trial and enhanced punishment
for criminals accused of committing sexual
assault against women.

 It submitted its report in 2013.

 Ranbir Singh committee- It was formed


in 2020 to review 3 codes of criminal law

o Indian Penal Code (IPC), 1860

CHENNAI |SALEM| MADURAI | COIMBATORE DELHI | BANGALORE | THIRUVANANTHAPURAM


www.shankariasacademy.com 184
www.iasparliament.com

o Code of Criminal Procedure (CrPC), o Example – Railway Convention


1973 and Committee, Joint Committee on Food
Management in Parliament House
o Indian Evidence Act, 1872. Complex etc also come under the
category of ad hoc Committees.
Old Acts Replaced by
 Broad classifications
Indian Penal Code (IPC), Bharatiya Nyaya Sanhita o Financial Committees
1860 Act, 2023
o Departmentally Related Standing
Bharatiya Nagarik Committees.
Criminal Procedure Code
Suraksha Sanhita
(CrPC), 1973 o Other Parliamentary Standing
Act, 2023
Committees; and

Bharatiya Sakshya Act, o Ad hoc Committees.


Indian Evidence Act 1872
2023
5. d

4. d India’s support for commercial crops

Parliamentary Committees India should optimise the potential of commercial


crops to provide income security to the farmers.
Recently, Home Minister has said that members had
complained about the inclusion of their names in the  Pradhan Mantri Fasal Bima Yojana
proposed Select Committee without their signatures. (PMFBY) – A government sponsored crop
insurance scheme that was launched in 2016.
 Parliament Committees – It means
a committee which is appointed or  National Edible Oil Mission-Oil Palm
elected by the House or nominated by (NMEO-OP) – It was launched in 2021 for a
the Speaker. period of 5 years for self-reliance in edible oil.

 It works under the direction of the Speaker and  Pradhan Mantri Kisan SAMPADA
presents its report to the House or to the Yojana (PMKSY) – A subcomponent under
Speaker and the Secretariat for which is the scheme of Integrated Cold Chain and Value
provided by the Lok Sabha Secretariat. Addition Infrastructure.

 India’s Parliament – It has 2 different  It aims to reduce post-harvest losses of


committees. horticulture and non-horticulture produce and
providing remunerative price to farmers for
 12 Standing Committees – They are their produce.
permanent and regular committees which are
constituted from time to time in pursuance of  National Food Security Mission
the provisions of an Act of Parliament or Rules (NFSM) – It has sub components such as
of Procedure and Conduct of Business in Lok NFSM – Oilseeds, NFSM-Oil palm, NFSM-
Sabha. Tree Borne Oilseeds, NFSM-Commercial Crops
etc.
 The work of these Committees is of continuous
nature.  Institutional support – Support is provided
through institutions such as
o Example – Privilege committee,
Financial Committee, etc. o Central Institute of Cotton Research -
Nagpur
 Ad hoc Committees – A temporary
committee, set up for a specific purpose, such o Central Tobacco Research Institute -
as examining a particular Bill. Andhra Pradesh

o Coconut Development Board – Kerala


 They are dissolved once that purpose has
been served. o Tobacco Board – Guntur, Andhra
Pradesh
 The principal Ad hoc Committees are the Select
and Joint Committees on Bills. o Tea Board – Kolkata, West Bengal

CHENNAI |SALEM| MADURAI | COIMBATORE DELHI | BANGALORE | THIRUVANANTHAPURAM


www.shankariasacademy.com 185
www.iasparliament.com

o Coffee Board – Bangalore,  NBRI – A premier plant-based,


Karnataka multidisciplinary, state-of-the-art National
R&D centre.
o Rubber Board – Kottayam,
Kerala  It is a constituent institution of the Council of
Scientific and Industrial Research (CSIR).
o Spices Board – Kochi, Kerala
8. a
6. d
Pradhan Mantri Uchchatar Shiksha Abhiyan
Vyommitra (PM-USHA) Scheme
Vyommitra will fly into space in 2024.  Pradhan Mantri Uchchatar Shiksha Abhiyan
(PM-USHA) Scheme is the new name for the
 It is an AI-enabled female robot, designed Ministry of Education’s scheme to improve the
to simulate human functions in the space quality of higher education in State
environment and interact with the life support Universities.
system.
 It aims to improve quality through
 Role – It will accompany astronauts on curricular & programme changes,
manned missions in addition to the unmanned teacher training, physical and digital
Gaganyaan mission. infrastructure, accreditation, and enhancing
employability, while ensuring equity, access,
 Gaganyaan mission will be India's 1st human- and inclusion.
manned space flight carrying Indian astronauts
into space.  States are required to sign a MoU with the
Centre.
 Name – ‘Vyoma’ meaning space and ‘Mitra’
meaning friend.  The MoU mandates the implementation of the
National Education Policy in order to avail
 Designed & developed by – ISRO Inertial
funds for the next 3 years, under the Centre’s
Systems Unit (IISU).
flagship scheme for State-run higher education.
 Aim – To perform specific tasks in order to
 Nearly 40% of the PM USHA budget must be
analyse how astronauts might behave. borne by the States themselves, and no extra
funds have been earmarked for NEP reforms.
 Functions – It is equipped with the capability
to monitor module parameters, issue alerts,
 The MoU makes it mandatory for States to
and execute life support operations. undertake the administrative, academic,
accreditation, and governance reforms detailed
 It has the ability to speak, see and make facial
in the NEP.
expressions, it has been created to resemble a
human.  These include an academic credit bank, entry
and exit flexibility, and the Samarth e-
 It will mimic every action that astronauts are
governance platform.
required to take and respond to them in 2
languages.  PM-USHA carries forward the vision of the
earlier Rashtriya Uchstar Shiksha Abhiyan
7. d
(RUSA), to improve the access, equity and
Namoh 108 quality of higher education in States.

Recently Ministry of Science and Technology unveiled  PM-USHA reduces the fragmentation of
the new variety of Lotus flower named “Namoh 108”. resources by streamlining the number of
(scheme) components to six.
 Namoh 108 – It is a lotus flower with 108
petals, the 1st flower whose genome is 9. c
completely sequenced for its characteristics.
Belem Declaration
 This variety flowers from March to December
 It was adopted during the Amazon Summit,
and is rich in nutrients.
2023 and recognizes Indigenous knowledge as
 Developed by – Lucknow based National a condition for biodiversity conservation.
Botanical Research Institute (NBRI).
 The declaration calls for ensuring full and
effective participation of Indigenous

CHENNAI |SALEM| MADURAI | COIMBATORE DELHI | BANGALORE | THIRUVANANTHAPURAM


www.shankariasacademy.com 186
www.iasparliament.com

Peoples in decision-making and public  The ECOWAS Commission and the ECOWAS
policy formulation processes. Bank for Investment and Development (The
Fund) are its two main institutions.
 The declaration promotes sustainable use of
biodiversity resources in the Amazon.  They are designed to implement policies,
pursue a number of programmes and carry out
 The Amazon Summit, 2023 was held at development projects in Member States.
Belem, Brazil with participation of 8 countries
that include Bolivia, Brazil, Colombia, Ecuador,  Single currency – It’s larger aim is to have a
Guyana, Peru, Suriname and Venezuela. single common currency and create a single,
large trading bloc in western Africa.
 The leaders from the 8 amazon countries failed
to agree on the goal to protect the rainforest at  The grouping has also intervened militarily in
the ongoing Amazon Summit organized by the the region in the past.
Amazon Cooperation Treaty Organization
(ACTO).  It works with the mandate of promoting
economic integration among its members.
10. a
 Members – Benin, Cape Verde, Cote d’ Ivoire,
Economic Community of West African States The Gambia, Ghana, Guinea Bissau, Liberia,
(ECOWAS) Nigeria, Sierra Leone, Senegal and Togo.
 The Economic Community of West African  Suspended Members – Guinea, Niger,
States (ECOWAS) is a regional group of 15 Burkina Faso and Mali
African countries founded in 1975.
 In 2024, Niger, Burkina Faso, and Mali
 Lagos Treaty – ECOWAS is also known as announced their withdrawal from the bloc.
CEDEAO in French and was established
through the Lagos Treaty.  Alliance of Sahel States - It is a mutual
defense pact created between Mali, Niger,
 Mission – To promote economic integration and Burkina Faso in 2023.
in all fields of economic activity, social and
cultural matters.  The Liptako-Gourma region is where the Mali,
Burkina Faso and Niger borders meet.

11. a

UDGAM Portal

 The Reserve Bank of India (RBI) has


unveiled a Centralised Web Portal
UDGAM (Unclaimed Deposits – Gateway to
Access inforMation).

 Purpose – For use by public to search their


unclaimed deposits across multiple banks at
one place.

 It enables to either claim the deposit amount or


make the deposit accounts operative at their
respective banks.

 RBI Technology Pvt. Ltd. (ReBIT), Indian


 The Institutions of the Economic Community of Financial Technology & Allied Services
West African States (ECOWAS) are: (IFTAS), and participating banks have
collaborated on developing the portal.
o The Commission
 Unclaimed Deposits – Balances in savings
o The Community Parliament
/ current accounts which are not
o The Community Court OF Justice operated for 10 years, or term deposits not
claimed within 10 years from date of maturity
o ECOWAS Bank for Investment and are classified as “Unclaimed Deposits”.
Development (EBID)
 Such deposits are transferred to the Depositor
Education and Awareness (DEA)

CHENNAI |SALEM| MADURAI | COIMBATORE DELHI | BANGALORE | THIRUVANANTHAPURAM


www.shankariasacademy.com 187
www.iasparliament.com

Fund maintained by the Reserve Bank of  Maritime cooperation began after the Indian
India. Ocean tsunami of 2004.

12. a  Quad works on a far broader agenda, which


includes tackling security, economic, and
Maya OS health issues.
 The Defence Ministry has developed 14. a
Maya OS with the help of various government
agencies, including Jaderi namakatti

o Defence Research and Development  These are white clay sticks. They are used to
Organisation (DRDO), adorn the foreheads of idols, men and temple
elephants.
o Centre for Development of Advanced
Computing (C-DAC), and  It is a GI product from Tamil Nadu.
o National Informatics Centre (NIC).  Composition - Rich deposit of hydrous
silicate minerals that form fine grain particles
 Maya is based on open-source Ubuntu that is of clay.
developed locally.
 The clay is processed and shaped in a finger like
 The Microsoft Operating System (OS) in all structure.
computers connected to the Internet will be
replaced with a new OS, Maya.  The production of namakatti depends on the
climatic condition as it needs a lot of sunlight to
 Indian Navy has decided to install Maya OS in dry.
their systems while the Army and the Air Force
are evaluating it.  Jaderi is a small village in Tiruvannamalai
district of Tamil Nadu.
 CERT-In - The Indian Computer Emergency
Response Team is the nodal agency to deal with  Other products from TN - Kanniyakumari
cyber security incidents. Matti banana and Chedibutta Saree are other
GI products from Tamil Nadu.
 It is an office within the Ministry of Electronics
and Information Technology. Jalesar Dhatu Shilp
13. c  It is a metal craft famous in the Jalesar in
Etah district, Uttar Pradesh.
Malabar Exercise
 It includes ghungrus (anklets), ghantis (bells)
 It is a multilateral naval exercise that includes and other decorative metal craft and brassware.
simulated war games and combat maneuvers.
 The bells casted in Etah district’s soil ring
 It aims to demonstrate the synergy,
better.
coordination and inter-operability among the
participating navies.  The Thatheras community, which resides in
Hathuras, involved in making of these
 Started in 1992 as a bilateral exercise between products.
the navies of India and the US.
Koftgari Metal Craft
 Current Members – United States of
America, Japan, Australia and India as  The Udaipur Koftgari Metal craftsmen
permanent partners. practices the ancient art of Koftgari used in
making ornamental weaponry at
 2023 Exercise – The Indian Navy was Udaipur, Rajasthan.
represented by indigenously built Destroyer
INS Kolkata, Frigate INS Sahyadri and P8I  Other GI products from Rajasthan –
Maritime Patrol Aircraft. Bikaner Kashidakari Craft, Jodhpur Bandhej
Craft, Bikaner Usta Kala Craft, Udaipur
 These countries are also members of the QUAD Koftgari Metal Craft and Bikaner Kashidakari
grouping Craft.
 QUAD – The Quad, officially the Quadrilateral
Security Dialogue, is a group of 4 countries:
Australia, India, Japan and the US.

CHENNAI |SALEM| MADURAI | COIMBATORE DELHI | BANGALORE | THIRUVANANTHAPURAM


www.shankariasacademy.com 188
www.iasparliament.com

15. c  The first 160 km stretch is called India-


Myanmar Friendship Highway (IMFH).
Convention on International Liability for
Damage Caused by Space Objects  The project has the power to revolutionise
tourism, cultural interchange and improve
 Recently ISRO rocket debris was found on regional trade among India, Myanmar and
Australian shore. Thailand.

 Article 7 of the Outer Space Treaty states that  The project was first proposed by former prime
the damages caused by a State Party that minister Atal Bihari Vajpayee and was
'launches or procures the launching of an approved at ministerial-level meeting in 2002.
object into outer space is internationally liable
for damage to another State Party to the 17. b
Treaty.
PM Young Achievers Scholarship Award
 Liability Convention was signed on 1972 and it Scheme for Vibrant India (PM –YASASVI)
expands the rules mentioned in the article 7 of
the Outer Space Treaty.  PM–YASASVI is Centrally Sponsored
Schemes for vulnerable groups comprising
 The Convention deals mainly with damage of Other Backward Class (OBCs),
caused by space objects to other space Economically Backward Class (EBC) and
assets, but it also applies to damage caused Denotified, Nomadic and Semi-Nomadic
by falling objects on earth. Tribes (DNT/S-NT).

 The amount of compensation is to be decided  The scheme is implemented by the Ministry


“in accordance with international law and the of Social Justice and Empowerment.
principles of justice and equity.
 The 5 sub-schemes under PM –YASASVI are:
 This provision of the Convention has resulted in
compensation payment only once so far — 1. Pre-Matric Scholarship for OBC, EBC
Canada sought damages from the then Soviet and DNT Students
Union in 1978.
2. Post-Matric Scholarship for OBC, EBC
16. c and DNT Students

Trilateral Highway Project 3. Top Class School Education for OBC,


EBC and DNT Students
It is also referred as “The Road to East” which
connects India, Myanmar and Thailand. 4. Top Class College Education for OBC,
EBC and DNT Students
 The project spans 1,400 kilometres and will
connect Moreh in Manipur with Mae Sot in 5. Construction of Hostel for OBC Boys
Thailand, while passing through Myanmar. and Girls

 It will be a gateway to ASEAN countries. 18. a

LockBit, Akira and Luna Moth

 The report named “Turning Tides Navigating


the Evolving World of Cybercrime” noted that
Ransomware attacks is one of the reasons for
disruption in the cybersecurity space in the first
half of 2023.

 LockBit ransomware was the most used,


accounting for 30.3% of observed ransomware
cases, cybercriminals were also found making
use of new variants, including Akira and
Luna Moth.

 Ransomwares like LockBit were also updated to


target newer operating systems including
Linux, and macOS.

 Ransomware-as-a-Service (RaaS) model has


dominated the cybercrime industry over the

CHENNAI |SALEM| MADURAI | COIMBATORE DELHI | BANGALORE | THIRUVANANTHAPURAM


www.shankariasacademy.com 189
www.iasparliament.com

past few years, Cybercrime-as-a-Service grew monitoring of the Child Care Institutions
parallel to it in H1 2023. (CCIs) and their inspection mechanism across
the country.
 WormGPT, a blackhat version of the popular
AI chatbot ChatGPT was found to be used to  It serves as a single platform for inspections of
generate malicious content, including phishing all the CCIs across the country by the following
emails, malware code, fake news, and social authorities:
media posts.
o Child Welfare Committees (CWCs),
19. b
o State Inspection Committees,
Wildfires and Heatwaves
o District Inspection Committees,
 Wildfires and heatwaves have been raging
across the Northern Hemisphere in the past o Members of Juvenile Justice Boards
month (July), even as many countries across (JJBs) and
Europe, Asia and northern America have been
experiencing record-high temperatures o State Commissions for Protection of
Child Rights (SCPCRs).
 Spain – Has been bearing the brunt of
wildfires, particularly in  National Commission for Protection of
the Canary Islands where 4,000 people to Child Rights (NCPCR) has developed
flee and forced others to wear face masks. this (MASI) application.

 Syria – The fires in Latakia raged for five  The app is linked to the monitoring Portal
days before rescuers could control it. where the automatic reports are generated.

 Turkey – The seaside town of Kemer was  The Juvenile Justice (Care and Protection of
affected by wildfire. Children) Act, 2015, makes it mandatory to
establish at least one Child Welfare Committee
 In the western coastal province of Izmir, towns (CWC) in each district.
of Kinik and Odemis were also affected by
wildfires. Flood-watch Mobile App

 Greece – Wildfires which were initially  The App aims to disseminate information
confined to the islands of related to the flood situation and forecasts up
Rhodes and Corfu spread towards the to 7 days on a real-time basis to the public.
mainland with the city of Volos facing a large
brunt of the crisis.  The App has readable and audio broadcast and
all the information is available in 2 languages
 Italy – Is facing extreme weather events that that is English and Hindi.
are occurring in both the northern and
 The App utilizes near real-time river flow data
southern regions of the country.
from various sources.
 The Palermo airport in Sicily has been closed
 The App is launched by Central Water
down due to raging wildfires.
Commission (CWC).
 Sardinia and Calabria have also reported
 Other key features include forecast using an
dozens of fires while evacuations were ordered.
Interactive Map where users can check the
 In the north, Lombardy is being battered by CWC Flood Forecast (up to 24 hours) or Flood
severe storms that have already killed a woman Advisory (up to 7 days).
and a sixteen-year-old girl.
Khanan Prahari App
 Croatia – In the country’s south along
 The mobile app Khanan Prahari allows citizens
the Adriatic Sea coast, a wildfire was raging a
to report incidents of illegal coal mining
few kilometres from the renowned walled town
through geo-tagged photographs and textual
of Dubrovnik.
information.
20. a
 It is an initiative of Ministry of Coal.
MASI Portal
 The corresponding web portal called as Coal
 MASI - Monitoring App Mine Surveillance & Management System
for Seamless Inspection for real time (CMSMS).

CHENNAI |SALEM| MADURAI | COIMBATORE DELHI | BANGALORE | THIRUVANANTHAPURAM


www.shankariasacademy.com 190
www.iasparliament.com

 The portal has been developed in association  Member – 19 countries (Canada, France,
with the Bhaskaracharya Institute of Germany, Italy, Japan, UK, US, Russia,
Space Application & Geoinformatics. Australia, Canada, Saudi Arabia, India, South
Africa, Turkey, Argentina, Brazil, Mexico,
21. b China and Indonesia), European Union and
African Union.
G20 Summit 2023
 It does not have permanent secretariat.
 Presidency – India
 Presidency – It is chosen by a rotation system
 2023 Theme – ‘One Earth One Family amongst the member countries.
One Future’ meaning ‘Vasudhaiva
Kutumbakam’ (the whole earth is a family).  India holds the Presidency of the G20 from
December 2022 to 30 November 2023.
 It was written in Devanagari script, drawn from
the ancient Sanskrit text of the Maha  Troika – It comprises of the past, present, and
Upanishad. future presidents.

 It resulted in adoption of New Delhi  Troika 2023 includes Indonesia, India, and
Declaration, which reflects India's motto Brazil.
of "Sabka Saath, Sabka Vikas, Sabka
Vishwas" –“Together with all, Development 22. b
for all, Trust of all”.
Women Reservation Act, 2023
 Outcome – The 55-member African Union
was formally made permanent President has given her assent to the Nari Shakti
member of the G20, on par with the European Vandan Adhiniyam (128th Amendment) Bill, 2023 and
Union. now it will be known as the Constitution (106th
Amendment) Act, 2023.
 G20 – It was established in 1999, after the
Asian financial crisis as a regular forum for  Provides 33% reservation for women in Lok
Finance Ministers and Central Bank Governors. Sabha, State Legislative Assembly, and
the Legislative Assembly of the National
 Aim – To study, review, and promote high level Capital Territory of Delhi.
discussion of policy issues pertaining to
international financial stability.  It will also apply to the seats reserved for SCs
and STs in Lok Sabha and states legislatures.

 Article 330A and Article 332A to be introduced


in the Constitution to propose changes for Lok
Sabha and Assemblies respectively.

 It can be implemented only after a delimitation


exercise is completed, and the next delimitation
exercise is scheduled to be held in 2026.

 Effectively, this means that the earliest


implementation of the women’s quota in the
Lok Sabha can be in the 2029 general election.

 Seats reserved for women will be rotated after


each delimitation, as determined by a law made
by Parliament.

 The Jammu & Kashmir Reorganisation


(2nd Amendment) Act 2023 – It seeks
to reserve one-third of all elected seats in the
Jammu and Kashmir Legislative Assembly for
women.

 The reservation for women will also apply to the


seats reserved for Scheduled Castes and
Scheduled Tribes in the J&K Assembly.

CHENNAI |SALEM| MADURAI | COIMBATORE DELHI | BANGALORE | THIRUVANANTHAPURAM


www.shankariasacademy.com 191
www.iasparliament.com

 The Government of Union Territories XI  Elaborate explanation of Dhamma


(Amendment) Act 2023 – It reserves one
third of all elected seats in the Puducherry  Appeal for developing tolerance
XII
Legislative Assembly for women. towards all religious sects

23. d  About Kalinga war and conquest


XIII
through dhamma
Rock Edicts of Ashoka
 Purpose of engraving inscriptions in
2 Ashokan era edicts were recently found in Delhi. XIV
various parts of the country
 Edicts – It is a decree or announcement of law,
which can be under any official authority and
24. c
can be issued in any number of physical forms
Sacred Ensembles of Hoysalas (Karnataka)
o Major and Minor Rock Edicts
Recently, Hoysala-era temples were added as to
o Major and Minor Pillar Edicts
UNESCO’s World Heritage List thus becoming the
Major 42nd site from India.
Rock About
 The Chennakeshava temple – It is located
Edict
in Belur.
 Prohibits animal slaughter
 Major shrine
I
of– Lord
 Bans festive gatherings
Vishnu
 Mentions the South Indian kingdoms  Time period –
II such as Cholas, Pandya, Satyaputas and Consecrated
Keralaputras around 1117
AD
 Yukatas, Pradesikas along with Rajukas
III shall visit all the areas of the kingdom to  Built by – Hoysala king Vishnuvardhana
spread Dhamma
 Nickname – Vijaya Narayana temple as it
 Preference of Dhammaghosa (Sound of was built by the king to mark his victories
IV peace) over Berighosa (Sound of against the Cholas.
wandrums)
 The Kesava Temple – It is located in
 Mentions every human as the child of Somanathapura (Mysore).
Ashoka
 Time period – 1268 C.E
 Humane treatment of slaves by their
V
masters  Built by –
Somanatha, a
 Mentions about the appointment of General of
Dhamma Mahamatras Hoysala King
Narasimha III
 King’s responsibility towards the
VI
welfare of the people  Major Shrine -
Vaishnava
 Tolerance towards all shrine
VII
religions and sects
 The Hoysaleswara temple – It is located on
 Ashoka’s 1st dhammayatra to Bodh the banks of Dwarasamudra tank in Halebidu.
VIII
Gaya and Bodhi tree
 Time period – 12th Century C.E.
 tresses on the moral conduct of people
IX  Major Shrine – Lord Shiva
 Condemns expensive ceremonies
 It is believed to be the largest Shiva
X  Condemns the desire for fame and glory temple built by the Hoysalas.

CHENNAI |SALEM| MADURAI | COIMBATORE DELHI | BANGALORE | THIRUVANANTHAPURAM


www.shankariasacademy.com 192
www.iasparliament.com

 Halebidu was sacked by Malik Kafur, a general o Proper special sessions – It


of the then Delhi Sultan Alauddin Khalji in the happens with debates or
early 1300s. discussions, and

o Midnight sessions – It
happens without any debates.

 Special sitting in Lok Sabha – In 2012, to


commemorate the 60th anniversary of the 1st
sitting of the Indian Parliament.

 Special sitting in Rajya Sabha – In 1977 and


1991 when the Lok Sabha was under dissolution
to decide on the President’s Rule.

25. a

Special Session of Parliament

A special session of Parliament was held in September


2023.

 Constitution of India does not use the term


“special session”.

 Article 352 – It refer to a “special sitting of the


House” to deal with proclamation of
emergency.

 This part was added to the article during 44th


Amendment Act to add safeguards to the power
of proclaiming emergency in the country.

 If a Proclamation of Emergency is issued and 26. c


Parliament is not in session, then one-tenth of
Lok Sabha MPs can ask the President to Abraham Accords
convene a special meeting to disapprove the September 2023 marks an important milestone for
Emergency. West Asia and North Africa, 3 years since the signing
of the Abraham Accords.
 Agenda – Either to celebrate a historical
legacy like the Indian freedom struggle and
 Signed in - 2020
Indian Independence or to pass a bill.
 Agreement between - United Arab Emirates,
 Summon – The President, who summons a Bahrain and Israel.
regular Parliamentary session will summon
this session also as per provisions of Article  Mediated by – The US
85(1) of the Constitution.

 Chair – For the 2 Houses to be in session, the


Presiding Officers should chair their
proceedings.

 The presiding officers can also direct that the


proceedings of their respective Houses would
be limited.

 The procedural devices like question hour


would not be available to MPs during the
session.

 2 divisions of the special sessions

CHENNAI |SALEM| MADURAI | COIMBATORE DELHI | BANGALORE | THIRUVANANTHAPURAM


www.shankariasacademy.com 193
www.iasparliament.com

 Objective- To normalise ties between


these Arab states and Israel.
Contention Area Disputed Parties
 Following the signing of the Abraham Accords
in 2020, Arab states including Egypt, Paracel Island China, Taiwan and Vietnam
Jordan, UAE, Morocco, Bahrain and
China, Taiwan, Vietnam,
Sudan have established diplomatic ties with Spratly island
Philippines and Malaysia
Israel.
Scarborough
 It is sometimes referred to as “the new dawn of China and the Philippines
Shoal
a new Middle East”.

27. a
28. d
South China Sea Dispute
Non Fungible Tokens (NFTs)
Recently, Philippines coast guard removed the floating
barrier placed by China that prevented Filipino fishing A crypto gambling platform dappGambl, concluded
boats from entering a disputed area in South China that most of the researched NFT collections were worth
Sea. close to nothing.

 South China Sea boundaries –Brunei, China,  NFTs – They are blockchain-based tokens that
Indonesia, Malaysia, Philippines, Taiwan and each represent a unique asset like a piece of art,
Vietnam. digital content, or media.

o Taiwan Strait – Connects South China  It is an irrevocable digital certificate of


Sea with East China Sea ownership and authenticity for a given
asset, whether digital or physical.
o Luzon Strait – Connects South China
Sea with Philippine Sea  Block chain is a distributed ledger where all
transactions are recorded.
 Rivers draining into it – The tributaries of the
Pearl (Zhu) River including the Xi River, the  Key characteristics – They are designed to be
Red River and the Mekong River. cryptographically verifiable, unique or scarce
and easily transferable.
 Senkaku or the Daioyu Islands dispute –
Between China and Japan in East China Sea  Creation – It is created by an artist, creator, or
license-holder through a process called
minting.

 Minting – It involves signing a blockchain


transaction that outlines the fundamental
token details, which is then broadcasted to the
blockchain to trigger a smart contract function
which creates the token and assigns it to its
owner.

 Thus a unique token identifier, or token ID is


mapped to an owner identifier and stored
inside a smart contract.

 When the owner of a given token ID wishes to


transfer it to another user, it is easy to verify
ownership and reassign the token to a new
owner.

 It cannot be exchanged or replaced with


tokens of same type as each are unique.

 Fractional NFTs (F-NFTs) – It allows multiple


people to own a piece of the same asset by
dividing the NFT into smaller fractions, each of
which can be purchased individually.

CHENNAI |SALEM| MADURAI | COIMBATORE DELHI | BANGALORE | THIRUVANANTHAPURAM


www.shankariasacademy.com 194
www.iasparliament.com

 World Air Quality Report – by IQAir, a


Swiss air quality technology company.

 Air Quality Index – by Central Pollution


Control Board

30. b

India’s Space Policy 2023

 Published by – ISRO

 It formally laid down the regulations


of privatizing space missions in India.

 It also denoted clear guidelines of operations


for New Space India Limited (NSIL) and Indian
National Space Promotion and Authorization
Centre (IN-SPACe).

 Vision – To augment space capabilities and to


enable, encourage and develop a flourishing
29. b commercial presence in space
Air Quality Life Index (AQLI) Report  To use space as a driver of technology
development and derived benefits in allied
According to the University of Chicago report, the air areas.
pollution has major impact on the life expectancy.
 To pursue international relations, and create an
 Prepared by – University of Chicago’s ecosystem for effective implementation of
Energy Policy Institute (UCEPI) space applications among all stakeholders.
 Aim – To quantify the causal relationship  Policy – ISRO will move out of
between long-term human exposure to air manufacturing space systems, and
pollution and life expectancy. instead focus only on advancing space R&D.
 Data – It uses highly localized satellite data,  Manufacturing and operations will be
making it is possible to report life expectancy handled by NewSpace India Limited
impacts at the county or similar level around (NSIL), a public sector unit set up in 2019
the world. under the Department of Space as the
commercial arm of ISRO.
 It is rooted in peer-reviewed research by an
international team of scholars.  IN-SPACe – Indian National Space Promotion
& Authorisation Centre is expected to create
 Findings – In 2021, the average person in the regulatory framework to ensure a level playing
world would have gained 2 years and 3 months field for the NGEs.
of life expectancy if fine particulate pollution
were reduced to meet the WHO guideline.  It will also handle transfer of technology
requests, in order to give private space firms
 The South Asian nation stands to lose 6.8 years access to already established ISRO-owned
of life on average per person. space infrastructure.
 The average person living in Bangladesh, India, 31. a
Nepal, and Pakistan is exposed to particulate
pollution levels that are 51.3% higher than at Statue of Oneness
the turn of the century.
Madhya Pradesh Chief Minister unveiled the 108-foot-
 Bangladesh remains the most polluted tall Statue of Oneness of Adi Shankaracharya at
country in the world. Omkareshwar.

 Significance – It is the 1st pollution index to  Statue of Oneness - Depicts Adi


show what the threat of air pollution means to Shankaracharya as a 12-year-old child.
a person’s life anywhere in the world.
 Location –Mandhata Parvat hill at
Omkareshwar, Madhya Pradesh.

CHENNAI |SALEM| MADURAI | COIMBATORE DELHI | BANGALORE | THIRUVANANTHAPURAM


www.shankariasacademy.com 195
www.iasparliament.com

 Adi Shankaracharya became a proponent o Ex – Angrites, Erg Chech 002.


of Advaita Vedanta at Omkareshwar.
 It contains abundant lead-206 and lead-207, as
 Statue of Unity – It is a statue of Sardar well as undecayed uranium-238 and uranium-
Vallabhai Patel, the Iron Man of India. 235.

 It is the world’s tallest statue (182 m), located  Findings – It is some 4.56556 billion years old
near the Sardar Sarovar Dam, in the banks of (around 120,000 years).
the Narmada River in Gujarat.
 Aluminium-26 was distributed quite unevenly
 Statue of Equality – It is the statue of Sri throughout the cloud of dust and gas which
Ramanujacharya. formed the solar system.

 It is the second-tallest statue in the world,  Conocarpus – It is a fast-growing exotic


located in Hyderabad in Telangana. mangrove species.

 Born in 1017 in Sriperumbudur in Tamil Nadu,  Proxima Centauri – A red dwarf star,
he is revered as a Vedic philosopher and social nearest to the Sun.
reformer who travelled across India, advocating
equality and social justice. 33. b

 Statue of Valour – A 125-foot bronze statue Warm Arctic Cold Eurasian (WACE) pattern
of 'Ahom general' Lachit
Borphukan in Assam's Jorhat was unveiled Together with rapid Arctic warming and sea ice
in 2024. decline, especially over the Barents–Kara seas,
extreme cold winters have occurred frequently in mid-
 Battle of Alaboi and Battle of Saraighat were latitudes, particularly in Central Eurasia.
fought between Mughalas and Ahoms.
 It is a pattern with 2 distinct winter
 Statue of Social Justice – A 206 feet statue temperature anomalies centered over the
of Dr. B. R. Ambedkar at Andhra Pradesh. Barents–Kara seas and Central Eurasia.

 Statue of Equality (In USA) – A 19-feet  2 distinct winter temperature anomalies


statue of Dr. B. R. Ambedkar, part of the
o Warm surface temperatures
Ambedkar International Centre (AIC) in
over the Arctic
Maryland, is the largest statue of Ambedkar
outside India. o Cold surface temperatures over
Eurasia
32. a

Erg Chech 002  It is accompanied by an anomalous high


pressure and anticyclonic circulation over the
In 2020, some unusual rocks containing distinctive Eurasian continent.
greenish crystals were found in the Erg Chech sand
sea.  The sea ice loss in the Barents and Kara seas
and the cooling of the Eurasian continent can
 Erg Chech – A sandy region of the Sahara in both be traced to anomalous atmospheric
western Algeria and northern Mali. circulations.

 It consists largely of shifting dunes.

 The rocks containing distinctive greenish


crystals, it turned out to be from outer space,
left over from the dawn of the Solar System.

 They were all pieces of a meteorite known as


Erg Chech 002, which is the oldest volcanic
rock ever found.

 Erg Chech 002 – It is an ancient andesite


meteorite, an “ungrouped achondrite”, which
are rocks formed from melted planetesimals,
which is what we call solid lumps in the cloud of
gas and debris that formed the Solar System.

CHENNAI |SALEM| MADURAI | COIMBATORE DELHI | BANGALORE | THIRUVANANTHAPURAM


www.shankariasacademy.com 196
www.iasparliament.com

 Impact in Indian Ocean – It is also associated 35. c


with upper air circulation changes that reach
into the Indian Ocean sector. Beijing convention on the judicial sale of ships

 The rapid increase in the cyclogenesis potential 15 countries join the Beijing convention on the judicial
over the Arabian Sea coincides with a shift in sale of ships.
the WACE pattern.
 It is a UN convention to promote legal
34. b certainty in judicial sale of ships.

GRIHA Norms  Developed by – United Nations Commission on


International Trade Law (UNCITRAL).
The Indian Army’s new Thal Sena Bhawan (TSB),
coming up with GRIHA-IV (Green Rating for  Aim - It addresses the problem of new owners
Integrated Habitat Assessment) norms. and those financing the purchase of vessels
who, for instance, find themselves dealing with
 Green Rating for Integrated Habitat previous creditors laying claim to the ship as
Assessment (GRIHA) - A national rating security for a loan.
system that evaluates the environmental
performance of a building holistically over its  International Maritime Organisation (IMO)
entire life cycle. was supporting the establishment of the
convention by acting as the repository for these
 Developed by TERI (The Energy and notices and certificates of judicial sale.
Resources Institute) and has been adopted
by the Ministry of New and Renewable Energy.  Information on pending and completed judicial
sales of ships will be accessible online on IMO’s
 3 stages for Evaluation – Pre-construction Global Integrated Shipping Information
stage, building planning & construction stages System (GISIS) platform.
and building operation & maintenance stage.
 Parties – As of now, 15 States signed, but not
 Assessment – Based on 34 criteria and awards signed by India yet.
points on a scale of 100.
 UNCITRAL – It is a subsidiary body of the U.N.
 Eligibility – A project must achieve at least 50 General Assembly responsible for helping to
points to get minimum of 1 star. facilitate international trade and investment.

 The criteria and appraisal norms is revised  Beijing Declaration and Platform for
every 3 years or sooner. Action (BPfA) – It is one of the key guiding
documents on gender equality and
 SVA GRIHA (Simple Versatile women's empowerment.
Affordable GRIHA) – It has been
developed by ADaRSH in collaboration  It was adopted at the 4th World Conference on
with TERI and is currently under pilot stage. Women in 1995.

 It is meant to simplify, and make the greening 36. d


of small buildings (less than 2500 sqm built-up
area) affordable. World Coffee Conference (WCC 2023)

 Energy Conservation Building Code The 5th edition of the World Coffee Conference held in
(ECBC) – Initiated by the Bureau of Bengaluru.
Energy Efficiency (BEE) in 2007 and
updated in 2017, sets minimum energy  WCC – It is a property of the London-based
standards for commercial buildings. International Coffee Organization (ICO), a
body set up in 1963 under the auspices of the
 Leadership in Energy and UN.
Environmental Design (LEED) - An
environmentally oriented building certification  5th WCC in Bengaluru – It is the 1st time
program run by the U.S. Green Building India is hosting the conference.
Council (USGBC).
 Organized by – The Coffee Board
 It aims to improve building and construction (Ministry of Commerce and
project performance across 7 areas of Industry) and International Coffee
environmental and human health. Organization.

CHENNAI |SALEM| MADURAI | COIMBATORE DELHI | BANGALORE | THIRUVANANTHAPURAM


www.shankariasacademy.com 197
www.iasparliament.com

 Theme – Sustainability through circular  Hence there is no national language for


economy and regenerative agriculture. India.

 Bengaluru is the coffee capital of India, 38. a


accounting for over 70% of the country’s total
coffee production. UPAg (Unified Portal for Agricultural Statistics
(UPAg)) Portal
 International Coffee Organization
(IOC) – An intergovernmental organization  UPAg Portal aims to address the complex
bringing together exporting and importing governance challenges India’s farm sector is
Governments to tackle the challenges in the facing now.
coffee sector.
 It is a platform designed to generate crop
 Members – They represent 98% of world coffee estimates and is integrated with other systems
production and 67% of world consumption generating agriculture statistics such as price,
and India is one among them. trade, procurement, stock.

37. a  It is a crucial component of the Digital Public


Infrastructure for Agriculture.
Munshi-Ayyangar formula
 It is an initiative of the Department of
 The Munshi-Ayyangar formula is named after Agriculture and Farmers’ Welfare under
Drafting Committee members K M Munshi and the Ministry of Agriculture.
N Gopalaswamy Ayyangar.
 It aids to streamline and enhance data
 The Munshi-Ayyangar Formula was proposed management in the agricultural domain.
in 1950, which recommended Hindi in
Devanagari Script to be the Official  Key Features
Language in India along with English for a
period of 15 years.  Data Standardization – It consolidates data
from various sources into a standardized
 As part of the Munshi-Ayyangar format for easier access and understanding.
formula, Article 343 of the Constitution was
adopted in 1950.  Data Analysis – It offers insights such as
production trends and consumption patterns
 September 14 is observed as Hindi Diwas or for making informed decisions.
Hindi Day, to commemorate the Constituent
Assembly of India making Hindi the official  Granular Production Estimates – It
language of the Union government. improves the government's ability to respond to
agricultural crises swiftly.
 Article 343
 Commodity Profile Reports – The reports
1. The official language of the Union shall will be produced using algorithms, and
be Hindi in Devanagari script. provides users with comprehensive insights.

2. The form of numerals to be used for the  Plug and Play – Users will have the flexibility
official purposes of the Union shall be to use the portal's data to prepare their own
the international form of Indian reports, promoting data-driven decision-
numerals. making.

3. For a period of 15 years from the 39. d


commencement of this Constitution,
the English language shall continue to Mithuns (Bos frontalis)
be used for all the official purposes of
the Union.  Scientific name - Bos frontalis. Family
– Bovidae.
 When the 15-year period came to an end,
protests broke out over the fear of imposition of  It is a ruminant (large group of herbivores with
Hindi in large parts of non-Hindi-speaking a 4-chambered stomach) species that is found
India. in India (Northeast India), Bangladesh
Myanmar, and China.
 The resistance resulted in the Centre passing
the Official Languages Act, which stated  It is the State animal of Arunachal Pradesh
that English would continue to be upheld as an and Nagaland.
official language along with Hindi.

CHENNAI |SALEM| MADURAI | COIMBATORE DELHI | BANGALORE | THIRUVANANTHAPURAM


www.shankariasacademy.com 198
www.iasparliament.com

 Habitat – The mithun is semi-domesticated  Nilgiri Tea – It is cultivated amidst the


and is reared in a free-range forest ecosystem. mountains’ lush terrain at an elevation of 1000-
3000 ft in southern India.
 Since the soil in these parts is acidic and low in
salt content, mithuns have an affinity for salt.  Araku Coffee – It is the world’s 1st terroir
mapped coffee, grown on organic plantations in
 Food Animal Tag - Food Safety and the Araku Valley of Andhra Pradesh.
Standards Authority of India
(FSSAI) recognized this bovine as a ‘food  It is Arabica variety, known for its unique
animal’. texture and a symphony of flavours.

 Food animals are those that are raised and used  Sundarbans Honey – It is home to wild
for food production or consumption by colonies of honeybees and is less viscous than
humans. other honey.

 The recognition aids to help farmers and tribal  Kashmiri Pashmina – It refers to the raw
village communities benefit commercially from unspun wool of the Changthangi goat (the
the sale and processing of mithun meat. world’s most unique Cashmere goat) found only
at a height of 14,000 feet above sea level.
 Other animals having food animal
tag – Himalayan yak.  Zighrana Attar – A fragrance from Kannauj
in Uttar Pradesh.
40. d
 Khadi Scarf – An eco-friendly clothing material
Psyche Mission most beloved for its beautiful texture and
versatility throughout the seasons.
 Launched by - NASA in 2023
 Banarasi silk stole - Handcrafted in Varanasi,
 Aim – To study the metallic asteroid luxurious silk threads are used to create
called ‘16 Psyche’, located in the main intricate patterns.
asteroid belt between Mars and Jupiter.
 Assam stole – A piece of clothing crafted using
 Type- Orbiter Muga silk.
 Mission – By 2029, the spacecraft will begin  Kadam – Burflower tree wood handcrafted by
exploring the asteroid and will spend about 2 Karnataka artisans.
years orbiting the asteroid.
 Kanchivaram stole – It is crafted from pure
 It will take pictures, map the surface, and mulberry silk threads in Tamil Nadu.
collect data to determine Psyche’s composition.
 Ikkat stole – It is crafted by artisans
 It uses solar electric propulsion and of Odisha using mulberry silk with the
communicates with Earth through the Deep exquisite Ikkat technique (dyeing process).
Space Network (DSN) of NASA.
 Sheeshamwood Sandook With Brass Patti – A
 Jupiter Icy Moons Explorer (JUICE) strong box made of solid old wood or metal,
- European Space Agency’s (ESA) mission to with a lid on top and embellishments all over.
explore Jupiter and three of its moons -
Ganymede, Callisto and Europa.  The Sandook was handcrafted using Sheesham
(Indian Rosewood).
41. d
 Kashmiri Saffron – Known for its unparalleled
Gifts to G20 leaders culinary and medicinal value due to the crisp
air, abundant sunlight and well-drained soil of
Different heads of states & leaders got a special gift
Kashmir.
hamper from the government of India.
42. a
 Pekoe Darjeeling Tea – It is a highly
aromatic located on the misty hills of West Hoysalas
Bengal at altitudes of 3000-5000 ft.
Recently, Hoysala-era temples were added as to
 It is 1st product in India to get GI tag in 2004- UNESCO’s World Heritage List thus becoming the
05. 42nd site from India.

CHENNAI |SALEM| MADURAI | COIMBATORE DELHI | BANGALORE | THIRUVANANTHAPURAM


www.shankariasacademy.com 199
www.iasparliament.com

 Origin – They were provincial governors  The consequence was killing of the then PM
under the Western Chalukyas Indira Gandhi, by 2 Sikh guards in 1984.
(Chalukyas of Kalyani).
 It was followed by wide-spread riots against the
 They established themselves as rulers as the Sikhs across India fuelling more anti-India
Western Chalukyas and Cholas crumbled. sentiment.

 Geography – Karnataka  Operation Metal – It was limited to Golden


Temple
 Time period – 10th century to 14th century
 Operation Shop – Capturing of suspects
 Capital – It was Belur earlier and moved to from outskirts of Punjab
Halebidu or Dwarasamudra later.
 Operation Woodrose – Carried out by
 Hoysala Emblem - Figure representing Sala Indian Army throughout Punjab
attacking the tiger.
 Operation Black Thunder – 2nd phase of
 Vishnuvardhan, Veera Ballala and Operation Blue Star that began in 1988 to
Ereyanga are important rulers of this remove Sikh militants from the Golden Temple.
dynasty.
 Operation Polo – India’s police action
43. d against the princely state of
Hyderabad in 1948.
Khalistan Movement
44. d
An unprecedented diplomatic crisis emerged, after
Canadian Prime Minister accused agents of the East Asia Summit
Government of India for killing Canadian Khalistani
leader Hardeep Singh Nijjar in June 2023. Recently, 18th East Asia Summit was held in Jakarta,
Indonesia for making it a forum for all in order to
 It is a Sikh nationalist movement. strengthen cooperation.

 Objective – To create an independent state for  A leader-led regional forum, held annually to
Sikh people inside the North-Western Republic discuss political, security and economic
of India. challenges facing Indo-Pacific.

 Evolution – 1947 Partition of India left Sikhs  Meetings – Held after the annual ASEAN
in a lot of discontentment as their traditional leaders' meetings.
lands being lost to Pakistan.
 1st summit – Held in Kuala Lumpur, Malaysia
 Punjabi Suba movement was initiated in 1955 in 2005.
under Akali Dal, for re-organisation of Punjab
along linguist lines.

 It resulted in trifurcation of Punjab into


Punjab, Haryana and Himachal Pradesh.

 In sharing the waters of Ravi-Beas and Sutlej


between the 2 states, Punjab and Haryana were
the foundation on which the Khalistan dispute
was created.

 Thus in early 1980s, it had emerged as a major


separatist movement that was fed on arms
under the patronage of Pakistan’s ISI.

 Emergence of Bhindranwale – He became


the extremist voice of Sikhs and declared
himself as the protector and arbiter of Sikh
rights.

 Operation Blue Star – To capture


Bhindrawale in Harmandir Sahib Complex, but
it resulted in a strong anti-India sentiment.

CHENNAI |SALEM| MADURAI | COIMBATORE DELHI | BANGALORE | THIRUVANANTHAPURAM


www.shankariasacademy.com 200
www.iasparliament.com

 Membership – 18 countries, originally 16  Observed on – The month of Bhadrapada or


countries based on the ASEAN plus 6 Bhadraba (August–September), the day after
mechanism the Ganesh Chaturthi festival.

 10 ASEAN countries were Brunei, Cambodia,  Soulung festival – It is a harvest


Indonesia, Laos, Malaysia, Myanmar (Burma), festival observed annually by the Adi tribes of
the Philippines, Singapore, Thailand, and Arunachal Pradesh to commensurate the birth
Vietnam. and arrival of Mithun (ruminant animal) on
this earth.
 ASEAN plus 6 includes China, Japan, South
Korea, Australia, New Zealand and India.  Mithun, which has got ‘food animal’ tag is a
state animal of both Arunachal Pradesh and
 Russia and the US were added at the 6th Nagaland.
EAS in 2011.
 Other harvest festivals in India - Onam,
 Coverage – East Asian, Southeast Asian, South Makar Sankranti, Baisakhi, Lohri, Ladakh
Asian and Oceanian regions. Harvest festival, Pongal, Ugadi, Bihu, Gudi
Padwa.
 Cooperation – Convergence between ASEAN
Outlook on Indo-Pacific (AOIP) and Indo-
Pacific Ocean’s Initiative (IPOI).

 Significance – The members represent 54%


of the world’s population and account for 62%
of global GDP.

 18th Summit 2023 – It adopted the EAS


Leaders’ Statement on Maintaining and
Promoting the Region as an Epicentrum of
Growth.

 It reaffirmed Bali Principles, a shared


commitment by the principles for friendly and
mutually beneficial relations.

 It affirmed to work on Master Plan on ASEAN


Connectivity (MPAC) 2025 and ASEAN’s
development of the Blue Economy.

 It welcomed the progress made in the


implementation of the ASEAN Work Plan on
Education 2021-2025 and its alignment
with Manila Plan of Action for education.

 It supported the ASEAN Centre for Public


Health Emergencies and Emerging Diseases
(ACPHEED).

45. b 46. d

Nuakhai Juhar Festival Naganathaswamy Temple

Prime Minister greeted the people on the auspicious The Tamil Nadu Department of Archaeology is set to
occasion of Nuakhai. take up restoration of the 1,000-year-old Chola-era
Temple in Thanjavur district.
 Nuakhai – It is an agrarian
festival originated during the Vedic period  Also known as – Kailasamudaiyar Temple.
where the sages or Rishis used to talk about
Panchyajna.  Built by – Rajendra Chola I (1012-1044CE).

 Regions – Western Odisha and Southern  Dedicated to – Lord Shiva


Chhattisgarh.
 Features – Exquisite stone sculptures,
 It is known as Navakhai Parv in Chhattisgarh. distinctive makara thoranas (ceremonial
arches), ekatala vimana and a mukha mandapa.

CHENNAI |SALEM| MADURAI | COIMBATORE DELHI | BANGALORE | THIRUVANANTHAPURAM


www.shankariasacademy.com 201
www.iasparliament.com

 Inscriptions of Rajendra Chola I – It refers  Composition – Chairperson, a member


to a grant of land free of taxes by the secretary and 3 expert members, all shall be
nagaratthar of Ilaichikkudi for raising a flower civil servants appointed by the Ministry of
garden named after the king for the use of the Environment Forest and Climate Change
temple. (MoEFCC).

 Inscriptions of Kulothunga I (1070-1120  The chairperson – A minimum of 25 years of


CE) – It is called as Tamizh Koothu and refers experience in environmental, forestry, or
to a grant of land called as “Koothu kaani”. wildlife fields or substantial administrative
expertise in government.
 Recognition – Declared as a protected
monument in 2014 under the Tamil Nadu  Term – 3 years.
Ancient and Historical Monuments and
Archaeological Sites and Remains Act, 1966.  The member secretary – He must hold a rank
not lower than deputy inspector general or
 Nataraja Sculpture – The world’s tallest 27- director in the government and
foot Nataraja sculpture has been installed at
Bharat Mandapam, venue of the G20 Leaders'  Possess at least 12 years of experience in
Summit. environmental, forestry, or wildlife matters.

 It is an ashtadhatu (8-metal alloy) piece of art.  3 expert members – One each from the
environment, forest, and wildlife sectors,
 The design draws inspiration from 3 revered should have a minimum of 20 years of
Nataraja idols of Cholas era temples expertise.

 The Thillai Nataraja Temple in 48. b


Chidambaram
Liptako-Gourma Charter
 The Uma Maheswarar Temple in
Konerirajapuram The military leaders of Mali, Burkina Faso and Niger
has recently signed a mutual defence pact, Liptako-
 The Brihadeeswara (Big) Temple, a Gourma Charter in Mali's capital Bamako.
UNESCO World Heritage Site, in Thanjavur.
 The Charter established the Alliance of Sahel
47. c States (AES).

Central Empowered Committee (CEC)  Aim - To establish an architecture of collective


defence and mutual assistance.
The Supreme Court recently hands over its green
watchdog committee, the Central Empowered  Signatories – Mali, Burkina Faso and
Committee (CEC) to the Environment Ministry. Niger

 Setup in – 2002, by Supreme Court of  The charter binds the signatories to assist one
India as per orders passed in Godavarman another in the event of an attack on any one of
case. them and to prevent or settle armed rebellions.

 It was reconstituted in 2008.

 Role – It serves as a watchdog for issues


pertaining to environmental
conservation and compliance.

 Jurisdiction – It extends to the whole of


India.

 Recent notification – Instead of an ad hoc


body, it should be instituted as a permanent
statutory body on environmental issues under
the administrative control of the
Environmental ministry.

 It completely removes non-


governmental members from the
committee.

CHENNAI |SALEM| MADURAI | COIMBATORE DELHI | BANGALORE | THIRUVANANTHAPURAM


www.shankariasacademy.com 202
www.iasparliament.com

 Liptako-Gourma Region – It is the meeting Global Security Initiative and Global


point of the borders of Mali, Burkina Faso and Civilization Initiative (GCI).
Niger.
 Five Eyes Intelligence – An intelligence-
 G5 SAHEL Alliance – Mali, Burkina Faso sharing alliance of the US, UK, Australia,
and Niger were the members of the G5 Sahel Canada and New Zealand.
alliance, supported by France, with Chad and
Mauritania. 50. a

 It was launched in 2017 to tackle armed groups Smart Lander for Investigating Moon (SLIM)
linked to al-Qaeda and ISIL (ISIS) groups.
 Japan’s space Agency Japan Aerospace
 MINUSMA - United Nations Exploration Agency (JAXA) has recently
Multidimensional Integrated Stabilization launched the SLIM moon lander.
Mission in Mali (MINUSMA) was instituted in
2013 by the UNSC Resolution 2100 to perform  SLIM – It is a small-scale exploration lander
security-related tasks and support political designed for pinpoint landings on the Moon’s
processes in Mali. surface and investigate into the Moon’s origins.

 ECOWAS – The Economic Community of  The mission was dubbed as the Moon Sniper.
West African States (ECOWAS) is a regional
group of 15 countries founded in 1975 through  It will also test technology fundamental to
Lagos Treaty. exploration in low-gravity environments, an
important requirement for future scientific
49. b investigation of the solar system.

Global Security Initiative (GSI)  The Launch vehicle is H-IIA rocket.

Nepal rejected calls from China to join Global Security  This is the 1st Moon-landing attempt being
Initiative (GSI). made by JAXA.

 GSI – It is a China-led framework to  It would be the smallest and lightest spacecraft


restore stability and security in Asia, to to land on the Moon.
counter-narrative to U.S. leadership.
 The chosen landing site for SLIM is near a small
 Aim – To eliminate the root causes of crater named Shioli in the equatorial region of
international conflicts, improve global security the Moon.
governance, encourage joint international
efforts and promote durable peace and  SLIM is set to touch down on the near side of
development in the world. the moon close to Mare Nectaris, a lunar sea
that, viewed from Earth, appears as a dark spot.
 To uphold the principle of "indivisible security”,
means that no country can strengthen its own  After landing, the craft aims to analyse the
security at the expense of others. composition of olivine rocks near the sites in
search of clues about the origin of the moon.
 Major Pillars – Mutual respect, Openness
and inclusion, Multilateralism, Mutual benefit,  There is no lunar rover is loaded on SLIM.
Holistic approach.
51. d
 Principles – Stay committed to the vision of
common, comprehensive, cooperative and Daivaradhane Tradition
sustainable security.
 Daivaradhane or Bhootaradhana is an art
 Abiding by the purposes and principles of the form for worship in Tulu Nadu with kola
UN Charter. and nema festivals being a part of it.

 Taking the legitimate security concerns of all  The dialogue, costume, facial paint and
countries seriously. performance by the ‘Daiva’ varies from each
demi-god (Koragajja or Guliga Daiva).
 Peacefully resolving differences and disputes
through dialogue and consultation.  It is done periodically once a year or once in two
years.
 3 key initiatives of China’s foreign
policy – The Global Development Initiative,  It was in practice even before temples for gods
came into being.

CHENNAI |SALEM| MADURAI | COIMBATORE DELHI | BANGALORE | THIRUVANANTHAPURAM


www.shankariasacademy.com 203
www.iasparliament.com

52. d  It provides mutual recognition systems which


reduce trade barriers and costs in a global
State of the Rhino Report, 2023 market.

 The State of the Rhino Report, 2023 is released  OIML has 63 Member States.
by the US-based International Rhino
Foundation (IRF).  It plays a crucial role in harmonising national
laws and regulations on the performance of
 The 2 most significant factors causing rhino measuring instruments such as:
populations to decline are poaching and habitat
loss but climate change is also increasingly o Clinical thermometers
impacting many facets of their survival.
o Alcohol breath analysers,
 Poaching still threatens all 5 rhino species,
which are: o Radar speed measuring instruments,

1. Black Rhino o Ship tanks found at ports and

2. White Rhino o Petrol dispensing unit.

3. Greater one-horned Rhino  OIML is a single certificate accepted


worldwide.
4. Sumatran Rhino
 India became a member of the OIML in
5. Javan Rhino 1956 and signed the metric convention in 1956.

 India is home to 3,262 rhinos.  India has recently became OIML certificate
issuing authority (only 13 countries are
 The greater one-horned rhino authorised for OIML certification).
population has steadily increased over
the last century and grown about 20 % the past 54. d
decade.
ciTRAN
 The black rhino population is increasing
despite constant poaching pressure. Recently the researchers at the Indian Institute of
Science Education and Research (IISER) in Bhopal
 Namibia is home to the largest number of black have identified a Circular RNA, ciTRAN.
rhinos in the world.
 ciTRAN is a circular RNA that modulates
 South Africa continues to battle devastating HIV-1 transcription.
poaching losses of its white rhinos due to
increasing poachers.  Circular RNA plays a pivotal role in regulating
gene expression and is essential for various
 Indonesia's Ujung Kulon National Park is home biological processes.
to the world's only population of Javan rhinos.
 The establishment of how the ciTRAN
53. a modulates the virus’s transcription process,
could potentially lead to novel drugs and
International Organisation of Legal Metrology therapies to fight HIV-1.
(OIML)
 Bhashini – It is an open-source AI-powered
 The International Organization of Legal translation tool that has been designed to
Metrology (OIML) is an intergovernmental overcome the language barrier in the country.
organisation and not a specialised agency
under the United Nations. 55. d

 It was established in 1955 to promote the global Nalanda University


harmonisation of legal metrology procedures
that underpin and facilitate international trade. The Backdrop of Nalanda, a UNESCO World Heritage
site, welcomed delegates at the G20 Summit for the
 The OIML is an international standard-setting President’s dinner.
body that develops model regulations,
standards and related documents for use  Nalanda – It was an acclaimed Mahavihara, a
by legal metrology authorities and industry. large Buddhist monastery in the ancient
kingdom of Magadha (modern-day Bihar).

CHENNAI |SALEM| MADURAI | COIMBATORE DELHI | BANGALORE | THIRUVANANTHAPURAM


www.shankariasacademy.com 204
www.iasparliament.com

 An ancient centre of higher learning, founded in On the side lines of the G20 Summit in New Delhi, a
the 5th century CE by Kumaragupta. Memorandum of Understanding (MoU) was signed to
create India Middle East Europe Economic Corridor
 Vikramshila - Situated in Bihar, the ancient which will be funded by PGII.
Vikramshila University was famous as a Centre
of learning during the Pala dynasty.  PGII is an initiative of G7 to fund
infrastructure projects across the world.
 The university was established by the King
Dharmpala.  It is a developmental initiative aimed at
narrowing the infrastructure gap in developing
 Odantapuri – It is a celebrated Buddhist countries as well as help towards accelerating
centre of learning (vihara) in India, identified progress on SDGs globally.
with modern Bihar.
 Its recent project is India Middle East
 It was founded in the 7th century CE Europe Economic Corridor (IMEC).
by Gopala, the first ruler of the Pala dynasty.
 The IMEC is being envisioned as a network of
56. a transport corridors, including railway lines and
sea lanes through integration between Asia, the
Scarborough Shoal Arabian Gulf and Europe.
Recently the Philippines have executed a special  The IMEC comprises of an Eastern Corridor
operation to remove a floating barrier installed by connecting India to the Gulf region and
China at the disputed island of Scarborough Shoal. a Northern Corridor connecting the Gulf
region to Europe.
 Scarborough Shoal is a rock located in the
South China Sea.  It is seen as a counter to China’s Belt and Road
 It is approximately 120 nautical miles west of Initiative (BRI).
the Philippine island of Luzon.
58. c
 There are no structures built on Scarborough
Shoal, but the feature is effectively controlled by WHO Traditional Medicine Global Summit,
China. 2023

 The strategic Scarborough Shoal, named after a World Health Organization (WHO) has released the
British cargo vessel that ran aground on the outcome document of 1st WHO Traditional Medicine
atoll in the 18th century, was seized in 2012 by Global Summit 2023 in form of “Gujarat Declaration”.
China.
 Organized by - World Health Organization
 It is disputed (WHO).
area
between  Co-hosted by - Ministry of AYUSH.
China and
Philippines.  Venue - Gandhinagar, Gujarat.

 China claims  India is the host of WHO-GCTM (Global Centre


by far the for Traditional Medicine).
largest portion
of territory in  Gujarat declaration - It reaffirms global
an area commitment and harness the potential of
demarcated by traditional medicine to achieve health and well-
its so-called being for all.
"nine-dash
line".  It will serve as a catalyst to harness the potential
of traditional medicine through the lens of
 The 38th Parallel passes between North science and integrate the traditional medicines
Korea and South Korea and do not pass in national health systems.
through Scarborough Shoal.
59. a
57. c
Perovskite
Partnership for Global Infrastructure and
Investment (PGII) The National Centre for Photovoltaic Research and
Education (NCPRE) at the Indian Institute of

CHENNAI |SALEM| MADURAI | COIMBATORE DELHI | BANGALORE | THIRUVANANTHAPURAM


www.shankariasacademy.com 205
www.iasparliament.com

Technology Bombay has developed a perovskite solar  Both committed to ASEAN-India Plan of Action
cell. to Implement the ASEAN-India Partnership for
Peace, Progress and Shared Prosperity (2021-
 Perovskite is a naturally occurring 2025).
mineral of calcium titanate with a
chemical formula of CaTiO3.  AOIP – ASEAN Outlook on Indo-Pacific is
ASEAN’s own initiative to enhance ASEAN
 It offers excellent light absorption, charge- Community building process, as well as
carrier mobilities, high device efficiency, strengthen and complement cooperation with
colossal magnetoresistance (their electrical all partners, as reflected in the East Asia
resistance changes when they are put in a Summit Plan of Action (2024-2028).
magnetic field).
 It is adopted in Ha Noi Declaration 2020.
 It is an industry-scalable technology with
potential applications in lasers, sensors,  ASEAN – It comprises of 10 Southeast Asian
catalyst electrodes, solar and fuel cells, memory states which promotes intergovernmental
devices and spintronics (study of intrinsic spin cooperation and facilitates economic
of electrons). integration amongst its members.

 Obelisks - A new form of life lying between  In 2006, ASEAN was given observer status at
viruses and viroids. the United Nations General Assembly.

 Zosurabalpin - A new class of antibiotics to  ASEAN plus Three – It was created to


tackle Acinetobacter baumannii. improve existing ties with the People's Republic
of China, Japan, and South Korea.
 C-Bot - A coral monitoring and surveillance
robot developed by the National Institute of  ASEAN Plus Six – ASEAN plus three with
Oceanography (NIO). additional countries Australia, New Zealand
and India.
60. d

20th ASEAN India Summit

Recently, the 20th ASEAN India summit was held to


provide opportunities for the leaders to engage in
dialogue and interactions.

 It is an annual summit held since its


establishment in 2002.

 Aim – To elevate political, security, economic


and socio-cultural ties.

 20th Summit– It was held in Jakarta,


Indonesia.

 Timor Leste participated as observer country.

 Outcomes – Enhance information-sharing


and capacity-building in the maritime domain.

 Promote cooperation in areas of blue economy.


61. d
 Ensure seamless connectivity in the Indo-
Pacific. Green Revolution
 AITIGA – The ASEAN-India Trade in Goods MS Swaminathan, who played a crucial role in
Agreement (AITIGA) entered into force in combating India’s food insecurity, passed away
2010. recently.
 It sets 2025 as target for concluding the review  Norman Borlaug – Father of Green
aimed at addressing the asymmetry in bilateral Revolution
trade.
 M.S. Swaminathan – Father of Indian Green
Revolution

CHENNAI |SALEM| MADURAI | COIMBATORE DELHI | BANGALORE | THIRUVANANTHAPURAM


www.shankariasacademy.com 206
www.iasparliament.com

 Need – Post- independence, India signed a  Objective – To facilitate the marriage of


long-term Public Law (PL) 480 agreement to couples professing different faiths and
get food aid under Government agricultural preferring a civil wedding.
trade development assistance, with the US in
1954.  Eligibility conditions – The man should
have completed 21 years of age and the woman
 The imported grains would directly go to feed 18.
the people and they called it a “ship-to-mouth’
existence.  Neither should have a spouse living.

 Green revolution – It refers to the  Both should be capable of giving valid consent
large increase in production of food and should not suffer from any mental disorder
grains resulting from the use of high yielding that renders them unfit for marriage.
variety (HYV) seeds especially for wheat and
rice.  They should not be within the degrees of
prohibited relationship in such a way that
 1st Phase – Mid 1960s to mid-1970s their religion does not permit such
marriages.
o The use of HYV seeds was restricted to
the more affluent states such as Punjab,  Procedure – Parties to an intended
Andhra and Tamil Nadu. marriage should give notice to the
marriage officer of the district in which
o Further, the use of HYV seeds primarily one of them had resided for at least 30 days.
benefited the wheat growing regions
only.  The marriage has to be solemnised within 3
months of the notice.
 2nd phase- Mid-1970s to mid-1980s
 Objections – Any person can object to the
o The HYV technology spread to a larger marriage within 30 days of the publication of
number of states and benefited more the notice on the ground that it contravenes one
variety of crops. of the conditions for a valid marriage.
 Main features  The marriage officer has to inquire into the
objection and give a decision within 30 days.
o The HYV seeds
 If he refuses permission for the marriage,
o Use of chemical fertilizers and an appeal can be made to the district
pesticides court and the court’s decision will be
final.
o Application of modern farm
machines
 Severance – When a member of an undivided
o Extensive irrigation facilities family who professes Hindu, Buddhist, Sikh or
Jaina religions, gets married under SMA, it
o Multiple cropping results in his or her severance (breaking off)
from the family.
o Improved credit facilities
 The Constitution under Articles 245 & 246
o Support price policy empowers both the Parliament and the State to
enact marriage regulations.
o Improved R&D and extension
infrastructure 63. c

 The success of ‘green revolution’ during 1960s United Nations (UN)


was only possible when the semi-dwarf gene or
sd1 gene (mutant allele of plant height) was UN have completed 78 years in 2023.
discovered in the Chinese cultivar ‘Dee-geo-
woo-gen’ (DGWG).  Precursor to UN – League of Nations was
created in 1919 after World War I, as a part of
62. c the Treaty of Versailles but failed.

The Special Marriage Act, 1954  The Atlantic Charter, a joint declaration by US
and UK in 1941 (during World War II) came
A 5-judge bench of the Supreme Court has ruled and the title “United Nations” was
against legalising same sex marriage in India. adopted for the 1st time to identify the
countries that allied against Axis powers.

CHENNAI |SALEM| MADURAI | COIMBATORE DELHI | BANGALORE | THIRUVANANTHAPURAM


www.shankariasacademy.com 207
www.iasparliament.com

 It was subsequently incorporated by reference  UN 75 declaration - The year 2020 marks


in the Declaration of the United Nations in the 75th anniversary of the founding of the
1942. United Nations.

 San Francisco Conference, the UN Conference 64. b


on International Organization determined the
main principles and structure of the UN South Lhonak Lake
Charter.
A cloud burst over Lhonak Lake in North Sikkim
 Formation – It was established in 1945. resulted in a flash flood in the Teesta River in Lachen
Valley killing around 14 people.
 Members – 193, India is a founding member.
 South Lhonak Lake – It is located at 17100
 There are 6 official languages in UN. feet above sea level in Sikkim.

 Achievements of UN - It passed a resolution  It is a glacial moraine dammed lake formed


to eliminate nuclear weapons in 1946. due to the
melting of the
 In 1948, it created the World Health Lhonak
Organisation (WHO) to deal with glacier.
communicable diseases like smallpox, malaria,
HIV.  It is one of the
14 potential
 In 1950, it created High Commissioner for lakes
Refugees to take care of millions who had been susceptible to
displaced due to World War II. Glacial Lake
Outburst
 In 1972, UN Environment Program (UNEP) Flood
was created to improve their quality of life (GLOF).
without compromising that of future
generations.  Impact of
the lake
 In 2002, it established the International outburst –
Criminal Court (ICC) to try those who have It led to the breach of the Chungthang dam,
committed war crimes, genocide, and other which is the largest hydropower project in
atrocities. Sikkim.

 The UN peacekeeping missions are authorized  Chungthang dam is also a part of the 1,200-
by the UNSC, currently 12 UN peacekeeping megawatt (MW) Teesta Stage III Hydro Electric
missions are active. Project.

CHENNAI |SALEM| MADURAI | COIMBATORE DELHI | BANGALORE | THIRUVANANTHAPURAM


www.shankariasacademy.com 208
www.iasparliament.com

 Bridges such as the Indreni bridge, Ritchu 66. c


Bridge, Sangkhalang bridge, etc. were washed
away in the floods. Tele MANAS

 Teesta – It is a major transboundary river that India’s round-the-clock mental health helpline, Tele-
flows through India and Bangladesh. MANAS has completed one year.

 It originates from the Pahunri (or Teesta  Tele MANAS – Tele Mental Health Assistance
Kangse) glacier and flows through Sikkim and and Networking across States
West Bengal before flowing into Bangladesh.
 It is the National Tele Mental Health
 It is a tributary of Jamuna Programme of India and is conceptualised as
River (Brahmaputra River). the “digital arm of the National Mental Health
Programme”.
65. b
 Launched in – 2022, on the World Mental
Alternate Dispute Resolution (ADR) Health Day (October 10).

 It denotes a wide range of dispute resolution  Guiding agency – Ministry of Health and
processes and techniques that parties can use to Family Welfare (MoHFW)
settle disputes with the help of a 3rd party.
 Nodal centre – National Institute of Mental
 ADR Mechanism – It includes arbitration, Health and Neuro Sciences (NIMHANS) in
mediation, conciliation and negotiation. Bengaluru

 Arbitration – The resolution parties refer  Technical support by – International


their dispute to one or more persons called Institute of Information Technology-
arbitrators. Bangalore (IIITB)

 It is brought before an Arbitral Tribunal, which  Objectives – To provide universal access to


renders a binding award on the parties equitable, accessible, affordable and quality
mostly except in few interim cases. mental health care through 24X7 tele-mental
health services (number - 14416)
 Negotiations - It is a voluntary, binding
procedure without the involvement of any 3rd  To scale up the reach of mental health services
party to obtain a negotiated settlement. to anybody who reaches out, across India, any
time
 It can occur in business, non-profit
organisation, government branches, legal  To implement a full-fledged mental health-
proceedings, International relation and service network
personal circumstances such as marriage,
parenting, divorce and regular life.  It is a 2-tier system.

 Mediations – A neutral person is known as a  Tier 1 – It will comprise the State Tele MANAS
‘mediator’ who assists the parties in attempting cells, which includes trained counsellors and
to obtain a mutually acceptable resolution of mental health specialists.
the disagreement through mediation.
 Tier 2 – It will comprise specialists at District
 The 3rd party does not make any decision for Mental Health Programme (DMHP)/Medical
the parties, instead, it serves as a facilitator who College resources for physical consultation
helps them improve their interaction, and thus and/or eSanjeevani for audio visual
it is also called as facilitated negotiation. consultation.

 Conciliation – It is a non-binding  It is implemented via a National


procedure. Technical Advisory Group (NTAG) and 3
technical advisory sub-committees that
 The parties are assisted by an impartial 3rd includes:
party, the Conciliator, in finding a mutually
satisfactory agreed settlement of the disputes. o Mental Health Service Delivery

 The parties have the right to accept or o Information Technology Architecture


reject the conciliator’s recommendations.
o Health Systems
 If both parties accept the conciliator’s
settlement, it will be final and binding.

CHENNAI |SALEM| MADURAI | COIMBATORE DELHI | BANGALORE | THIRUVANANTHAPURAM


www.shankariasacademy.com 209
www.iasparliament.com

 MANOASHRAYA – Dashboard on  Financing bodies – Asian Infrastructure


Rehabilitation Homes (RH)/ Halfway Homes Investment Bank (AIIB) and Silk Road
(HH) in the country. Fund (SRF).

 Other alternatives for BRI are

o Build Back Better World


Initiative (B3W) – Launched by G7
in 2021, later renamed as Partnership
for Global Infrastructure and
Investment (PGII).

o Global Gateway Programme


(GGP) – By EU in 2023

o India-Middle East-Europe
(IMEC) corridor – Proposed at the
New Delhi G20 summit, 2023.
67. d

China’s Belt and Road Initiative (BRI)

The year 2023 marks a decade since China’s ambitious


infrastructure funding project, the BRI, was first
outlined by President Xi Jinping.

 Launched in – 2013

 ‘Belt’ Plan – It was to revitalise a series of


trading and infrastructure routes between Asia
and Europe.

 Connectivity through Central Asia was a key


element of the initiative.

 ‘Road’ Plan – A sea trade infrastructure called


‘Road’ was announced to connect China with 68. d
Southeast Asia, Europe and Africa.
Sveriges Riksbank Prize in Economic Sciences
 The major focus has been to build ports, (Nobel Prize for Economics) 2023
bridges, corridors, etc. throughout South East
Asia and Indian Ocean. Recently, Ms. Claudia Goldin won the 2023 Nobel Prize
for economics
 Together, these initiatives were referred to as
the One Belt One Road Initiative (OBOR).  Award for – The research work which is
the 1st comprehensive account of women’s
 Aim – To resolve China’s capital surplus and earning and labour market participation
industrial overcapacity and to increase its through the centuries.
political influence in broader regions.
 Work of Claudia Goldin – She studied the
 Partnership – about 147 countries have archives of about 200 years of United
signed on to projects or indicated an interest in States of America to demonstrate how and
doing so. why gender differences in earnings and
employment rates have changed over time.
 Italy is the only G7 country to sign up for
this.  Understanding the Gender Gap: An
Economic History of American Women
 It has 6 international economic
corridors, spanning across Asia, Europe and  Significance – She is only the 3rd women to
Africa. have won the prize (for Economics) after Elinor
Ostrom in 2009 and Esther Duflo in 2019.
 Principles of BRI – Policy coordination,
infrastructure connectivity, trade, financial  She is 1st women to get Economic Nobel as a
integration, people-to-people connections and solo winner.
industrial cooperation

CHENNAI |SALEM| MADURAI | COIMBATORE DELHI | BANGALORE | THIRUVANANTHAPURAM


www.shankariasacademy.com 210
www.iasparliament.com

69. a  Statutory backing by – Environment


Protection Act, 1986.
Environment Impact Assessment (EIA)
 In 2006, State governments were also
Recent Teesta dam breach in Sikkim and the flood and given powers to issue Environmental
landslides in Himachal Pradesh are stark reminders Clearances in certain cases.
for the need for a different set of environmental
standards and clearances in Indian Himalayan  Only projects enumerated in the schedule
Region. attached to the notification require prior EC.
 EIA – UNEP defines EIA as a tool used to  Categorisation – Mining, extraction of
identify the environmental, social and natural resources and power generation and
economic impacts of a project prior to decision- physical infrastructure.
making.
 The threshold limit beyond which EIA is
 It compares various alternatives for the warranted for all these projects is the same
proposed project and analyses all possible across the country.
environmental repercussions in various
scenarios  Benefits – It helps to decide appropriate
mitigation strategies and also to reduce cost
and time of project implementation.

70. a

White Phosphorus

Amnesty International and Human Rights Watch have


accused the Israel Defense Forces of using white
phosphorus munitions in Gaza and Lebanon.

 White phosphorus – It is a pyrophoric


substance that ignites when exposed to
oxygen and is one among the most unstable
pyrophoric substances.
 Objectives – To predict environmental
impacts at an early stage in project planning  Properties – It emits a distinct garlic-like
and design odour and produces thick, light smoke with
intense 8150 Celsius heat.
 To find ways to reduce adverse impacts and to
shape projects to suit the local environment.  Usage – It is dispersed in artillery shells,
bombs, rockets and via felt (textile) wedges
 To present the predictions and options to soaked in the chemical.
decision-makers.
Military applications
 In India – EIA was introduced in 1978 with
respect to river valley projects.  As smokescreen, it hides ground troop
movement

 As an incendiary weapon to start fire

 Confuses the infrared optics and weapons


tracking systems & protects the forces from
guided missiles.

 Impact on humans – Can cause burns


even up to bones.

 Inhalation causes respiratory damage, harm


internal organs.

 Impact on Environment – It can devastate


infrastructure and property, damage crops and
kill livestock.

CHENNAI |SALEM| MADURAI | COIMBATORE DELHI | BANGALORE | THIRUVANANTHAPURAM


www.shankariasacademy.com 211
www.iasparliament.com

 Regulation – It is not explicitly banned by  It reflects chronic undernutrition.


international conventions but their use is
regulated under the International  Child wasting – Share of children under age 5
Humanitarian Law (IHL). who have low weight for their height.

 It is not considered a chemical  It reflects acute undernutrition.


weapon because its operational utility is
primarily due to heat and smoke, rather than  Child mortality – Share of children who die
toxicity. before their 5th birthday.

 IHL – It lays down the responsibilities of states  It reflects inadequate nutrition and unhealthy
and non-state groups in an armed conflict, and environments.
is also called ‘the law of war’ or ‘the law of
armed conflict’.  Scores – It is calculated on a 100-point scale
reflecting the severity of hunger based on the 4
 Protocol III of the 1980 Convention on Certain indicators.
Conventional Weapons prohibits incendiaries
or the use of other substances to attack civilian  Zero “0” is the best possible score (no hunger)
populations. and “100” is the worst.

 Palestine and Lebanon have joined Protocol III,  2023 Report – Global hunger remains too
while Israel has not ratified the protocol. high, and progress on reducing hunger has
largely stalled.
71. a

Global Hunger Index (GHI), 2023

India ranks 111 out of a total of 125 countries in the


Global Hunger Index 2023.

 GHI – A tool for comprehensively measuring


and tracking hunger at global, regional, and
 It is due to the combined effects of overlapping
national levels.
crises like COVID-19 pandemic, Russia-
Ukraine war, economic stagnation, impacts of
 Launched in – 2006
climate change and conflicts among many
 Published by – Concern countries.
Worldwide & Welthungerhilfe
 India – With a score of 28.7, hunger level is
 4 Indicators serious.

 Undernourishment – Share of the  It has the highest child wasting rate in the
population with insufficient caloric intake. world, at 18.7%, reflecting acute
undernutrition.
 It is based on an opinion poll conducted on a
72. a
very small sample size.
World Energy Outlook, 2023
 Child stunting – Share of children under age
5 who have low height for their age.  It is an annual report.

 Launched in – 1998, by International


Energy Agency (IEA)

 2023 findings for India– Residential


ownership of Air-conditioner usage to see 9-
fold increase by 2050.

 India’s power consumption for ACs by 2050


will exceed the whole of Africa’s demand
existing today.

 Energy supply to rise to 60.3 exajoules (EJ) by


2050.

CHENNAI |SALEM| MADURAI | COIMBATORE DELHI | BANGALORE | THIRUVANANTHAPURAM


www.shankariasacademy.com 212
www.iasparliament.com

 Annual CO2 emissions to rises nearly 30% by longer buffer risks and provide its expected
2050. functions.

 Over the past 5 decades, more than 700 73. d


heatwave events occurred.
Moscow Format
International Migration Outlook, 2023
 The Moscow format is one of the
 It is an annual publication. several dialogue platforms on
Afghanistan, which began before the
 Published by – Organisation for Taliban takeover of Kabul.
Economic Co-operation and
Development (OECD)  Members - Russia, China, Pakistan, Iran,
Kazakhstan, Tajikistan, Kyrgyzstan,
 2023 findings – India saw the highest Uzbekistan, Turkmenistan and India.
migration flows to OECD countries in 2021 and
2022.  Introduced by - The Russian government in
2017.
 India replaced China as the main country of
origin of new migrants to OECD countries in  Aim - To encourage a peaceful conclusion to
2020. the violence and instability in Afghanistan

 India was followed by China and Romania in  The Kazan Declaration is a document that
terms of new migrants. resulted from the 5th regional consultation on
Afghanistan, known as the Moscow Format,
 Due to the ongoing Russia-Ukraine war, inflows which took place in Kazan, Russia.
of refugees from Ukraine to OECD nations
reached the highest level on record.  The Kazan Declaration underscores the call
for the Afghan authorities to create conditions
Interconnected Disaster Risks report, 2023 that improve the well-being of the
Afghan people, deter further migration, and
 It is released annually. facilitate the return of refugees.

 Launched in – 2021, by Institute for 74. b


Environment and Human Security (UNU-
EHS), an arm of UN University at Germany. Mussel Species

 2023 findings – World is inching closer to 6 Recently National Green Tribunal has sought a report
interconnected risk tipping points, describing on removal of invasive mussel species from Ennore-
immediate and increasing risks across the Pulicat wetland of Tamil Nadu.
world.
 ‘Mytella strigata’ is an invasive South
 Aquifers are depleting faster than they can be American mussel species.
replenished.
 Mytella strigata is referred locally as ‘kaaka
 In last 100 years, more than 400 vertebrate aazhi’.
species were wiped out.
 It is wiping out the locally prevalent and
 Melting of glaciers is at double the speed commercially valuable yellow clams (manja
relative to past 2 decades. matti) and green mussels (pachai aazhi).

 Around 50% of glaciers (excluding Greenland  It has spread over the riverbed, preventing
and Antarctica) would be lost by 2100 prawns from grazing or burying themselves in
irrespective of any current efforts. the sediment.

 About 75% objects orbiting around earth are  Because of the invasive mussels, the river
junks. bottom is suffocated with one-foot-deep sludge
of black, foul-smelling slimy excreta.
 High heat with humidity, hinders sweat
evaporations and may cause organ failure and  kaaka aazhi is not an alien species as it has an
brain damage. entry in the Wildlife Protection Act, 1972
(WPA).
 Risk tipping point – It is the moment at
which a given socio-ecological system can no  ‘Unionidae’ is the largest family of freshwater
mussels that is found mostly in USA.

CHENNAI |SALEM| MADURAI | COIMBATORE DELHI | BANGALORE | THIRUVANANTHAPURAM


www.shankariasacademy.com 213
www.iasparliament.com

75. d  Focus – Water conservation and afforestation.

SIMBEX  Digital platform – It will streamline


registration of projects, its verification, and
 It is a bilateral naval exercise between India issuance of Green Credits.
and Singapore.
 Green Credit Registry and trading
 The 30th Edition of the Singapore India platform – It allows buying and selling of
Maritime Bilateral Exercise (SIMBEX), 2023 Green Credits.
was held in the southern parts of the South
China Sea. 77. d

SAMPRITI  Operation Kachchhap - Operation


Kachchhap is part of a series of determined
 It is a joint army exercise between India and efforts by the Directorate of Revenue
Bangladesh. Intelligence (DRI) to combat illegal wildlife
trafficking and preserve the environment.
 11th edition (SAMPRITI-XI) of annual joint
military exercise was held at Meghalaya, India.  The rescued turtles are Indian Tent Turtle,
Indian Flapshell Turtle, Crown River Turtle,
CHAKRAVAT Black spotted/Pond Turtle and Brown Roofed
Turtle.
 CHAKRAVAT is Annual Joint HADR
(Humanitarian Assistance and Disaster  Operation Sarvashakti - An anti-terror
Relief) Exercise. campaign in Jammu and Kashmir's Pir Panjal
Range.
 The exercise involves multi agency
participation.  Conducted by - The Indian Army
 The 2023 edition would further synergise  Project Udbhav - The Indian Army has
efforts at the national level among all started an initiative, named Project Udbhav in
stakeholders, as well as witness participation collaboration with the United Service
from 8 countries of the Indian Ocean Region. Institution (USI) of India.
 The exercise has been conducted by  It aims to rediscover the Indic heritage of
Indian Army, Indian Navy (IN) and statecraft and strategic thoughts derived from
Indian Air Force (IAF) in rotation since ancient Indian texts of statecraft, warcraft,
2016. diplomacy and grand strategy.
 The 2022 edition of the exercise was conducted  It focuses on indigenous military systems,
at Agra by IAF. historical texts, regional texts and kingdoms,
thematic studies, and intricate kautilya Studies
 The 2023 edition of the exercise is being hosted from 4th century BCE to the 8th century CE.
by the Indian Navy at Goa in the month of
October.  The first scholarly outcome of the initiative is
the Paramparik Bhartiya Darshan Ranniti aur
76. c
Netriyta ke Shashwat Niyam.
Green Credit Program (GCP)
78. c
 It is an innovative market-based mechanism. Eastern Ghats
 Objective – Incentivize voluntary  It is a discontinuous range of
environmental actions across diverse mountains along India’s eastern coast.
sectors by various stakeholders like individuals,
communities, private sector industries, and  It is an ancient orogenic belt formed from the
companies. collision of crustal rocks during the Archean
Era.
 GCP's governance framework – Inter-
ministerial Steering Committee and GCP  Geographical location – Odisha, Andhra
administrator. Pradesh, Tamil Nadu, Karnataka and
Telangana.
 GCP Administrator – Indian Council of
Forestry Research and Education  Major rivers – Godavari, Mahanadi, Krishna,
(ICFRE). and Kaveri

CHENNAI |SALEM| MADURAI | COIMBATORE DELHI | BANGALORE | THIRUVANANTHAPURAM


www.shankariasacademy.com 214
www.iasparliament.com

 The Eastern Ghats are lower in height than the Recently the Cabinet approved royalty rates for
Western Ghats. mining of 3 critical and strategic minerals such as
Lithium, Niobium and Rare Earth Elements (REEs).
 Important hills - Shevaroy hills, Bilirangan
hills, Nallamala Hills, Mahendragiri, Javadi  Niobium also known as columbium, is a
hills, etc. chemical element with the symbol Nb (formerly
Cb) and atomic number 41.
 Rocks – Made up of charnockite, granite
gneiss, khondalite, granite and quartzite  Niobium is a light grey, crystalline and ductile
transition metal.
 Minerals found– Limestone, bauxite and
iron ore.  Pure niobium has a hardness similar to that of
pure titanium, and it has similar ductility to
 Madhurawada Dome – It was formed by a iron.
tectonic arrangement with the khondalite suite
and quartzite Archean rocks north of  Niobium is used in alloys, tools and dies and
Visakhapatnam. superconductive magnets.

79. d  Niobium in one of the Critical Minerals in


India and not one of the 17 rare earth
United Nation Forum on Forests (UNFF) elements (REE).
The Ministry of Environment, Forest and Climate  The main source of Niobium element is the
Change has organised a Country-Led Initiative (CLI) mineral columbite.
event as part of the United Nations Forum on Forests
(UNFF) in 2023 at the Forest Research Institute (FRI)  This mineral also contains tantalum and the
in Dehradun, Uttarakhand. two elements are mined together.

 It is a subsidiary body within the UN  Columbite is found in Canada, Brazil, Australia,


Organization. Nigeria and elsewhere.

 Formation Year - 2000  Some niobium is also produced as a by-product


of tin extraction.
 Objectives – To promote management,
conservation& sustainable development of all  The Mines and Minerals (Development and
types of forests. Regulation) Amendment act, 2023 delisted 6
minerals, including Lithium and
 Formed by - Resolution 2000/35 of Niobium, from the list of atomic
Economic and Social Council of the United minerals.
Nations (ECOSOC)
81. a
 Membership - Universal membership that
comprises all member states of the UN and its Habitat rights to PVTGs
specialized agencies
Recently Baiga PVTG in Chhattisgarh become the 2nd
 India’s Membership - Founding member in the State to get habitat rights after Kamar PVTG.
of UNFF
 Habitat – It includes the area comprising the
 The 19th session of UNFF is scheduled at UN customary habitat and such other habitats in
Headquarters, New York in May 2024. reserved forests and protected forests of
primitive tribal groups and pre-agricultural
 Target basis – Based on Rio Declaration, Forest communities and other forest dwelling
Principles, Agenda 21 and outcome of the Scheduled Tribes.
Intergovernmental Panel on Forests (IPF)/
Intergovernmental Forum on Forests (IFF) and  Habitat rights – Rights over their customary
other key milestones of international forest territory of habitation, socio-cultural practices,
policy. economic and livelihood means, intellectual
knowledge of biodiversity and ecology,
 Collaborative Partnership on Forests was traditional knowledge of use of natural
established in 2001 to Support the work of resources, as well as protection and
UNFF and its member countries. conservation of their natural and cultural
heritage.
80. d

Niobium

CHENNAI |SALEM| MADURAI | COIMBATORE DELHI | BANGALORE | THIRUVANANTHAPURAM


www.shankariasacademy.com 215
www.iasparliament.com

 However, it excludes any traditional right of  History – Vohra committee report submitted
hunting or trapping or extracting a part of the in 1995 pointed out the nexus between criminal
body of any species of wild animal gangs, police, bureaucracy and politicians have
come clearly out in various parts of the country.
 They are given to PVTGs under Section 3(1) (e)
of The Scheduled Tribes and Other Traditional Ethics Committee
Forest Dwellers (Recognition of Forest Rights)
Act, 2006 (Forest Rights Act, 2006). About Lok Sabha Rajya Sabha

 The title may not be an ownership title in the Established 2000 1997
nature of a private property owner, but consent
and consultation of the gram sabha will be Should not 10 members
needed for any developmental activity. Members contain more including
than 15 members Chairman
 Coverage – As of April 2024, out of 75 PVTG
in India, only 3 have habitat rights. Nominated by Speaker Chairman

o 1st one was Bharia PVTG in It is appointed by


It is from the
Madhya Pradesh Speaker amongst
Chairperson largest party in
the Committee
the House
o The Kamar and the Baiga tribe in members
Chhattisgarh.
Not exceeding 1 Not exceeding 1
Term
 PVTGs – There are 75 PVTGs out of 705 STs, year year
spread over 18 states and 1 Union Territory
(A&N Islands).  It enforces the code of conduct
of members of Parliament.
 Highest PVTGs is present in Odisha followed by
Andhra Pradesh.  It examines the cases of
misconduct and recommends
Functions
 The Baiga – An ethnic group found in Central appropriate action.
India primarily in the state of Madhya Pradesh,
and in smaller numbers in Uttar Pradesh,  It is engaged in maintaining
Chhattisgarh and Jharkhand. discipline and decorum in
Parliament.

83. a

The Mediation Act, 2023

Recently, some portions of the Mediation Act of 2023


came into force.

 Mediation – It is a voluntary, low-cost, and


82. c confidential process where disputing parties
mutually find a solution with the help of an
Ethics committee impartial third-party mediator.

The Lok Sabha Ethics Committee inquired regarding a Alternate


complaint of cash for query against a sitting Member Dispute Statutory backing of Mediation
of Parliament. Resolution

 An allegation of corruption against an MP – It  Legal Services Authority Act of


can be sent to either Ethics or Privileges  Mediation
1987
Committee, but usually more serious
accusations go to the latter.  Arbitration
 MSME Development Act of 2006

 Privileges Committee – It can take up cases  Conciliation


 Companies Act of 2013
of misconduct that involve both MPs and
non- MPs.  Negotiation
 Commercial Courts Act, 2015

 Ethics Committee – It can take up only  Mediation Act of 2023


cases of misconduct that involve MPs.

CHENNAI |SALEM| MADURAI | COIMBATORE DELHI | BANGALORE | THIRUVANANTHAPURAM


www.shankariasacademy.com 216
www.iasparliament.com

 Features of the Mediation Act – It aims to foster  It may be challenged on grounds of fraud,
a link between the mediation and the corruption, impersonation, or related to
arbitration of commercial disputes. disputes not fit for mediation.

 Parties must attempt to settle civil or  Community mediation – It will be


commercial disputes by mediation before conducted by a panel of 3 mediators to resolve
approaching any court or certain tribunals. disputes likely to affect the peace and harmony
amongst residents of a locality.
 Even if they fail to reach a settlement through
pre-litigation mediation, the court or tribunal Arbitration
may at any stage refer the parties to mediation and Mediation Act
if they request for the same. About
Conciliation 2023
Act 2021
 Excluded list of disputes – Not fit for
mediation
Arbitration
o Related to claims against minors or Institutional Council of India Mediation
persons of unsound mind, mechanism (Yet to be Council of India
implemented)
o Involving criminal prosecution, and

o Affecting the rights of third parties. Mediation service


Service Mediation &
 The central government may amend this list. providers services secretarial
assistance
 Applicability – To mediations conducted in
India Mediated
Verdict Arbitral award settlement
o Involving only domestic parties agreement

o Involving at least one foreign party and


relating to a commercial dispute (i.e., 84. b
international mediation)
Household Savings
o If the central or state government is a
party, the Act will apply to commercial Household sector’s savings in financial assets has
disputes, and other disputes as shown a sharp decline to 5.1% of GDP in 2022-23.
notified.
 Household savings – It refer to money left
 Mediation process – It must be after the household pays taxes and
completed within 180 days (may be extended spends on the consumption of goods and
by 180 days by the parties). services.

 A party may withdraw from mediation after two  3 components


sessions.
o Financial assets – Currency, bank
 Court annexed mediation – It must be deposits, pension, insurance, equity
conducted as per the rules framed by the and related products.
Supreme Court or High Courts.
o Physical assets – Investment in
 Mediators – They may be appointed by fixed assets of construction

o The parties by agreement, or o Gold and silver ornaments

o Mediation service provider (an  Household savings rate = (Household


institution administering mediation). savings/Personal disposable income) x 100%

 Mediation Council of India – Established by  Factors influencing Household Savings –


Central government. Income, interest rate, future income
expectation, wealth and tax.
 Nature of the mediation – Agreements
resulting from mediation (other than  Status of Household Savings – The
community mediation) will be final, binding, Reserve Bank of India (RBI) data showed that
and enforceable in the same manner as court household financial savings were at 5.1% of
judgments. GDP in FY23, almost a 40-year low.

CHENNAI |SALEM| MADURAI | COIMBATORE DELHI | BANGALORE | THIRUVANANTHAPURAM


www.shankariasacademy.com 217
www.iasparliament.com

 The annual financial liabilities of households  Earlier Amendments – 1982, 1991, 1993, 2002
increased by 5.8% of GDP in FY23, signalling an and 2006.
unusually high reliance on loans for
consumption and real estate purchases.  2022 Amendment – It empowers Centre
to notify a conservation reserve.
 This rate of increase in financial liabilities was
the 2nd highest since India's Independence.  Rationalised Schedules – Reduces the number
from 6 to 4
 Household assets saw a sharp decline from Rs
22.8 trillion in FY21, to Rs 13.76 trillion in o Schedule I — Animal species with
FY23. highest level of protection including
those which are critically endangered.
 Household debt, as measured by the stock of
financial liabilities, remained significantly o Schedule II – Animal species with a
elevated at 37.6% of GDP in FY23. lesser degree of protection

 The primary reasons behind the subdued o Schedule III – Protected Plant
savings and increased borrowing seem to species
be stagnant or declining incomes for
households and small and medium-sized o Schedule IV – Specimens listed in the
enterprises (SMEs), occurring in the midst of Appendices under CITES (scheduled
high inflation. specimens)

 Implementation of CITES – Introduced of new


chapter for this and enables Centre to designate
a management and scientific authority.

 Prohibit invasive alien species – Empowers


Centre to regulate or prohibit the import, trade,
possession or proliferation of such species.

 Definition of Vermin – It means any wild


animal notified under Section 62 of Wildlife
Protection Act.

 Decentralisation of power – Allow the state to


form an Advisory Committee consisting of
Chief Wildlife Warden, members of legislature,
wildlife NGOs and Panchayat Raj to manage
wildlife sanctuaries.

 Empowers the Gram Sabha and other local


85. b village institutions to collectively protect the
forests, wild animals and biodiversity and take
Wildlife Protection (Amendment) Act, 2022 action against any activity posing threat to
wildlife.
Excessive number of species have been included in the
new schedules of the Wildlife Protection (Amendment)  The State may declare areas adjacent to
Act, 2022, with no consultation or process. national parks and sanctuaries as a
conservation reserve, for protecting flora and
 Enacted in – 1972. fauna, and their habitat.
 Objectives – To prohibit the hunting of wild  Surrender of captive animals – Persons shall
animals and to protect and manage of wildlife voluntarily surrender any captive animals or its
areas and the species in these areas. products to the Chief Wild Life Warden.
 To establish new protected areas such as o No compensation will be paid to the
National Parks and Wildlife Sanctuaries. person for surrendering and such items
become property of the State
 Importance – It paved the way for government.
establishment of The National and State Board
for Wildlife, Central Zoo Authority and  Extended functions of Zoos – It will also
National Tiger Conservation Authority serve as the areas for ex-situ
(NTCA). conservation, rescue centers and breeding
centers apart from exhibition centres.

CHENNAI |SALEM| MADURAI | COIMBATORE DELHI | BANGALORE | THIRUVANANTHAPURAM


www.shankariasacademy.com 218
www.iasparliament.com

 Dilution of conservation – It allows low o Smaller dots – Emits bluer light


intensity, small-scale fishing in rivers and other
water bodies in and around protected areas. o Larger dots – Emits redder light

 Relaxes the norms on grazing of cattle and use  This happens because light shone on the dot
of drinking water by the local communities energises some electrons to jump from one
living inside the protected areas until they are energy level to a higher one, before jumping
settled elsewhere. back and releasing the energy at a different
frequency.
 Allows transfer or transport of a captive
elephant for a religious or any other purpose 87. b
subject to such terms and conditions as may be
prescribed by the Central Government. Phonotaxis

 Animal Welfare Board of India – It is a  Phonotaxis is the movement of an animal


statutory advisory body on Animal Welfare in response to a sound.
Laws.
 It has mostly been observed among crickets,
 It was established in 1962 under Section 4 of moths, frogs, and toads, among a few other
the Prevention of Cruelty to Animals Act, creatures.
1960.
 The click of crickets in the evening or frogs
86. c croaking during the monsoons are due to
Phonotaxis.
Quantum Dots
 There are 2 types of Phonotaxis:
Nobel Prize for Chemistry 2023 has been awarded to
Alexei I. Ekimov, Louis E. Brus, and Moungi G. 1. Positive
Bawendi for the discovery and synthesis of quantum
dots. 2. Negative

 Quantum dots – They are man-made  The purpose of positive phonotaxis is


nanoscale crystals that exhibit unique attraction.
optical and electronic properties, including the
ability to transport electrons and emit light of  It usually happens when the females of a
various colors when exposed to UV light. particular species are attracted to the sounds
made by the males.
 Effect of light– When light is shined on a
quantum dot, it absorbs and then re-emits it at  Negative Phonotaxis indicate to repel or warn
a different frequency. or move away.

 By manipulating their size, one can  Crickets in particular have been found to steer
precisely control their properties and make themselves away from low-intensity ultrasound
them emit light of specific colours when typically associated with bats (which use it for
they are excited by light echolocation).

88. b

Dancing Frogs

The Wildlife Trust of India after analysing the second


edition of the Global Amphibian Assessment said that
the dancing frogs are the most threatened amphibian
genus of India.

 Dancing frogs are members of the Micrixalus


genus and there are about 24 frog species in this
family.

 Their preferred habitats are shola grasslands,


myristica swamps and evergreen forests within
the Western Ghats, where they mainly reside
near slow-moving perennial streams.

CHENNAI |SALEM| MADURAI | COIMBATORE DELHI | BANGALORE | THIRUVANANTHAPURAM


www.shankariasacademy.com 219
www.iasparliament.com

 The males stretch up their hind legs one at a  QR codes can hold more data than barcodes.
time and wave their webbed toes in the air in a
rapid motion akin to a dance.  They are less likely to be damaged than
barcodes.
 The signature dance move called ‘foot-
flagging’ serves the dual purpose of attracting  QR codes can be read from all directions,
a female while sending out a warning signal to while barcodes can only be read from one
other male frogs in the area. direction.

 According to the 2nd edition of the Global  QRs have improved security with encryption.
Amphibian Assessment, the dancing frogs are
the most threatened amphibian genus of India. 90. a

 It is also the 5th most threatened genus in the Pichwai Painting


world with 92% of its species in the threatened
category.  Origin – It is a 400-year-old art whose roots
are from Nathdwara town in Rajasthan.
 Budgett Frog - Other common names are
Hippo Frog and the Freddie Kruger Frog.  Theme – Various stages of Lord Krishna’s
life, including his childhood, youth, and
 Habitat – In inland waters in Paraguay, adulthood.
Argentina and Bolivia.
 It is typically done on cloth, usually khadi but
 They are highly intelligent and very aggressive. now being produced on various mediums such
as paper, canvas, and silk.
 When frightened, they inflate themselves,
stand up on their short legs and attack the  Different schools of Pichwai — Nathdwara,
potential with an open huge mouth followed up Kishangarh and Bundi in Rajasthan and Deccan
by a high-pitched scream. school

 Purple Frog - Also known as pignose frog or  Materials used – Natural colors made from
Mahabali frog. minerals and plant extracts.

 It is believed to have co-existed with  Stone pigments are used for gold and silver
dinosaurs. tones.

 It remains underground for almost a  Usage – As a backdrop for hindu deities in


year except for 2-3 weeks during the monsoon temples.
as it comes out to mate.
 They are typically hung behind the idol of
Shrinathji, a local form of Krishna and the
centre of Pushtimarg worship.

Dogra architecture

 Dogra architecture is a distinct culture that


formed in Jammu, where cultures mixed
together.

 Dogra Dynasty – Dynasty of Hindu Rajputs


who ruled Jammu & Kashmir from 1846 to
89. b 1947.

Quick Response (QR) Codes  Some examples of Dogra architecture include:


Recently the Cybersecurity experts have found hackers o Mubark mandi – A hub of Dogra
using Quick response (QR) codes for phishing culture until 1947, with a jharokha style
activities. of balcony.

 QR was invented by Masahiro Hara, a Japanese o Maharaj Gunj – A market area with a
engineer in 1994. variety of colonial and vernacular
architecture.
 QR Code is a type of 2D barcode or printed
representation of data that can be scanned for o Rani Charak Mahal – A mahal with
data retrieval whereas barcode has 2 types - 1 phenomenal architecture and interiors.
dimensional and 2 dimensional.

CHENNAI |SALEM| MADURAI | COIMBATORE DELHI | BANGALORE | THIRUVANANTHAPURAM


www.shankariasacademy.com 220
www.iasparliament.com

Vajra Mushti Kalaga  It is situated between Weddell Sea and


Ross Sea.
 The ‘’vajra mushti kalaga’’ is a form of
wrestling or a traditional Indian martial  Cause of melting – Due to the warming of the
art that incorporates various hand-to-hand Southern Ocean mainly the Amundsen Sea
combat techniques, such as grappling, region.
wrestling, and striking.
 Amundsen Sea, off the coast of West
 It entails two jettys taking a swipe at each Antarctica, will warm roughly 3 times faster
other’s head with a small metal weapon called a than the historical rate through the rest of this
knuckleduster. century which will lead to much more rapid
melting of ice shelves.
 Vajra Mushti matches are still held during
the annual Mysore Dasara festival at  Impacts – Sea-level rises of around 1 metre
Mysore Palace. may threat hundreds of millions of people
worldwide at risk of coastal flooding.
 The tradition dates back to the Wadiyar
dynasty in 1610. 92. a

91. c Certification and Certification Agencies

West Antarctic Ice Shelf Melt Certification


Purpose Issued by
Mark
 West Antarctic Ice Sheet – It is Antarctica’s
largest contributor to sea-level rise which has ISI Mark Certifies industrial
enough ice to increase the global mean sea level (Mandatory products comply Bureau of Indian
by as much as 5 metres (m). for certain with Indian Standards (BIS)
products) Standards
 Thwaites Glacier, widest glacier on Earth is
located in this region. BIS Certifies purity of
Bureau of Indian
Hallmark gold and silver
Standards (BIS)
(Voluntary) jewellery

Directorate of
Marketing and
Inspection, Depar
tment of
Certifies Agriculture,
agricultural Cooperation and
AGMARK
products conform Farmers Welfare,
(Voluntary)
to approved Ministry of
standards Agriculture &
Farmers Welfare
under the
Agricultural
Produce Act,1937

Certifies processed
fruit products were
made in a hygienic
FPO Mark Fruit Products
‘food-safe’
(Mandatory) Order, 1955
environment and
are fit for
consumption

Indicates whether
Green and the food item is

Brown Dot vegetarian or non-
vegetarian

India Testing centers


Organic Certifies organic accredited by
Certification farm products meet the Agricultural
(Voluntary) National Standards and Processed
Food Products

CHENNAI |SALEM| MADURAI | COIMBATORE DELHI | BANGALORE | THIRUVANANTHAPURAM


www.shankariasacademy.com 221
www.iasparliament.com

for Organic Export critical minerals as essential for its economic


Products Development development and national security.
Authority
(APEDA) under 94. d
the National
Program for Cricket in Olympics
Organic Production
of the Government International Olympic Committee approves cricket
of India among other 5 sports for 2028 Los Angeles Olympics
whose final voting will be held soon.
Certifies products
meet standards to Bureau of  The 5 new sports under consideration are:
Eco mark
cause the least Indian
(Voluntary) 1. Twenty20 cricket
impact on the Standards (BIS)
ecosystem 2. Baseball/softball
Non- Certifies motor 3. Flag football (non-contact American
Polluting vehicles conform to football)
Bharat Stage
Vehicle relevant Bharat
emission standards
Mark Stage emission 4. Squash
(Mandatory) standards
5. Lacrosse

93. b  1900 Paris Olympics – Cricket was last


featured at this event.
Critical Minerals and Clean Energy Summit,
2023  Roadmap of Cricket in Global stage

 The 1st edition of the International Energy o In 2019, International Cricket Council
Agency’s Critical Minerals and Clean Energy (ICC) took a decisive move to make
Summit was held at Paris, France. cricket a global sport.

 It aims to discuss the challenges and o In 2022, it returned to the


opportunities behind rising demand for critical Commonwealth Games 2022, when
minerals by sharing experiences. women's T20s team played.

 Critical minerals are minerals like copper,  Cricket is an optional sport at the quadrennial
lithium, nickel and cobalt that are essential for Commonwealth Games. It first appeared at
developing clean energy technologies, the 1998 Commonwealth Games, with a
economic development and national security. men's tournament.

 6 Key Action Areas of the Summit  BCCI agreed to come under the ambit of India's
National Anti-Doping Agency, an affiliate of the
1. Accelerating progress towards World Anti-Doping Agency (WADA).
diversified minerals supplies.
o IOC mandates that every global sport
2. Unlocking the power of technology and body must be WADA compliant.
recycling.
 T20 cricket is the sport’s shortest international
3. Promoting transparency in the format.
markets.
95. c
4. Enhancing the availability of reliable
information. UNEP Cool Coalition

5. Creating incentives for sustainable and The COP28 Presidency held by the United Arab
responsible production. Emirates is leading the Global Cooling Pledge
alongside the UNEP Cool Coalition.
6. Strengthening efforts on international
collaboration.  Cool Coalition is dedicated to inspiring
ambition, mobilizing action, and accelerating
 Based on the ‘Report of the Committee on progress toward UN 2019 Climate Change
Identification of Critical Minerals’ constituted Summit.
by the Ministry of Mines, India classifies 30

CHENNAI |SALEM| MADURAI | COIMBATORE DELHI | BANGALORE | THIRUVANANTHAPURAM


www.shankariasacademy.com 222
www.iasparliament.com

 The Cool Coalition is a global multi-  Tangsa textile – Textile products of the
stakeholder network that connects a wide Tangsa tribe of Changlang district, Arunachal
range of key actors towards a rapid global Pradesh are famous for their exotic designs
transition to efficient and climate-friendly and colours.
cooling.
97. c
 It is a unified initiative that links action across
the Kigali Amendment, Paris Agreement and Question Hour
Sustainable Development Goals.
 Question Hour – It is the 1st hour of every
 The 3 pillars of the cool coalition includes parliamentary sitting.
Advocacy, Action and Knowledge Exchange.
 Duration – 1 hour
 The Cool Coalition is led by:
 MPs ask questions to ministers and also to
1. UN Environment, the private members (MPs who are not
ministers).
2. Climate and Clean Air Coalition,
Procedure for Questionnaire
3. Kigali Cooling Efficiency
Program and  Governed by

4. Sustainable Energy for All o Rules of Procedure and Conduct


(SEforALL). of Business in Lok Sabha

 Climate Action Network is not part of this o Directions by Speaker, Lok Sabha
initiative.
 Prior notice – Addressed to the lower house’s
 Cool Champions are thought leaders from Secretary-General.
government, private sector and civil society to
raise awareness about the need for efficient and  Submission of notices – Either
climate friendly cooling and mobilize their through Member’s Portal or through the
peers to join this effort. printed forms as in the Parliamentary Notice
Office.
96. d
 It should not be more than 5 notices/ day.
Yak Churpi, Khamti Rice & Tangsa Textile
 Admissibility of question – Decided by Lok
Three indigenous products of Arunachal Pradesh such Sabha Speaker
as Yak churpi, Khamti rice & Tangsa textile received GI
tag.  Conditions for admissibility – It shouldn’t
allow questions containing
 Yak churpi - It is the first ever yak milk
product, Arunachal Yak Churpi, to receive o More than 150 words and on matters
the Geographical Indication. which may weaken the unity and
integrity of the country
 Churpi is prepared from the milk of Arunachali
yak, a unique breed found in West Kameng and o Arguments, defamatory statements or
Tawang districts of the state. referring to the character of any person
except in their official capacity
 It is reared by tribal yak pastoralists known
as Brokpas who migrate along with their yaks o Queries raising larger issues of policy
to higher reaches during summers and descend
to mid-altitude mountainous regions in o Any subject pending judgment before
winters. any court/tribunal/body or under
Parliamentary Committee
 Churpi is an excellent source of proteins and is
frequently used as a substitute for vegetables by 98. b
tribal yak herders in the vegetation-starved cold
UMMEED
and hilly mountainous regions of the state.
Recently, the Union Ministry of Education (MoE)
 Khaw Tai – Is a chewy sticky rice variety of
released the draft guidelines for schools to prevent
Namsai region being cultivated by traditional
suicide among students.
Khampti tribal farmers in Arunachal
Pradesh. It is also called as Khamti rice.

CHENNAI |SALEM| MADURAI | COIMBATORE DELHI | BANGALORE | THIRUVANANTHAPURAM


www.shankariasacademy.com 223
www.iasparliament.com

 UMMEED – Understand, Motivate, Manage,  A dynamic inter-governmental organisation


Empathise, Empower, and Develop and a regional forum.
(UMMEED).
 It is a tri-partite forum that brings together
 It is released by Department of School representatives of government, business and
Education, Ministry of Education. academia.

 The Objectives include:  Established in – 1997.

o To prevent suicides among  The vision for IORA originated during a visit by
school students late President Nelson Mandela of South Africa
to India in 1995.
o To serve as directions to schools for
enhancing sensitivity, understanding,  Permanent secretariat –
and providing support in case of Ebene, Mauritius
reported self-harm.
 Objective – Strengthening regional
o To de-stigmatise mental health cooperation and sustainable development
concerns through storytelling, rallies, within the Indian Ocean Region.
posters, exhibitions and other
activities.  Apex body – Council of (Foreign) Ministers
(COM) which meets annually
 These guidelines align perfectly with the NEP’s
(National Education Policy) vision of holistic  Membership – 23 Member States and 12
education. Dialogue Partners

 The Plan of Action is setting up of School o India is a member and France was the
Wellness Teams (SWT), orientations, and latest member.
immediate response to vulnerable students.
o Non-members – Pakistan,
 Composition of SWT – School counsellors, Myanmar
students, teachers, supporting staff, a
representative of the school management  Dialogue Partners – Italy, Japan, Germany,
committee under the leadership of the school China, USA, UK, Russia, Turkey, Korea, Egypt
principal. and Saudi Arabia

 Functions of SWT – It implements school  Troika – Established by the Council of


activities in creating awareness about mental Ministers, which consists of
well-being.
 Chair - Sri Lanka for 2 years (2023-2025)
99. b
 Vice Chair – India
Indian Ocean Rim Association (IORA)
 Previous chair – Bangladesh
Recently Indian Ocean Rim Association’s Council of
Ministers meeting was held under the theme  2 Specialised agencies of IORA
‘Reinforcing Indian Ocean Identity’.
o Regional Centre for Science and
Technology Transfer (RCSTT) at
Tehran, Iran.

o Fisheries Support Unit (FSU) at Oman.

CHENNAI |SALEM| MADURAI | COIMBATORE DELHI | BANGALORE | THIRUVANANTHAPURAM


www.shankariasacademy.com 224
www.iasparliament.com

 G20 – It is the premier forum for international


economic cooperation.

 It has 19 members and 2 regional forums, while


Pakistan and Myanmar are not its members.

 It does not have a permanent secretariat.

 The Presidency is supported by the Troika –


previous, current and incoming Presidency.

100. c

Setu Bandhan Scheme

Recently the ministry of road transport and highways


approved 7 bridge projects in Arunachal Pradesh
under the Setu Bandhan Scheme.

 The Setu Bandhan scheme have been


introduced to improve inter-state connectivity,
especially on the rural areas at the borders
where state roads do not get the required
attention.

 The scheme is an initiative of the Ministry of


Road Transport and Highways
(MoRTH).

 The scheme is different from “Setu Bharatam”


scheme.

 Setu Bharatam is an ambitious programme


with an investment of Rs. 50,000 crore to build
bridges for safe and seamless travel on National
Highways.

 The programme aims at making all national


highways Railway Level Crossing free by 2019.

CHENNAI |SALEM| MADURAI | COIMBATORE DELHI | BANGALORE | THIRUVANANTHAPURAM


www.shankariasacademy.com 225
www.iasparliament.com

TEST – V Select the answer using the code given below:

1. In India, which of the following are the eligibility a. 1 only


criteria of Prospective adoptive parents (PAPs) for
b. 2 only
adoption of a child?
c. Both 1 and 2
1. The consent of either of the spouses is needed
in cased of married couple. d. Neither 1 nor 2
2. They should not have been accused or convicted
in civil act of any nature.
4. Consider the following with respect to System of Air
3. They should not have a life-threatening medical Quality and Weather Forecasting (SAFAR).
condition.
1. It is a numerical model-based framework to
4. A single female can adopt a child of any gender. forecast Delhi’s air quality.
Select the correct answer using the codes given below: 2. It is developed by Indian Meteorological
Department (IMD).
a. 1, 2, 3 only
Which of the above statement(s) is/are incorrect?
b. 1, 3, 4 only
a. 1 only
c. 2, 3, 4 only
b. 2 only
d. 1, 2, 3, 4
c. Both 1 and 2

d. Neither 1 nor 2
2. Consider the following statements with respect to
Comptroller and Auditor General of India (CAG).

Statement-I: As per Article 148 of Indian 5. ‘Bletchley Declaration, sometimes seen in the news
Constitution, CAG is the guardian of the public purse. recently, refers to?
Statement-II: CAG is responsible for auditing all the a. The commitment to create a resilient and
expenditure of both the Central and the State sustainable future for all member economies.
Governments as well as of those organisations which the
government significantly funds. b. An agreement that aims to establish a shared
understanding of the risks and opportunities posed
Select the correct answer using the codes given below: by “frontier AI.”
a. Both Statement-I and Statement-II are correct c. An arrangement on Sustainable transport,
and Statement-II is the correct explanation for Enforcement, Post-crash response, Pedestrian and
Statement-I cyclist safety.
b. Both Statement-I and Statement-II are correct d. An agreement under World Trade Organisation
and Statement-II is not the correct explanation for that aims to deliver a free, fair, transparent,
Statement-I inclusive trade and investment environment.
c. Statement-I is correct but Statement-II is
incorrect
6. Consider the following pairs.
d. Statement-I is incorrect but Statement-II is
correct Tribal leaders Regions

1. Rani Gaidinliu – Assam


3. Which of the following statement(s) regarding 2. Rani Kamalapati – Maharashtra
Biosphere Reserves is/are correct?
3. Birsa Munda – Jharkhand
1. It is an international designation by United
Nations Environment Programme. 4. Shaheed Veer Narayan Singh – Bihar
2. Bhutan and Nepal do not have any biospheres How many of the above pairs are correct?
under the World Network of Biosphere
Reserves. a. Only one

CHENNAI |SALEM| MADURAI | COIMBATORE DELHI | BANGALORE | THIRUVANANTHAPURAM


www.shankariasacademy.com 226
www.iasparliament.com

b. Only two 3. The Constitution has not prescribed a


minimum age for appointment as a judge of
c. Only three Supreme Court.

d. All four 4. The Constitution allows Chief Justice of India


to appoint other place as a seat of Supreme
Court with the approval of the Parliament.
7. Consider the following statements. How many of the above statements is/are not correct?
1. It was formally inaugurated in 1963 as the 16th a. Only one
state of Indian Union.
b. Only two
2. It conducts an annual event called ‘festival of
festivals’. c. Only three
3. It is also called as the ‘land of festivals’. d. All four

The above given statements are correct with respect to


which of the following Indian States?
10. Consider the following statements with respect to
a. Uttar Pradesh the classification of Micro, Small and Medium
Enterprises (MSMEs).
b. Nagaland
1. Enterprises having not more than Rs. 5 Crore as
c. West Bengal investment and not more than Rs. 10 Crore as
annual turnover are classified as Micro
d. Manipur enterprises.

2. Enterprises having not more than Rs. 50 Crore


as investment and not more than Rs. 250 Crore
8. Consider the following statements regarding the
as annual turnover are classified as Medium
UNESCO Creative Cities Network (UCCN).
enterprises.
1. Only the cities of UNESCO’s member states and
Which of the above statement(s) is/are correct?
associate members are eligible.
a. 1 only
2. The cities are categorised under 8 creative
fields. b. 2 only
3. India has cities in all fields of the network. c. Both 1 and 2
4. Kozhikode is the 1st Indian city to get City of d. Neither 1 nor 2
Literature tag.

Which of the above statement (s) is/are correct?


11. Consider the following statements.
a. All four
1. It is a colour coded index showing the measure
b. All except 1 of air quality.
c. All except 2 & 3 2. It was launched as a part of Swachh Bharat
Campaign.
d. All except 2 & 4
3. It aims to communicate the current level and
future projections of air pollution to the public
9. Consider the following statements regarding the and the government.
Supreme Court of India.
Which of the following initiative corresponds to the
1. At present, 35 judges are present in the above given features?
Supreme Court.
a. Air Quality Index
2. It replaced the British Privy Council as the
b. Air Quality Life Index
highest court of appeal.
c. Graded Response Action Plan

CHENNAI |SALEM| MADURAI | COIMBATORE DELHI | BANGALORE | THIRUVANANTHAPURAM


www.shankariasacademy.com 227
www.iasparliament.com

d. System of Air Quality and Weather Forecasting to study the interactions between terrestrial
and Space weather.

How many of the statements given above


12. Kra Isthmus, occasionally seen in the news, is are incorrect?
located in?
a. Only one
a. Sinai Peninsula
b. Only two
b. Malay Peninsula
c. All three
c. Nagasaki Peninsula
d. None
d. Kamchatka Peninsula

16. Consider the following statements with respect to


13. Consider the following statements regarding the the Compressed Bio-Gas Blending Obligations (CBO).
Elephant Corridor Report, 2023.
1. It is implemented by the Ministry of
1. At present, there are 150 elephant corridors in Environment, Forest and Climate Change.
India.
2. CBO will be 5% of total CNG/PNG consumption
2. Kerala has the most elephant corridors in India. from 2028-29 onwards.

3. Elephant corridors situated within revenue Which of the above statement (s) is/are correct?
lands and private lands are required to be
notified under the Wild Life (Protection) Act, a. 1 only
1972.
b. 2 only
How many of the statements given above are correct?
c. Both 1 and 2
a. Only one
d. Neither 1 nor 2
b. Only two

c. All three
17. Consider the following statements with respect to
d. None of the above Cryosphere.

1. Cryosphere includes snow and ice on both land


and sea.
14. Commission on Social Connection, sometimes seen
in the news, is an initiative by? 2. The State of the Cryosphere Report is released
by the Intergovernmental Panel on Climate
a. World Trade Organisation (WTO) Change (IPCC).

b. World Health Organisation (WHO) 3. The Global Cryosphere Watch (GCW) is an


international mechanism of United Nations
c. International Organization for Migration (IOM) Environment Program (UNEP) for supporting
all key cryospheric in-situ and remote sensing
d. UN Department of Economic and Social Affairs observations.
(UN DESA)
How many of the statements given above are correct?

a. Only one
15. Consider the following statements with respect to
Atmospheric Gravity Waves (AGW). b. Only two
1. It is the horizontal displacement of stable air c. All three
that occurs when there is an extreme weather
event. d. None

2. In a gravity wave, the sinking region is the most


favorable region for cloud development.
18. Which of the following statements is correct with
3. Atmospheric Waves Experiment (AWE) is a respect to Amaterasu?
first-of-its-kind experimental attempt by ISRO

CHENNAI |SALEM| MADURAI | COIMBATORE DELHI | BANGALORE | THIRUVANANTHAPURAM


www.shankariasacademy.com 228
www.iasparliament.com

a. It is an extremely high-energy particle emitted by a. Both Statement-I and Statement-II are correct
the sun. and Statement-II is the correct explanation for
Statement-I
b. It is the most powerful cosmic rays detected so
far. b. Both Statement-I and Statement-II are correct
and Statement-II is not the correct explanation for
c. It was discovered by the National Aeronautics Statement-I
and Space Administration (NASA).
c. Statement-I is correct but Statement-II is
d. Its energy is higher than the energy imbued in incorrect
protons by the Large Hadron Collider.
d. Statement-I is incorrect but Statement-II is
correct

19. Consider the following statements with respect to


Sangai Deer.
22. Arrange the following from North to South:
1. It is the State animal of Manipur.
1. Gulf of Suez
2. It is found exclusively in Khawnglung wildlife
sanctuary. 2. Port Said

3. Phumdi is the most important and unique part 3. Port Al Qantara


of Sangai’s habitat.
4. Port Suez
How many of the statement (s) given above is/are
correct? Select the correct answer using the code below:

a. Only one a. 4-1-3-2

b. Only two b. 4-1-2-3

c. All three c. 2-3-1-4

d. None d. 2-3-4-1

20. A23a, sometimes seen in the news, is related to? 23. Consider the following statements with respect to
Central Adoption Resource Authority (CARA)
a. A protein that edits the faulty gene responsible
for hemoglobinopathy. 1. It is a statutory body established by the Juvenile
Justice (Care and Protection of Children) Act,
b. An armed drone that India has recently procured 2015.
from the USA.
2. It deals with both intra-country and inter-
c. A large iceberg that is drifting away beyond country adoption of Indian children.
Antarctic waters.
How many of the above statements is/are incorrect?
d. The fastest growing blackhole ever recorded.
a. 1 only

b. 2 only
21. Consider the following statements
c. Both 1 and 2
Statement-I: In India, Governor of the state has the
power to reserve a money bill for the consideration of d. Neither 1 nor 2
the President.

Statement-II: Once President approves, the Governor


can give assent to the money bill and thus it becomes an 24. Consider the following statements with respect to
act. Other Backward Class (OBC) commissions

Select the correct answer using the codes given below: 1. None of the recommendation of Kaka Kalelkar
Commission were implemented.

CHENNAI |SALEM| MADURAI | COIMBATORE DELHI | BANGALORE | THIRUVANANTHAPURAM


www.shankariasacademy.com 229
www.iasparliament.com

2. Mandal Commission recommended 27% 5. Traditional craftsmanship


reservation for about 52% of the OBC
population. Select the correct code

3. Article 341 of Indian Constitution provides for a. All five


the appointment of a Commission to investigate
the conditions for the improvement of socially b. All except 4
and educationally backward class.
c. All except 5
How many of the above statements is/are not correct?
d. All except 4 & 5
a. Only one

b. Only two
28. Consider the following statements:
c. All three
1. Chavittu Natakam is a theatre form that
d. None of the above flourished in Tamil Nadu with the spread of
Christianity.

2. Pahari painting is a style of miniature painting.


25. ‘Reverse flipping’, sometimes seen in the news
recently, refers to? 3. Basohli painting is a school of Pahari painting
that flourished in the Indian hill states.
a. It is the trend of overseas start-ups shifting their
domicile to India and listing on Indian stock How many of the above statements is/are correct?
exchanges.
a. Only one
b. It refers to a situation where a private company
becomes a public company by purchasing control of b. Only two
the public company.
c. All three
c. It is a strategic business practice, involves
d. None of the above
outsourcing tasks to neighboring or nearby
countries rather than distant offshore locations.

d. It refers to sudden shift in economic phase from 29. Consider the following pairs
the state of inflation to the state of recession.
Terms in Sikhism About

1. Udaasis – A sect within Sikhism


26. “Emission Gap Report” is published annually by?
2. Janamsakhis – Biographies of Guru Nanak
a. UN Environment Programme
3. Simran – Service
b. World Meteorological Organization
4. Dan – Charity
c. United Nations Carbon Offset Platform
How many of the above pairs are correct?
d. Intergovernmental Panel on Climate Change
a. Only one

b. Only two
27. UNESCO’s 2003 Convention for Safeguarding of the
Intangible Cultural Heritage proposes which of the c. Only three
following domains that manifest intangible cultural
heritage? d. All four
1. Oral traditions and expressions

2. Performing arts 30. “Nidirana Noadihing”, sometimes seen in the news,


is related to?
3. Social practices, rituals and festive events
a. A festival celebrated annually in the State of
4. Knowledge and practices concerning nature Nagaland.
and the universe
b. A martial art practised in the State of Manipur.

CHENNAI |SALEM| MADURAI | COIMBATORE DELHI | BANGALORE | THIRUVANANTHAPURAM


www.shankariasacademy.com 230
www.iasparliament.com

c. A Meghalaya based dance-drama that was d. None of the above


recognised as an important cultural heritage by the
state.

d. A new species of music frog discovered in 34. Consider the following statements with respect to
Arunachal Pradesh. National Efficient Cooking Programme:

1. It aims to promote affordable and energy-


efficient coking coal.
31. Consider the following
2. It is an initiative of Ministry of Petroleum and
1. Oman Natural Gas (MOPNG).

2. Yemen Which of the above statement(s) is/are incorrect?

3. Qatar a. 1 only

4. Bahrain b. 2 only

5. United Arab Emirates c. Both 1 and 2

Which of the following are members are Gulf d. Neither 1 nor 2


Cooperation Council (GCC)?

a. All five
35. Consider the following pairs:
b. All except 1
Reports/ Indices Organisations
c. All except 3
1. Adaptation Gap Report – UNEP
d. All except 2
2. Composite Water Management Index – NITI Aayog

3. Global Plastics Outlook – OECD


32. “Varaha”, sometimes seen in the news recently, is
associated with? How many of the pair(s) given above is/are correctly
matched?
a. Chandrayaan-3 Mission
a. Only one
b. Samudrayaan Mission
b. Only two
c. Western Ghats Spatial Decision Support System
c. All three
d. AGNIi (Accelerating Growth of New India’s
Innovations) d. None of the above

33. Consider the following statements with respect to 36. UNESCO Michel Batisse Award, sometimes seen in
Road Accidents in India Report, 2022 the news is related to which of the following?

1. The report is released annually by the Ministry a. To recognize operations that have a significant
of Road Transport and Highways. impact on the humanitarian assistance provided
during war.
2. Road accidents and accident-related killings
are higher in rural areas than urban areas. b. For a political person who provide Policy
leadership for conservation of the nature.
3. Tamil Nadu recorded the highest number of
road accidents on National Highways in 2022. c. Highest civilian award by UNESCO for person in
any field of human endeavour.
How many of the statement(s) given above is/are
correct? d. For outstanding achievement in management of
biosphere reserves across the globe.
a. Only one

b. Only two
37. Consider the following statements with respect to
c. All three the Advocate-on-Record System

CHENNAI |SALEM| MADURAI | COIMBATORE DELHI | BANGALORE | THIRUVANANTHAPURAM


www.shankariasacademy.com 231
www.iasparliament.com

1. Advocate-on-Record (AoR) are a pool of Delhi- 40. Consider the following statements with respect to ‘E
based lawyers and only they are allowed to file Prime Layer’
cases before the Supreme Court.
1. It is a new earth layer that is formed in the inner
2. AOR system is being governed by the Supreme most part of the earth’s core.
Court Rules, 2013.
2. It is formed as a result of surface water
3. Advocate-on-Record (AoR) can be appeared penetrating deep into the planet which alters
only before the Supreme Court and are barred the composition of liquid core's outermost
to appear before the other courts. region.

Which of the above statement(s) is/are incorrect? 3. It is characterized by high silicon content and
low hydrogen concentration.
a. Only one
How many of the statements given above are correct?
b. Only two
a. Only one
c. All three
b. Only two
d. None of the above
c. All three

d. None of the above


38. Consider the following pairs:

Volcanoes Countries
41. Consider the following statements regarding Jammu
1. Mount Etna – Greece and Kashmir Reorganization Act, 2019

2. Fagradalsfjall – Hawaii, USA 1. The law bifurcated the State into two Union
Territories, Ladakh without legislature and
3. Kilauea – Iceland J&K with legislature.
4. Dukono – Indonesia 2. The Act amended the Representation of People
Act, 1950 which reserved certain seats for
How many of the pair(s) given above is/are correctly Scheduled Tribes.
matched?
3. National Integration Day was observed on
a. Only one October 31, 2019, the day the two union
territories (J&K and Ladakh) were established.
b. Only two
How many of the above statement(s) is/are incorrect?
c. Only three
a. Only one
d. All four
b. Only two

c. All three
39. Identify the characteristic features of
‘Tantalum’ from the options given below: d. None
1. Low density

2. Critical mineral 42. Consider the following statement about volcanoes:


3. Low melting point 1. A volcano is an opening in the earth's crust
through which magma, gases and ash are
4. High corrosion resistance
released to the earth's surface
Select the answer using the code given below:
2. There are no volcanoes in the Himalayan region
a. 1 and 3 only of India.

b. 2 and 4 only 3. Narcondam in Andaman and Nicobar Islands is


an active volcano in India since 1991.
c. 1, 2 and 4
How many of the above statement(s) is/are incorrect?
d. 1, 2, 3 and 4

CHENNAI |SALEM| MADURAI | COIMBATORE DELHI | BANGALORE | THIRUVANANTHAPURAM


www.shankariasacademy.com 232
www.iasparliament.com

a. Only one 2. Mashelkar Committee recommended a


roadmap for implementation of Euro norms
b. Only two based on emission standards for India.

c. All three 3. BS VI restricts the Sulphur emission to 500


ppm.
d. None
How many of the statement(s) given above is/are
correct?
43. Consider the following statements with respect to a. Only one
Bay of Bengal Initiative for Multi-Sectoral Technical
and Economic Cooperation (BIMSTEC) b. Only two
1. It was established with the signing of Dhaka c. All three
declaration.
d. None
2. BIMSTEC energy centre is located in Uttar
Pradesh.

Which of the above statement(s) is/are correct? 46. Consider the following statements regarding Global
Partnership on Artificial Intelligence (GPAI) summit
a. 1 only
1. The summit concluded with the adoption of
b. 2 only Bletchley declaration.

c. Both 1 and 2 2. GPAI values is based on World Economic


Forum recommendations.
d. Neither 1 nor 2
Which of the above statement(s) is/are incorrect?

a. 1 only
44. Consider the following statements.
b. 2 only
Statement-I: Dollarisation is the process by which a
country decides to use two currencies the local currency c. Both 1 and 2
and generally a stronger, more established currency like
the US dollar. d. Neither 1 nor 2

Statement-II: It will have positive impact on


seigniorage.
47. A stone deity ‘Kakad Bhairav’ worshipped by the
Select the correct answer using the codes given below: Bhils tribe was recently proved to be a fossilized
dinosaur egg. In light of this, which one of following
a. Both Statement-I and Statement-II are correct statement (s) is/are correct?
and Statement-II is the correct explanation for
Statement-I 1. Dinosaur Fossils National Park is located in the
state of Madhya Pradesh.
b. Both Statement-I and Statement-II are correct
and Statement-II is not the correct explanation for 2. Dinosaur Fossils National Park is the first site
Statement-I to receive UNESCO’s Global Geopark tag.

c. Statement-I is correct but Statement-II is Select the answer using the code given below:
incorrect
a. 1 only
d. Statement-I is incorrect but Statement-II is
correct b. 2 only

c. Both 1 and 2

45. Consider the following statements with respect to d. Neither 1 nor 2


Bharat Stage Emission Standards

1. The standards and timelines for the BS


standards were introduced by the Central 48. “Karan Singh’s Proclamation”, sometimes seen in
Pollution Control Board (CPCB), Ministry of the news recently, refers to?
Environment, Forest and Climate Change.

CHENNAI |SALEM| MADURAI | COIMBATORE DELHI | BANGALORE | THIRUVANANTHAPURAM


www.shankariasacademy.com 233
www.iasparliament.com

a. A proclamation to cease all violent activities in b. It is based on the Vienna Declaration and
India by Khalistani protestors. Program of Action of 1993.

b. It reflected the full and final surrender of Jammu c. It consists of a preamble and 30 articles setting
& Kashmir's sovereignty to India. out fundamental rights and freedoms.

c. It refers to donation of forest lands to British for d. It was proclaimed through United Nations
commercial cultivation. General Assembly resolution 217 A.

d. It relates to idea of Khalistan by separating the


land from Indian Union.
52. Consider the following pairs.

Places in News About


49. Consider the following statements with respect to
Pradhan Mantri Anusuchit Jaati Abhyuday Yojana (PM- 1. Shidaowan Nuclear Plant – Rajasthan
AJAY)
2. Gelephu Smart city – Assam
1. It was launched by the Department of
Pharmaceuticals under Ministry of Chemicals 3. Ennore creek – Tamil Nadu
and Fertilisers.
How many of the above pairs are correct?
2. It aims to distribute low-cost generic medicines
to vulnerable populations. a. Only one

Which of the above statement(s) is/are correct? b. Only two

a. 1 only c. All three

b. 2 only d. None

c. Both 1 and 2

d. Neither 1 nor 2 53. India during its G20 presidency in 2023, had
launched an initiative named ‘MAHARISHI’ that deals
with which one of the following?

50. The Kashi Tamil Sangamam 2.0 that aims to revive a. Yoga
the bonds between Varanasi and Tamil Nadu through
cultural exchange and knowledge sharing involves b. Millets
which of the following entities in organising it?
c. Mangroves
1. The Union Ministry of Education
d. Biofuels
2. IIT Madras

3. Banaras Hindu University


54. Consider the following statements with respect to
4. The Union Ministry of Culture Indian Ocean Naval Symposium (IONS)

Select the answer using the code given below: 1. It aims to increase maritime co-operation
among navies of the littoral states of the Indian
a. 4 only Ocean Region.

b. 2 and 3 only 2. It has its own designated flag, which was


designed by India.
c. 1, 2 and 3 only
3. India is the current chair of IONS.
d. 1, 2, 3 and 4
Which of the above statement(s) is/are correct?

a. Only one
51. Which of the following is incorrect with respect to
Universal Declaration of Human Rights (UDHR) b. Only two

a. It is a treaty which is legally binding on all of its c. All three


members.
d. None

CHENNAI |SALEM| MADURAI | COIMBATORE DELHI | BANGALORE | THIRUVANANTHAPURAM


www.shankariasacademy.com 234
www.iasparliament.com

55. “VISION 2020: The Right to Sight” is an initiative 1. The Codex Alimentarius is a collection of
of? internationally adopted food standards and
related texts presented in a uniform manner.
a. Doctors Without Borders
2. Member countries are mandated to enforce the
b. Helen Keller International food standards set by the commission.

c. World Health Organization 3. India became the member of Codex


Alimentarius in 1964.
d. The Global Fund to Fight AIDS, Tuberculosis
and Malaria How many of the statements given above are correct?

a. Only one
56. Consider the following statements. b. Only two
Reports Published by c. All three

1. World Malaria Report 2023 - WHO d. None

2. Composite Water Management Index - CPCB

3. Logistics Ease Across Different State - NITI Aayog 59. Permakore, sometimes seen in the news, is related
to which of the following?
4. Migration and Development Brief - OECD
a. A permanently frozen layer of soil in Antarctica.
How many of the above statements is/are correct?
b. A musical instrument that is made from Indian
a. Only two Bison.
b. Only three c. One of the layers present in the internal structure
of Earth
c. All four
d. Most sacred stone worshipped by the Vaishnavas
d. None during ancient times

57. Match the following: 60. Consider the following statements with respect to
Bonnet Macaque
Missions/ Launched
Significance 1. It is a primate that is endemic to the southern
by
Rockets India.
p. First methane-liquid 2. They are highly arboreal and are strong
1. XPoSat a. ISRO
oxygen rocket swimmers.
2. Zhuque- q. To study astronomical X- 3. It has been listed as Vulnerable under the
b. NASA
2 ray sources IUCN’s Red list of threatened species.
3.TEMPO c. LandSpace r. Monitoring of Pollution How many of the statements given above
are incorrect?
Select the correct answer using the codes given below:
a. Only one
a. 1-b-q; 2-c-p; 3-a-r
b. Only two
b. 1-a-q; 2-c-r; 3-b-p
c. All three
c. 1-c-q; 2-b-p; 3-a-r
d. None
d. 1-a-q; 2-c-p; 3-b-r

61. Consider the following statements with respect to


58. Consider the following statements with respect to Crimes in India Report, 2022
Codex Alimentarius Commission

CHENNAI |SALEM| MADURAI | COIMBATORE DELHI | BANGALORE | THIRUVANANTHAPURAM


www.shankariasacademy.com 235
www.iasparliament.com

1. There was slight decline in various types of 64. Consider the following statements.
crimes in 2022 compared to 2021.
Statement-I: Decentralized Autonomous
2. Mumbai has been recognised as the safest city Organizations is a non-legal structure that operates
in India for the 3rd consecutive year. without centralized control.

Which of the above statement(s) is/are incorrect? Statement-II: It uses blockchain technology and
smart contract to take decisions in a bottom-up
a. 1 only management approach.
b. 2 only Select the correct answer using the codes given below:

c. Both 1 and 2 a. Both Statement-I and Statement-II are correct


and Statement-II is the correct explanation for
d. Neither 1 nor 2 Statement-I

b. Both Statement-I and Statement-II are correct


and Statement-II is not the correct explanation for
62. Consider the following statements regarding PM Statement-I
JANMAN scheme
c. Statement-I is correct but Statement-II is
1. The scheme aims to uplift the PVTGs by incorrect
ensuring their access to essential infrastructure
and services. d. Statement-I is incorrect but Statement-II is
correct
2. The scheme comprises both central sector and
centrally sponsored schemes.

3. In 1973, the Lokur Commission created the 65. Consider the following statements with respect to
Primitive Tribal Groups (PTGs) as a separate Unlawful Activities (Prevention) Act (UAPA), 1967
category.
1. The Act permits keeping a person in prison for
How many of the above statement(s) is/are correct? up to one year without even filing a charge
sheet.
a. Only one
2. National Investigation Agency is authorized to
b. Only two attach and forfeit property derived from or used
in unlawful or terrorist activities.
c. All three
Which of the above statement(s) is/are correct?
d. None
a. 1 only

b. 2 only
63. Consider the following statements regarding Zero
Coupon Zero Principle bonds c. Both 1 and 2
1. They are financial instrument that are included d. Neither 1 nor 2
in the list of securities under Securities
Contracts (Regulation) Act, 1956.

2. They can be issued only by the Non-Profit 66. Consider the following statements.
Organisations that are registered with Social
Stock Exchanges (SSEs). 1. The Governor of the state is the ex-officio
chancellor of the universities in that state.
3. They are available for trading in the secondary
market. 2. The Governor as a chancellor is not bound by
the aid and advice of the council of ministers
How many of the above statement(s) is/are correct? and shall act only on his personal capacity.
a. Only one 3. The Governor of a state appoints the Vice-
Chancellors of state universities.
b. Only two
4. The President is the Chancellor of Central
c. All three universities as per Central Universities Act.
d. None How many of the above statements is/are correct?

CHENNAI |SALEM| MADURAI | COIMBATORE DELHI | BANGALORE | THIRUVANANTHAPURAM


www.shankariasacademy.com 236
www.iasparliament.com

a. Only one b. VVPAT

b. Only two c. ENCORE

c. Only three d. SAMARTH

d. All four

70. Food and Agricultural Organisation (FAO) in its


recent report had suggested to use which one of the
67. Consider the following statements with respect to following traditional farming practices to reduce the
Youth for Unnati and Vikas with AI (YUVAi) escalating human and environmental costs of producing
food?
1. The program aims to equip undergraduate
students studying technical education across a. Akkadi Saalu
the nation with relevant mindset and skill sets.
b. Lantana
2. It was launched jointly by the National e-
Governance Division (NeGD), Ministry of c. Bhattarakas
Electronics and Information Technology
(MeitY), and Intel India. d. Nishidis

Which of the statement(s) given above is/are correct?

a. 1 only 71. Consider the following statements.

b. 2 only Statement I: Noma disease, which was officially


added in the list of neglected tropical diseases, is also
c. Both 1 and 2 known as the Face of Poverty.

d. Neither 1 nor 2 Statement II: Effective drugs and surgical treatments


for Noma remains inaccessible for many.

Select the correct answer using the codes given below:


68. Consider the following pairs.
a. Both Statement-I and Statement-II are correct
Portals Purposes and Statement-II is the correct explanation for
Statement-I
1. Bhoomi Rashi – Spatial Planning
b. Both Statement-I and Statement-II are correct
2. Gram Manchitra – Land Acquisition and Statement-II is not the correct explanation for
Statement-I
3. e-jagrti – Dispute resolution
c. Statement-I is correct but Statement-II is
How many of the above pair(s) given above is/are incorrect
correctly matched?
d. Statement-I is incorrect but Statement-II is
a. Only one correct
b. Only two

c. All three 72. Consider the following.


d. None 1. Hydrogen

2. Carbon monoxide
69. Consider the following passage. 3. Carbon dioxide
It was launched by the Election Commission of India 4. Oxygen
(ECI) in 2009 to build a truly participative democracy
in India. It provides for voter education, spreading voter 5. Methane
awareness and promoting voter literacy in India.
Which of the following are constituents of Synthesis
Identify the correct answer using the passage given (Syn) gas?
above.
a. All except 3
a. SVEEP

CHENNAI |SALEM| MADURAI | COIMBATORE DELHI | BANGALORE | THIRUVANANTHAPURAM


www.shankariasacademy.com 237
www.iasparliament.com

b. All except 4 c. It is more than a survival strategy that plays a


pivotal role in carbon sequestration.
c. All except 5
d. This migration follows the directions of local
d. All except 2 winds prevailed in a region.

73. Consider the following statements with respect to 76. Consider the following statements with respect to
Nuclear Plants in India? Paat-Mitro
1. Kudankulam Plant is India's largest nuclear 1. It is a mobile application that provides
power plant, which is built with the assistance information about Minimum Support Price
of Russia. (MSP) and agronomy to jute farmers.

2. Kakrapar Atomic Power Project is based on 2. It was developed by the National Informatics
indigenous Pressurised Heavy Water Reactors. Centre (NIC) of Ministry of Electronics &
Information Technology.
Which of the above statement(s) is/are correct?
Which of the statements given above is/are correct?
a. 1 only
a. 1 only
b. 2 only
b. 2 only
c. Both 1 and 2
c. Both 1 and 2
d. Neither 1 nor 2
d. Neither 1 nor 2

74. Consider the following statements with respect to


Global TB Report, 2022 77. Consider the following statements with respect to
Srimukhalingam Temple
1. It is an annual report published by the World
Health Organization (WHO). 1. It is constructed by the King Kamarnava-II,
similar to the Architecture of the Lingaraja
2. WHO’s Global Tuberculosis strategy aims to temple of Odisha.
end the TB epidemic globally by 2030.
2. The temple is located on the banks of River
3. Government of India has set the target of Swarnamukhi.
eradicating TB by the year 2025.
3. It is one of the World Heritage Sites of India
How many of the statements given above are correct? under cultural category.
a. Only one How many of the statement(s) given above is/are
correct?
b. Only two
a. Only one
c. All three
b. Only two
d. None
c. All three

d. None
75. Which of the following statements
is incorrect with respect to the Diel Vertical Migration
(DVM)?
78. Consider the following statements:
a. It is a pattern of migration followed by the deep-
sea marine animal especially tiny free-floating 1. The Panama Canal connects the Atlantic and
zooplanktons. Pacific Ocean.
b. DVM is a synchronized, daily movement of 2. The Bab-el-Mandeb separates the Sinai
animals driven by the need for food and to avoid Peninsula from East Africa.
predators.
3. The Taiwan Strait separates mainland China
and the island of Taiwan.

CHENNAI |SALEM| MADURAI | COIMBATORE DELHI | BANGALORE | THIRUVANANTHAPURAM


www.shankariasacademy.com 238
www.iasparliament.com

How many of the statement(s) given above is/are a. Only one


correct?
b. Only two
a. Only one
c. All three
b. Only two
d. None
c. All three

d. None
82. Consider the following pairs.

Important
79. Global Drought Snapshot Report, 2023 is published Related to
Judgments
by?
Joseph Shine
a. UN Convention to Combat Desertification v Union Demonetisation of currency notes of
(UNCCD) 1
Rs. 500 and Rs.1000 denominations
of India
b. International Drought Resilience Alliance
(IDRA) Vivek Narayan

c. UN Environment Programme (UNEP) Sharma v. Armed forces can take action against
2
Union of their officers for adulterous acts.
d. Jointly by a and b
India

Anoop
80. Consider the following statements with respect to Appointments of Chief Election
Baranwal v. Commissioner (CEC) and Election
Tax Inspectors without Borders (TIWB) Initiative 3
Commissioners (ECs)
Union of India
1. It aims to transfer technical know-how and
skills to tax auditors of developing countries How many of the pair(s) given above is/are correctly
and share general audit practices to them. matched?
2. It is a joint initiative of United Nations a. Only one
Development Programme and Organisation for
Economic Cooperation and Development. b. Only two

Which of the statement(s) given above c. All three


is/are incorrect?
d. None
a. 1 only

b. 2 only
83. Consider the following statements with respect to
c. Both 1 and 2 Defection.

d. Neither 1 nor 2 1. The presiding officer has to decide a defection


case within 6 months of reporting.

2. The decision of the presiding officer in cases of


81. Consider the following statements regarding defection cannot be challenged in any court.
Tsunami waves.
3. The President is empowered to make rules to
1. Volcanic eruption has generated about 80% of give effect to the provisions of the 10th schedule
all tsunami events recorded globally. of Indian Constitution.
2. The wavelength is a prime factor which How many of the above statements is/are correct?
distinguishes tsunamis from wind waves.
a. Only one
3. When the sea depth decreases, the wavelength
of tsunami wave decreases and it caused the b. Only two
increase in the height of waves.
c. All three
How many of the above statements is/are correct?
d. None

CHENNAI |SALEM| MADURAI | COIMBATORE DELHI | BANGALORE | THIRUVANANTHAPURAM


www.shankariasacademy.com 239
www.iasparliament.com

84. Consider the following statements regarding the c. 5-4-3-2-1


Annual Status of Education Report (ASER) 2023.
d. 1-5-4-6-3
1. It is a nationwide citizen-led annual household
survey that provides a snapshot of the status of
children’s schooling and learning.
87. Consider the following statements
2. It was conducted by the National Council of
Educational Research and Training (NCERT). Statement-I: Nutrient Based Subsidy (NBS) Scheme
aims to encourage soil-balanced fertilization and
Which of the statement(s) given above is/are correct? increase agricultural productivity.

a. 1 only Statement-II: Nutrient Based Subsidy (NBS) Scheme


provides subsidies for urea and non-urea-based
b. 2 only fertilizers based on the nutrients they contain.

c. Both 1 and 2 Select the correct answer using the codes given below:

d. Neither 1 nor 2 a. Both Statement-I and Statement-II are correct


and Statement-II is the correct explanation for
Statement-I
85. Consider the following with respect to Geographical b. Both Statement-I and Statement-II are correct
Indication (GI) Tag: and Statement-II is not the correct explanation for
Statement-I
1. Every State in India has at least one GI tag.
c. Statement-I is correct but Statement-II is
2. Tamil Nadu has the highest number of GI tags incorrect
in India.
d. Statement-I is incorrect but Statement-II is
3. Among varied categories of GI tag, Handicrafts correct
tops the list with higher number of products.

How many of the above statements is/are correct?


88. “Maasir-i-Alamgiri” is a Persian book written by?
a. Only one
a. Ibn Battuda
b. Only two
b. Amir Khusrau
c. All three
c. Saqi Musta'd Khan
d. None
d. Niccolao Manucci

86. Identify the correct sequence of the proposed


trading route with respect to ‘Trilateral Transit and 89. Consider the following pairs.
Trade Agreement’ between India, Afghanistan and
Iran. Channel Locations

1. Kandla 1. 8 Degree Channel – Maldives & Minicoy Island

2. Gwadar Port 2. 9 Degree Channel – Minicoy Island & Laccadive

3. Chabahar port 3. 10 Degree Channel – Little Andaman & Nicobar

4. Zahedan How many of the pair(s) given above is/are correct?

5. Kabul a. Only one

6. Zaranj b. Only two

Select the correct answer using the codes given below: c. All three

a. 1-2-3-4-6 d. None

b. 5-6-4-3-1

CHENNAI |SALEM| MADURAI | COIMBATORE DELHI | BANGALORE | THIRUVANANTHAPURAM


www.shankariasacademy.com 240
www.iasparliament.com

90. Which of the following with reference to ‘Amrit 93. Consider the following passage.
Dharohar Scheme’ is correct?
He founded the first-ever Kirtan Ghar at Bordowa,
a. It is an initiative of Union Government to raise Assam in 1494 AD which intended to practice and
awareness on central sector schemes. preach the neo Vaishnavite faith, and propagated the Ek
Saran Naam Dharma.
b. Its goal is to promote unique conservation values
of Ramsar Sites. Identify the personalities associated with the above
passage.
c. It is an initiative that aims to empower and
enhance the skills of the craftsman community. a. Saint Kapila

d. It aims to address the needs of the most b. Saint Sankaradeva


vulnerable, ensuring efficient implementation of
developmental programs for rural development. c. Maharshi Patanjali

d. Saint Ramanujacharya

91. Consider the following statements.

1. It aims to map the land parcels in rural 94. Consider the following statements with respect to
inhabited areas using drone technology. criteria for classical language.

2. It was launched by the Ministry of Panchayati 1. High antiquity of its early texts/recorded
Raj. history over a period of 1500-2000 years.

3. It provides property cards to facilitate 2. The literary tradition should be original and not
monetization of properties, enabling bank loans borrowed from another speech community.
and comprehensive village-level planning.
3. The language should have a body of ancient
All the above given statements are the features of which literature/texts which is considered a valuable
of the following schemes? heritage by generation of speakers.

a. SARATHI How many of the above statement(s) is/are correct?

b. PRERANA a. Only one

c. SAMARTH b. Only two

d. SVAMITVA c. All three

d. None

92. Consider the following pairs.

GI Tagged products States 95. The terms Valabhi, Phamsana, Latina, Shekhari,
Bhumija is associated with which of the following?
1. Adi Apong - Assam
a. Vesara style of architecture
2. Kachchhi Kharek - Gujarat
b. Nagara style of architecture
3. Kalajeera Rice - West Bengal
c. Dravidan style of architecture
4. Singpho Phalap - Mizoram
d. Maru-Gurjara style of architecture
How many of the pair(s) given above is/are correctly
matched?

a. None 96. The term “Camptothecin (CPT)” seen in news, best


refers to which of the following?
b. Only two
a. It is a chemical compound used to make twine,
c. Only three cloth and carpets.

d. Only one b. It is used in the treatment of bronchitis, cough,


wounds ulcers, diarrhea, and dysentery.

CHENNAI |SALEM| MADURAI | COIMBATORE DELHI | BANGALORE | THIRUVANANTHAPURAM


www.shankariasacademy.com 241
www.iasparliament.com

c. It is an anti-cancer drug derived from 100. The terms “Amini and Kalpeni” sometimes seen in
Nathapodytes nimmoniana, native to India. the news were associated with?

d. It is a compound found in coral that could a. Islands of Lakshadweep


survive heat stress.
b. Indigenous cattle breeds of Gujarat

c. Eastern flowing rivers in India Nepal border


97. “VijayRaghavan Panel Report” often seen in news
was tasked with which of the following? d. Particularly Vulnerable Tribal Groups of Odisha

a. To revive telecom sector

b. To review the functioning of DRDO

c. To formulate food pricing policy in the long term

d. To examine and suggest measures to encourage


the MSMEs

98. Consider the following statements with respect to


National Multidimensional Poverty Index (MPI)

1. It is released by the NITI Aayog.

2. Health, education and standard of living are the


three dimensions used in MPI.

3. Assets and Bank accounts are among the


indicators measured.

How many of the above statements is/are correct?

a. Only one

b. Only two

c. All three

d. None

99. Consider the following statements with respect to


Bharat Ratna awards.

1. The award list is recommended by a committee


to the President of India.

2. It is not mandated to award every year.

3. The original statute did not provide for


awarding posthumously.

4. The award does not carry any monetary grant.

How many of the statement(s) given above


is/are incorrect?

a. Only one

b. Only two

c. Only three

d. All four

CHENNAI |SALEM| MADURAI | COIMBATORE DELHI | BANGALORE | THIRUVANANTHAPURAM


www.shankariasacademy.com 242
www.iasparliament.com

Answer Key - Test - V


1 2 3 4 5 6 7 8 9 10
B A B C B A B C B B
11 12 13 14 15 16 17 18 19 20
A B A B C B A D B C
21 22 23 24 25 26 27 28 29 30
C D D D A A A B C D
31 32 33 34 35 36 37 38 39 40
D B C C C D A A B A
41 42 43 44 45 46 47 48 49 50
B A D C B C A B D C
51 52 53 54 55 56 57 58 59 60
A A B B C A D B B D
61 62 63 64 65 66 67 68 69 70
C B B D B C C A A A
71 72 73 74 75 76 77 78 79 80
A B C C D A A B D C
81 82 83 84 85 86 87 88 89 90
B A D A C B C C C B
91 92 93 94 95 96 97 98 99 100
D D B C B C B C A A

EXPLANATION o Any PAP, irrespective of their marital


status and whether or not they have
1. b biological son or daughter are eligible.
Child Adoption in India  Ineligible - Having life-threatening
medical conditions.
The Supreme Court has expressed concern over the
delay in India’s system of child adoption.  Have been accused or convicted in
criminal act of any nature.
 Adoption – A legal process that allows someone
to become the parent of a child, even though the  A single male cannot adopt a girl child
parent and child are not related by blood.
2. a
 Legal framework – Hindu Adoption and
Maintenance Act, 1956 (for Hindus, Jains, Comptroller and Auditor General of India
Sikhs and Buddhists) and the Juvenile Justice
(Care and Protection of Children) Act, 2015.  Constitutional body – Article 148 provides for
an independent office of the CAG.
 Eligibility conditions for Prospective
Adoptive Parents (PAPs)  Financial administration – It is the head of
Indian Audit and Accounts Department.
o Physically, mentally and emotionally
stable.  Guardian of the public purse – CAG is
responsible for inspecting and auditing all
o Financially capable the expenditure of both the Central and
the State Governments as well as of
o The consent of both
those organisations which the
spouses required in case of married
government significantly funds.
couples.

o A single female can adopt a child  Appointed by – President by a warrant under


of any gender. his hand and seal

CHENNAI |SALEM| MADURAI | COIMBATORE DELHI | BANGALORE | THIRUVANANTHAPURAM


www.shankariasacademy.com 243
www.iasparliament.com

 Tenure – 6 years or up to age of 65 years  Areas potential for preservation of traditional


whichever is earlier tribal or rural modes of living for harmonious
use of environment.
 Resignation – Can resign any time by
addressing the resignation letter  World Network of Biosphere Reserves –
to President. It is part of UNESCO’s Man and Biosphere
(MAP) Programme.
 Removal – Same manner as a judge of Supreme
Court  Formed in – 1971, as a backbone for
biodiversity conservation, ecosystem
 Removed by the President on the basis of a restoration, and living in harmony with nature.
resolution passed to that effect by both the
Houses of Parliament with special majority.  There are now 738 properties in 134 countries.

 Reason for removal – Proved misbehaviour or  There are 18 biosphere reserves in India, out of
incapacity which 12 are recognized internationally under
the MAB programme.
 Reappointment – Not eligible for further office,
either under the Government of India or of any  Spain is one of the lead participating WNBR
State. countries globally, with 53 properties.

 Salary & service conditions – Determined by  The 1st biosphere reserve in South Asia was the
Parliament. Hurulu Biosphere Reserve, in Sri Lanka.

 Salary – Equal to judge of Supreme Court.  Bangladesh, Bhutan and Nepal do not
have biospheres.
 Administrative expenses – Charged upon
Consolidated Fund of India

 Criticism – It essentially acts as an auditor and


not the role of comptroller.

 It audits the accounts after the expenditure is


committed.

3. b

Biosphere Reserves

Recently, The Ministry of Environment, Forests and


Climate Change and the National Centre for
Sustainable Coastal Management, concluded the 10th
South and Central Asian Biosphere Reserve Network
Meeting (SACAM) in Chennai, India

 Biosphere Reserve (BR) –


An international designation by
UNESCO for representative parts of natural
and cultural landscapes extending over large
area of terrestrial or coastal/marine ecosystems
or a combination thereof.

 They are living examples of how human beings


and nature can co-exist while respecting each
other’s’ needs.
4. c
 Criteria for designation – It must contain an
effectively protected and minimally disturbed System of Air Quality and Weather Forecasting
core area of value of nature conservation. (SAFAR)

 The core area should be typical of a bio- According to the SAFAR-India, the overall air quality
geographical unit and large enough to sustain in the national capital New Delhi is in the 'poor'
viable populations. category with an AQI of 286.

 It is an advanced air quality early warning


system in India.

CHENNAI |SALEM| MADURAI | COIMBATORE DELHI | BANGALORE | THIRUVANANTHAPURAM


www.shankariasacademy.com 244
www.iasparliament.com

 Launched by – Ministry of Earth Sciences  Frontier AI – It refers to highly advanced


(MoES). generative AI models with potentially
dangerous capabilities that can pose significant
 Developed by – Indian Institute of risks to public safety.
Tropical Meteorology (IITM), Pune.
 Golden Gate Declaration – The 30th
 Aim – To reduce air pollution in India and to edition of Asia-Pacific Economic Cooperation
improve the air quality index of metropolitan (APEC) Economic Leaders' Meeting concluded
cities. in San Francisco with the adoption of the
Golden Gate Declaration.
 To provide location specific information on air
quality in near real time.  It commits to create a resilient and sustainable
future for all member economies.
 Significance – It forecasts 1-3 days in
advance for the 1st time in India.  It commits to deliver a free, open, fair,
transparent, inclusive trade and investment
 The World Meteorological Organization has environment.
recognised SAFAR as a prototype activity on
the basis of the high quality control and  Putrajaya Vision 2040 and the Aotearoa Plan
standards maintained in its implementation. of Action guide to attain the goals of APEC
mission.
 Decision Support System (DSS) – It is
a numerical model-based framework to 6. a
forecast Delhi’s air quality and sources of
local and regional pollution which can impact Janjatiya Gaurav Diwas (Tribal Pride Day)
the air.
The Janjatiya Gaurav Divas is celebrated every year
 It is developed by Indian Institute of Tropical on November 15 since 2021 to recognize the efforts of
Meteorology (IITM) Pune. the brave tribal freedom fighters.

5. b  It is to mark the birth anniversary of Sri Birsa


Munda who is revered as Bhagwan by tribal
Bletchley Declaration communities.

In the AI Safety Summit 2023, India have signed the  Objective – To recognize the efforts of the
Bletchley Declaration that addresses the risks of AI. tribals for preservation of cultural heritage and
promotion of Indian values of valour,
 It is the world’s first declaration for the hospitality and national pride.
responsible AI development.
Tribal
 Bletchley Park is a historic site in UK where the Location Contributions
leaders
Enigma code was cracked during World War II.
 Formed a new
 Aim – To establish a shared understanding religion, Birsait.
of the risks and opportunities posed by
“frontier AI.” Birsa
Jharkhand  Lead Munda
Munda
Rebellion (Ulgulan
 Signatories – 28 major countries, including Revolt) against
India, the US, China, and the European Union. British.

 2 pronged focus  Looted and


distributed grains
o Identifying risks of shared concern, Shaheed
amongst the poor
building a scientific understanding of Veer
Chhattisgarh after the 1856 famine.
them. Narayan
Singh
 Attained martyr in
o Developing cross-country policies to 1857 revolt.
mitigate these risks.
Alluri  Lead Rampa
 It encourages transparency and accountability Seetha Ram
Andhra
Rebellion against
from actors developing frontier AI technology Pradesh
Raju British.
concerning their plans to measure, monitor,
and mitigate potentially harmful capabilities.

CHENNAI |SALEM| MADURAI | COIMBATORE DELHI | BANGALORE | THIRUVANANTHAPURAM


www.shankariasacademy.com 245
www.iasparliament.com

 Revolted against Mimkut Kukis


British rule.
Rani Bushu Kacharis
Gaidinliu Manipur  Joined in Heraka
movement to Sekrenyi Angami
establish Naga Raj
(Rule of Nagas). Aoling Konyak

Sidhu and  Lead Santhal Moatsu Aos


Jharkhand and
Kanhu Rebellion (Hul
Bengal Tuluni Sumis
Murmu Revolt) in 1855-56.

 Committed ‘Jal Nyaknylum Changs


Jauhar’ (ritual
8. c
suicide) to save her
Rani honour.
Madhya UNESCO Creative Cities Network (UCCN)
Kamalapa
Pradesh
ti  The pride of the Gond 7 Creative
tribe and the last Fields of Indian Cities in UCCN
Hindu queen of UCCN
Bhopal.
Crafts &
 Loot the British and Jaipur (2015), Srinagar (2021)
Folk Arts
distribute the stolen
wealth among the Design -
Madhya
Tantia Bhil needy
Pradesh
Film Mumbai (2019)
 ‘Robin Hood of
India’. Gastronomy Hyderabad (2019)

7. b Literature Kozhikode (2023)


Nagaland Media Arts -
The 24th Hornbill Festival, a vibrant celebration of Chennai (2017), Varanasi (2015),
culture, music and dance will be held in Nagaland’s Music
Gwalior (2023)
misty mountains in 2023.
UNESCO has published the list of 55 creative cities on
 Formation – Formally inaugurated in World Cities Day (Oct 31st), in which 2 Indian cities
1963 (1st December), as the 16th state of Kozhikode and Gwalior were included.
Indian Union.
 Launched in – 2004
 It consists of 16 (Sixteen) Administrative
Districts, inhabited by 17 major tribes along  Objective – To strengthen cultural activities,
with other sub-tribes. goods, services and international cooperation
for sustainable development.
 Hornbill Festival – It is an annual
event which is called as the festival of  Eligibility – UNESCO’s member states
festivals. and associate members.

 It unites 16 Naga tribes of Nagaland to celebrate  Categories – 7 creative fields


in colourful costumes, pulsating rhythm, and
musical stories.  Updating Time – Once every 2 years

 Land of Festivals – It is called so as many  Total Cities (2023) – 350, in more than 100
festivals are celebrated by different tribes of the countries
state.
 Kozhikode is the 1st Indian city to get
Nagaland – The Land of Festivals City of Literature tag.

Festival Tribes  No Indian cities were placed under “Design


and Media Arts” field.
Tsukhenyie Chakhesangs
 UN SDG 11 – Sustainable Cities and
Communities.

CHENNAI |SALEM| MADURAI | COIMBATORE DELHI | BANGALORE | THIRUVANANTHAPURAM


www.shankariasacademy.com 246
www.iasparliament.com

9. b 10. b

Supreme Court Classification of MSMEs

The Supreme Court needs a structural overhaul to MSMEs, the economic fabric of India, needs digital
transform the Constitutional setup. empowerment which would help expand the market
reach and also make access to easy finance.
 Inaugurated on – January 28, 1950.
 MSME – Micro Small and Medium Enterprises
 It succeeded the Federal Court of (MSMEs).
India and enforces both Central as well
as the State laws.  Classification – Based on the investment in
plant/machinery/equipment and annual
 It has replaced the British Privy Council as the turnover.
highest court of appeal.
 It includes both manufacturing and service
 Constitution – Articles 124 to 147 in Part V of rendering enterprises.
the Constitution deal with the organisation,
independence, jurisdiction, powers, procedures Classification Micro Small Medium
etc., of the Supreme Court (SC).
Investment Not more Not more Not more
 It acts as the custodian of the Indian in plant & than Rs.1 than Rs.10 than Rs.50
Constitution and the protector of the machinery crore crore crore
Fundamental Rights.
Not more Not more Not more
Annual
 3 jurisdictions – Original, appellate and than Rs. 5 than Rs. 50 than Rs.
turnover
advisory. crore crore 250 crore

 It serves as a Constitutional Court as well as a  Significance


Court of Appeal.
o Contributes about 30% to India’s GDP
 Composition – Article 124 deals with
composition, at present 34 judges are o Employs around 11 crore people
present in the Supreme Court.
o Constitutes around 40% of total
 Qualifications – The Constitution has not exports
prescribed a minimum age for
appointment as a judge of Supreme Court. o More than half of them located in rural
India.
o He should be a citizen of India.
11. a
o He should have been a judge of a High
Court (or high courts in succession) for Air Quality Index (AQI)
five years; or
As the air quality in Delhi has plummeted to ‘severe’
o He should have been an advocate of a category with the AQI breaching the 400 mark, the
High Court (or High Courts in stage 3 of the Graded Response Action Plan (GRAP)
succession) for ten years; or was set in motion.

o He should be a distinguished jurist in  AQI is a colour coded index showing the


the opinion of the president. measure of air quality.

 Appointed by – President.  Launched in – 2014, as a part of the Swachh


Bharat campaign.
 Removal by – Impeachment, as per the
procedures laid in The Judges Enquiry Act  Aim – To communicate to the public and
(1968). the government, the current level and
future projections of air pollution.
 Seat – The Constitution declares Delhi as the
seat of the Supreme Court.  Represented pollutants – Around 6 to 8 air
pollutants like ground level Ozone (O3),
 It also authorises the Chief Justice of particulate matter (PM 10 and PM 2.5), carbon
India to appoint other place or places as monoxide (CO), sulphur dioxide (SO2),
seat only with the approval of the nitrogen dioxide (NO2), etc.
President.

CHENNAI |SALEM| MADURAI | COIMBATORE DELHI | BANGALORE | THIRUVANANTHAPURAM


www.shankariasacademy.com 247
www.iasparliament.com

 AQI transforms air quality data of these  The isthmus lies between the Gulf of
pollutants into 1 number (index value), Thailand to the east and the Andaman Sea to
nomenclature and colour. the west.

 6 Categories – Good (0-50), satisfactory (50-  Since the late 19th century, the Isthmus of Kra
100), moderately polluted (100-200), poor has frequently been proposed for canalization
(200-300), very poor (300-400) and severe to decrease the shipping time between Europe
(400-500). and East Asia by eliminating the voyage around
Singapore.
 The higher the AQI, the worse the air and large
percentage of the population is likely to 13. a
experience adverse health effects.
Elephant Corridor Report, 2023

In the recently held standing committee meeting of the


National Board for Wildlife, Raman Sukumar, an
Indian ecologist reported multiple inconsistencies in
the elephant corridor report, 2023.

 A corridor is a small patch of land that provides


connectivity for elephant movement across
habitats, largely within a landscape of the
elephant reserve.

 According to the report, there is an increase of


62 elephant corridors in the country since
2010, bringing the total to 150.

 West Bengal has the most elephant corridors


12. b (26) that amounts to 17% of the total elephant
corridors.
Kra Isthmus
 Elephants have seen increased presence in
The Thailand Prime Minster has proposed a route Madhya Pradesh found in Sanjay Tiger reserves
across the Isthmus of Kra as an alternative to the route and Bandhavgarh.
that passes through Strait of Malacca.
 The elephant corridor in forest areas shall be
 Kra Isthmus is narrow neck of southern notified under the rules mentioned under
Myanmar and Thailand, connecting the Wild Life (Protection) Act, 1972.
the Malay Peninsula to the Asian mainland.
 The elephant corridor areas within the revenue
 An isthmus is a narrow strip of land that lands and private lands must be notified under
connects two larger landmasses and separates the Environment (Protection) Act, 1986.
two bodies of water.
14. b

Commission on Social Connection

Commission on Social Connection was recently


launched to address the increasing disease of
loneliness.

 Launched by – World Health


Organisation (WHO)

 It will promote social connection as a priority


and accelerate the scaling up of solutions in
countries of all incomes.

 The Commission will consider how connection


enhances the well-being of our communities
and societies and helps foster economic
progress, social development, and innovation.

CHENNAI |SALEM| MADURAI | COIMBATORE DELHI | BANGALORE | THIRUVANANTHAPURAM


www.shankariasacademy.com 248
www.iasparliament.com

 The Commission is to be co-chaired by US o Saving in Forex, promoting circular


Surgeon General, Dr Vivek Murthy and African economy and to assist in achieving the
Union Youth Envoy, Chido Mpemba, and will target of net zero emission.
have 11 leading policy-makers, thought leaders
and advocates.  It will be monitored and implemented by
the Central Repository Body
 This WHO Commission will help (CRB), Ministry of Petroleum and
establish social connection as a global health Natural Gas.
priority and share the most promising
interventions.  CBO will be voluntary till FY 2024-2025
and mandatory blending obligation would
15. c start from FY 2025-26.

Atmospheric Gravity Waves (AGW)  CBO shall be kept as 1%, 3% and 4% of total
CNG/PNG consumption for FY 2025-26, 2026-
NASA is set to launch the Atmospheric Waves 27 and 2027-28 respectively.
Experiment (AWE) to study the important drivers of
Space weather including Atmospheric Gravity Waves  From 2028-29 onwards CBO will be 5%.
(AGW).
17. a
 All air motions are influenced by gravity and it
can have two motions, either straight or wave Cryosphere
and these waves can be vertical or horizontal.
 The cryosphere contains the frozen parts of the
 AGW is vertical wave that is generated when planet. It includes snow and ice on land, ice
there is an extreme weather event or a sudden caps, glaciers, permafrost, and sea ice.
disturbance leading to a vertical displacement
of stable air.  The State of the Cryosphere Report,
2023 - Released by the International
 Natural phenomena like thunderstorms, Cryosphere Climate Initiative (ICCI), a
hurricanes, tornadoes, regional orography and network of policy experts and researchers
others have the potential to send out a variety working to preserve the Earth’s cryosphere.
of periodic waves, including AGWs.
 Key findings of the report – The Himalayas is
 In a gravity wave, the upward moving expected to lose 50% of today’s ice if global
region is the most favorable region for cloud average temperatures touch 2°C.
development and the sinking
region favorable for clear skies.  When permafrost thaws, it releases CO2 and
methane emissions, which will cause a spike in
 AGW moves through a stable layer of the temperatures even if human emissions reach
atmosphere and continue all the way to Space, zero.
where they contribute to the Space weather.
 The Earth’s ice sheets lost 7,560 billion tonnes
 Atmospheric Waves Experiment (AWE) is of ice between 1992 and 2022.
a first-of-its-kind NASA experimental
attempt aimed at studying the interactions  Ice sheets in Greenland and parts of Antarctica
between terrestrial and Space weather. could contribute between 12-20 metres of sea-
level rise at 2°C.
16. b
 The Global Cryosphere Watch (GCW) -
Compressed Bio-Gas Blending Obligation An international mechanism of World
(CBO) Meteorological Organization (WMO) for
supporting all key cryospheric in-situ and
 CBO aims to promote production and remote sensing observations.
consumption of Compressed Bio-Gas (CBG) in
the country. 18. d

 The key objectives of the CBO include: Amaterasu

o To stimulate demand for CBG in City  It is an extremely high-energy


Gas Distribution sector, particle that exceeds 240 exa-electron volts
(EeV) and seemingly arrived from a void in
o Import substitution for Liquefied space where nothing is known to exist.
Natural Gas (LNG) and

CHENNAI |SALEM| MADURAI | COIMBATORE DELHI | BANGALORE | THIRUVANANTHAPURAM


www.shankariasacademy.com 249
www.iasparliament.com

 This energy is millions of times more  The IUCN Status of Sangai is Critically
powerful than the particles produced by Endangered.
the Large Hadron Collider, the most
powerful accelerator ever built.  It is included under the Schedule-1 of Wildlife
(Protection) Act 1972.
 It is second only to the “Oh-My-God”
particle, another high-energy cosmic ray 20. c
detected in 1991 that came in at 320 EeV.
A23a
 Ultra-high energy particles like Amaterasu
usually travel through space quite smoothly The British Antarctic Survey recently found that one of
since they don’t bounce off magnetic fields, like the world’s largest icebergs is drifting beyond
low-energy cosmic rays but it is not in case of Antarctic.
Amaterasu.
 A23a is one of the world’s largest icebergs
 It was discovered by a Japanese scientist that is drifting beyond Antarctic waters,
Toshihiro Fujii in May 2021. after being grounded for more than 3 decades.

 The scientists propose 3 explanations for the  The iceberg split from the Antarctic’s Filchner
enigmatic origin of the particle. Ice Shelf in 1986 and it became stuck to the
ocean floor and had remained for many years in
o It could be from a source that we have the Weddell Sea.
not yet identified
 The iceberg is about 3 times the size of New
o It might have been magnetically York City and more than twice the size of
deflected much higher than current Greater London, measuring around 4,000
models predict sq.kms.

o Scientists may have to rethink their  The iceberg is moving past the northern tip of
understanding of high-energy particle the Antarctic Peninsula, by wind and ocean
physics current.

19. b

Sangai Deer

The recent proposal of hydro-electric modernisation


plan in the Manipur’s Loktak Lake could be
detrimental to the Sangai deer.

 Sangai Deer also called as the Brow-antlered


deer and Dancing Deer is a unique species
found exclusively in Manipur's Keibul
Lamjao National Park.

 Phumdi is the most important and unique


part of Sangai’s habitat.

 It is the state animal of Manipur.

21. c

Governor’s grant of assent to Bills

Recently, the Tamil Nadu government has challenged


the actions of its Governor, R.N. Ravi, in the Supreme
Court.

CHENNAI |SALEM| MADURAI | COIMBATORE DELHI | BANGALORE | THIRUVANANTHAPURAM


www.shankariasacademy.com 250
www.iasparliament.com

 Article 200 – When a Bill passed by the 22. d


legislature of a State is presented to the
Governor, he/she has 4 options. Suez Canal

Israel has come up with plans to create an alternative


to the Suez Canal.

 Suez Canal – A man-made canal that


connects the Mediterranean Sea to the Red Sea.

 Need – A crucial shortcut for shipping between


Europe and Asia.
 Legislature’s overriding powers – If the
bill returned by the governor is passed again by  Geography – It cuts north-south across the
the assembly with or without amendments, Isthmus of Suez in Egypt.
then the governor must give his assent to the
bill.  North to south – Port Said, Port Al
Qantara, Port Ismailia, Port Fai’d, Port Suez
 The Governor enjoys only a ‘suspensive veto’. and Gulf of Suez.
 Money Bills – He has 3 alternatives

o Give his assent to the bill.

o Withhold his assent to the bill, which


then ends there.

o Reserve the bill for the


consideration of the president but
President cannot return it for the
reconsideration of State legislature.

 He cannot return a money bill for the


reconsideration of the state legislature.

 Reservation for President – It is done when


the governor feel that the provisions of the Bill
will contravene the provisions of the
Constitution.

o Bills reducing the power of the High


Court

o Bills on concurrent list that are


repulsive to a Union law based on
ministerial advice.

 Governor will not have any further


role when the bill is reserved for President’s
consideration.
 Year of construction – Began in 1859 and
 Shamsher Singh Case – The Governor does got completed in 10 years.
not exercise their discretionary powers while
withholding assent or returning a bill.  Owned and maintained by – Suez Canal
Authority (SCA), which is under the
 Rameshwar Prasad Case – Governor’s government of Egypt.
refusal to give assent to the bill is subjected to
judicial scrutiny and can be struck down as  Convention of Constantinople- Signed in
unconstitutional. 1888 by the maritime powers, it stated that the
canal should be open to ships of all nations in
 Nabam Rebia case – The discretionary times of both peace and war.
powers of the Governor to withhold assent are
amenable to judicial review.  Monopoly – Egypt’s control over the
waterway has been a source of conflict for
almost 70 years now.

CHENNAI |SALEM| MADURAI | COIMBATORE DELHI | BANGALORE | THIRUVANANTHAPURAM


www.shankariasacademy.com 251
www.iasparliament.com

 Ben Gurion  OBC – It was coined to denote backward/


Canal marginalised communities and castes that were
Project – A not Scheduled Castes (SCs) or Scheduled Tribes
proposal to (STs).
create an
alternate to Suez  Article 340 – It provides for the appointment
Canal. of a Commission to investigate the conditions
for the improvement of socially and
 It passes educationally backward class.
through
the Negev 1st OBC Commission
Desert in Israel, connecting the Gulf of Aqaba
and the Eastern Mediterranean.  It was formed in 1953, headed by Kaka
Kalelkar.
23. d
 Objective – To identify socially and
Central Adoption Resource Authority (CARA) educationally backward classes

 It is a statutory body of Ministry of Women  Report – It prepared a list of 2,399 backward


& Child Development, Government of India. castes or communities in the country,
categorised 837 of them as “most backward”.
 Enacted under – Juvenile Justice (Care
and Protection of Children) Act, 2015.  Recommendations – Enumerating castes in
the 1961 census
 India became a signatory to the Hague
Convention on Protection of Children and Co-  Providing 25-40% reservation at different
operation of 1993 and India ratified the levels of government jobs
convention in 2003.
 70% reservation for admission to technical and
Functions of CARA professional institutions

 Adoption  Outcome – It was never discussed in the


Parliament and never implemented.
o To promote in-country adoptions
2nd OBC Commission
o To facilitate inter-state adoptions in
coordination with State Agency  It was formed in 1979 by B.P.Mandal
o To regulate inter-country  Findings – Identified 3,743 castes and
adoptions communities as OBCs
 Regulations – To frame regulations on o Estimated their population at 52%
adoption and related matters from time to time
as may be necessary  Recommendations – 27% reservation in
government jobs and admissions to all
 Monitor – It regulates bodies like government-run scientific, technical, and
professional institutions
o State Adoption Resource Agency
(SARA)  Outcome – Within the 27% reserved for
Socially and Economically Backward Castes,
o Specialised Adoption Agency (SAA) preference shall be given to candidates
belonging to the poorer sections of the SEBCs.
o Authorised Foreign Adoption Agency
(AFAA) 3rd OBC Commission
o Child Welfare Committees (CWCs) and
 It was formed in 2017, headed BY
o District Child Protective Units (DPUs) Justice G.Rohini.

24. d  Objectives – To examine the inequitable


distribution of reservation benefits among
OBC Commissions OBCs in the Central List.

Recently, the Minister for Backward Classes Welfare in o To work out the mechanism, criteria,
Andhra Pradesh said that the state will begin a norms and parameters in a scientific
backward class census. approach

CHENNAI |SALEM| MADURAI | COIMBATORE DELHI | BANGALORE | THIRUVANANTHAPURAM


www.shankariasacademy.com 252
www.iasparliament.com

o To identify the respective castes and  It is an annual assessment.


classify them into their respective sub-
categories  Published by – United Nations
Environment Program (UNEP)
 Recommendations – It suggested breaking
the caste groups into broad categories,  Aim – To evaluate the gap between the amount
of global greenhouse gas emissions now and
o With the dominant castes getting the what is necessary to meet the objectives of the
smallest share of the 27% reservation Paris Agreement.
o The historically crowded-out caste  It tracks the greenhouse gas emission
groups getting the largest share of the projection for 2030 and what is
reservation pie. necessary to limit global warming to
well below 2°C or 1.5°C.
25. a
 The report assessed countries’ Nationally
Reverse Flipping Determined Contributions (NDCs), which they
are required to update every 5 years.
Economic Survey 2023-24 acknowledged the concept
of reverse flipping and this trend has been gaining
 2023 Theme – “Broken Record –
attention in the recent years.
Temperatures hit new highs, yet world fails to
cut emissions (again)”.
 Flipping – The process of transferring the
entire ownership of an Indian company to an
 2023 Report – The planet-warming GHG
overseas entity, accompanied by a transfer of
emissions must fall by 42% by 2030 to hold
intellectual property rights and data owned by
warming at 1.5 C.
the Indian company.
 GHG emissions across the G20 also increased
 Indian company becomes a 100% subsidiary of
by 1.2 % in 2022.
a foreign entity, with the founders and investors
retaining the same ownership via the foreign
 India accounts for 18% of the world population,
entity, having swapped all shares.
but to date only contributed 5% of warming.
 Reasons for flipping - Taxation benefits, 27. a
ease of setting up in places like Singapore, the
US and ease of attracting foreign capital Intangible Cultural Heritage of UNESCO
 Reverse flipping – It is the trend Ramlila is featured in the list of the Intangible Cultural
of overseas start-ups shifting their Heritage of UNESCO.
domicile to India and listing on Indian stock
exchanges.  It includes traditions or living expressions
inherited from our ancestors and passed on to
 Reasons for reverse flipping to India – our descendants.
India’s favourable economic policies (tax
breaks, funding assistance), expanding  UNESCO’s 2003 Convention for the
domestic market and growing investor Safeguarding of the Intangible Cultural
confidence in the country's start-up ecosystem. Heritage proposes 5 broad ‘domains’

 Reverse Merger - It refers to a situation o Oral traditions and expressions,


where a private company becomes a public including language as a vehicle of the
company by purchasing control of the public intangible cultural heritage
company.
o Performing arts
 Nearshoring - It is a strategic business
practice, involves outsourcing tasks to o Social practices, rituals and
neighbouring or nearby countries rather than festive events
distant offshore locations.
o Knowledge and practices
26. a concerning nature and the universe

Emission Gap Report 2023 o Traditional craftsmanship

The Emission Gap Report 2023 has revealed that the  Features – It also includes contemporary
world is set to get warmer by at least 3 degree Celsius rural and urban practices in which diverse
by the end of the century with the current climate cultural groups take part.
policies of the countries.

CHENNAI |SALEM| MADURAI | COIMBATORE DELHI | BANGALORE | THIRUVANANTHAPURAM


www.shankariasacademy.com 253
www.iasparliament.com

 It is not merely valued as a cultural good, on a  Themes – Religious epics such as Mahabharat
comparative basis, for its exclusivity or its and Ramayana, Puranas, and Gita were
exceptional value. frequently painted in the style.

 It thrives on its basis in communities.  Basohli painting – A school of Pahari


miniature painting that flourished in
 It can only be heritage when it is recognized as the Indian hill states during the late 17th
such by the communities, groups or individuals and the 18th centuries, known for its bold
that create, maintain and transmit it. vitality of colour and line.

 Ramlila – It is a performance of Ramayana 29. c


epic in a series of scenes that include song,
narration, recital and dialogue. Terminologies related to Sikhism

 This staging of the Ramayana is based on the Recently, the birth anniversary of Guru Nanak was
Ramacharitmanas, composed by Tulsidas in celebrated Guru Nanak Jayanti (Gurpurab).
the 16th century in a form of Hindi in order to
make the Sanskrit epic available to all.  Guru Nanak Dev (1469–1539) was the founder
of Sikhism.
28. b
 Nanak was a Bhakti saint who preached that the
Chavittu Natakam most important form of worship is Bhakti, or
devotion to Waheguru.
 It is a colourful, vigorous theatre form that
flourished in Kerala with the spread of Terminologies related to Sikhism
Christianity.
It is a religious, ascetic sect, focus
 Portuguese are considered to have introduced Udaasis on the teachings of its founder, Sri
this art form here. Chand , son of Guru Nanak Dev.

 Influenced from – Opera, Kathakali and Sikhs celebrate Guru nanak jayanti
Nagar Kirtan
Kalaripayattu. with a procession called Nagar Kirtan.

 Stamping Drama- The artists produce Janamsakhis Biographies of Guru Nanak


resonating sounds by stamping the floor while
dancing. It is a cube-shaped structure in Mecca
Kaba
that is considered holy.
 Stage is named as ‘thattu’, which is laid with
planks of wood. It is a place where people contribute
their labour and help the needy
Langar
 Instruments used – Chenda, Padathamber, through acts such as cooking food for
Maddalam and Ilathalam, provides background them at the gurudwaras.
music.
Meaning - My own limb, Guru Nanak
 These days Tabala, Fiddle, Flute and Bulbul are Dev said that Lehna (one of his
also played. Angad disciple) as Angad instead
of his own son who is not inclined
Pahari painting towards spirituality.

 Pahari denotes ‘hilly or mountainous’ in origin. Ntim Word

 It is a style of miniature painting that Dan Charity


developed in the independent states of the
Himalayan foothills in India. Isniin Ablution

 Origin – It goes back to the Buddhist Pala Sevii Service


dynasty, which ruled Bengal and Bihar during
Simran Prayer
8th – 11th century.

 Features – It demonstrate challenges in their


territorial classification and refers both 30. d
religious and secular subjects.
Nidirana Noadihing

Nidirana Noadihing is a new species discovered in


Arunachal Pradesh.

CHENNAI |SALEM| MADURAI | COIMBATORE DELHI | BANGALORE | THIRUVANANTHAPURAM


www.shankariasacademy.com 254
www.iasparliament.com

 It is called as a 'music frog' 32. b

 Features – Both the male and female are Varaha


vocal.
 Varaha is ocean mining vehicle, part of Deep
 It has a unique call pattern consisting of two- Ocean Mission (or) Samudrayaan Mission.
three notes.
 MATSYA-6000 is also a part of the Deep Ocean
 Nidirana are known to inhabit swamps, ponds Mission.
and paddy fields, and they often construct nests
to lay their eggs.  Varaha is a self-propelled track-based seabed
mining system and operates on the flexible riser
31. d technique.

Gulf Cooperation Council (GCC)  Flexible riser is a class of flexible pipes, which is
used to connect subsea pipelines to floating
A Qatari court sentenced 8 Indian Navy veterans to offshore installations.
death on charges of spying for Israel, thus straining the
bilateral relations and also impacting India’s relation  Varaha is lowered to the ocean bed from a
with GCC countries. surface ship using a high-strength flexible cord
system.
 GCC – It is a regional political and economic
union.  Once the vehicle reaches the ocean bed, it will
be able to move around while the surface ship
 Established in – 1981. moves in tandem.

 Headquarters – Riyadh, Saudi Arabia 33. c

 Aim – To promote cooperation among Road Accidents in India Report, 2022


member states.
The Ministry of Roads, Transport and
 Initially proposed as a “Gulf Union” for deeper Highways recently released the Road Accidents in
coordination, its military arm, the Peninsula India Report, 2022 which indicate that the number of
Shield Force, was created in 1984. accidents in 2022 have increased by 11.9% compared
to 2021.
 Members – 6 (Bahrain,
Kuwait, Oman, Qatar, Saudi Arabia, and  This report is based on the data/information
the United Arab Emirates). received from police departments of
States/UTs on calendar year basis.
 India and Yemen are not a member.
 The data provided are in conducive with
 Al-ula declaration – During the 41st GCC standardized formats provided in Asia Pacific
summit, it was signed based on greater Road Accident Data (APRAD) base project.
cooperation within the Council.
 Asia Pacific Road Accident Data (APRAD) base
 Significance – They have over 8.5 million project is under United Nations Economic and
non-resident Indians, constituting around 65% Social Commission for Asia and the Pacific
of total NRIs. (UNESCAP).

Key highlights of the report

 Tamil Nadu recorded the highest number of


road accidents on National Highways in 2022,
whereas, the number of persons killed in road
accident was the highest in Uttar Pradesh.

 Both road accidents and accident-related


killings are more of a rural phenomenon than
an urban phenomenon.

 In 2022, about 68% of road accidents death


took place in rural area where as urban
area accounted for 32% of total accidents
death in the country.

CHENNAI |SALEM| MADURAI | COIMBATORE DELHI | BANGALORE | THIRUVANANTHAPURAM


www.shankariasacademy.com 255
www.iasparliament.com

 Among vehicle categories involved in road  The cost of adaptation is the amount needed for
accidents, two-wheelers for the second planning, preparing for, facilitating and
consecutive year, accounted for the highest implementing measures to reduce harm or
share in total accidents and fatalities during exploit beneficial opportunities arising from
2022. climate change.

 The Electronic Detailed Accident Report  Adaptation measures such as river flood
(e-DAR) is a portal developed by the Ministry protection, infrastructure and coastal
of Roads, Transport and Highways (MoRTH). protection demand the highest adaptation costs
in regions of East Asia, Pacific and Latin
 e-DAR project is aided by World Bank America and the Caribbean.
Assistance to provide an integrated and
systemic solution to problem of accidents in  The Climate adaptation finance flows from
line with international practice. public multilateral (like World Bank) and
bilateral sources (from a developed to a
34. c developing nation) declined by 15 % to around
USD 21 billion in 2021.
National Efficient Cooking Programme
 Domestic budgets seem to be the largest source
Recently the Union Minister for Power and New & of funding for adaptation in many developing
Renewable Energy unveiled the National Efficient countries.
Cooking Programme to revolutionize cooking practices
in India. Composite Water Management Index (CWMI)

 The National Efficient Cooking Programme  The index is released by the NITI
(NECP) introduces induction-based cook- Aayog and upgraded version has not been
stoves, offering a cost advantage of 25-30% over released by the Niti Aayog.
traditional cooking methods.
 The index is the first comprehensive collection
 It aims to promote both energy savings and of countrywide water data in India, based on
cost-effective cooking solutions by introducing in-depth structured questionnaires followed by
affordable and energy-efficient induction- focus group discussions to generate qualitative
based cook stoves. information.
 It was launched by the Energy Efficiency  The 1st edition was published in 2018.
Services Limited (EESL), which is
under Ministry of Power and New &  The Index has been developed in close
Renewable Energy. collaboration with multiple national and state
stakeholders and involved a robust data
 EESL seeks to reduce the environmental impact validation process.
of cooking methods, ensuring cleaner air and
improved health for citizens.  It ranks states in terms of efficacy based on 28
parameters.
 EESL has also partnered with Modern Energy
Cooking Services (MECS) for the large-scale  The index, published by NITI Aayog, was
deployment of induction cooktops. prepared in association with three ministries
that includes Water Resources, Drinking Water
 Energy Efficient Fans Programme & Sanitation and Rural Development.
(EEFP) focuses on deploying energy-
efficient ceiling fans, which was also  The Index comprises 9 themes (each having an
launched along the NECP. attached weight) with 28 different
indicators.
35. c

Adaptation Gap Report 2023

 The report identifies 7 ways to increase


financing, including through domestic
expenditure and international and private
sector finance.

 The Adaptation Gap Report 2023 was


released by the United Nations
Environment Programme (UNEP).

CHENNAI |SALEM| MADURAI | COIMBATORE DELHI | BANGALORE | THIRUVANANTHAPURAM


www.shankariasacademy.com 256
www.iasparliament.com

Global Plastics Outlook: Policy Scenarios to  Advocate-on-Record (AoRs) are a pool of elite
2060 Report Delhi-based lawyers whose legal practice is
mostly before the Supreme Court.
The Global Plastics Outlook: Policy Scenarios to 2060
Report was recently released by the Organisation  They can appear before other courts too.
for Economic Co-operation and Development
(OECD)  Only an AoR can file cases before the
Supreme Court.
 The report states that if the present trends
persist, the global plastic production will triple  An AoR might engage other lawyers including
and exceed 1 billion tonnes by the year 2060. senior counsels to argue before the Court but
the AoR is essentially the link between the
 The report indicates that the plastic pollution litigant and the highest court of the country.
will rise in almost threefold increase in plastics
use driven by rising populations and incomes  The Supreme Court Rules, 2013
without any radical action taken. prescribe eligibility criteria for an AoR.

 The report estimates that almost 2/3rd of  An advocate has to clear an examination set by
plastic waste in 2060 will be from short-lived the Court itself, the advocate has to meet
items such as packaging, low-cost products and specific criteria to be eligible to appear for the
textiles. exam.

 Globally, plastic leakage to the environment is  According to Section 30 of the Advocates Act of
seen doubling to 44 million tonnes (Mt) a year. 1961, any lawyer enrolled with the Bar Council
is entitled to practice law before any Court or
 The build-up of plastics in lakes, rivers and tribunal in the country.
oceans will more than triple, as plastic waste
balloons from 353 Mt in 2019 to 1,014 Mt in  Article 145 of the Constitution states that the
2060. Supreme Court is empowered to make rules and
regulate its own procedure for hearing cases.
 The incineration and landfilling of plastic waste
will continue to account for around 20% and  The AoR system is broadly based on the British
50% of plastic waste respectively. practice of barristers and solicitors.

36. d 38. a

UNESCO Michel Batisse Award Volcanoes

The director of Gulf of Mannar biosphere reserve was  They are openings, or vents where lava, tephra
selected for UNESCO Michel Batisse Award for 2023. (small rocks), and steam erupt onto the Earth’s
surface.
 UNESCO Michel Batisse Award is a 12,000
USD award that is given every 2 years during  Volcanoes can be on land and in the ocean and
the Man and the Biosphere Programme formed through the movement and collision of
(MAB) Council, in memory of Dr Michel tectonic plates.
Batisse.
 There are 4 main types of volcanoes that
 It is awarded for outstanding achievements in includes:
the management of the biosphere
reserves in line with the recommendations of 1. Cinder cones
the Seville Strategy.
2. Composite or stratovolcanoes
 The Seville Strategy provides
recommendations for developing effective 3. Shield volcanoes
biosphere reserves and for setting out the
conditions for the appropriate functioning of 4. Lava domes
the World Network of Biosphere Reserves.
 Some of the most active volcanoes are located
37. a in the Pacific Ring of Fire, which includes New
Zealand, Southeast Asia, Japan and the western
Advocate-on-Record System coast of the Americas.

Recently the Supreme Court recently stopped an  About 90% of all earthquakes worldwide strike
Advocate-on-Record (AoR) for filing a frivolous case within Pacific Ring of Fire.
and dismissed the public interest litigation.

CHENNAI |SALEM| MADURAI | COIMBATORE DELHI | BANGALORE | THIRUVANANTHAPURAM


www.shankariasacademy.com 257
www.iasparliament.com

 Italy’s Mount Etna – Is one of the world’s 40. a


most active volcanoes, was causing public
concern as it started erupting the year before. E Prime Layer

 It is the Europe’s most active volcano and one Researchers revealed the formation of a new enigmatic
of the largest in the world. layer – E prime layer at the outermost part of Earth's
core.
 Fagradalsfjall – Located in the Iceland.
 Earth comprises 4 primary layers that includes:
 Kilauea – It is located in Hawaii, United
States. 1. An ‘inner core’ at the planet's centre

 Dukono – Located in Indonesia started 2. ‘Outer core’ that surrounds the inner
erupting in August 1933 and is continuing. core

39. b 3. Mantle

Tantalum 4. Crust

Researchers from the Indian Institute of Technology  A new enigmatic layer or E prime layer is
(IIT), Ropar has found the presence of tantalum, a rare formed at the outermost part of Earth's core.
metal, in the Sutlej River.
 It is formed as a result of surface water
 Tantalum is a rare metal that was discovered penetrating deep into the planet altering
in 1802 by the Swedish chemist Anders Gustaf the composition of the metallic liquid core's
Ekeberg. outermost region.

 It is bright, very hard, silver-grey metal of  The material exchange between the core and
Group 5 of the periodic table. mantle is small, however, the experiments
revealed that when water reaches the core-
 It is characterized by its high mantle boundary, it reacts with silicon in the
density, extremely high melting point, and core, forming silica.
excellent resistance to all acids except
hydrofluoric at ordinary temperatures.  It is a hydrogen-rich, silicon-depleted
layer.
 It has 3rd highest melting point next to
tungsten and rhenium.  Layer formation process – Tectonic plates
carrying surface water have transported it deep
 As tantalum has a high melting point, it is into the Earth over billions of years.
frequently used as a substitute for
platinum, which is more expensive.  Upon reaching the core-mantle boundary about
1,800 miles below the surface, this water
 It possesses high corrosion resistance, initiates significant chemical changes,
because when exposed to air, it forms an oxide influencing the core's structure.
layer that is extremely difficult to remove, even
when it interacts with strong and hot acid  This reaction leads to the formation of a
environments. hydrogen-rich, silicon-depleted layer at the
outer core, resembling a film-like structure.
 Pure tantalum is ductile, meaning it can be
stretched, pulled, or drawn into a thin wire or  Silica crystals generated by this process ascend
thread without breaking. and blend into the mantle, impacting the
overall composition.
 It is almost completely immune to chemical
attack at temperatures below 150°C and is  These modifications in the liquid metallic layer
attacked only by hydrofluoric acid, acidic could potentially result in reduced density and
solutions containing the fluoride ion and free altered seismic characteristics.
sulphur trioxide.
41. b
 Tantalum is one of the Critical Minerals.
Jammu and Kashmir Reorganization Act, 2019
 It is mined in many places including Australia,
 The law bifurcated the State into two Union
Canada and Brazil.
Territories (UTs)
 Uses – One of the main uses of tantalum is in o Ladakh (without Legislature) -
the production of electronic components. Comprises Kargil and Leh districts

CHENNAI |SALEM| MADURAI | COIMBATORE DELHI | BANGALORE | THIRUVANANTHAPURAM


www.shankariasacademy.com 258
www.iasparliament.com

o J&K (with Legislature) - Comprises all  Scientific evidences also show their presence in
the remaining territories of the other planets like Mars and Venus.
erstwhile State of J&K.
 Formation – They are formed when material
 The Act amended the 2nd Schedule of the significantly hotter than its surroundings is
Representation of People’s Act, 1950 to specify erupted onto the surface of the Earth.
the total number of seats in the Jammu and
Kashmir Legislative Assembly to be 83.  Earth Core – Earth’s interior has outer crust,
middle mantle and inner core layer.
 It reserved six seats for Scheduled Castes. No
seats were reserved for Scheduled Tribes.  Mantle is denser than that of the crust and
contains a weaker zone called asthenosphere
 The Governor of the existing State of Jammu from which the molten rock materials find their
and Kashmir became the common Lieutenant way to the surface.
Governor for both the Union territories.
 Liquid rock is known as magma when it is
 The Union Territory of Jammu and Kashmir underground and called as lava when it breaks
will have a Council of Ministers of not more through the surface.
than 10% of the total number of members in the
Assembly.  3 ways of magma rise

 The Council will aide and advice the Lieutenant o Divergence of tectonic plates –
Governor on matters that the Assembly has Here, the magma rises up to fill in the
powers to make laws. space and when this happens
underwater volcanoes can form.
 The Chief Minister will communicate all
decisions of the Council to the Lieutenant o Convergence of tectonic plates –
Governor. When this happens, part of Earth’s
crust can be forced deep into its
 The High Court of Jammu and Kashmir will be interior, which under high heat and
the common High Court for the Union pressure melts, and rise as magma.
Territories of Ladakh, and Jammu and
Kashmir. o At hotspots – They are hot areas
inside of the Earth, where magma gets
 The Legislative Council of the state of Jammu heated up and it becomes less dense,
and Kashmir will be abolished. leading to its rise.

 The Act made amendments to the First  There are no volcanoes in the Himalayan
Schedule of the Indian Constitution to reflect region of India.
the new status of Jammu and Kashmir and
Ladakh.  However, Barren Island, lying 135 km north-
east of Port Blair became active in 1991 and
 National Unity Day was observed on 1995.
October 31, 2019, the day the two union
territories (J&K and Ladakh) were established.  The other volcanic island in Indian Territory
is Narcondam (Andaman and Nicobar
 National Integration Day, also known as Islands) which is probably extinct. Its crater
Quami Ekta Divas, is celebrated in India wall has been completely destroyed.
on November 19 each year which
commemorates the birthday of Indira Gandhi, 43. d
India’s first-ever female Prime Minister.
BIMSTEC
42. a
 It is a regional organization group of littoral
Volcanoes countries encircling the Bay of Bengal with own
flag and emblem.
 According to the US Geological Survey,
volcanoes are openings, or vents where lava,  Established in – 1997 with the signing of
tephra (small rocks), and steam erupt onto the the Bangkok Declaration.
Earth’s surface.
 2022 marked the 25th anniversary of BISTEC
 Occurrence – It can be on land and in the formation.
ocean in Earth.
 Secretariat- At Dhaka, Bangladesh.

CHENNAI |SALEM| MADURAI | COIMBATORE DELHI | BANGALORE | THIRUVANANTHAPURAM


www.shankariasacademy.com 259
www.iasparliament.com

 Aim – To foster economic and social  Dollarisation can impair the lender of the last
development among member countries. resort’s ability to provide emergency liquidity to
the financial system in the time of crisis.
 Members – 7 members including Bangladesh,
Bhutan, India, Nepal, Sri Lanka, Myanmar and  Seigniorage loss in dollarization- When a
Thailand. country dollarizes, it gives up the option to print
its own money. As a result, the central bank
 BIMSTEC Charter – It was signed and loses its ability to directly influence the
adopted during the fifth BIMSTEC economy through monetary policy.
Summit in Sri Lanka in 2022.
 Seigniorage is the profit that a
 BIMSTEC Centres- government or a central bank makes by
issuing money
o BIMSTEC Energy Centre - AT
Central Power Research Institute 45. b
in Bengaluru, India
Bharat Stage Emission Standards
o BIMSTEC Centre on Weather and
Climate - It is located in the National  Objective- To regulate the output of air
Centre for Medium Range Weather pollutants from compression and spark-
Forecasting in Uttar Pradesh, India. ignition engines equipment, including motor
vehicles.
o BIMSTEC Cultural Industries
Observatory - Bhutan  They are in line with the European emission
norms with a time lag of 5 years.
o BIMSTEC Technology Transfer
Facility - The entry into force is  Mashelkar Committee recommended a
awaited. roadmap for implementation of Euro norms
based on emission standards for India.
44. c
 Launched by- Central Pollution Control
Dollarisation Board (CPCB) under the Ministry of
Environment, Forest and Climate Change
 It is the process by which a country decides to (MoEFCC).
use two currencies the local currency and
generally a stronger, more established currency  In 1999, Supreme Court mandated that all
like the US dollar. vehicles in India have to meet the Euro I or
India 2000 standard.
 Occurrence- It occurs in developing countries
with a weak central monetary authority or an
unstable economic environment.

 It usually happens when a country’s currency


becomes unstable or loses its value due to high
inflation or other economic problems.

 Example- Zimbabwe ran a dollarization test to


see if the adoption of foreign currency could
reduce high inflation and stabilize its economy.

 Dollarised economies- Ecuador, Panama


and El Salvador have had successful economic
outcomes following dollarisation.

 Significance- It fosters economic stability,


promote investment, boost trade, encourage
foreign direct investment and promotes fiscal
discipline.

 Issues- It leads to monetary 46. c


autonomy, seigniorage loss, the country
may face a loss of public support or legitimacy Global Partnership on Artificial Intelligence
and it increases the country’s vulnerability to
external shocks.  Launched in – 2020

CHENNAI |SALEM| MADURAI | COIMBATORE DELHI | BANGALORE | THIRUVANANTHAPURAM


www.shankariasacademy.com 260
www.iasparliament.com

 It is a multi-stakeholder initiative with experts  The park is renowned for its rich fossil deposits,
from industry, civil society, governments, which provide valuable insights into the
international organisations and academia. prehistoric world.

 GPAI Secretariat – OECD  UNESCO Global Geopark tag – Awarded


since 2015, is given to sites with global
o The 1st 3 GPAI summits were held in relevance to geology, geomorphology or
Montreal, Paris and Tokyo, paleontology.
respectively.
 India has sent a proposal to recognise
 Aim – To bridge the gap between theory and Bagh site where Dinosaur fossils National
practice on AI by supporting cutting-edge Park as global Geopark which is yet to receive
research and applied activities on AI-related the tag.
priorities.
 If selected, it will become the 1st Indian site to
 Mission – To share multidisciplinary join a select club of 195 UNESCO Geoparks in
research and identifying key issues among AI 48 countries.
practitioners and to understand AI
impacts.  Bhil Tribes – Popularly known as the bow
men of Rajasthan, are the most widely
 4 working groups – Responsible AI, data distributed and the largest tribe of the South
governance, the future of work, and innovation Asia.
and commercialization.
 Spread – Madhya Pradesh, Maharashtra,
 Values – Based on OECD Gujrat, and Rajasthan and also in the north
Recommendation on Artificial eastern parts of Tripura.
Intelligence grounded in the principles of
human rights, inclusion, diversity, innovation  The endogamous tribe speak Bhili, an Indo-
and economic growth. Aryan language.

 Membership – 29 members.  Economy – Earlier great hunters but they now


practice agriculture.
 It is open to countries, including emerging and
developing countries.  They are also demanding a separate state of
Bhil Pradesh.
 GPAI summit 2023- It adopted New Delhi
Declaration on advancing safe, secure and 48. b
trustworthy AI and commitment to supporting
the sustainability of GPAI projects. Karan Singh’s Proclamation of 1949

 Bletchley declaration- It is the outcome of Supreme Court cited Karan Singh’s Proclamation of
UK AI safety summit which mainly focused on 1949 while upholding the abrogation of Article 370.
security and safety risks related to AI.
 Yuvraj Karan Singh was the heir to the throne
 The GPAI summit contrasts with Bletchley of Jammu and Kashmir after his father
declaration as GPAI summit finds a balance ‘Maharaja Hari Singh’.
between innovation and the risks associated
with AI systems.  Repeal of Government of India Act, 1935

47. a  Applicability of Indian Constitution –


Enforced by the king, his heirs and successors
Kakad Bhairav or Bhilat Baba in accordance with the tenor of its provisions.

A stone deity worshipped by the Bhils in Madhya  It shall, as from the date of its commencement,
Pradesh’s Bagh (Narmada Valley) was recently proved supersede and abrogate all other constitutional
to be a fossilized dinosaur egg. provisions inconsistent therewith which were
in force in J&K.
 Just like their ancestors worshipped ancient
stones as deities, generations of Bhils had  Thus, the paragraph 8 of the Instrument of
worshipped this sunbaked, reddish-brown Accession ceased to be of legal
Kakad Bhairav or Bhilat Baba. consequence reflecting the full and final
surrender of J&K's sovereignty to India.
 Dinosaur Fossils National Park – It is a
protected area located in Madhya Pradesh,  Instrument of accession (IoA) – Maharaja
India. Hari Singh, then ruler of J&K signed IoA in

CHENNAI |SALEM| MADURAI | COIMBATORE DELHI | BANGALORE | THIRUVANANTHAPURAM


www.shankariasacademy.com 261
www.iasparliament.com

1947 and agreed to the accession into Indian 50. c


union.
Kashi Tamil Sangamam 2.0
 Article 370 – It accorded a special status to
J&K by limiting Parliament’s power to legislate Recently, the 2nd edition of Kashi Tamil Sangamam
for the State as compared to other States. was conducted.

 This was revoked by a Constitution Order of  It is an annual month-long programme started


2019. in 2022.

49. d  It was organised as part of the ‘Azadi ka Amrit


Mahotsav’ to uphold the spirit of 'Ek Bharat
PM Anusuchit Jaati Abhyuday Yojana (PM- Shreshtha Bharat'.
AJAY)
 2nd edition at – Kasi, Uttar Pradesh in 2023.
 It is a 100% Centrally Sponsored Scheme for the
welfare of Scheduled Caste (SC) population.  Aim – To revive the bonds between Varanasi
and Tamil Nadu through cultural exchange and
 Launched in – 2021-22. knowledge sharing.

 Launched by – Ministry of Social Justice  Implementing entities


& Empowerment.
o The Union Ministry of Education
 It has been framed after merging the 3
erstwhile schemes o IIT Madras

o Pradhan Mantri Adarsh Gram Yojana o Banaras Hindu University


(PMAGY)
 Participants – They have been divided into 7
o Special Central Assistance to groups, each named after a ‘sacred’ Rivers such
Scheduled Caste Sub Plan (SCA to as Ganga, Yamuna, Saraswati, Sindhu,
SCSP) Narmada, Godavari and Cauvery.

o Babu Jagjivan Ram Chatrawas Yojana 51. a


(BJRCY)
Universal Declaration of Human Rights
 Objectives – To increase the income of SC (UDHR)
population by income generating schemes,
skill and infrastructure development, reduce Human Rights 75 is an initiative to mark the 75th
the poverty and increase literacy and anniversary of Universal Declaration of Human
enrolment of SCs in education institutions. Rights.

 3 components  Human Rights 75 is an initiative whose main


goals focus on universality, progress and
o Development of SC dominated villages engagement under the leadership of UN
into an ‘Adarsh Gram’. Human Rights, together with its partners.

o Grants-in-aid for district/state-level  UDHR – A global document that establishes


projects for socio-economic betterment the human rights and civil liberty of every
of SCs. person in the world.

o Construction of hostels in higher  It consists of a preamble and 30


educational institutions. articles setting out fundamental rights and
freedoms.
 Eligibility Criteria – For income generating
and skill development schemes, SC persons  Basis – Vienna Declaration and
belonging to BPL category are eligible. Program of Action of 1993.

 In case of infrastructure development, the  Proclaimed by – The United Nations General


villages having 50% or more SC population are Assembly in Paris on 10 December 1948
eligible. (UNGA resolution 217 A).

 Budget – It is Rs. 2050 crore for FY 2023-24.  Legality – It isn’t a treaty and isn’t legally
binding in itself, but it is viewed as the basis for
international human rights law.

CHENNAI |SALEM| MADURAI | COIMBATORE DELHI | BANGALORE | THIRUVANANTHAPURAM


www.shankariasacademy.com 262
www.iasparliament.com

 Achievements – It have inspired and paved  Technical Support – International Crops


the way for more than 70 human rights treaties. Research Institute for the Semi-Arid Tropics
(ICRISAT), 1 CGIAR Centres, and other
 It inspired the decolonization movement, anti- international organizations.
apartheid movement, gender issues and the
LGBTIQ+ issues.  Goals

 India is a signatory to this declaration. o Enhance food security and nutrition

52. a o Strengthen agricultural resilience

Shidaowan Nuclear Plant o Digital transformation

 China starts up world's 1st 4th generation o Public-private partnerships


nuclear reactor in Shidaowan that uses a
high-temperature gas-cooled nuclear reactor 54. b
with 1,150MW.
Indian Ocean Naval Symposium (IONS) – 2023
 4th generation reactors can operate at higher
The 8th edition of Indian Ocean Naval Symposium
temperatures than most other reactors, which
(IONS) Conclave of Chiefs (CoC) was conducted by
allows them to generate both electricity and
Royal Thai Navy at Bangkok, Thailand.
hydrogen.

Gelephu Smartcity Project  It is a voluntary initiative that was inaugurated


in 2008 at New Delhi, with the Indian Navy as
 It is a mega project in Bhutan to have 1,000- the Chair for 2 years.
sq. km. green city along Assam border
connecting South Asia to Southeast Asia.  Objectives – To increase maritime co-
operation among navies of the littoral states
 There are also discussions about a 58-km rail (states with coast) of the Indian Ocean Region.
link between Gelephu and Kokrajhar in Assam.
 To provides an open and inclusive forum for
Ennore Creek discussion of regionally relevant maritime
issues.
 It is a backwater located in Thiruvallur
District, Tamil Nadu.  IONS Flag – It was designed by India.

 As an arm of the Kosathalaiyar River, the Creek  Membership – 34 (25 Members & 9
meets the Bay of Bengal at Mugathwara Observers).
Kuppam, while the northern channel of the
creek connects to the Pulicat Lake.  The members were grouped based on 4
sub-regions such as:
 This creek has been demarcated as Coastal
Regulation Zone IV (Water Body) in the coastal o South Asian Littorals
zone management plan by the Tamil Nadu State
o South East Asian & Australian
Coastal Zone Management Authority.
Littorals
 Undertaking any reclamation, bunding, o East African Littorals
construction or altering the water bodies is
illegal under Water (Prevention and Control of o West Asian Littorals
Pollution) Act 1974 & Environment Protection
Act, 1986.

53. b

Millets and other Ancient Grains International


Research Initiative (MAHARISHI)

 Proposed by – India

 Aim – To advance research in the field of


millets and other ancient grains.

 Secretariat at – Indian Institute of Millets


Research (IIMR) in Hyderabad.

CHENNAI |SALEM| MADURAI | COIMBATORE DELHI | BANGALORE | THIRUVANANTHAPURAM


www.shankariasacademy.com 263
www.iasparliament.com

 India – It also took over as the co-Chair of Composite Water Management Index (CWMI)
IONS Working Groups on Maritime Security.
NITI weighs discontinuing key water report launched
 India is scheduled to take over as the Chair of 5 years ago.
IONS (2025-27) during 9th CoC planned to be
conducted in India in end 2025.  Prepared by – NITI Aayog in association
with 3 ministries of central government (Water
55. c Resources, Drinking Water & Sanitation, and
Rural Development).
VISION 2020: The Right to Sight
 Objective – It ranks states in terms of efficacy
 It is a global program. of water management on the basis of 28
parameters.
 Established by – World Health
Organization (WHO) and  Reports – Only 2 editions were
the International Agency for the published so far (Dec, 2023)
Prevention of Blindness (IAPB) in 1999.
o 1st edition provided data for 2015-16
 Aim – To eliminate avoidable blindness and 2016-17 in 2018
by 2020.
o 2nd edition was launched in 2019 for
 Supported by – Governments and health 2017-18.
ministries, non-governmental organizations
(NGOs) and Eye care professionals, program  Unpublished reports – The performance of
managers and industry. states for 2018-19 and 2019-20 (CWMI 3.0 and
4.0) are yet to be released but it points out that
 Causes of vision impairment and water scarcity is a national problem.
blindness – Refractive errors, Cataract,
Diabetic retinopathy, Glaucoma (a group of eye Logistics Ease Across Different State (LEADS)
diseases that can damage the optic nerve, which 2023
connects the eye to the brain) Age-related
macular degeneration. The 5th edition of the LEADS annual exercise provides
insights into improvement of logistics performance at
56. a State/UT level.

World Malaria Report 2023  Basis – Logistics Performance Index


(LPI) of World Bank.
According to World Malaria Report 2023, malaria
cases continue to dip in India while it is increasing  Unlike LPI that relies entirely on perception-
globally. based surveys, LEADS incorporates both
perception and objective data.
 Published by – World Health
Organisation (WHO)  Launched in – in 2018

 Aim – To provide a comprehensive and up-to-  Launched by – The Ministry of


date assessment of trends in malaria control Commerce and Industry
and elimination.
 Objectives – To assess the performance of
 2023 report – For the 1st time, it includes a logistics eco-system across States and Union
dedicated chapter on the intersection between Territories (UTs) and to promote competitive
climate change and malaria. federalism for development of logistics sector.

 Key findings – Globally, the malaria deaths  Data source – A pan-India primary survey
were higher than in 2019 but the malaria conducted across 36 States/UTs.
mortality rate decreases.
 3 pillars of logistics performance - Quality
 This year alone, 3 more countries were certified of Logistics Infrastructure, Quality of Logistics
by WHO as malaria-free (Azerbaijan, Belize, Services, and Operating & Regulatory
and Tajikistan). Environment.

 India – There were around 5,000 deaths in Migration and Development Brief
India last year, a decrease of 30% and 34 %
respectively compared with 2021. The latest report reveals a continuing growth in
remittance flows to low- and middle-income countries
(LMICs) in 2023, albeit at a slower pace compared to
previous years.

CHENNAI |SALEM| MADURAI | COIMBATORE DELHI | BANGALORE | THIRUVANANTHAPURAM


www.shankariasacademy.com 264
www.iasparliament.com

 The brief is a biannual report. o XSPECT – X-ray Spectroscopy and


Timing, which will provide
 Prepared by – The Migration and spectroscopic information within the
Remittances Unit, Development Economics energy range of 0.8-15 keV.
(DEC).
Zhuque-2 Rocket
 Released by – World Bank.
 Zhuque 2 is a launch vehicle powered by a
 Objectives – To provide the information about mixture of methane and liquid oxygen.
migration and remittance flows and related
policies over the past 6 months and to provide  With this, LandSpace becomes the first
medium-term projections of remittance flows company in the world to launch a
to developing countries. methane-liquid oxygen rocket ahead of
U.S. rivals including Musk's SpaceX and Jeff
 2023 report – Remittances grew by an Bezos' Blue Origin.
estimated 3.8% in 2023, reaching a total of
USD669 billion in LMICs.  Capacity – Currently, the rocket can carry a
payload of up to 1.5 metric tons into a 500-
 A 7.2% increase in South Asia with notable kilometre-orbit.
contribution by India.
 Advantages – The use of methane in
 India – It remained as the largest commercial launches could help slash costs and
recipient with an estimated USD125 billion in make it easier to reuse rockets.
2023.
 Zhuque-3 – It would use stainless steel
57. d propellant tanks and clusters of methane-liquid
oxygen propellant engines.
X-Ray Polarimeter Satellite (XPoSat)
Tropospheric Emissions: Monitoring of
Indian Space Research Organisation (ISRO) will soon Pollution (TEMPO) Satellite
launch X-Ray Polarimeter Satellite (XPoSat) to
investigate the polarisation of intense X-Ray sources.  TEMPO Satellite was launched by the NASA to
provide hourly updates on atmospheric
 X-Ray Polarimeter Satellite (XPoSat) is the 1st pollutants.
such satellite by ISRO that aims
to investigate the polarisation of intense  The Tropospheric Emissions: Monitoring of
X-Ray sources. Pollution (TEMPO) satellite is a spectrometer
that monitors air pollution over North America.
 It focuses predominantly on imaging, time-
domain studies and spectroscopy.  It provides hourly updates on atmospheric
pollutants.
 The Mission objectives include:
 It is an initiative of National
o Measurement of X-Ray polarization in Aeronautics and Space Administration
the energy band of 8-30 keV emanated (NASA).
from X-Ray sources,
 It measures the major elements, directly or by
o Long-term spectral and temporal proxy, of the diurnal tropospheric ozone
studies of cosmic X-Ray sources in the chemistry cycle.
energy band of 0.8-15 keV.
58. b
 The XPoSat will be launched by the Polar
Satellite Launch Vehicle (PSLV) and is Codex Alimentarius Commission
designated for observation from Low Earth
Orbit (LEO). Recently Codex Alimentarius Commission praised India
for its standards on Millets.
 The mission life is expected to be approximately
5 years.  The Codex Alimentarius Commission (CAC) is
an international food safety and quality
 The payloads of the XPoSat includes: standard-setting body established in 1963.

o POLIX – Polarimeter Instrument in  It is created by Food and Agriculture


X-rays, designed to measure Organization (FAO) and the World Health
polarimetry parameters. Organization (WHO).

CHENNAI |SALEM| MADURAI | COIMBATORE DELHI | BANGALORE | THIRUVANANTHAPURAM


www.shankariasacademy.com 265
www.iasparliament.com

 The objective of it includes protecting 60. d


consumer’s health and ensuring fair practices
in food trade. Bonnet Macaque

 Presently the Commission has 189 Codex The recent discovery of carcasses of 27 bonnet
Members made up of 188 Member Countries macaque monkeys in Dakshina Kannada district,
and 1 Member Organization (The European Karnataka has highlighted the human-monkey conflict
Union). in the region.

 Each member of the Commission is responsible  The bonnet macaque (Macaca radiata), also
for identifying any new scientific and other known as zati, is a species of macaque
relevant information, which may warrant endemic to southern India.
revision of any existing Codex standards or
related text.  Bonnet macaques are diurnal, arboreal and
terrestrial quadrupeds, although they spend
 India became the member of Codex much of their time on the ground.
Alimentarius in 1964.
 They are also are also strong swimmers.
 The Codex Alimentarius includes standards for
all the principal foods, whether processed,  The International Union for Conservation of
semi-processed or raw for distribution to the Nature (IUCN) lists the bonnet macaque
consumer. as vulnerable.

 Codex standards and related texts are voluntary  Distribution – It is distributed throughout
in nature not a substitute or alternative to southern India in a territory, restricted to the
national legislation. Indian Ocean on three sides as well as the
Godavari and Tapti Rivers.
 Codex texts are voluntary and non-
binding. 61. c

Crime in India 2022


 Codex currently has standards for Sorghum
and Pearl Millet.
 As per the recent National Crime Records
59. b Bureau (NCRB) report on the crime situation in
India, there was an increase in various
Permakore types of crimes in 2022 compared to 2021.

In recent times, the Koya tribe have moved on from  Kolkata for the 3rd consecutive year has been
using traditional Indian Bison horns to palm leaves as recognised as the safest city in India, it recorded
a gesture of conservation of the Indian Bison to craft the least number of cognisable offences per lakh
their traditional flute, Permakore. population among other major cities in India.

 Permakore is traditional flute that is  Delhi registered highest rate of crime against
made from the Indian Bison horn. women in the country.

 It is used by the indigenous Koya  Highest charge sheeting rate under IPC crimes
tribe inhabiting the Papikonda hill range in are Kerala, Puducherry and West Bengal.
Andhra Pradesh.
 Offences against the State- It is reported in
 Another name for the Papikondalu hill range is Jammu and Kashmir, Chhattisgarh, Manipur,
the’ Bison hill range,’ which is derived from the Assam, and Nagaland.
fact that it is home to the Indian Bison.
 Crime against foreigners- Theft and rape,
 In the Koya language, ‘Permam’ stands for with most of the victims are from Asian
Indian Bison or Guar, and ‘Kore’ stands for continent followed by African countries.
‘horn’ and thus, the flute made of Bison horn is
called Permakore.  Male tend to be suicide victims than female.

 Permakore is played at a designated time and 62. b


place to give a call to the entire village to go
hunting in the forest. PM JANMAN Scheme

 Launch- On Janjatiya Gaurav Diwas (Birth


 The flute made of the leaf is sustainable for a
few years but the one that is made of Bison horn anniversary of Birsa Munda) in Jharkhand.
can be used for generations if preserved.

CHENNAI |SALEM| MADURAI | COIMBATORE DELHI | BANGALORE | THIRUVANANTHAPURAM


www.shankariasacademy.com 266
www.iasparliament.com

 Objective- To provide essential amenities  ZCZP can only be issued for a specific project
(road connectivity, electricity, secure housing, with specific tenure.
clean drinking water, sanitation) and improved
access to education, health, nutrition,  The project must fall under the list of eligible
sustainable livelihood opportunities. activities under SEBI (ICDR) Regulations,
2018.
 The government will showcase and spread
awareness of various government schemes  Issuance - They are issued through private
made for the tribals. placement or public issuance.

 The scheme will be implemented by 9 o Minimum issue size – Rs. 50 Lakh


ministries with Ministry of Tribal Affairs (originally Rs.1 crore)
playing the key role.
o Minimum application size – Rs.10,
 Beneficiary- Government has identified 75 000 (originally Rs.2 lakh)
tribal communities and primitive tribes with a
population of lakhs who reside in more than 22 o Minimum subscription required -
thousand villages in the country. 75% of the funds proposed

 Focus- 11 critical interventions for  Trade – They shall be issued in dematerialized


Particularly Vulnerable Tribal Groups (PVTG). form only.

 The scheme comprises both central sector and  They are not available for trading in the
centrally sponsored schemes. secondary market, but they can be
transferred to legal heirs.
 Ayush Wellness Centres- It will be set up by
the Ministry of AYUSH and AYUSH facilities  Maturity – It will mature when the project for
will be extended to PVTG habitations through which they are raised terminate, or 12 months
mobile medical units. from the date of allotment.

 Skill enhancement- Ministry of Skill Unnati ZCZP Bonds


Development and Entrepreneurship will
facilitate skill and vocational training in PVTG  Issuer – SGBS Unnati Foundation, an NPO
habitations, multipurpose centres and hostels formed in November 2011.
according to suitable skills of these
o Unnati – Vocational training program
communities.
for underprivileged and unemployed
Particularly Vulnerable Tribal Groups (PVTGs) youth in the age group of 18 to 25 years.

o UNXT – 1 month training programme


 In 1973, the Dhebar
for final year students at government
Commission created Primitive Tribal
colleges.
Groups (PTGs) as a separate category, who are
less developed among the tribal groups. o Sugam – A grassroot
entrepreneurship accelerator helping
 In 2006, the Government of India renamed the non-urban youth to develop scalable
PTGs as Particularly Vulnerable Tribal Groups businesses.
(PVTGs).
64. d
63. b
Decentralized Autonomous Organizations
Zero Coupon Zero Principle bonds
 It is a legal structure that operates without
 Zero Coupon Zero Principle are financial centralized control and is governed by smart
instrument that are included in the list of contracts and the consensus of its members.
securities under Securities Contracts
(Regulation) Act, 1956.  Decision making- It uses blockchain
technology and smart contract to take
 They do not give any interest, and investors will decisions in a bottom-up
not get any money back on the maturity of the management approach.
bond.
 Community driven-It is represented by rules
 Eligibility criteria for issuance – Only by encoded as a computer program that
the Non-Profit Organisations (NPO) that is transparent, controlled by the respective
are registered with Social Stock Exchange organization members, and not influenced by a
(SSE). government.

CHENNAI |SALEM| MADURAI | COIMBATORE DELHI | BANGALORE | THIRUVANANTHAPURAM


www.shankariasacademy.com 267
www.iasparliament.com

 Smart contracts- It executes predefined rules or used for, any unlawful or terrorist activity by
without the need for intermediaries, ensuring NIA.
trust through code rather than traditional
authorities.  Special courts- It is established by the Central
or State government to try any offence under
 Enhanced cooperation- They enable global, the Act.
borderless cooperation on an unprecedented
scale.  Appeal and revision of the judgments of
the special courts - by the High Court and
 Constructive engagement-Participants, the Supreme Court.
often referred to as token holders, can propose
and vote on decisions related to the  The act permits keeping a person in prison
organization's goals and resources. for upto 180 days (6 months), without even
filing a charge sheet.
 Self-executing systems- This decentralized
decision-making process ensures that no single
entity holds undue influence.

65. b

Unlawful Activities Prevention Act (UAPA),


1967

 Aim- To prevent and punish unlawful and


terrorist activities that threaten the sovereignty
and integrity of the country.

 Definition- The Act defines unlawful activities


as any action that

o supports, propagates, or advocates


secession from India, 66. c

o disclaims or questions the territorial Governor’s role as Chancellor


integrity of India,
Recently, a Supreme Court bench reiterated that the
o causes or intends to cause disaffection governor acting as Chancellor are not bound by the aid
against India, and advice of Council of Ministers.

o supports any terrorist organization or  In most cases, the Governor of the state is
individual the ex-officio chancellor of the
universities in that state.
 Declaration of unlawful associations -
The Act empowers the Central government to  Powers of Chancellor – He is the Head of
declare any organisation or individual as the University, and vested with power to
unlawful.
o Appoint the Vice-Chancellors.
 Tribunal- The declaration is subject to judicial
o Annul decisions of the various
review by a tribunal constituted by a sitting or
university bodies.
retired judge of a High Court.
o Conduct inspections in the university
 Punishment -
in some states (such as Bihar, Gujarat,
o Unlawful activity- Imprisonment and Jharkhand).
ranging from 5 years to life, and fine.
o Preside over the convocation of the
o Terrorist activity- Death penalty or university, and confirms proposals for
imprisonment for life, and fine conferring honorary degrees.

 National Investigation Agency (NIA)- It is  Supreme Court’s ruling – Governor as a


authorized to investigate any offence under the chancellor are not bound by the aid and
Act, and to arrest any person who is reasonably advice of the council of ministers and
suspected of having committed such offence. shall act only on his personal capacity.

 It also provides for the attachment and


forfeiture of any property that is derived from,

CHENNAI |SALEM| MADURAI | COIMBATORE DELHI | BANGALORE | THIRUVANANTHAPURAM


www.shankariasacademy.com 268
www.iasparliament.com

 The powers as a Chancellor have no relation to e-Jagriti Portal


the exercise of the powers and duties as the
State Governor.  An initiative of the Department of Consumer
Affairs, Ministry of Consumer Affairs, Food,
 Central Universities – Under the Central and Public Distribution.
Universities Act, 2009, and other statutes,
the President of India shall be the Visitor of a  Aim –To revolutionize the landscape of
central university who presides over dispute resolution and grievance
convocations. redressal.

 Chancellors in central universities –  It will help reduce the number of pending cases
They are titular heads, who are appointed by in consumer courts.
the President in his capacity as Visitor.
69. a
 The Vice Chancellor too are appointed by the
Visitor. Systematic Voters' Education and Electoral
Participation (SVEEP)
 The Act adds that the President, as Visitor, shall
have the right to authorise inspections of  It is a program by the Election Commission
academic and non-academic aspects of the of India to educate voters, spread awareness,
universities and also to institute inquiries. and promote voter literacy since 2009.

 Education comes under Concurrent List, but  It is working towards preparing India’s electors
entry 66 of the Union List gives the Centre and equipping them with basic knowledge
substantial authority over higher education. related to the electoral process.

67. c  Primary goal – To build a truly


participative democracy in India by
Youth for Unnati and Vikas with AI (YUVAi) encouraging all eligible citizens to vote
and make an informed decision during
 It is a national program that aims to teach AI the elections.
skills to school students in grades 8–12.
 To build a stronger democracy through greater
 Collaboration between – National e- synergy with Civil Society Organisations, Media
Governance Division (NeGD), Ministry of and Corporate Houses and even greater
Electronics and Information Technology queries, suggestions, and participation from
(MeitY), and Intel. individuals.

68. a ENCORE

Bhoomi Rashi Portal  The Election Commission of India has designed


in-house software for complete Candidate and
 An initiative of Ministry of Road Transport and election management through a software called
Highways. ENCORE.

 Aim – It is a single point platform for online  ENCORE stands for Enabling Communications
processing of land on Real-time Environment.
acquisition notifications to accelerate
highway infrastructure development projects in  This provides a seamless facility for Returning
India. Officer to process candidate nomination,
affidavit, Voter turnout, counting, results and
Gram Manchitra data management.

 It is a Geographic Information System cVIGIL (Citizen Vigilance)


application
 It is a mobile application designed by the
 Launched by – Ministry of Panchayati Raj Election Commission of India.

 Aim – To encourage the Spatial Planning  It provides opportunity to report election code
by the Gram Panchayat. violations directly by citizens.

 It uses the National Informatics Centre's (NIC)  It is widely available, easy to use, generates
Geographic Information System (GIS) platform legally tenable and prosecutable information to
of Bharat Maps. administrators.

CHENNAI |SALEM| MADURAI | COIMBATORE DELHI | BANGALORE | THIRUVANANTHAPURAM


www.shankariasacademy.com 269
www.iasparliament.com

70. a  It is primarily a human disease but in recent


years, it has been identified in the animals,
Akkadi Saalu particularly dogs.
The recently released report by the FAO suggests that  A few days to hours before the worm come out
use of traditional framing practices including Akkadi of the skin, the person may develop a fever,
Saalu could reduce the escalating human and swelling, and pain in the area.
environmental costs of producing food.
 Snakebite Envenoming – A potentially life-
 It is an intercropping system that has been in threatening disease following the bite of a
traditional practice in Karnataka. venomous snake.
 It helps to preserve life in the soil by reducing  WHO formally lists the snakebite envenoming
the use of pesticides. as a highest priority Neglected Tropical Disease
in 2017.
 It is a traditional, biodiversity based ecological
farming practice with zero chemical fertilisers & 72. b
pesticides and minimal use of ground water.
Synthesis (Syn) gas
 It involves intercropping with a combination of
legumes, pulses, oilseeds, trees, shrubs and  A combustible mixture
livestock that improves the health of the soil. of hydrogen and carbon monoxide, often
containing carbon dioxide and methane.
 Lantana – It is a heavily branched shrub that
can grow as compact clumps, dense thickets  It is derived from carbon-containing feedstocks
and as a climbing vine. like biomass, natural gas, heavy oil, and coal.

 It is an invasive species that is not native to  It is primarily produced by coal gasification,


India, and was introduced in the country as an wherein oxygen and steam react with coal.
ornamental plant by the British.
 Uses – In producing methanol or ammonia,
 Basadis and Nishidis – It refers to Jain and can also be used as a fuel.
shrine or temple.
 India have launched the scheme for promotion
 Moodbidri town in Karnataka is known as ‘Jain of Coal/Lignite Gasification Projects that is
Kashi’ which is home to Jain temples. targeting to gasify 100 million tonnes (MT) of
coal by 2030.
 Bhattarakas – They are the Heads of
Digambara Jain institutions. 73. c

71. a Kudankulam Plant

Noma  It is located in Tamil Nadu.

Recently, the World Health Organization (WHO) has  Developed by – Nuclear Power Corporation
added noma to its official list of neglected tropical of India (NPCIL)
diseases (NTD).
 It is built with the technical assistance of
 It is a severe gangrenous disease of the mouth Russia.
and face, which is also known as cancrum oris
or gangrenous stomatitis.  The construction began in 2002 and is expected
to start operating at full capacity in 2027.
 It is called as ‘Face of poverty’ as effective
drugs like sulfonamides and penicillin and  It is India's largest nuclear power plant.
adequate surgical treatment remain
inaccessible for many. Kakrapar Atomic Power Project (KAPP-4)

 Guinea Worm Disease – Also known  The 4th unit of the Kakrapar Atomic Power
as Dracunculiasis is an infection caused by Project (KAPP-4) started controlled fission
the parasite Dracunculus medinensis. chain reaction and thus became critical
recently.
 It is considered as a neglected tropical disease
(NTD) and it is the 1st parasitic disease targeted  It is a nuclear power plant situated in Gujarat.
for eradication.

CHENNAI |SALEM| MADURAI | COIMBATORE DELHI | BANGALORE | THIRUVANANTHAPURAM


www.shankariasacademy.com 270
www.iasparliament.com

 It is the 2nd of 16 indigenous Pressurised reduction in the number of people newly


Heavy Water Reactors (PHWR) of 700 diagnosed with TB in 2020 and 2021.
MW.
 TB remains the world’s 2nd leading
 Operated by – NPCIL. cause of death from a single infectious
agent, and global TB targets either have been
missed or remain off track.

 The World Health Organization (WHO) has a


Global Tuberculosis (TB) strategy called End
TB, which aims to end the TB epidemic globally
by 2030.

 Government of India has set the target


of eradicating TB by the year 2025.

75. d

Diel Vertical Migration (DVM)

 It is the daily, synchronized movement of


marine animals between the surface and
deep layers of the open ocean.

 It is also known as diurnal vertical


migration.

 This type of migration is the largest animal


migration on the planet and is undertaken
every single day by trillions of animals in every
ocean.

 This type of migration is followed by deep-


74. c sea marine animal especially tiny free-
floating zooplanktons.
WHO Global TB Report, 2022
 It is more than a survival strategy that plays
Recently the WHO Global TB Report, 2022 was a pivotal role in carbon sequestration.
released which indicates that globally, the estimated
number of deaths from TB increased between 2019 and  The timing of this migration is tuned to the
2021. natural rhythms of sunrise and sunset.
 Tuberculosis (TB) is a communicable disease
that is a major cause of ill health and one of the
leading causes of death worldwide.

 TB is caused by the bacillus Mycobacterium


tuberculosis, which is spread when people who
are sick with TB expel bacteria into the air (e.g.
by coughing).

 The World Health Organization (WHO) has


published a global TB report every year since
1997.

 It is annual report that is done in the context


of global TB commitments, strategies and
targets.
76. a
 Key findings – In 2022, 202 countries and
Paat-Mitro
territories with more than 99% of the world’s
population and TB cases reported data. In the recently held ‘Jute Symposium’, the ministry of
textiles launched ‘Paat-Mitro’ that provides
 India, Indonesia and the Philippines, which important information about Minimum
collectively accounted for nearly 60% of the

CHENNAI |SALEM| MADURAI | COIMBATORE DELHI | BANGALORE | THIRUVANANTHAPURAM


www.shankariasacademy.com 271
www.iasparliament.com

Support Price (MSP) and agronomy to jute  There has been a long-standing request to
farmers. add the temple to the UNESCO Heritage List.

 The mobile application is developed by UNESCO World Heritage Site


the Jute Corporation of India Limited
(JCI).  These are the sites are designated as having
outstanding universal value under the
 It will also feature information on jute Convention Concerning the Protection of the
gradation parameters, farmer-centric schemes World Cultural and Natural Heritage.
like ‘Jute-ICARE’, weather forecasts and
procurement policies.  It was adopted by UNESCO in 1972 and
formally took effect in 1975.
 The app helps farmers to track status of their
payments for the raw jute sold to JCI under  The list of WHS is maintained by the
MSP Operation. International World Heritage Programme,
administered by the UNESCO World Heritage
 Jute Corporation of India Limited Committee.
(JCI) – It was incorporated in 1971 as a price
support agency with a clear mandate for the  The three types of sites are Cultural, Natural,
procurement of raw jute. and Mixed.

 The procurement is done without any 78. b


quantitative limit from the growers at MSP,
which is based on the recommendations of Bab-el-Mandeb
Commission for Agricultural Cost & Prices
(CACP).  It is a strait that separates the Arabian
Peninsula from East Africa and is just 29-
 It is the sole implementing agency of the jute I- km wide at its narrowest point.
CARE project.
 Throughout 19th and until mid-20th century,
77. a Britain protected it through the island of Perim
(Mayyum) in the Strait but in 1990, it
Srimukhalingam Temple came under the control of Sana’a, the capital of
the united country of Yemen.
ASI assures to send note to UNESCO over inclusion of
Srimukhalingam temple in Andhra Pradesh in world  As Houthis captured Sana’a in 2015, they pose
heritage structures’ list. a greater threat due to their close proximity to
Bab el-Mandeb strait.
 Sri Mukha Lingam Shiva temple is very old
temple where Shiva lingam is appeared as Panama Canal
Mukha (Face) Avatar.
 It is a lock-type canal, owned and administered
 It was constructed during the 8th by the Republic of Panama.
century by Kamarnava-II when the region
was under the control of the East Ganga  Lock-type canal is a system in which an
Dynasty. enclosure or basin located in the course of a
canal or a river (or in the vicinity of a dock) with
 Trinity – Srimukhalingam temple houses 3 gates at each end, within which the water level
ancient temples at one location - may be varied to raise or lower boats.
Madhukeswara, Someswara and Bheemeswara
Temples.  The Panama Canal connects the Atlantic
and Pacific oceans through the narrow
 The temple is popularly known as Dakshina Isthmus of Panama.
Kaasi (Varanasi of South India).
 From its opening in 1914 until 1979, the
 Architecture – Similar to Lingaraja Panama Canal was controlled solely by the
temple located in Bhubaneswar of United States, which built it.
Odisha.
 In 1979, however, control of the canal passed to
 Location – The temple is located on the Panama Canal Commission, a joint agency
the banks of River Vamsadhara, which of the United States and the Republic of
was the capital of Kalinga Ganga Kings for Panama, and complete control passed to
over 600 years. Panama in 1999.

CHENNAI |SALEM| MADURAI | COIMBATORE DELHI | BANGALORE | THIRUVANANTHAPURAM


www.shankariasacademy.com 272
www.iasparliament.com

Taiwan Strait 80. c

 The Taiwan Strait is also known as the Tax Inspectors without Borders (TIWB)
Formosa Strait that separates mainland Initiative
China and the island of Taiwan.
 It aims to transfer technical know-how and
 The strait is part of the South China Sea and skills to developing countries’ tax auditors and
connects to the East China Sea to the north. share general audit practices.

 It is a joint initiative of United Nations


Development Programme and
Organisation for Economic Cooperation
and Development.

 The host administrations of developing


countries are the lead partners in TIWB
programmes, clearly specifying their needs and
scope of work.

 The 7th TIWB programme is held at Saint


Lucia, an island country of the West Indies with
India being chosen as the Partner
Administration and will provide Tax Experts for
this programme.

79. d  The focus of the programme will be on effective


use of automatic exchange of information
Global Drought Snapshot Report under the Common Reporting Standard (CRS)
framework.
 Recently the Global Drought Snapshot Report
was launched at the COP 28 by the United  The partnership by the member countries is
Nations Convention to Combat voluntary in nature.
Desertification (UNCCD) in collaboration
with the International Drought 81. b
Resilience Alliance (IDRA).
Tsunami
 The Global Drought Snapshot Report highlights
Tsunami waves hit several parts of Japan’s coastal
the need for urgent action to address the
areas and urgent evacuation warnings were issued
increasing frequency and severity of droughts.
after a 7.6-magnitude earthquake shook the country’s
north-central region.
 Some of the human activities that can trigger
droughts include are over farming, Excessive
 Tsunami– A Japanese word meaning harbour
irrigation, Deforestation and Erosion.
wave. Tsu – harbour; nami - wave.
 Some climate patterns that can cause droughts
 It is a series of giant ocean waves caused by
include El Nino and La Nina.
earthquakes or volcanic eruptions under the
ocean.
 Key highlights of the report – The
drought emergency was declared in 23 Tsunami types
countries including India.

 The Europe had the highest number of


countries (8) in drought
emergencies that include Spain, Italy, United
Kingdom Greece, Portugal, Romania and
Serbia.

 Humans are responsible for all global heating


over the past 200 years leading to a current
temperature rise of 1.1°C above pre-industrial
levels.

 The Horn of Africa faced its worst drought in 40


years, with Ethiopia, Kenya and Somalia
particularly hard hit.

CHENNAI |SALEM| MADURAI | COIMBATORE DELHI | BANGALORE | THIRUVANANTHAPURAM


www.shankariasacademy.com 273
www.iasparliament.com

 Wind wave vs Tsunami wave – Armed forces can take action


The wavelength is a factor which against their officers
distinguishes tsunamis from wind for adulterous acts.
waves, a tsunami wavelength can be more
than 200 km long which is considerably longer Joseph Shine v Union of
than a wind wave wavelength. Joseph Shine v Union India, 2018 that
of India decriminalised adultery does
not prevent court martial
proceedings initiated
against members of the
armed forces for adulterous
conduct.

The Chief Election


Commissioner (CEC)
and Election
 Lifetime of a tsunami – It can be divided Commissioners (ECs) will
into 3 stages namely generation, propagation, be appointed by the
and run-up. President on the advice of a
Anoop Baranwal v.
committee comprising the
 The power of a tsunami – It is highly Union of India
Prime Minister, the Chief
dependent on 2 factors. Justice of India (CJI) and the
Leader of Opposition (LoP)
o The tide – At high tide the tsunami in the Lok Sabha or the
will be able to do much more damage leader of the single largest
than at low tide. party in opposition.
o Seafloor morphology – It alters the Opened the doors
height by changing the ratio between for disqualification
their wavelength and wave height. proceedings against
Maharashtra Chief
 In general, this ratio decreases as the wave Minister Eknath
travels into shallower water, causing the Subhash Desai versus
Shinde for defection from
tsunami to grow in size. Principal Secretary,
the Shiv Sena party, and held
Governor of
that the then-Governor
 Sea depth – As the sea depth decreases, Maharashtra and Ors
Bhagat Singh Koshyari
the wavelength of tsunami wave “erred” in calling for a trust
decreases and thus the height increases. vote which triggered the fall
of the Uddhav Thackeray-led
 Size – It is also influenced by the topography of government in mid-2022.
the coastline.
Upheld the Prevention of
 Numbers – There may be more than one wave Cruelty to Animals (Tamil
and the succeeding one may be larger than the Nadu Amendment) Act of
one before. 2017 and Prevention of
Cruelty to Animals (Conduct
82. a Animal Welfare Board v.
of Jallikattu) Rules of 2017,
Union Of India
Key Supreme Court Judgments in 2023 saying that the traditional
bull-taming
 The Supreme Court has delivered several
sport ‘Jallikattu’ has been
landmark judgments in 2023.
going on in Tamil Nadu for
Case Judgement the last century.

Upheld the decision taken by A child born of a void or


the Union Government voidable marriage can inherit
Vivek Narayan to demonetise currency the parent’s share in a joint
Sharma v. Union of notes of Rs. 500 and Hindu family property
India Rs.1000 denominations and Revanasiddappa v. governed by the Mitakshara
held that it satisfies the test Mallikarjun law. However, such a child
of proportionality. would not be entitled
to rights in or to the
property of any other
person in the family.

CHENNAI |SALEM| MADURAI | COIMBATORE DELHI | BANGALORE | THIRUVANANTHAPURAM


www.shankariasacademy.com 274
www.iasparliament.com

Declined a married woman’s 83. d


plea to medically terminate
her 26-week Tenth Schedule
pregnancy, and it opposed
X v. Union of India Maharashtra Speaker’s interpretation of the anti-
to order doctors to stop the
heartbeat of the foetus when defection law could be challenged in the Supreme
medical reports say she will Court.
give birth to a viable baby.
 Anti-defection law – It punishes individual
Non-heterosexual couples MPs/MLAs for leaving one party for another.
Supriyo v. Union of cannot claim an
India unqualified right to  It was added by 52nd Constitutional
marry. Amendment Act as the Tenth Schedule in 1985.

Issued directions to  It was a response to the toppling of multiple


expedite adoption state governments by party-hopping MLAs
processes and directed after the general elections of 1967.
Temple of Healing v.
States to conduct drives to
Union of India  Deciding authority – The Presiding
identify adoptable children
and establish adoption Officer of the House is empowered to
agencies. make rules to give effect to the provisions of
the 10th schedule.
Unanimously upheld the
power of the President  Role of judiciary – The decision can be
to abrogate Article challenged before the higher judiciary.
370 via an executive order in
Tenth Schedule
August 2019, leading to the
Re Article 370 of the
reorganisation of the State of Maharashtra Speaker’s interpretation of the anti-
Constitution of India
Jammu and Kashmir into 2 defection law could be challenged in the Supreme
Union Territories and Court.
denuding its special
privileges under the  Anti-defection law – It punishes individual
Constitution. MPs/MLAs for leaving one party for another.
Central agencies cannot deny  It was added by 52nd Constitutional
accused persons their right Amendment Act as the Tenth Schedule in 1985.
to default bail by filing
multiple supplementary  It was a response to the toppling of multiple
chargesheets and seeking state governments by party-hopping MLAs
renewed custody. after the general elections of 1967.
Later, the Supreme Court, in  Deciding authority – The Presiding
a rather unusual order, Officer of the House is empowered to
Ritu Chhabaria v.Union directed lower courts to make rules to give effect to the provisions of
of India decide pending default bail the 10th schedule.
applications without relying
on its own judgment.  Role of judiciary – The decision can be
challenged before the higher judiciary.
Article 141 of the
Constitution provides that
Type Defection
the law declared by the
Supreme Court shall be
 Voluntarily giving up the
binding on all courts within
membership of political
India.
party (or)
Member of
The Supreme Court can
political party  Voting or abstain from
exercise its plenary power to
voting in that House
do ‘complete justice’
contrary to the direction
Shilpa Sailesh v. Varun under Article 142(1) of the
issued by the political party
Sreenivasan Constitution to dissolve a
marriage on the ground Independent
that it had broken down  If an MP/MLA who has been
candidate
irretrievably. elected as an independent

CHENNAI |SALEM| MADURAI | COIMBATORE DELHI | BANGALORE | THIRUVANANTHAPURAM


www.shankariasacademy.com 275
www.iasparliament.com

candidate joins a political 84. a


party later
Annual Status of Education Report (ASER)
 If nominated legislators joins 2023
Nominated a political party after 6
member months of being appointed The Annual Status of Education Report 2023 has been
to the House published recently by Pratham, a civil society
organisation.
Exemptions
 It is a nationwide citizen-led household
 The law allows a group of two-third survey that provides a snapshot of the status
MP/MLAs to join (merger) another political of children’s schooling and learning in rural
party without inviting the penalty for defection. India.

 If a person is elected as the Speaker of Lok  Conducted by – Pratham, a non-


Sabha or the Chairman of Rajya Sabha, governmental organisation.
then he could resign from his party and re-join the
party once he demits the post.  Launched – 2005

 91st Amendment Act, 2005 deleted the  Published – Annually until 2014, but became
exemption from disqualification in case of split by alternate year cycle since 2016.
1/3rd members of legislature.
o It collects information about enrolment
in pre-school and school for children in
 Concerns – It does not provide a time-
the age group of 3 to 16.
frame within which the presiding officer has to
decide a defection case. o It assesses children aged 5 to 16 one-
on-ones to understand their
 It only punishes legislators and not political foundational reading and arithmetic
parties who are at the heart of the politics have abilities.
no liability under the law.
 ASER, 2017 – For the first time it focused on the
 The exception provision for merger youth aged 14 to 18 in 28 districts of the
is based on the number of members country.
rather than the reason behind the
defection.  ASER 2023 – It revisits 14-18 age group on
collecting data and information about new
 The law focuses on voluntary defection and domain that have emerged as important in the
remain silent about expulsion of member from post pandemic world.
the party.
 It was conducted in 28 districts across 26 states
o If expelled then such member would be
in India.
an independent in the House with the
option of joining another party.  2023 findings – Most of the people in this age
group were enrolled in the Arts/Humanities
 Concerns – It does not provide a time-
streams.
frame within which the presiding officer has to
decide a defection case.  Females are less likely to be enrolled in
the Science, Technology, Engineering, and
 It only punishes legislators and not political
Mathematics (STEM) stream (28.1%) than
parties who are at the heart of the politics have
males (36.3%).
no liability under the law.
 Only 5.6% of surveyed youth report taking
 The exception provision for merger
vocational training or other related courses
is based on the number of members
currently.
rather than the reason behind the
defection.  About 25% still cannot read a Class II level text
fluently in their regional language.
 The law focuses on voluntary defection and
remain silent about expulsion of member from  More than half struggle with division (3 digit by
the party. 1 digit) problems.
o If expelled then such member would be
 A little over half can read sentences in English
an independent in the House with the
(57.3%).
option of joining another party.

CHENNAI |SALEM| MADURAI | COIMBATORE DELHI | BANGALORE | THIRUVANANTHAPURAM


www.shankariasacademy.com 276
www.iasparliament.com

 Females can do better than males in  5 Major Categories (Highest to Lowest in


reading a Class II level text in their tag) – Handicrafts-Agriculture-Foodstuff-
regional language. Manufactured-Natural.

 Males are more than twice as likely to have their


own smartphone as females.

 Across all digital tasks, such as finding and


sharing videos, searching the internet, setting
an alarm, and using Google maps, boys
outperformed girls.

 Close to 90% of all youth have a smartphone in


the household and kn0w how to use it.

 About two-thirds of the youth reported using


smartphones for educational purposes, such as
watching online videos related to studies,
solving doubts or exchanging notes.

 There is a huge skill deficit among adolescents,


many of them only a few years away from
entering the job market.

85. c

Status of GI tag in India

17 products from across 6 States/Union Territories got


the geographical Indications (GI) tag recently.
86. b
 GI tag – A sign used on products that have a
specific geographical origin and possess unique Chabahar Port
qualities due to that origin.
India and Iran are set to hold talks to establish "long-
 Ownership – Any trader's body, association, term cooperation framework" on Chabahar port.
or organization can apply for a GI tag.
 Chabahar Port – It is Iran’s first Deepwater
 Conditions – Uniqueness of the item with port and sits at the mouth of the Gulf of Oman.
historical records proof and a complete
breakdown of how the product is made.  It is Iran's only oceanic port situated in Sistan
and Baluchistan Province, on the Makran coast.
 Raw materials – They do not necessarily
have to come from that region, unless it is an  2 main ports – Shahid Kalantari and Shahid
agricultural tag. Beheshti.

o For instance, the mulberry silk used in


Kancheepuram sarees comes from
Karnataka, and the gold zari from
Surat.

 Status – Every State in India has at least


1 GI tag.

 Tamil Nadu has the highest number of


GI-tags.

 Variety of products – There are 34 classes of


products that can get GI tags ranging from
chemicals and paint to foodstuffs, handicrafts,
musical instruments, and even firearms and
locomotives.

CHENNAI |SALEM| MADURAI | COIMBATORE DELHI | BANGALORE | THIRUVANANTHAPURAM


www.shankariasacademy.com 277
www.iasparliament.com

 Chabahar port project – Signed in 2003  Cost basis – Domestic and international cost
between India and Iran. of fertilizers, the country's inventory levels, and
the currency exchange rate.
 Trilateral Transit and Trade
Agreement – Signed in 2016 between India,  Additional subsidies – For fertilizers that
Afghanistan and Iran. are enriched with secondary and
micronutrients like zinc and molybdenum
o Kabul-Zaranj-Zahedan- (Mo).
Chabahar port-Kandla
 New guidelines – The MRPs are supposed to
(or) be market-determined and set by the individual
companies selling them.
o Kandla-Chabahar port-Zahedan-
Zaranj-Kabul  The government merely pays a fixed per-tonne
subsidy on each of these fertilisers, linked to
 India was granted a 10-year lease and access to their nutrient content.
the Chabahar free trade zone.
 But the Department of Fertilisers has issued
 India got the opportunity to build the 628 detailed guidelines for the evaluation of
km rail line from Chabahar to Zahedan, just “reasonableness” of the MRPs for all non-urea
across the border from Afghanistan. fertilisers covered under NBS.
 India is restricted to the Shahid Beheshti port 88. c
that is undertaken by India Ports Global
Limited (IPGL) which is established for the Maasir-i-Alamgiri
development of ports overseas.
 It is a Persian-language book written by Saqi
87. c Musta'd Khan that details the reign
of Emperor Aurangzeb (1620-1707).
Nutrient Based Subsidy (NBS) Scheme
 The book was written after the emperor's death,
The Centre has introduced price capping formula for at the request of Inayetullah Khan Kashmiri,
phosphatic and potassic (P&K) fertilizers under the emperor's last secretary.
Nutrient Based Subsidy scheme through guidelines
issued under the Essential Commodity Act.  Sir Jadunath Sarkar translated the book into
English and labeled it a "Gazetteer of Mughal
 Launched in – 2010 State".
 Aim – To encourage soil-balanced  It cited that a temple that stood adjacent to the
fertilization and increase agricultural existing Kashi Vishwanath temple have been
productivity. razed on the orders of the Aurangzeb, and the
Gyanvapi mosque erected on its ruins.
 It provides subsidies for non-urea-based
fertilizers based on the nutrients (N, P, K, and 89. c
S) they contain.
Nine-degree Channel
 Administered by – Department of Fertilizer,
Ministry of Chemicals and Fertilizers. The Indian Air Force (IAF) has proposed to set up a
forward fighter airbase in Minicoy Islands, to
 Objectives – To improve availability of safeguard the sea lanes of communications and to keep
fertilizers to farmers and promote balanced use the Arabian Sea free from non-state actors in the
of fertilizers. region.

 To reduce subsidy burden on the Government  Location – In the Indian Ocean between the
and encourage competition among fertilizer Laccadive Islands of Kalpeni and Suheli Par,
companies. and Maliku Atoll of Minicoy.

 Coverage – Phosphatic and Potassic (P&K)  Features – Approximately 200 km wide with
Fertilizers. a depth of 2597 metres.

 Subsidy – It is a fixed amount, decided on  Lakshadweep – An archipelago consisting of


annual basis. 36 islands.

o Capital - Kavaratti

CHENNAI |SALEM| MADURAI | COIMBATORE DELHI | BANGALORE | THIRUVANANTHAPURAM


www.shankariasacademy.com 278
www.iasparliament.com

o India’s smallest and uni-district Union  Timeline –June 5, 2023 to June 5, 2026.
Territory.
 Amrit Dharohar Capacity Building
o Almost all the inhabited islands are Scheme – It aims to enhance livelihood
coral atolls. opportunities for local communities.
o The territory falls under the  Implementation – Indian Institute of
jurisdiction of the Kerala High Court. Tourism and Travel Management (IITTM), an
autonomous body under Ministry of Tourism,
Channels in India in collaboration with MoEFCC.
 8 Degree Channel – Between Minicoy  Training programmes – Alternative
Island and Maldives Livelihood program (ALP) and Paryatan Navik
Certificate (PNC).
 9 Degree Channel – Between Laccadive
Islands and Minicoy Island  Implementation – In 1st phase, 5 Ramsar
sites were identified.
 10 Degree Channel – Between Little
Andaman and Nicobar o Sultanpur National Park - Haryana

o Sirpur wetland – Madhya Pradesh

o Yashwant Sagar – Madhya Pradesh

o Bhitarkanika National Park –


Odisha

o Chilika Lake – Odisha

Viksit Bharat Sankalp Yatra (VBSY)

 It is Centre’s flagship initiative to raise


90. b awareness on its schemes.

Amrit Dharohar Scheme  Launched in – December 2023 and will run


till January 2024.
Recently, Ministry of Tourism started training in
Bhitarkanika National Park, Odisha under Amrit  Aim – To ensure that all intended beneficiaries
Dharohar Capacity Building Scheme. are covered under various flagship schemes of
the Central government and thereby achieving
 Launched in – 2023 beneficiary saturation under government
schemes.
 Aim – To promote unique conservation
values of Ramsar Sites’. SARATHI app

 Ramsar Convention 1971, an  Launched by – Ministry of Rural


intergovernmental treaty for conservation and Development in partnership with the Nudge
wise use of wetlands, was ratified by India in Institute.
1982.
 Aim – To address the needs of the most
vulnerable, ensuring efficient implementation
of developmental programs for rural
development.

 The app will be used in 6 states to support


special projects for vulnerable households.

PM Vishwakarma

 Jammu and Kashmir has become the 1st Union


Territory (UT) to implement PM Vishwakarma
Yojana (PMVY).

 It is an initiative that aims to empower and


enhance the skills of the craftsman
community (Vishwakarmas).

CHENNAI |SALEM| MADURAI | COIMBATORE DELHI | BANGALORE | THIRUVANANTHAPURAM


www.shankariasacademy.com 279
www.iasparliament.com

 18 traditional trades will be covered in the first  Geographical indications assign products to the
instance under PM Vishwakarma. place of its origin, conveying an assurance of
quality and distinctiveness for such goods.
91. d
 GI is defined under Article-23 and 24 of the
SVAMITVA Trade-Related Aspects of Intellectual Property
Rights (TRIPS) Agreement of the World Trade
SVAMITVA Scheme wins Best Innovation Award for Organization (WTO).
Innovation Sandbox presentation during Public Policy
Dialogues–2024 at Indian School of Business,  Under the Paris Convention for the Protection
Hyderabad. of Industrial Property, GIs are covered as an
element of Intellectual Property Rights
 SVAMITVA – Survey of Villages and Mapping
with Improvised Technology in Village Areas.  India is a member of the WTO and thus enacted
the Geographical Indications of Goods
 Initiative by – Ministry of Panchayati (Registration & Protection) Act, 1999 that came
Raj, State Panchayati Raj Departments, State into force from 2003.
Revenue Departments and Survey of India.
 GI tagged products can be both Natural and
 Launched in – 2021, as a Central Sector Man-made and is valid for ten years.
Scheme.
 The first product in India to be accorded with
 Aim – For mapping the land parcels in GI tag was Darjeeling Tea in the year 2004-05.
rural inhabited areas using drone technology
and Continuously Operating Reference Station GI Tagged products
(CORS).
 Adi Apong is a traditional rice beer brewed by
 To provide an integrated property validation the Adi community from Arunachal
solution for rural India. Pradesh that has been recognized for its
unique flavour and brewing techniques
 SVAMITVA Property Cards –
They facilitate monetization of  Kachchi Kharek is a variety of dates grown in
properties, enabling bank loans and the Kutch region of Gujarat, they have a soft
comprehensive village-level planning. and chewy texture known for their natural
sweetness and rich, caramel like flavour.
 Target year – 2024–25.
 Koraput Kalajeera rice is known as the
 Coverage Area – The scheme has been ‘Prince of Rice’, it has an aromatic variety
saturated with generation of property cards of originating from Koraput district
all inhabited villages of Haryana, Uttarakhand, of Odisha. The rice looks like coriander seeds
Puducherry, Andaman and Nicobar Islands and it is known for its nutritional and cooking
Goa. quality.
PRERANA program  Singhpho Phalap is a distinctive tea
produced by the Singpho tribe in Arunachal
 It is a week-long residential program for 20 Pradesh.
selected students (10 boys and 10 girls) of class
IX to XII.  Arunachal Pradesh- Recently 12 products
namely – Apatani, Monpa, Adi, Galo, Tai
 Aim – To empower the selected students with Khamti and Nyishi textiles, Monpa handmade
leadership qualities. paper, Singpho Phalap (Singpho tea), Adi
Apong, Dao (machete), Angnyat millet and
 Ministry – Ministry of Education. Marua Apo (Marua millet beverage) have been
given GI tag.
 Prerana program will run from a Vernacular
School, established in 1888, in one of the oldest  Adis, Apatanis, Singphos, Monpas, Nyishis,
living cities of India, Vadnagar, in Gujarat. Khamtis are some of the tribes of Arunachal
Pradesh
 The curriculum of Prerana School has been
prepared by IIT Gandhi Nagar and is rooted in 93. b
9 value-based themes.
Saint Srimanta Sankaradeva
92. d
 Sankardeva founded the first-ever Kirtan
Geographical Indication (GI) Tag Products Ghar at Bordowa in 1494 AD which intended

CHENNAI |SALEM| MADURAI | COIMBATORE DELHI | BANGALORE | THIRUVANANTHAPURAM


www.shankariasacademy.com 280
www.iasparliament.com

to practice and preach the neo Vaishnavite  Benefits of Recognition – 2 major annual
faith, and propagated the Ek Saran Naam international awards for scholars of eminence
Dharma. in classical Indian languages

 It focussed on worship in the form of bhakti  A Centre of Excellence for studies in Classical
(devotion) to Lord Krishna, through singing Languages is set up.
and congregational Ek Saran Naam
Dharma listening of his name and deeds.  The UGC is requested to create, to start with at
least in the Central Universities, a certain
 Sankardeva espoused a society based on number of Professional Chairs for the Classical
equality and fraternity, free from caste Languages so declared.
differences, orthodox Brahmanical rituals and
sacrifices. 95. b

 His teaching focused on prayer and chanting Nagara architecture


(naam) instead of idol worship.
 Depending on the period and geography, there
 His dharma was based on 4 components - deva are 5 modes of Nagara architecture - Valabhi,
(god), naam (prayers), bhakats (devotees), and Phamsana, Latina, Shekhari, and
guru (teacher). Bhumija.

 The Neo-Vaishnavite reformist


movement that Sankardeva started is behind
the monastic institutions called Thans/Sattras
that dot Assam.

 Batadrava Than- It is a temple complex,


located in Nagaon District, Assam, the
birthplace of Vaishnavite reformer-saint
Srimanta Sankardeva

94. c
 The first two are associated with what scholars
Classical language criteria have classified as Early Nagara Style.
 Guidelines for declaring a language as  The Valabhi begins as a masonry rendering of
‘Classical’ - the barrel-roofed [wooden] structure, simple or
with aisles, familiar through chaitya halls
o High antiquity of its early [prayer halls, most associated with Buddhist
texts/recorded history over a period shrines].
of 1500-2000 years.
 A formalisation of multi-eave towers, wedded
o A body of ancient literature/texts,
to a piling up of slabs, leads to the Phamsana.
which is considered a valuable heritage
by generations of speakers  From these modes emerged the Latina - a
single, slightly curved tower with 4 sides of
o The literary tradition be original and
equal length.
not borrowed from another speech
community. Comparison to Dravida style
 The classical language and literature being  The Dravida counterpart to the shikhara is
distinct from modern, there may also be a the vimana.
discontinuity
between the classical  In Dravida style, vimanas are typically smaller
language and its later than the great gatehouses or gopurams.
forms or its offshoots.

 Currently, 6
languages enjoy the
‘Classical language
status.

CHENNAI |SALEM| MADURAI | COIMBATORE DELHI | BANGALORE | THIRUVANANTHAPURAM


www.shankariasacademy.com 281
www.iasparliament.com

 While shikharas are mentioned in south Indian Camptothecin (CPT)


sources, they refer to only the dome-shaped
crowning cap atop the vimana.  Only 2 CPT analogs (irinotecan and topotecan)
have been approved for cancer treatment.
 The existence of gopurams also points to
another unique feature of the Dravida style, the  The allopathic medicine is produced using
presence of a boundary wall. Few Nagara style Nathapodytes nimmoniana.
temple complexes are lined with distinctive
boundary walls that are a part of the temple’s  Camptothecin is majorly produced in Southeast
design. Asian region, with the plant being largely found
only in China and India.
 This is one of Ayodhya’s Ram temple’s hybrid
features, although no elaborate gopuram has 97. b
been built, a 732m long wall runs around the
temple compound. VijayRaghavan Panel Report

96. c  Aim - To review the functioning of the Defence


Research and Development Organisation
Nathapodytes nimmonana (DRDO).

 It is an endangered medicinal plant widely  Earlier in 2008, Dr P Rama Rao Committee was
distributed throughout the Western Ghats of constituted to review the functioning of the
India. DRDO and all the recommendations by the
Committee were accepted by the DRDO except
 The plant contains camptothecin (CPT) which the creation of a Board of Research for
is renowned anticancer drugs. Advanced Defence Sciences (BRADS).

 Though, CPT found in many plant species but Recommendations of the committee
maximum amount of CPT has been reported
from N. nimmoniana.  Focus on research - The DRDO should focus
on its original goal of research and development
 The plant has been overharvested, pushing it to for defence.
the endangered list.
 It should refrain from involving itself in
 The Chinese variety of the plant used to extract productization, production cycles, and product
the cell is listed as critically endangered. management, tasks that are more suitable for
the private sector.
 Researchers at the Indian Institutes of
Technology Madras and Mandi have  Defence Technology Council - The
metabolically engineered plant cells to increase committee has also suggested a Defence
production of anti-cancer drug camptothecin Technology Council (DTC) chaired by the Prime
(CPT). Minister, with the defence minister and the
National Security Advisor as its Vice Presidents.

 It will also have an executive committee chaired


by the Chief of Defence Staff (CoDS).

 It will include representation from academia


and industry, with two members from each
sector.

 The Council will determine the country’s


defence technology roadmap and decide on
major projects and their execution.

 Separate Department - The panel has also


suggested the creation of the Department of
Defence Science, Technology, and Innovation
under the Defence Ministry.

 This department will be headed by a technocrat


and it will serve as the secretariat for the
Defence Tech Council (DTC).

CHENNAI |SALEM| MADURAI | COIMBATORE DELHI | BANGALORE | THIRUVANANTHAPURAM


www.shankariasacademy.com 282
www.iasparliament.com

 Centralized Laboratory - The committee  Sub-indices of National MPI:


has recommended the setting up of close to 10
national-level laboratory facilities instead of the o Headcount ratio (H): How many are
existing 40 or more DRDO labs across the poor?
nation.
o Intensity of poor (I): How poor are
Other reports the poor?

 Abhijeet Sen Committee- It was constituted o Formula - MPI= H*I


to formulate food policy in the long term.
 If the deprivation score (sum of the weighted
 Aruna Sundararajan Committee- It was status of all the indicators) for an individual is
formed to revive telecom sector. more than 0.33, then an individual is
considered multidimensionally poor.
 K V Kamath Panel- To examine the MSME
sector.  Findings – As many as 24.82 crore people
moved out of multidimensional poverty in 9
98. c years to 2022-23, with Uttar Pradesh, Bihar
and Madhya Pradesh registering the largest
National Multidimensional Poverty Index decline.

Over 24.8 crore people moved out of poverty in India  All 12 indicators of MPS have shown
in 9 years. remarkable improvement during this period.

 According to the World Bank, those who are 99. a


unable to earn 2.15 dollar per day are living in
extreme poverty. Bharat Ratna Awards

 Released by – NITI Aayog, in collaboration Recently, the President of India has conferred Bharat
with United Nations Development Programme Ratna posthumously to former Bihar CM Karpoori
(UNDP), and Oxford Poverty and Human Thakur.
Development Initiative (OPHI).
 It is the highest civilian Award in India,
 Purpose – Plays a role in assessing instituted in 1954.
advancements towards Sustainable
Development Goals (SDGs) 1.2.  Eligibility – Any person without distinction of
race, occupation, position or sex is eligible for
 It is based on 3 dimensions and 12 these awards.
indicators.
 For recognition of exceptional
service/performance of the highest order in any
field of human endeavour.

 Recommendation – By Prime
Minister himself to the President.

 Numbers – Restricted to a maximum of 3 in a


particular year, but not mandatory to
award every year.

 Award – A Sanad (certificate) signed by the


President and a medallion but no monetary
grant is given.

 Title – Cannot be used as a prefix or suffix to


the recipient's name as per Article 18 (1) of
the Constitution.

 However, one may use the expression


“Awarded Bharat Ratna by the President” or
“Recipient of Bharat Ratna Award”.

 Posthumous awards – Though original


statutes did not provide for this, 1955
amendment permitted them.

CHENNAI |SALEM| MADURAI | COIMBATORE DELHI | BANGALORE | THIRUVANANTHAPURAM


www.shankariasacademy.com 283
www.iasparliament.com

 It has been granted to 49 individuals so far


(2024), with 17 posthumously.

 It was previously awarded in 2019 to Shri


Nanaji Deshmukh (posthumously), Dr
Bhupendra Kumar Hazarika (posthumously)
and Shri Pranab Mukherjee.

100. a

Amini and Kalpeni

The Kochi-Lakshadweep islands submarine optical


fiber connection was recently inaugurated by Prime
Minister, Narendra Modi that connects eleven
Lakshadweep Islands including Agatti, Amini and
Kalpeni.

 The tiniest Union Territory of India,


Lakshadweep is an archipelago consisting of 12
atolls, 3 reefs and 5 submerged banks.

 The inhabited islands of Lakshadweep


are Kavaratti, Agatti, Amini, Kadmat, Kiltan,
Chetlat, Bitra, Andrott, Kalpeni and Minicoy.

 More than 93% of the population who are


indigenous, are Muslims and majority of them
belong to the Shafi School of the Sunni Sect.

 Kochi-Lakshadweep islands submarine


optical fibre connection (KLI-SOFC)
Project – It covers a wide range of sectors
including technology, energy, water resources,
healthcare and education.

 The project submarine cable connectivity from


Mainland (Kochi) to 11 Lakshadweep Islands,
that includes:

o Kavaratti, Agatti, Amini, Kadmat,


Chetlet, Kalpeni, Minicoy, Androth,
Kiltan, Bangaram and Bitra.

 It will lead to an increase in internet speed


unlocking new possibilities & opportunities.

 The project is funded by Universal Services


Obligation Fund (USOF), Department of
Telecommunication.

 Bharat Sanchar Nigam Limited (BSNL) was the


Project Executing Agency and the work was
awarded to M/s NEC Corporation India Pvt Ltd
through Global Open Tendering process.

CHENNAI |SALEM| MADURAI | COIMBATORE DELHI | BANGALORE | THIRUVANANTHAPURAM


www.shankariasacademy.com 284
www.iasparliament.com

TEST – VI
1. Consider the following. a. All except 3 & 5

1. Low temperatures b. All except 4

2. High wind speed c. All except 2

3. Moisture availability d. All five

4. Plenty of aerosols

How many of the above factors contribute to the 4. Consider the following statements with respect to
formation of dense fog in North India in winter? Foreign Contribution Regulation Act (FCRA), 2010

a. Only one 1. Ministry of External Affairs and Ministry of


Finance oversee the implementation of this act.
b. Only two
2. Foreign funds can be availed only through the
c. Only three State Bank of India’s New Delhi branch.

d. All four 3. It bars the sub-granting by NGOs to smaller


NGOs which works at the grass roots level.

4. Administrative expenses was capped at 50% of


2. Consider the following statements with respect to the total foreign funds received, earlier the
minority educational institutions. upper limit was 70%

1. Article 30 of the Indian Constitution confers on How many of the above statements is/are correct?
the minority, the discretion to choose the
administrators of the institution. a. Only one

2. The minority character of an educational b. Only two


institution will be revoked, if the administrators
from other communities run the institution. c. Only three

Which of the above statement(s) is/are correct? d. All four

a. 1 only

b. 2 only 5. Consider the following statements with respect to


Square Kilometre Array (SKA) Observatory
c. Both 1 and 2
1. It is a single largest telescope in the world.
d. Neither 1 nor 2
2. It is an infrared wave telescope used to detect
infrared waves emitted by the celestial objects.

3. Consider the following statements. 3. India is a part of this observatory.

1. Every individual has the right to move the How many of the statement(s) given above is/are
Supreme Court for the enforcement of their correct?
fundamental rights.
a. Only one
2. Special leave to appeal by the Supreme Court.
b. Only two
3. Law declared by Supreme Court to be binding
on all courts. c. Only three

4. Enforcement of decrees and orders of Supreme d. None


Court.

5. Power of superintendence over all courts by the


6. Consider the following statements
High Court.
Statement-I: United Liberation Front of Assam
In the above statements, which are the components of
(ULFA) aims to establish a sovereign Assamese nation
Basis Structure doctrine in India?
by an armed struggle against the Indian state.

CHENNAI |SALEM| MADURAI | COIMBATORE DELHI | BANGALORE | THIRUVANANTHAPURAM


www.shankariasacademy.com 285
www.iasparliament.com

Statement-II: United Liberation Front of Assam 9. Who among the following nominates both the
(ULFA) was born due to the failure of Assam Accord of Chairman and other members of Supreme Court Legal
1985. Services Committee (SCLSC)?

Select the correct answer using the codes given below: a. The President

a. Both Statement-I and Statement-II are correct b. The Parliament


and Statement-II is the correct explanation for
Statement-I c. Chief Justice of India

b. Both Statement-I and Statement-II are correct d. Supreme Court Collegium


and Statement-II is not the correct explanation for
Statement-I

c. Statement-I is correct but Statement-II is 10. Consider the following with respect to PRITHvi
incorrect Vigyan:

d. Statement-I is incorrect but Statement-II is 1. It aims to observe the atmosphere, ocean,


correct geosphere, cryosphere and solid earth.

2. It is implemented by Ministry of Science and


Technology.
7. Consider the following pairs.
3. PACER Mission is a sub-scheme under this
Places in News Location program.

1. Pong dam – Uttarakhand How many of the above statement(s) is/are correct?

2. Kalpasar Project – Rajasthan a. Only one

3. Dahej – Gujarat b. Only two

4. Ratle Project – Himachal Pradesh c. Only three

How many of the pair(s) given above is/are correctly d. None


matched?

a. Only one
11. Consider the following pairs.
b. Only two
Monument/Temple Significance
c. Only three
1. Kalaram Temple - Mahad Satyagraha
d. All four
2. Gol Gumbaz - Whispering gallery

3. Somnath Temple - Solanki style


8. Consider the following statements with respect to
copper reserves. How many of the pair(s) given above is/are correctly
matched?
1. Rajasthan has the largest copper reserves in
India. a. All three

2. Chile has the world's largest copper reserves. b. Only one

3. Australia has the world’s largest production of c. Only two


copper.
d. None
How many of the statement(s) given above
is/are incorrect?

a. Only one 12. The term “Rattapparai” seen in news is related to


which of the following?
b. Only two
a. It is a volcanic rock that originate from molten
c. Only three magma deep within the Earth's mantle.

d. None

CHENNAI |SALEM| MADURAI | COIMBATORE DELHI | BANGALORE | THIRUVANANTHAPURAM


www.shankariasacademy.com 286
www.iasparliament.com

b. It is a rock art painting predominately painted in c. Both 1 and 2


red ochre.
d. Neither 1 nor 2
c. It is a meteorite that formed the Ramgarh crater
due to its streak in India.

d. It is a red fossil rock found near Godaveri basin 16. Consider the following regarding All India Survey of
in Maharashtra. Higher Education (AISHE) 2021–22 Report:

1. There is higher share of enrolment in


government institutions.
13. Consider the following statements with respect to
Maru-Gurjara style 2. A marginal decrease in enrolment of females
across India.
1. It is also called as Indo-Saracenic architecture.
3. Tamil Nadu has the highest Gross Enrolment
2. It orginated as a regional style in Gujarat and Ratio (GER).
Rajasthan, later it become popular in Buddhism
temples. 4. Graduation rate in arts and social sciences
streams is higher than others.
Which of the above statement(s) is/are incorrect?
How many of the statement(s) given above is/are
a. 1 only correct?

b. 2 only a. Only one

c. Both 1 and 2 b. Only two

d. Neither 1 nor 2 c. Only three

d. All four

14. “Project Kusha” often seen in the news is related to


which of the following?
17. Consider the following statements with respect to
a. It is a connectivity project along Assam border Global Snow Leopard and Ecosystem Protection
connecting South Asia to Southeast Asia. Programme (GSLEP)

b. It is an initiative to halve the number of deaths 1. It is an alliance between 12 snow leopard range
and disability due to snakebite envenoming by countries, organizations and communities to
2030. protect the snow leopard and its mountain
ecosystems.
c. It was launched to develop New Generation
Launch Vehicles and thereby to build the country's 2. It aims to achieve the goals of the Bletchley
own space station and send Indians to the Moon. Declaration.

d. It is an indigenous project to develop Long 3. India is one among the 12 snow leopard range
Range Surface to Air Missiles countries and a member of GSLEP.

How many of the statement(s) given above is/are


correct?
15.Consider the following statements
regarding Cervavac a. Only one

1. It is India’s first indigenous and only gender- b. Only two


neutral quadrivalent HPV (Human Papilloma
Virus) vaccine. c. Only three

2. It is developed by Serum Institute of India d. None


which is yet to get approval for
commercialisation.

Which of the above statement(s) is/are correct? 18. Consider the following statements with respect to
Wetland City Accreditation
a. 1 only

b. 2 only

CHENNAI |SALEM| MADURAI | COIMBATORE DELHI | BANGALORE | THIRUVANANTHAPURAM


www.shankariasacademy.com 287
www.iasparliament.com

1. It is a voluntary system that recognizes cities 3. Karaivetti Bird Sanctuary –Kerala


which have taken exceptional steps to safeguard
their urban wetlands. 4. Longwood Shola Reserve Forest –Tamil Nadu

2. It is established under the Ramsar Convention. How many of the pair(s) given above is/are correctly
matched?
3. No Indian city have been formally accredited in
the Wetland City Accreditation. a. Only one

How many of the statements given above are correct? b. Only two

a. Only one c. Only three

b. Only two d. All four

c. All three

d. None 22. Consider the following statements with respect to


High Altitude Pseudo Satellite Vehicle (HAPS).

1. They are capable of achieving altitudes nearly


19. NUTEC Plastics, sometimes seen in the news, is an twice as high as those typically reached by
initiative of? commercial airplanes.

a. International Atomic Energy Agency (IAEA) 2. HAPS possess the capability to travel at
supersonic speeds.
b. United Nations Environment Programme
(UNEP) 3. Unlike conventional drones, it can sustain flight
for extended periods, often months, due to their
c. International Union for Conservation of Nature solar-powered systems.
(IUCN)
How many of the statements given above are correct?
d. World Wide Fund for Nature (WWF)
a. Only one

b. Only two
20. Which of the following countries are part the
Lithium Triangle? c. All three

1. Argentina d. None

2. Chile

3. Bolivia 23. Consider the following pairs:

4. Peru AI Models Developed by

5. Brazil 1. LaMDA – Google

Select the correct answer using the codes given below: 2. Llama – OpenAI

a. 1, 2 and 3 only 3. Sora – Meta AI

b. 2, 3 and 4 only 4. Hanooman – Anthropic

c. 1, 2 and 5 only How many of the pair(s) given above is/are correctly
matched?
d. 1, 2, 3, 4 and 5
a. Only one

b. Only two
21. Consider the following pairs:
c. Only three
Ramsar Wetlands States
d. All four
1. Magadi Kere Conservation Reserve–Karnataka

2. Ankasamudra Bird Reserve –Telangana

CHENNAI |SALEM| MADURAI | COIMBATORE DELHI | BANGALORE | THIRUVANANTHAPURAM


www.shankariasacademy.com 288
www.iasparliament.com

24. Greening India's Wastelands with Agroforestry 2. They have either DNA or RNA as their genetic
(GROW) Report, sometimes seen in the news, is material.
released by?
3. Unlike viruses, viroids lack a lipid layer or a
a. Ministry of Environment, Forest and Climate protein coat.
Change
How many of the statements given above are correct?
b. Ministry of Agriculture & Farmers' Welfare
a. Only one
c. Centre for Environment Education
b. Only two
d. NITI Aayog
c. All three

d. None
25. Consider the following statements with respect to
Nilgiri Marten.

1. It is endemic to the Western Ghats. 28. Consider the following statements with respect to
Virtual Private Network (VPN).
2. They are burrowing animals and spend most of
their time on ground. 1. VPNs operate without the involvement of any
physical cables.
3. It is classified under Schedule II of the Wildlife
Protection Act, 1972. 2. VPNs ensure that the user's data or browsing
activity remains inaccessible to unauthorized
How many of the statements given above parties.
are incorrect?
Which of the statement(s) given above
a. Only one is/are incorrect?

b. Only two a. 1 only

c. All three b. 2 only

d. None c. Both 1 and 2

d. Neither 1 nor 2

26. Consider the following statements with respect to


Multiple Independently targetable Re-Entry Vehicle
(MIRV). 29. Morodharo, sometimes seen in the news recently, is
a?
1. It possess the capability to overwhelm anti-
ballistic-missile defence systems by deploying a. Sangam Port
multiple warheads simultaneously.
b. Harappan Site
2. Russia was the first country to develop MIRV
technology. c. World Heritage Site

3. In India, Agni-P is equipped with MIRV feature. d. Buddhist Monastery

How many of the statements given above are correct?

a. Only one 30. Consider the following countries:

b. Only two 1. Denmark

c. All three 2. Norway

d. None 3. Finland

4. Sweden

27. Consider the following statements with respect to 5. Latvia


Viroids.
Which of the above countries are members of Nordic-
1. Viroids infect both plants and animals. Baltic Eight (NB8)?

CHENNAI |SALEM| MADURAI | COIMBATORE DELHI | BANGALORE | THIRUVANANTHAPURAM


www.shankariasacademy.com 289
www.iasparliament.com

a. All except 2 34. Consider the following passage:

b. All except 3 It is an early visual art form closely linked to the


Manipur Meitei community's cultural history. The
c. All except 5 paintings are based on stories and events from the
Hindu epic and Puranas.
d. All of the above
The above passage best describes which of the following
paintings?
31. Consider the following statements with respect to a. Thangka paintings
Snow Leopard Population Assessment in India (SPAI)
Program. b. Basohli painting
1. It is the first ever population estimation of c. Subika painting
Snow Leopard in India.
d. Pichwai painting
2. The Wildlife Institute of India (WII) serves as
the national coordinator for this program.

3. Jammu and Kashmir possesses the highest 35. Consider the following statements with respect to
estimated snow leopard population, followed the C-CARES.
by Uttarakhand and Himachal Pradesh.
1. It is a web portal for Citizen Assistance and
How many of the statements given above Relief in Emergency Situations.
are incorrect?
2. It is designed by the Centre for Development of
a. Only one Advanced Computing (C-DAC).

b. Only two How many of the above statement(s) is/are incorrect?

c. All three a. 1 only

d. None b. 2 only

c. Both 1 and 2

32. World Restoration Flagships, sometimes seen in the d. Neither 1 nor 2


news recently, is an initiative of?

a. World Economic Forum


36. Consider the following statements with respect to
b. United Nations Environment Programme the Payment Banks.

c. Organisation for Economic Co-operation and 1. The minimum paid-up capital for payments
Development bank is Rs 100 crore.

d. United Nations Educational, Scientific and 2. These banks are not allowed to lending loans
Cultural Organization and issue credit cards.

3. These banks were originated on the basis of the


Nachiket Mor Committee.
33. Which of the following statement
is incorrect regarding parthenogenesis? How many of the statement(s) given above is/are
correct?
a. It is a form of asexual reproduction in which a
female produces an embryo without fertilization. a. Only one

b. An egg produced parthenogenetically may be b. Only two


either haploid (n) or diploid (2n).
c. All three
c. Parthenogenesis may occur in both vertebrates
and invertebrates. d. None

d. Parthenogenesis has more genetic variation than


sexual reproduction.
37. Ilsenhohle Cave Site, sometimes seen in the news
recently, is located in?

CHENNAI |SALEM| MADURAI | COIMBATORE DELHI | BANGALORE | THIRUVANANTHAPURAM


www.shankariasacademy.com 290
www.iasparliament.com

a. England c. All except 3

b. Germany d. All except 4

c. Greece

d. Italy 41. Kerch Strait, sometimes seen in the news recently,


connects which of the following?

a. Black Sea and Caspian Sea


38. Jigarthanda, sometimes seen in the news recently,
refers to? b. Black Sea and Sea of Azov

a. A GI tagged food product from the state of Tamil c. Black Sea and Mediterranean Sea
Nadu
d. Mediterranean Sea and Red Sea
b. India’s first hypervelocity expansion tunnel test
facility

c. A new fish species that was recently discovered 42. Consider the following statements with respect to
near Lakshadweep Interpol Notice system.

d. An indigenous technology developed for early 1. Interpol notices are international requests for
detection of cancer cooperation allowing legislatures in member
countries to share critical information.

2. It can be issued only based on the request of


39. Consider the following pairs. member country’s INTERPOL National Central
Bureau.
Organisms Relationship
3. Bureau of Police Research and Development
1. Barnacles and Swimming Crabs – Parasitism (BPRD) is the national central bureau of India
to liaison with Interpol.
2. Clownfish and Sea Anemones – Commensalism
4. Most of these notices are for police use only and
3. Barnacles and Humpback Whales – Mutualism are not available to the public.
4. Corals and Sponges – Competition How many of the statement(s) given above
is/are incorrect?
How many of the pair(s) given above is/are correctly
matched? a. Only one
a. Only one b. Only two
b. Only two c. Only Three
c. Only three d. All four
d. All four

43. Consider the following statements.


40. Which among the following are known as Super Statement-I: Fair and Remunerative Price is the
Pollutants? minimum price that sugar mills have to pay to farmers
for sugarcane.
1. Methane
Statement-II: Minimum Selling Price (MSP) for
2. Black Carbon Sugarcane is the minimum price at which sugar mills
can sell sugar to the market.
3. Nitrogen dioxide
Select the correct answer using the codes given below:
4. Ground-level ozone
a. Both Statement-I and Statement-II are correct
Select the correct answer using the codes given below:
and Statement-II is the correct explanation for
a. All except 1 Statement-I

b. All except 2

CHENNAI |SALEM| MADURAI | COIMBATORE DELHI | BANGALORE | THIRUVANANTHAPURAM


www.shankariasacademy.com 291
www.iasparliament.com

b. Both Statement-I and Statement-II are correct 2. Taj Mahal, Akbar’s Tomb and Fatehpur Sikri
and Statement-II is not the correct explanation for are the three world heritage sites that are
Statement-I covered under this zone.

c. Statement-I is correct but Statement-II is Which of the statement(s) given above is/are correct?
incorrect
a. 1 only
d. Statement-I is incorrect but Statement-II is
correct b. 2 only

c. Both 1 and 2

44. Consider the following statements with respect to d. Neither 1 nor 2


Elephant Reserves in India.

1. India’s 33rd elephant reserve Terai is located in


Uttar Pradesh. 47. Which of the following is/are correct regarding
Paruveta Utsavam?
2. The State of Karnataka has the maximum
number of elephant reserves in India. 1. It is an annual mock hunting festival.

Which of the above statement(s) is/are correct? 2. It is celebrated by Chenchu tribes in Odisha.

a. 1 only 3. It is one among the list of UNESCO’s Intangible


cultural heritage.
b. 2 only
Select the answer using the code given below:
c. Both 1 and 2
a. Only one
d. Neither 1 nor 2
b. Only two

c. All Three
45. Which of the following statements is/are correct
with respect to Outer Space Treaty, 1967? d. None of the above

1. It was adopted by United Nations Security


Council (UNSC).
48. Consider the following pairs.
2. It is also called as Magna Carta of space law.
Languages Regions
3. There shall be free access to all areas of celestial
bodies. 1. Santali – Jharkhand

4. India signed the treaty in 1967 but yet to ratify 2. Kokborok – Tripura
it.
3. Badaga – Odisha
Select the correct answer using the codes given below:
How many of the pair(s) given above is/are correctly
a. 1 and 3 only matched?

b. 2 and 3 only a. Only one

c. 2 and 4 only b. Only two

d. All four c. All Three

d. None of the above

46. Consider the following statements with respect to


Taj Trapezium Zone (TTZ).
49. Consider the following pairs.
1. It is geographically spread in the states of
Martial Arts Practiced in
Rajasthan and Uttar Pradesh.
1. Kalaripayattu – Kerala

2. Gatka – Punjab

CHENNAI |SALEM| MADURAI | COIMBATORE DELHI | BANGALORE | THIRUVANANTHAPURAM


www.shankariasacademy.com 292
www.iasparliament.com

3. Khukuri Dance – Sikkim 52. Consider the following statements with respect to
Genome India Project
4. Krav Maga – Arunachal Pradesh
1. It aims to collect 10,000 genetic samples from
How many of the pair(s) given above is/are correctly citizens across India, to build a reference
matched? genome grid.

a. Only one 2. It is based on Next-Generation Sequencing


(NGS) platform.
b. Only two
3. It is a national project funded by Department of
c. Only three Biotechnology (DBT), Government of India.
d. All four How many of the statements given above are correct?

a. Only one
50. Consider the following pairs. b. Only two
Projects States c. All Three
1. Kaleshwaram Project – Andhra Pradesh d. None of the above
2. Nokhra Solar Project – Rajasthan

3. Kiru Hydel Project – Jammu & Kashmir 53. Consider the following statements with respect to
the Unified Payments Interface (UPI)
How many of the pair(s) given above is/are correctly
matched? 1. It is a system that powers multiple bank
accounts into a single mobile application.
a. Only one
2. The fund transfer through UPI is faster than
b. Only two National Electronic Fund Transfer (NEFT).
c. All Three 3. UPI is regulated by the National Payments
Corporation of India Limited (NPCIL).
d. None of the above
How many of the above statement(s) is/are correct?

a. Only one
51. Consider the following statements with respect to
Global Mercury Partnership b. Only two
1. It aims to protect human health and the c. All Three
environment from the releases of mercury to
air, water and land. d. None of the above
2. It was launched by the World Economic Forum.

3. It extends financial assistance to developing 54. Consider the following statements.


nations to address and mitigate mercury
pollution issues. It was reconstructed in 1565 by Chilarai, who was the
reigning king of the Koch dynasty. This temple is
How many of the statement(s) given above is/are dedicated to different forms of Mother Shakti.
correct? Ambubachi Mela is one of the major festivals of this
temple.
a. Only one
Identify the temple using the description given above.
b. Only two
a. Somnath Temple
c. All Three
b. Kalaram Temple
d. None of the above
c. Kamakhya Temple

d. Guruvayur Temple

CHENNAI |SALEM| MADURAI | COIMBATORE DELHI | BANGALORE | THIRUVANANTHAPURAM


www.shankariasacademy.com 293
www.iasparliament.com

55. Consider the following statements with respect to How many of the statement(s) given is/are above are
the Central Asian Flyway (CAF): correct?

1. It lies entirely within the Northern Hemisphere. a. Only one

2. India is located within the Central Asian b. Only two


Flyway.
c. All three
3. It is the shortest flyway in the world.
d. None
How many of the statement(s) given above is/are
correct?

a. Only one 59. Consider the following statements with respect to


Kala-Azar
b. Only two
1. It is a parasitic infection transmitted by
c. All Three sandflies.

d. None of the above 2. India has recently become the first country in
the world to be officially validated by the WHO
for eliminating it as a public health problem.

56. MakhaBucha, sometimes seen in the news recently, 3. The National Health Policy (2002) originally
is associated with? sought to eliminate Kala Azar by 2010.

a. Jainism How many of the statements given above are correct?

b. Islamism a. Only one

c. Buddhism b. Only two

d. Vaishnavism c. All three

d. None

57. Which of the following best describes the


term ‘Micellar Water’?
60. Consider the following statements with respect to
a. An alkaline solution that is infused with fulvic IBSA Fund
acid and contain several nutrients.
1. It aims to identify replicable and scalable
b. A domestic wastewater that has less nitrogen and projects that can be disseminated to interested
phosphorous than black water. developing countries.

c. A water formed by special molecules known as 2. It was established jointly by India, Brazil and
surfactants that can help cleanse and tone the skin. South Africa.
d. A water that has a higher pH level than that of 3. New Development Bank (NDB) serves as the
plain tap water and can neutralize acid in your Fund Manager and Secretariat of the IBSA
bloodstream. Fund.

How many of the statement(s) given above is/are


correct?
58. Consider the following statements with respect to
Indian Skimmer a. Only one

1. It is a migratory species that breeds in Russia b. Only two


and East Asia.
c. All three
2. They spend most of their life cycle above the
treeline. d. None

3. It has been listed under the Convention of


Migratory Species (CMS) of wild animals.

CHENNAI |SALEM| MADURAI | COIMBATORE DELHI | BANGALORE | THIRUVANANTHAPURAM


www.shankariasacademy.com 294
www.iasparliament.com

61. Consider the following statements with respect to 4. Germanium – Optical fibers, satellites and solar
VSHORADS Missile cells.

1. It is a Man Portable Air Defense System How many of the pair(s) given above is/are correct?
(MANPAD) developed by India in collaboration
with Israel. a. Only one

2. This missile system is engineered to neutralize b. Only two


hostile aircraft, drones, and helicopters within
a range of up to 60 kilometers. c. Only three

Which of the statement(s) given above is/are correct? d. All four

a. 1 only

b. 2 only 65. India has recently inaugurated the World's first


portable hospital. It was launched under which of the
c. Both 1 and 2 following projects?

d. Neither 1 nor 2 a. Project BHISHM

b. Project VARTAK

62. Planum Boreum, sometimes seen in the news c. Project JEEVAN


recently, is associated with?
d. Project RAAHAT
a. A rare event of Auroras formed in the mid-
latitudes

b. A pesticide induced disease prevalent among the 66. Consider the following statements with respect to
cattles Hangul.

c. A species of perennial tropical grass native to the 1. It is the only surviving Asiatic member of the
African grasslands red deer family.

d. The layers on the North Pole formed from the 2. It is classified as critically endangered in the
gradual deposition of dust IUCN Red List.

3. The population of Hangul has steadily


decreased in the recent years.
63. Agalega Island, sometimes seen in the news
recently, is latitudinally lies between? How many of the statement(s) given above is/are
correct?
a. Seychelles and Maldives
a. Only one
b. Maldives and Mauritius
b. Only two
c. Madagascar and South Africa
c. All Three
d. Maldives and Lakshadweep
d. None of the above

64. Consider the following pairs.


67. Swachhata Green Leaf Rating (SGLR) Initiative,
Critical Minerals Major Applications sometimes seen in the news recently, is associated with?

1. Beryllium – Manufacture of computer, a. To improve sanitation in railway stations in


electronic and optical products. India

2. Tellurium – Solar power, thermoelectric b. To improve sanitation practices in India's


devices and Rubber vulcanizing. tourism sector

3. Zircon – High value chemical c. To rank and provide incentive to the start-ups
manufacturing and electronics sector. that supports sustainable green energy

d. To provide rating to the hospitals, based on the


maintenance of cleanliness in their campus

CHENNAI |SALEM| MADURAI | COIMBATORE DELHI | BANGALORE | THIRUVANANTHAPURAM


www.shankariasacademy.com 295
www.iasparliament.com

68. Which of the following statements regarding ADITI 71. Consider the following countries:
Scheme is incorrect?
1. Iran
a. It aims to support innovation in critical and
strategic deep-tech technologies which are in the 2. Iraq
nascent stage of research & development.
3. India
b. It operates within the iDEX (Innovations for
Defence Excellence) framework under the 4. United States of America
Department of Defence Production (DDP).
5. United Arab Emirates
c. Under the scheme, startups will be eligible for
grant up to 50% of product development budget How many of the above-mentioned countries are full-
(PDB). fledged members of the Gas Exporting Countries Forum
(GECF)?
d. Startups recognized by the Ministry of Micro
Small and Medium Enterprises will be eligible for a. Only two
funding under the scheme.
b. Only three

c. Only four
69. Which one of the following World Heritage Sites is
d. All five
located in the Red Sea and Gulf of Aden (RSGA)
Ecosystems?

1. Socotra Archipelago 72. IRIS, sometimes seen in the news recently, is related
to?
2. Dungonab Bay
a. An application used to detect frauds in bitcoin
3. Senganeeb Atoll
mining
4. Malpelo Island
b. An humanoid Artificial Intelligence robot
Select the correct answer using the codes given below: teacher

a. All expect 1 c. A ballistic missile defence interceptor developed


by the DRDO
b. All expect 2
d. A methane-powered rocket developed by a
c. All expect 3 private Chinese company

d. All expect 4
73. Consider the following pairs.

70. Consider the following statements with respect to Women schemes States
Orans
1. Orunodoi Scheme – Assam
1. Orans are traditional sacred groves found in
Odisha. 2. Ammavodi Scheme – AndhraPradesh

2. It is home to the endangered white cheeked 3. Gruha Lakshmi Scheme – Karnataka


macaque.
4. Vidiyal Payanam Scheme–Tamil Nadu
Which of the statement(s) given above is/are correct?
How many of the pair(s) given above is/are correctly
a. 1 only matched?

b. 2 only a. Only one

c. Both 1 and 2 b. Only two

d. Neither 1 nor 2 c. Only three

d. All four

CHENNAI |SALEM| MADURAI | COIMBATORE DELHI | BANGALORE | THIRUVANANTHAPURAM


www.shankariasacademy.com 296
www.iasparliament.com

74. Ammonium Phosphate Sulphate, sometimes seen in a. 1 only


the news recently, is mainly used in?
b. 2 only
a. Fertilizers
c. Both 1 and 2
b. Fire crackers
d. Neither 1 nor 2
c. Cloud seeding

d. Chemical weapons
78. Kasarkod Tonka, an important nesting site for Olive
ridley turtle, is located in?

75. Consider the following passage. a. Kerala

He encouraged all devotees to wear turbans and dhotis b. Odisha


to promote equality. He initiated the Thuvayal Panthy
programme, teaching vegetarianism and discipline to c. Karnataka
followers. He established Nizhal Thangals as
community worship spaces, which did not have any idol d. Tamil Nadu
or deity.

Identify the personality associated with the above


passage. 79. Consider the following pairs:

a. Ayyankali Terms in news Relevance

b. Ayya Vaikundar 1. Catalhoyuk – Neolithic Site

c. Ramalinga Swamigal 2. Nandankanan – Zoological Park

d. Chaitanya Mahaprabhu 3. Raghanesda – Solar Park

How many of the pair(s) given above is/are correctly


matched?
76. Consider the following statements with respect to
Golden Langur a. Only one

1. It is endemic to Western Ghats. b. Only two

2. They are fossorial and diurnal in nature. c. All Three

3. It is listed as critically endangered under the d. None of the above


IUCN red list.

How many of the statement(s) given above is/are


80. Consider the following statements with respect to
correct?
Gulf of Mannar Biosphere Reserve
a. Only one
1. It is an international bird and biodiversity area
b. Only two and a part of Central Asian Flyway.

c. All Three 2. It has been recognized by the Man and


Biosphere (MAB) Programme of UNESCO.
d. None of the above
Which of the statement(s) given above is/are correct?

a. 1 only
77. Consider the following statements with respect
to Pushpak b. 2 only

1. It is the reusable launch vehicle of Indian Space c. Both 1 and 2


Research Organisation (ISRO).
d. Neither 1 nor 2
2. The vehicle is designed as an all-rocket, fully
reusable single-stage-to-orbit (SSTO) vehicle.

Which of the statement(s) given above is/are correct?

CHENNAI |SALEM| MADURAI | COIMBATORE DELHI | BANGALORE | THIRUVANANTHAPURAM


www.shankariasacademy.com 297
www.iasparliament.com

81. Consider the following statements with respect to a. Only two


Elections in Rajya Sabha
b. Only three
Statement-I: The provisions of the Tenth Schedule,
with respect to voting against the instruction of the c. Only four
party, will not be applicable for a Rajya Sabha election.
d. All five
Statement-II: The elections to Rajya Sabha are not
treated as a proceeding within the Legislative
Assembly.
84. Match the following with reference to the
Select the correct answer using the codes given below: three stages of India’s Nuclear Program.

a. Both Statement-I and Statement-II are correct Types of nuclear


Stages By-Product
and Statement-II is the correct explanation for reactor
Statement-I
a. Pressurized Heavy
1. Stage-
b. Both Statement-I and Statement-II are correct Water Reactors p. Plutonium-239
I
and Statement-II is not the correct explanation (PHWRs
for Statement-I
2. b. Fast Breeder q. Energy,Uranium-233
c. Statement-I is correct but Statement-II is Stage-II Reactor and Plutonium-239
incorrect
3. c. Advanced Heavy
r. Energy and Uranium-
d. Statement-I is incorrect but Statement- II is Stage- Water Reactors
233
correct III (AHWR)

Select the correct answer using the codes given below:

82. Consider the following pairs with respect to WTO’s a. 1- b-p: 2-c-q: 3-a- r
Subsidies.
b. 1- c-q: 2-a-r: 3-b-p
WTO Subsidies Limits
c. 1-a-p: 2-b-q: 3-c-r
1. Green box – Subjected to reduction commitments
d. 1- b- q: 2-c-r: 3-a-p
2. Blue box – Trade distortive subsidies

3. Amber box – No limits


85. Consider the following statements with respect to
How many of the pair(s) given above is/are correctly Prototype Fast Breeder Reactor (PFBR)
matched?
1. India’s first indigenous Fast Breeder Reactor
a. Only one was recently commenced core loading at
Kudankulam Nuclear Power Plant, Tamil Nadu.
b. Only two
2. PFBR has been fully designed and constructed
c. All Three indigenously by BHAVINI.
d. None 3. Once commissioned, India will only be the
second country after USA to have commercial
operating Fast Breeder Reactor.
83. Consider the following countries. How many of the statement(s) given above is/are
correct?
1. Argentina
a. Only one
2. Brazil
b. Only two
3. India
c. All three
4. Ukraine
d. None
5. Pakistan

How many of the above countries are member of Cairns


Group?

CHENNAI |SALEM| MADURAI | COIMBATORE DELHI | BANGALORE | THIRUVANANTHAPURAM


www.shankariasacademy.com 298
www.iasparliament.com

86. How cruise missiles are different from ballistic 3. Most SHGs are women’s groups with
missiles? membership ranging between 10 and 20.

1. Ballistic missiles are powered throughout its How many of the statements given above are correct?
flight whereas cruise missiles are powered only
in the first phase of flight. a. Only one

2. Cruise missiles travel at low altitudes whereas b. Only two


ballistic missiles travelled at high altitudes.
c. All Three
Which of the statement(s) given above is/are correct?
d. None of the above
a. 1 only

b. 2 only
90. Consider the following statements with respect to
c. Both 1 and 2 the Cervical Cancer.

d. Neither 1 nor 2 1. Almost all cervical cancer cases are caused due
to high-risk Human papillomavirus (HPV).

2. The 90-70-90 targets are a set of goals set by the


87. Consider the following statements with respect to World Health Organization (WHO) in 2020 to
Missile Technology Control Regime (MTCR) eliminate cervical cancer by 2030.

1. It aims to limit the spread of ballistic missiles Which of the statement(s) given above is/are correct?
and other unmanned delivery systems that
could be used for chemical, biological and a. 1 only
nuclear attacks.
b. 2 only
2. It was established by the G-7 industrialized
countries. c. Both 1 and 2

3. India is a part of the regime since 2016. d. Neither 1 nor 2

How many of the statements given above are correct?

a. Only one 91. Which of the following regulates the State’s


borrowing in India?
b. Only two
1. Article 293
c. All Three
2. 7th Schedule
d. None of the above
3. Finance Commission

4. Fiscal Responsibility and Budget Management


88. Shanan Hydropower Project, sometimes seen in the Act, 2003
news recently, is located at Uhl River. Uhl River is a
tributary of? Select the correct answer using the code given below:

a. Ravi River a. Only one

b. Indus River b. Only two

c. Beas River c. Only three

d. Sutlej River d. All four

89. Consider the following statements with respect to 92. Consider the following statements with respect to
Self Help Groups (SHGs) Cinematograph (Certification) Rules, 2024

1. SHGs are self-managed and have well-defined 1. It overhauled the Cinematograph


rules and by-laws. (Certification) Rules, 1983 to improve and
contemporize the entire process of certification
2. SHGs can be formal or informal. of films for public exhibition.

CHENNAI |SALEM| MADURAI | COIMBATORE DELHI | BANGALORE | THIRUVANANTHAPURAM


www.shankariasacademy.com 299
www.iasparliament.com

2. The UA category is classified into UA+, UA13+ 95. Consider the following pairs.
and UA16+, aiding parents in content
suitability decisions for children. Portal Launched by

3. According to the rules, two-third of the Central 1. SheRNI Portal - University Grants Commission
Board of Film Certification (CBFC) Board shall (UGC)
be women.
2. PM-SURAJ Portal - Ministry of Social Justice and
How many of the statements given above Empowerment.
are incorrect?
Which of the pair(s) given above is/are correctly
a. Only one matched?

b. Only two a. 1 only

c. All Three b. 2 only

d. None of the above c. Both 1 and 2

d. Neither 1 nor 2

93. Consider the following statements with respect to


Atmospheric Research Testbed
96. Consider the following statements with respect to
1. It aims to study the vital cloud processes Status of Leopards in India Report
associated with the Indian monsoon,
particularly focusing on the Monsoon Core 1. It is the outcome of the 5th cycle of leopard
Zone (MCZ) in central India. population estimation carried out by the
National Tiger Conservation Authority and
2. It is a component under AWaRe (Access, Wildlife Institute of India.
Watch, Reserve) program of Indian
Meteorological Department (IMD). 2. According to the report, the number of leopards
declined rapidly when compared to 2018 levels.
3. It is being funded by the Ministry of Earth
Sciences (MoES). 3. Madhya Pradesh is having the largest leopard
population followed by Maharashtra and
How many of the statements given above are correct? Karnataka.

a. Only one How many of the statement(s) given above is/are


correct?
b. Only two
a. Only one
c. All Three
b. Only two
d. None of the above
c. All Three

d. None of the above


94. DIANA, sometimes seen in the news recently, is an
initiative of?

a. North Atlantic Treaty Organization (NATO) 97. Dogleg manoeuvre, sometimes seen in the news
recently, is associated with?
b. Shanghai Cooperation Organization (SCO)
a. A rare atmospheric phenomenon occurred in
c. Collective Security Treaty Organization (CSTO) the Pacific Ocean

d. Bay of Bengal Initiative for Multi-Sectoral b. A practice of austerity policy that aims to reduce
Technical and Economic Cooperation government budget
(BIMSTEC)
c. A sharp turn to deviate from a straight flight
path in space programme

d. None of the above

CHENNAI |SALEM| MADURAI | COIMBATORE DELHI | BANGALORE | THIRUVANANTHAPURAM


www.shankariasacademy.com 300
www.iasparliament.com

98. Consider the following pairs.

GI Products Origin

1. Risa Textile - Tripura

2. Ambaji White MarblE - Gujarat

3. Ratlam Riyawan Lahsun - Madhya Pradesh

4. Narasapur crochet lace - Andhra Pradesh

How many of the pair(s) given above is/are correctly


matched?

a. Only one

b. Only two

c. Only three

d. All four

99. Women, Business and the Law Report 2024 was


published recently by?

a. World Bank

b. World Economic Forum

c. International Monetary Fund

d. International Court of Justice

100. Consider the following statements with respect to


Asiatic Lions

1. The Asiatic lion is a subspecies of lion, slightly


smaller than African lions.

2. At present, Gir National Park, Gujarat is the


only abode of the Asiatic lion.

3. The International Union for Conservation of


Nature (IUCN) has recently re-categorized the
Asiatic lion’s conservation status as vulnerable
from endangered.

How many of the statement(s) given above is/are


correct?

a. Only one

b. Only two

c. All three

d. None

CHENNAI |SALEM| MADURAI | COIMBATORE DELHI | BANGALORE | THIRUVANANTHAPURAM


www.shankariasacademy.com 301
www.iasparliament.com

Answer key - Test - VI


1 2 3 4 5 6 7 8 9 10

C A D B A C A A C B

11 12 13 14 15 16 17 18 19 20

C B C D A B B C A A

21 22 23 24 25 26 27 28 29 30

B B A D A B A D B D

31 32 33 34 35 36 37 38 39 40

A B D C A C B B B C

41 42 43 44 45 46 47 48 49 50

B B B A B A A B C B

51 52 53 54 55 56 57 58 59 60

A C B C C C C D B B

61 62 63 64 65 66 67 68 69 70

D D B D A B B D D D

71 72 73 74 75 76 77 78 79 80

A B D A B D C C C C

81 82 83 84 85 86 87 88 89 90

A D C C A B C C C C

91 92 93 94 95 96 97 98 99 100

D A B A C B C D A C
 Fog materialises whenever there is a
temperature disparity between the ground and
EXPLANATION the air.
1. c
 Dense fog in India – The fog in entire Indo-
Fog Gangetic plains is due to the presence of

Dense fog covered the better part of north India during o Low temperatures
the last days of December and the 1st couple of days of
2024. o Low wind speed

 Fog – It is a collection of small droplets of


water produced when evaporated water has
cooled down and condensed.

 It is nothing but a thick cloud, but very close to


the earth’s surface.

 Conditions for a thick fog

o Lower temperatures

o Abundant moisture near the surface

o Higher humidity

CHENNAI |SALEM| MADURAI | COIMBATORE DELHI | BANGALORE | THIRUVANANTHAPURAM


www.shankariasacademy.com 302
www.iasparliament.com

o Moisture availability

o Plenty of aerosols

2. a

Aligarh Muslim University (AMU)

Recently the Supreme Court was hearing a reference


related to the minority status of Aligarh Muslim
University.

 Established in – 1875 as Muhammadan


Anglo-Oriental (MAO) College

 Founder – Sir Syed Ahmad Khan

 Pre-independence - Aligarh Muslim


University (AMU) Act, 1920 was passed during
colonial era which enabled MAO and a few  It is a form of judicial review that is used to test
other colleges to be affiliated with it. the legality of any legislation by the courts.

 Post-independence – 2 amendments to  Components – Supremacy of the


1920 Act in 1951 and 1965 changed the Constitution
structure of the governing body of the
university and gave powers to President of o Powers of the Supreme Court (SC)
India to nominate its members. under

 S. Azeez Basha vs Union of India, 1967 – o Art 32 – Every individual has the right
The Court ruled that the AMU was not a to move the SC for the enforcement of
minority institution as there was a central their fundamental rights.
legislation governing the same.
o Art 136 – Special leave to appeal by
the SC.
 Supreme Court – The minority character
of an educational institution is not lost o Art 141 – Law declared by SC to be
if its founders, who belong to a particular binding on all courts.
minority community, chose administrators
from other communities to run the o Art 142 – Enforcement of decrees and
institution. orders of SC.
 Article 30 – All religious and linguistic  Powers of the High Courts under
minorities in India have the right to establish
and manage their own educational institutions o Art 226 – Power of High Courts to
including schools, colleges and other issue certain writs
educational facilities.
o Art 227 – Power of superintendence
 It confers on the minority the discretion over all courts by the High Court.
to choose the administrators of the
institution.

3. d

Basic Structure Doctrine

 Propounded on – 24th April 1973 in the


Kesavananda Bharati case.

 It is neither mentioned in the Indian


Constitution nor openly defined.

 It is a constitutional principle that limits the


power of Parliament to amend the Constitution
and preserves its essential features.

CHENNAI |SALEM| MADURAI | COIMBATORE DELHI | BANGALORE | THIRUVANANTHAPURAM


www.shankariasacademy.com 303
www.iasparliament.com

4. b 5. a

Foreign Contribution Regulation Act (FCRA) Square Kilometre Array (SKA) Observatory

The Foreign Contribution Regulation Act, 2010 (FCRA) India had decided to formally join the Square
registration of 2 prominent NGOs — Centre for Policy Kilometre Array (SKA) project, an international
Research (CPR) and World Vision India (WVI) have scientific collaboration working to build the world’s
been cancelled recently. largest radio telescope.

 FCRA – It regulates foreign donations and  It is not a single large telescope, but
ensures that such contributions do not a collection of thousands of dish
adversely affect internal security antennas operating as a single unit.

 Established in – 1976 during Emergency  SKA Observatory Convention – The


period amidst the apprehensions that foreign international treaty that established the facility
powers were interfering in India’s affairs by as an intergovernmental organisation.
pumping money through independent
organisations  Objective – To create 1 square
kilometre of effective area for collecting
 Implementation by – Ministry of Home radio waves using radio telescopes.
Affairs
 A radio telescope is a specialized type of
 Applicability – To all associations, groups antenna and receiver system used to detect and
and NGOs which intend to receive foreign collect radio waves emitted by celestial
donations. objects.

 The annual returns must be filed on the lines of  Mission – To build and operate cutting-edge
Income Tax. radio telescopes to transform our
understanding of the Universe, and deliver
 Foreign contribution – It means the benefits to society through global collaboration
donation, delivery or transfer made by any and innovation.
foreign source
 Headquarters– United Kingdom (UK)
 Availing foreign funds – In a
designated bank account at SBI’s New  Implementation – By installing 1000’s of
Delhi branch. smaller antennas in a specific array design that
would make them function like a single radio
 Utilisation of foreign funds – Only for the telescope.
purpose for which they have been received and
as stipulated in the Act.  Budget – USD 2.4-billion project

 Eligible – It must have a definite cultural,  Distribution of antennas – About 200 of


economic, educational, religious or social them in South Africa and more than 130,000 in
programme with prior FCRA registration/ Australia.
permission from the Central Government.
 Installation location – In sparsely
 Ineligible – The candidates for elections, populated areas in order to minimize signal
journalists or newspaper and media broadcast interference from undesirable Earth-based
companies, judges and Government servants, sources.
members of legislature and political parties or
their office-bearers, and organisations of a  Consortium Members – It includes 16
political nature. member countries, such as Australia, South
Africa, Canada, China, India, Japan, and
 Aadhaar provision are mandatory for all several European nations.
office-bearers, directors and other key
functionaries of an NGO.  Working – It receive radio waves from space
using antennas which are sent through a
 Administrative expenses was capped processing chain that enables the astronomical
at 20% of the total foreign funds received, data ultimately to be turned into an image of the
earlier the upper limit was 50% sky.

 Bar on sub-granting by NGOs to smaller  Advantage – Unlike optical telescopes, radio


NGOs who work at the grass roots level. telescopes can be used even in cloudy skies, as
the longer wavelengths can pass through clouds
unhindered.

CHENNAI |SALEM| MADURAI | COIMBATORE DELHI | BANGALORE | THIRUVANANTHAPURAM


www.shankariasacademy.com 304
www.iasparliament.com

 They can detect invisible hydrogen gas,  During the Kargil War, ULFA openly
the most abundant element in the Universe, supported Pakistan in its monthly newsletter
which emits in the radio band at 1420 MHz. Swadhinata.

 They can also reveal areas of space that may be  ULFA peace accord – ULFA cadres have
obscured by cosmic dust. agreed to surrender arms and ammunition,
vacate their camps.
 Significance – It will be the world's biggest
and most advanced radio telescope ever  They have decided to join the mainstream and
constructed. to engage in the peaceful democratic process.

 India – Being involved, since its inception in  It would be made a time-bound programme by
1990s, led by Pune-based National Centre for the Ministry of Home Affairs to fulfil the
Radio Astrophysics (NCRA). demands of the ULFA.

 India’s contribution – In design and  97 out of 126 Assembly seats in Assam would be
development of the telescope and the reserved for indigenous people.
development, and operation, of the Telescope
Manager, the ‘neural network’ or the software  Rs. 1.5 lakh crore investment had been pledged
that will run the entire facility. in the peace accord.

 Benefits – Though none of the SKA facilities  Constituency safeguards would be ensured by
would be located in India, there are immense protecting land rights and putting restrictions
science and technology gains to the Indian on migration from one constituency to another.
scientific community.
7. a
6. c
Pong dam
ULFA Peace Accord
Recently the draft policy was prepared to declare Pong
Recently, the pro-talks faction of the United Liberation Dam Wildlife Sanctuary and the surrounding area an
Front of Asom (ULFA) have signed a historic tripartite eco-sensitive zone.
peace deal with the Government of India and the State
government of Assam.  Built in – 1975, named after Maharana Pratap
and called as Maharana Pratap Sagar.
 Historical background – There was an
influx of migrants from all over since 19th  Location – Over Beas River in Himachal
century into Assam. Pradesh.

 It was further exacerbated by the Partition in  It is the highest earth-filled dam in India and is
1947 and the subsequent exodus of refugees one of India’s largest man made reservoir.
from the erstwhile-East Pakistan.
 In 1983, Pong Dam Wildlife Sanctuary was
 It resulted in a 6-year long mass movement declared by the state government.
from 1979.
 In 1994, the Government of India declared
 ULFA – United Liberation Front of Assam Pong Dam Lake Bird Sanctuary as a ‘wetland of
was born during the anti-foreigner’s national importance’ and in 2002, it was
movement of 1979. declared as a Ramsar Site.

 Aim – To establish a sovereign Assamese  It is the largest congregation of the ‘Bar headed
nation through an armed struggle geese’.
against the Indian state.
 Vulture Cafe – A place where cattle carcasses
 Geographical spread – It still has camps in are made available at one spot, for vultures to
Myanmar, and previously had camps in both ‘feast’ (scavenge) upon them.
Bangladesh and Bhutan.
Kalpasar Project
 Links to other insurgents – Outfits in the
Northeast, Myanmar and also with Islamic Local fishers of Gujarat’s Baruch district suffer from
terror outfits like Al-Qaeda. lack of fish catch due to Bhadbhut barrages and others
dams across River Narmada.
 It also has links to Pakistan’s Inter-Services
Intelligence (ISI).  It is known as ‘The Gulf of Khambhat
Development Project’.

CHENNAI |SALEM| MADURAI | COIMBATORE DELHI | BANGALORE | THIRUVANANTHAPURAM


www.shankariasacademy.com 305
www.iasparliament.com

 Mission – To make a sustainable water state  Zambia has about 6% of the world’s copper
where there is ‘water for all, water forever and reserves and was the 8th largest producer of
more crops per drop’. copper in 2022.

 Vision – To store Gujarat’s 25% average  Reserves – Chile, Australia, Peru, Russia and
annual surface water by 30 km long dam across Mexico.
the Gulf of Khambhat.
 Production – Chile, Peru, the Democratic
 It will be the world’s largest fresh water Republic of the Congo, China, the United
reservoir in sea. States.

Dahej Composition Alloys

Industrial pollution has destroyed local fisheries in Copper +iron +nickel Stainless steel
Dahej near Bharuch, Gujarat.
Copper + nickel Morel metal
 Location – Near the ancient port site of
Barygaza (near today’s Bharuch) on the Gulf of Copper + aluminium Duralumin
Khambhat, Gujarat.
Copper + zinc Brass
 It began to be developed as a Special Economic
Zone (SEZ) in 1990. Copper + tin Bronze

 It is one of the 4 Petroleum, Chemical and


Petrochemicals Investment Regions (PCPIR) in
the country.

 Ecological significance - The freshwater of


the Narmada enters the saline waters of the
Gulf of Khambhat in Dahej.

 Many types of sea life including shrimp, bumla,


salmon, boi, makul are found here.

 It is also the breeding ground of the Hilsa


(‘palwa’ in Gujarat) fish.

Ratle Hydro Electric Project

Government diverts Chenab River water to expedite


hydroelectric project in Jammu and Kashmir. 9. c

 It is an 850 MW hydroelectric power project Supreme Court Legal Services Committee


under construction on the Chenab River (SCLSC)
in Jammu and Kashmir.
Recently, Department of Justice have announced
 The project is scheduled to open in 2026. nomination of Supreme Court judge Justice BR Gavai
as the Chairman of the Supreme Court Legal Services
8. a Committee (SCLSC).

Copper reserves in India  Constituted under – Section 3A the Legal


Services Authorities Act, 1987 which states that
 Copper – It is a good conductor of electricity the Central Authority (NALSA) shall constitute
and is ductile (able to be drawn out into a thin the committee.
wire).
 Objective – To provide free and competent
 Reserves in India – Rajasthan legal services to the weaker sections of society,
(50%); Madhya Pradesh (24%); Jharkhand in cases falling under the top court’s
(19%); 7% in AP, Gujarat, Haryana, Karnataka jurisdiction.
etc.
 Composition – It consists of a sitting SC
 Reserves across globe – Chile has judge, who is the chairman, along with other
the world's largest copper reserves and is members possessing the experience and
also the world's largest copper producer. qualifications prescribed by the Centre.

CHENNAI |SALEM| MADURAI | COIMBATORE DELHI | BANGALORE | THIRUVANANTHAPURAM


www.shankariasacademy.com 306
www.iasparliament.com

 Selection criteria – Under Section 27 of the  To explore polar and high seas regions of the
1987 Act, the Centre is empowered to make Earth.
rules in consultation with the CJI, by
notification.  To do sustainable harnessing of oceanic
resources.
 Rule 10 of the NALSA Rules, 1995, entails the
numbers, experience, and qualifications of the  To translate the knowledge of Earth systems
SCLSC members. science into services for societal, environmental
and economic benefit.
 Appointment – CJI nominates both the
chairman and other members of  Earth System Sciences deal with all the 5
SCLSC and appoints the Secretary to the components of the earth system namely
Committee. atmosphere, hydrosphere, geosphere,
cryosphere, and biosphere and their complex
 Powers – It can appoint officers and other interactions.
employees as prescribed by the Centre, in
consultation with the CJI. 11. c

 Legal Services Authorities Act 1987 – Kalaram temple


Gives a statutory base to legal aid programmes
and to provide free and competent legal  It is built in 1792, with the efforts of
services to eligible groups. one Sardar Rangarao Odhekar, it derives
its name from a black statue of the Lord Kala
 National Legal Services Authority Ram translates literally to Black Ram.
(NALSA) – It was constituted in 1995 to
monitor and evaluate the implementation of  Location – On the banks of the River
legal aid programmes and to lay down policies Godavari in the Panchavati area of the Nashik
for making them available. in Maharashtra. It was a part of Dandakaranya
(dense forest in central India).
10. b
 Historical importance of Kalaram
PRITHvi VIgyan (PRITHVI) temple - In 1930 Nashik Satyagraha is led
by B R Ambedkar along with the Marathi
The Prime Minister of India has approved the teacher and social activist Pandurang Sadashiv
overarching scheme ‘PRITHvi Vigyan’ (PRITHVI). Sane, known as Sane Guruji.

 Implemented by – Ministry of Earth  Ambedkar organised a large protest outside the


Sciences (MoES). Kalaram temple with many dalit protesters to
demand access for Dalits to Hindu temples.
 Timeline – 2021-26, at an overall cost of Rs.
4,797 crore.  Mahad satyagraha or Chavdar Tale
Satyagraha was held in 1927 to assert the rights
 It encompasses 5 ongoing sub-schemes namely of Dalits to use the water in public places.

o ACROSS – Atmosphere & Climate Gol Gumbaz


Research-Modelling Observing
Systems & Services  It is the located in Karnataka, built in Indo-
Islamic style of architecture.
o O-SMART – Ocean Services,
Modelling Application, Resources and  It is the tomb of Mohammed Adil
Technology Shah (ruled 1627–1657).
o PACER – Polar Science and  It is the 2nd largest dome ever built, next to St
Cryosphere Research Peter’s Basilica in Rome.
o SAGE – Seismology and Geosciences  It includes a mosque, a Naqqar Khana (a hall for
the trumpeters), a central chamber, where
o REACHOUT – Research, Education, every sound is echoed 7 times, Whispering
Training and Outreach Gallery, where even minute sounds can be hear
clearly 37 metres away.
Objectives
 It is a protected monument administered by the
 To observe the atmosphere, ocean,
Archeological Survey of India.
geosphere, cryosphere and solid earth.

 To predict weather, ocean and climate hazards.

CHENNAI |SALEM| MADURAI | COIMBATORE DELHI | BANGALORE | THIRUVANANTHAPURAM


www.shankariasacademy.com 307
www.iasparliament.com

Somnath Temple  Ramgarh crater- It is located in Rajasthan,


it was formed approximately 165 million years
 Located in - Prabhas Patan, Veraval, in ago due to a meteor impact.
Gujarat.
 Inside the crater, there is a lake
 It is the holy place of the 1st Aadi Jyotirling called Pushkar Talab, which contains both
Shree Somnath Mahadev, among the 12 saline and alkaline water.
Jyotirlinga shrines of Shiva.
 Waddham, a reserve forest, is a well-known
 It is also the place where Lord Shri Krishna took fossil site where a full-fledged dinosaur
his last journey. skeleton was found near the Godavari basin in
Maharashtra.
 Built by - In 1782, Maratha queen Ahalyabai
Holkar built a small temple at the site. 13. c

 Somanatha temple or Deo Patan faced several Maru-Gurjara Style


attacks from raiders, with the most damaging
by Mahmud of Ghazni in 1026 CE.  It is also known as Solanki style, the style of
West Indian temple architecture.
 Under Mughals - Akbar permitted the
worship of the linga in the Somanatha temple  The style originated in Gujarat and Rajasthan
and appointed desais /officers to administer it. from the 11th to 13th centuries, under the
Chaulukya dynasty (Solanki dynasty).
 Aurangzeb gave orders for its destruction and
its conversion into a mosque in 1706 just before  Although originating as a regional style in
he died. Hindu temple architecture, it became especially
popular in Jain temples, and mainly under
 After Independence – The contemporary Jain patronage later spread across India.’
Somnath temple was reconstructed in the
Maru-Gurjara style under the orders of  Indo-Saracenic architecture- It is also
Vallabhbhai Patel and completed in 1951. called as Indo-Saracenic Revival architecture”
which was created by British architects, it
 Maru-Gurjara style is also known as Solanki combines exotic Islamic and Indian
style, the style of West Indian temple architecture elements with neo-classical and
architecture Gothic revival elements from the British
Victorian era.
12. b
 The Chepauk Palace, finished in 1768, is
Rattaparai frequently cited as the first Indo-Saracenic
construction.
 The word, Rattapparai, has become
synonymous with the rock art painted in
red ochre.

 They are located in three hillocks: Melvalai,


Alampadi and Sethavarai in Villupuram
district of Tamil Nadu.

 Alampadi - The paintings are in red and white


ochre.

 Melvalai – A nondescript village whose 14. d


paintings dates back to 3000 B.C. and majority
of etchings was in red ochre. Painting depicting Project Kusha
a group of men rowing a boat, indicates a link
between the Indus Valley and the southern  Aim – To develop India’s own Long-Range
region. Surface-to-Air Missiles (LR-SAM)

 Sethavarai - It has paintings of animals,  Jointly developed with - Israel Aerospace


especially a deer and a fish. The outlines are in Industries, Israel’s major aerospace and
red ochre, while the inner portion is filled with aviation manufacturers
white ochre.
 Range - Hit hostile targets at 150 km, 250 km,
and 350 km ranges.

CHENNAI |SALEM| MADURAI | COIMBATORE DELHI | BANGALORE | THIRUVANANTHAPURAM


www.shankariasacademy.com 308
www.iasparliament.com

 Target – Nearly 8 crore children between the


ages of 9 and 14 years will be eligible and when
divided over 3 years.

 At least 14 HPV types have been identified as


oncogenic and among these, HPV types 16 and
18 considered to be the most oncogenic.

 Benefits – It offers protection against the HPV


 Features – The LR-SAM system will be a strains that cause cancer of the anus, vagina and
mobile platform equipped with long range oropharynx and genital warts.
surveillance and fire control radars.
Cervical Cancer
 It seeks to establish a formidable 3
layered defense system to detect and destroy  It is the 4th most common cancer in
incoming stealth fighters, aircraft, drones, women which develops in a woman's cervix (the
cruise missiles and precision-guided entrance to the uterus from the vagina).
munitions.
 Cause – Almost all (99%) are linked to
 It will be made capable of interacting with infection with high-risk human
an integrated command and control system papillomaviruses (HPV), an extremely common
(IACCS), an automated air defence command virus transmitted through sexual contact.
and control centre used by the Indian Air Force
(IAF).  Prevention – There are effective primary
(HPV vaccination) and secondary prevention
 The ground systems include Igla, OSA-AK-M, approaches.
Pechora missiles, Spyder quick-reaction
missiles, indigenous Akash area defence  Treatment – It is a treatable form of cancer,
missiles and the Barak-8 medium-range SAM as long as it is detected early and managed
systems. effectively

 The Defence Research and Development 16. b


Organisation (DRDO) is working on the Project
Kusha to operationally deploy its own long- AISHE
range air defense system by 2028-29.
All India Survey of Higher Education (AISHE) 2021–
Missile Defence System Country 22 was made public recently.

S-400 Triumf Russia  Initiated in – 2011.

Patriot United States  Aim – To give complete picture of Higher


Education in the country.
Iron Dome Israel
 3 Categories – Universities, Colleges and
15. a Stand-alone institutions.

Cervavac  Data source – Voluntary uploading of data by


institutions of Higher Education listed in
 It is India’s first indigenous and only www.aishe.gov.in portal.
gender neutral quadrivalent HPV
vaccine.  AISHE 2021-22 – It was the 11th Survey and
1st Survey conducted through Web Data
 Cervavac is an indigenously developed Capture Format (DCF).
quadrivalent vaccine by the Serum Institute of
India (SII) in Pune, is already available Key takeaways of AISHE 2021-22
commercially.
 There is increase in enrolment – 4.33
 Immunisation drive – It will be conducted crore students, up from 2020-21 level, captured
through schools and existing vaccination in 8 different levels.
points, planned in 3 phases over 3 years and is
likely to start from the 2nd quarter of 2024 for  Primacy of government
free. institutions – 73.7% of all students
attend government universities, where
 Currently, the 2-dose HPV vaccine is available state public universities have the largest share
commercially for about Rs 2,000 per dose. of enrolment, accounting for around 31%.

CHENNAI |SALEM| MADURAI | COIMBATORE DELHI | BANGALORE | THIRUVANANTHAPURAM


www.shankariasacademy.com 309
www.iasparliament.com

 Higher enrolment of females – It has leopard landscapes across the cat’s range by
steadily increased to 2.07 crores in 2021-22, a 2020.
32% jump from 2014-15 levels with the highest
numbers at the post graduate level, 55.4% are  The 12 snow leopard range countries
women. are Afghanistan, Bhutan, China, India,
Kazakhstan, Kyrgyzstan (Kyrgyz Republic),
 Gender Enrolment Ratio – It is 28.4 for the Mongolia, Nepal, Pakistan, Russia, Tajikistan
age group 18-23 years at all India level and Uzbekistan.
(population data from the 2011 census) and is
in favour of women in 26 states and Union  Bletchley Declaration aims to enhance
Territories. global collaboration on Artificial
Intelligence (AI) safety.
 Chandigarh boasts of the highest GER
(64.8%), followed by Puducherry, Delhi and
Tamil Nadu.

 Gender Parity Index – It is 1.01 at all-India


level, and for SC and ST categories, it is 1.01 and
0.98 respectively.

 Higher preferences for Arts – In UG,


Bachelor of Arts (BA) and in PG, Master of Arts
(MA) programme has the highest enrolment.

 Demographics of students graduating –


Among the graduated students, roughly 50.8%
are women and the graduation rate in arts
and social sciences streams is higher
than others.

 Category-wise, around 35% of the students


belong to OBC, 13% are from SC and 5.7% are
from ST community.
18. c
GER & GPI
Wetland City Accreditation
 Gross Enrolment Ratio (GER) indicates how The Ministry of Environment, Forest, and Climate
many students are part of the higher education Change selected Indore, Bhopal and Udaipur to get
system in a given population. accredited for Wetland City Accreditation under the
Ramsar Convention.
 Gender Parity Index (GPI) shows the ratio of the
female GER to male GER. A GPI of 1 indicates  Wetland City Accreditation (WCA) aims to
parity between the two genders, GPI greater than 1 promote the conservation and wise use of urban
indicates a disparity in favour of females. and peri-urban wetlands, as well as sustainable
socio-economic benefits for local populations.
17. b
 The Accreditation seeks to encourage cities that
Global Snow Leopard and Ecosystem Protection
are close to and dependent on wetlands,
Programme (GSLEP)
primarily Wetlands of International
Importance.
 GSLEP is an alliance of 12 snow leopard
range countries, multi-lateral institutions,
 The Ramsar Convention during COP12 held
NGOs, scientists and local communities.
in the year 2015 approved this voluntary WCA
system that recognizes cities that have taken
 It seeks to address high-mountain development
exceptional steps to safeguard their urban
issues using the conservation of the charismatic
wetlands.
and endangered snow leopard as a flagship.
 This voluntary scheme provides an
 The Bishkek Declaration is a 2013 pledge
opportunity for cities that value their natural or
by 12 countries to ensure the well-being of snow
human-made wetlands to gain international
leopards and the people who live among them.
recognition.
 Secure 20 by 2020 is an initiative of GSLEP that
identified and aims to secure at least 20 snow

CHENNAI |SALEM| MADURAI | COIMBATORE DELHI | BANGALORE | THIRUVANANTHAPURAM


www.shankariasacademy.com 310
www.iasparliament.com

 No Indian city have been formally  India's Khanij Bidesh India Limited (KABIL)
accredited in the Wetland City and Argentina's state-owned enterprise of
Accreditation. Catamarca province have forged an agreement
to explore lithium.
 Criteria – A city to get accredited formally for
Wetland City Accreditation should satisfy the
standards used to implement each of the 6
international criteria mentioned Operational
Guidance for WCA of the Ramsar Convention
on Wetlands.

 The Amrit Dharohar – Is an initiative of the


MoEF&CC that aims to achieve similar goals by
promoting unique conservation values of
Ramsar Sites.

19. a

NUTEC Plastics Initiative

International Atomic Energy Agency and the


Argentine Antarctic Institute (IAA), under the IAEA's
NUTEC Plastics Initiative conducted a study about
21. b
effects of micro-plastics in the Antarctic.
New Ramsar Wetlands
 NUTEC Plastics is a flagship initiative
by the International Atomic Energy 5 more Indian wetlands have been added to the global
Agency (IAEA). list of wetlands of international importance under the
Ramsar Convention taking Indian Ramsar Sites to 80.
 It aims to address the global challenge of plastic
pollution.  Magadi Kere Conservation Reserve
 It brings together countries and partners from  Ankasamudra Bird Conservation
around the world to fight plastic pollution on 2 Karnataka
Reserve
fronts:
 Aghanashini Estuary.
o At point of source, by introducing new
technologies to improve plastic
 Karaivetti Bird Sanctuary
recycling. Tamil Nadu
 Longwood Shola Reserve Forest
o In the ocean, where the bulk of plastic
waste ends up.
 The total number of Ramsar sites in Tamil
 63 countries are participating in marine Nadu is 16, highest in India.
monitoring of micro-plastics and 30 around the
 The United Kingdom has most number of
world are developing innovative recycling
Ramsar sites in the world.
technology.
 The world’s 1st Ramsar Site was the Cobourg
 India is part of the NUTEC Plastics Initiative.
Peninsula in Australia, designated in 1974.
20. a
 The largest site is Rio Negro in Brazil (120,000
Lithium Triangle square kilometres).

India is all set to invest about INR 200 crore to explore  World Wetland Day is celebrated every year on
and develop 5 lithium blocks in Argentina which is a February 2.
part of the Lithium Triangle.
22. b
 Lithium Triangle includes countries such
as Argentina, Chile and Bolivia. High Altitude Pseudo Satellite Vehicle (HAPS)

Recently, the National Aerospace Laboratories (NAL)


 It accounts for more than half of the world’s
has successfully completed the 1st test of high altitude
total lithium resources.
pseudo satellite vehicle.
 Argentina has the second largest lithium
resources in world.

CHENNAI |SALEM| MADURAI | COIMBATORE DELHI | BANGALORE | THIRUVANANTHAPURAM


www.shankariasacademy.com 311
www.iasparliament.com

 HAPS – It is a solar-powered pseudo satellite, Other Indic AI


a new age unmanned aerial vehicle (UAV) models – It includes
which is also called as stratospheric vehicle. Ola’s Krutrim,
SaravamAI’s
 Features – It can fly at altitudes of 18-20 OpenHathi and IIT-
km from the ground, almost double the heights Madras’ Airavata
attained by commercial airplanes. model.

 Unlike drones, they can remain in air for A group of LLMs that
months for being solar powered and can cover is capable of handling
larger area. text, voice messages,
and documents. It has
 They move slowly, at just about 80-100 km Anthropic, a San
some limitations
Claude Francisco-based
per hour. compared
AI startup.
to ChatGPT or Google
 Unlike satellites, it has lower operating cost as Gemini, regarding
it does not require a rocket to get into space. access to Internet and
extensions.
 Unlike geostationary satellites, can be easily
redeployed over another location or re- An LLM that powers Pi
equipped with different payload. chatbot with world-
class real-time web
 India’s prototype – A scaled-down version, search capabilities to
one-third in size to the eventual aircraft had a Inflection
Inflection AI ensure that users get
wing-span of about 12 metres, remain in air for 2.5
access to high-quality
about 8 and a half hours, achieving an altitude and up-to-date
of about 3 km. information in real-
time.
 The full-scale machine that NAL is trying to
build, by 2027, would be aiming to remain in Biotechnology India-specific model
the air for 90 days at a stretch. Research and developed
Innovation to determine the age
23. a Council - of a
Garbhini-
Translational foetus (Gestational
AI models in news GA2
Health Science Age) in a pregnant
and Technology woman in the second
AI model Developed by Purpose Institute & IIT and third trimesters
Madras precisely.
A collection of
conversational LLMs An Artificial
that work together to AIIMS Delhi &
Intelligence (AI)
LaMDA Google complete different Centre for
iOncology platform designed for
types of generative AI Development of
ai the early detection of
tasks that involve Advanced
breast and ovarian
dialogue. Computing
cancer
A LLM that can use LLM based on GPT-
text prompts to 3.5 (Generative
Llama Meta AI
generate and discuss Pretrained
code. Transformer 3.5)
ChatGPT OpenAI model that can be used
AI model that can
for natural language
create near flawless 1
Sora OpenAI processing tasks such
minute long videos
as text generation &
based on text prompt.
language translation.
A series of Indic large
Seetha
language models
Mahalaxmi 24. d
trained Eighth
Healthcare
Schedule languages for Greening India's Wastelands with Agroforestry
Hanooman (SML) & IIT
generating text-to- (GROW) Report
Bombay-
text, text-to-speech,
led BharatGPT
text-to-video and vice  GROW Report provides state-wise and district-
ecosystem.
versa content. wise analysis, supporting government

CHENNAI |SALEM| MADURAI | COIMBATORE DELHI | BANGALORE | THIRUVANANTHAPURAM


www.shankariasacademy.com 312
www.iasparliament.com

departments and industries for greening and  It looks like a civet or a mongoose and it most
restoration projects. prefers higher altitudes (300 to 1200 m).

 It is released by Niti Aayog.  Life style – They are omnivorous, diurnal,


and mainly arboreal but descend to the
 NITI Aayog utilized remote sensing and GIS to ground occasionally. They are social and
assess agroforestry suitability across all spend their time and even hunt in groups.
districts in India.
 Threats – Habitat loss & fragmentation and
 Using thematic datasets, an Agroforestry poaching.
Suitability Index (ASI) was developed for
national-level prioritization.  Protection status

 The GROW initiative aligns with national o IUCN Status – Vulnerable


commitments, aiming to restore 26 million
hectares of degraded land by 2030 and create o Wildlife Protection Act,
an additional carbon sink of 2.5 to 3 billion 1972 – Schedule II
tonnes of carbon dioxide equivalent.
26. b
 Agroforestry integrates trees, crops, and
livestock, addressing food, nutrition, energy, Multiple Independently targetable Re-Entry
employment and environmental challenges. Vehicle (MIRV)

25. a The Agni VI missile will have the feature of MIRV.

Nilgiri Marten  MIRV is a ballistic missile payload that contains


multiple warheads, each capable of being aimed
Tamil Nadu Government has planned to conserve the at a different target.
lesser-known species such as Nilgiri Marten under its
new “TN Endangered Species Conservation Fund”.  They can confuse anti-ballistic-missile
protection systems by releasing multiple
 It is a rare species of marten, a small carnivores warheads, some of which may be dummies,
belonging to the weasel and badger families. with different targets.

 Scientific name – Martes gwatkinsii  The United States was the 1st country to
develop MIRV technology, deploying a MIRVed
 Native – Endemic to the Western Ghats. Intercontinental Ballistic Missile (ICBM) in
1970 and a MIRVed Submarine-Launched
 Distribution – It spans across the states of Ballistic Missile (SLBM) in 1971.
Karnataka, Kerala and Tamil Nadu.
 The United States, the United Kingdom, and
France use MIRV technology on SLBMs.

 In India, Agni-P is equipped with MIRV

 Habitat – 5 clusters in the Western Ghats


namely Brahmagiri, Nilgiris, Anamalai,
Cardamom Hills and Agasthyamalai.

 They are seen in Shola ecosystems, typically


semi-evergreen forests, evergreen forests and
forest patches interspersed with grasslands and
also in tea, acacia, coffee, cardamom, and wattle
plantations.

 Features – It is a small mustelid with a


chocolate coloured fur and a canary yellow
throat.

CHENNAI |SALEM| MADURAI | COIMBATORE DELHI | BANGALORE | THIRUVANANTHAPURAM


www.shankariasacademy.com 313
www.iasparliament.com

27. a (ISP's) tracking, and keeps your personal data


private.

Virus Viroid Connection


Name Use Case
Method
 Discovered – 1898
 Discovered - 1971
 Composed For remote workers to
of – Nucleic acid  Composed Connect to a
Client-to- access company files
(DNA or RNA) core of - Viroids private
Site VPN over a private
that serves as the circular single network or
(Remote connection (or) for users
genetic material. stranded RNA. 3rd party
Access VPN) who wish to browse the
server
public Internet securely.
 Surrounded by – A  Surrounded
protein coat, and, in by – Did not contain
some cases, a lipid the lipid layer or
(fat) layer outside that protein coat.
coat. Network For organizations to link
 Size – Smaller than connects to their internal networks
Site-to-Site
 Size – Larger in than viruses. another across multiple sites in
VPN
viroids network via different locations
 Host – Only LAN, WAN securely.
 Host – It infects both plants are known to be
plants and animals. infected by viroids.
Connect to a For mobile users while
 Example – Tobacco  Example – Potato VPN private on the go, or while
mosaic virus, spindle tuber viroid. Applications network from experiencing an unstable
Hepatitis- C. smartphone Internet connection.

28. d

Virtual Private Network (VPN)

Mumbai cops arrested a 21 year old student who


allegedly sent threatening emails by masking his IP
address using a Virtual Private Network service.

 Virtual – No physical cables are involved.

 Private – No one else can see your


data or browsing activity.

 Networked – Multiple devices work together


to maintain an established link.
29. b
 Working – It establishes a digital
connection between your computer and Morodharo
a remote server owned by a VPN provider,
creating a point-to-point tunnel that encrypts Morodharo is the recently discovered Harappa site
your personal data, masks your IP address. near Dholavira that provides insight into the
expansion and influence of the ancient civilization.
 Secures the data – VPN scrambles personal
data into code and renders it unreadable to  Morodharo is a Harappa site dated back to
anyone without an encryption key. around 4,500
years ago,
 Bypass censorship and flourished during
surveillance – Location spoofing allow users 2,600-1,900 BCE
the ability to circumvent firewalls and view to late 1,900-
blocked websites. 1,300 BCE period.

 Prevent tracking – It masks your IP  It is located near


address, prevents Internet service providers the village of

CHENNAI |SALEM| MADURAI | COIMBATORE DELHI | BANGALORE | THIRUVANANTHAPURAM


www.shankariasacademy.com 314
www.iasparliament.com

Lodrani near the World Heritage Site of 31. a


Dholavira.
Snow Leopard Population Assessment in India
 It is situated close to the Rann desert and it is (SPAI) Program
interpreted that the region, which eventually
turned into a desert, was likely navigable Recently, Indian government released the report,
during the Harappan era. ‘Status of Snow Leopards in India’ under Snow
Leopard Population Assessment in India.
 The settlement features a large fortification,
platform, well and burial cairns.  It is the 1st ever population estimation of
Snow Leopard.
 The discovery included a substantial quantity of
Harappan pottery reminiscent of artefacts  Study period – From 2019 to 2023.
found in Dholavira.
 Study area – It systematically covered over
 Artefacts found at Morodharo are similar to 70% of the potential snow leopard range in the
those found at Dholavira, indicating cultural country, approximately 120,000 sq km of snow
connections. leopard habitat across the trans-Himalayan
region.
30. d
 National Coordinator – Wildlife
Nordic-Baltic Eight (NB8) Institute of India (WII), supported by all
range states and 2 conservation partners,
The India-Nordic-Baltic Eight (NB8) meeting was held Nature Conservation Foundation of Mysuru
recently on the sidelines of the Raisina Dialogue, 2024 and WWF-India.
in New Delhi.
 Techniques – Methods as outlined under
 NB8 - A regional framework consisting of 8 the Global Snow Leopard and Ecosystem
countries from the Northern Europe. Protection Program’s (GSLEP) Population
Assessment of the World’s Snow leopards
 Members (PAWS) initiative.
o 5 Nordic Countries – Denmark,  Findings – There are around 718 snow
Finland, Iceland, Norway, and leopards in India. Only 34% of the habitat in
Sweden. India legally protected.
o 3 Baltic countries – Estonia,  Ladakh has the highest estimated
Latvia, and Lithuania. population (477) followed by Uttarakhand,
Himachal Pradesh, Arunachal Pradesh, Sikkim,
 Established in – 1992 and Jammu and Kashmir.
 Aim – To
discuss
important
regional and
international
issues in an
informal
atmosphere.

 Chair – In
2024, NB8 is
chaired
by Sweden which was taken over Snow leopard
from Latvia.
 Habitat – High rocky areas, alpine meadows,
 NB8 Wise Men Report – Also known as
alpine steppe shrub, and high altitude forests,
Birkavs-Gade Report, it provides concrete
generally live above the tree line at elevations of
guidelines for promoting cooperation between
2,700-5,000 m.
the 8 countries.
 Range – 12 countries - Afghanistan, Bhutan,
 e-PINE – Enhanced Partnership in Northern
China, India, Kazakhstan, Kyrgyzstan, Mongolia,
Europe (e-PINE) was launched by the United
Nepal, Pakistan, Russia, Tajikistan & Uzbekistan.
States in 2003.

CHENNAI |SALEM| MADURAI | COIMBATORE DELHI | BANGALORE | THIRUVANANTHAPURAM


www.shankariasacademy.com 315
www.iasparliament.com

 Global population – Estimated at 3020 to 5390. 6. Regreening Africa’s Agriculture (Ethiopia,


Ghana, Kenya, Mali, Niger, Rwanda,
 IUCN Status – Vulnerable Senegal, Somalia)

 Threats – Overgrazing, human wildlife conflict, 7. Growing Forests in Africa's Drylands


poaching, free-ranging dogs, habitat degradation, Initiative (Mali, Senegal, Tanzania,
and climate change impacts. Uganda, Kenya)

 Importance – Being an apex predator of  Namami Gange (listed in 2022) is the only
Himalayan ecosystem, snow leopard exerts top programme from India that is being listed
down regulation of prey populations and thus under this world restoration flagships initiative.
influence in ecological equilibrium of its habitat.
33. d
32. b
Parthenogenesis
World Restoration Flagships
Charlotte, a female round stingray has become the
UN has announced 7 World Restoration Flagships topic of international attention after getting pregnant
which will be awarded during the 6th UN Environment despite not having a male ray companion for at least 8
Assembly. years.

 World Restoration Flagships is an initiative  Parthenogenesis – In Greek, it means ‘virgin


under the UN Decade on Ecosystem creation’.
Restoration, a global movement coordinated
by UN Environment Programme  It is a rare form of asexual reproduction in
(UNEP) and UN Food and Agriculture which a female produces an embryo
Organization (FAO). without fertilization by a male’s sperm.

 The winning initiatives were announced ahead  An egg produced parthenogenetically may
of the 6th session of the UN Environment be either haploid (n) or diploid (2n).
Assembly, which is the world’s highest-level
decision-making body for matters related to the  Parthenogenesis occurs in many types
environment. of invertebrates including scorpions,
nematodes, mites, water fleas, wasps, some
 The award conferred by the UNEP and FAO of bees, etc.
the UN makes the initiatives eligible for
technical and financial support from the  It may also occur in some vertebrate animals
organisation as well, such as amphibians, some fish, reptiles,
and in few bird species.
 The awards serve as a part of the UN Decade on
Ecosystem Restoration, led by the two agencies.  Parthenogenesis has less genetic
variation than sexual reproduction.
 The campaign aims to prevent, halt and reverse
the degradation of ecosystems on every  Parthenogenic species may be
continent and in every ocean.
o Obligate – They are incapable of
 The projects/ initiatives of the World sexual reproduction.
Restoration Flagships that are honoured are:
o Facultative – They are capable
1. The Restoring Mediterranean Forests of switching between parthenogenesis
Initiative (Lebanon, Morocco, Tunisia, and sexual reproduction depending
Türkiye) upon environmental conditions.

2. The Living Indus initiative (Pakistan)  Apomixis – A process where the egg is
fertilised with cells from the mother rather than
3. The Acción Andina Social Movement by a male.
(Argentina, Bolivia, Chile, Ecuador, Perú)
 Automixis - The process by which a mammal
4. Sri Lanka Mangrove Regeneration can undergo virgin birth. Here, egg would need
Initiative (Sri Lanka) to double its genetic content, divide, and then
re-combine.
5. Terai Arc Landscape(TAL) Initiative
(Nepal)  The egg would end up with the normal number
of chromosomes without the need for any
additional genetic material.

CHENNAI |SALEM| MADURAI | COIMBATORE DELHI | BANGALORE | THIRUVANANTHAPURAM


www.shankariasacademy.com 316
www.iasparliament.com

 A smaller cell known as a “polar body,” which  The portal is developed by the Centre for
forms at the same time as the egg and contains Development of Advanced Computing
DNA similar to the mother’s, merges with the (C-DAC), an R&D organization under the
fertile egg. Ministry of Electronics and Information
Technology.
 The offspring’s are similar to the mother but
not exact clones.  The portal marks a significant leap in the
digitization journey of CMPFO aiming to
 It is more common in low-density populations, address the long-standing issue of digitizing its
such as those on the verge of extinction. records and work processes.

34. c  Coal Mines Provident Fund Organization


(CMPFO) – An autonomous organization
Subika Painting under the aegis of Ministry of Coal.
 Subika paintings are from Manipur, which  It was established in 1948 for administering
are based on stories and events from the Hindu provident fund and pension schemes for
epic and Puranas. providing social security to the coal sector
workers.
 The style is intricately linked to the Meitei
community’s cultural history through its 6 36. c
surviving manuscripts that include:
Payment Banks
1. Subika
 Payments banks are a new model of banks,
2. Subika Achouba conceptualised by the Reserve Bank of India.
3. Subika Laishaba  These banks can accept a restricted deposit,
which is currently limited to rupees 2,00,000
4. Subika Choudit
per customer and may be increased further.
5. Subika Cheithil
 These banks cannot issue loans and credit
6. Thengrakhel Subika cards.

 Basohli painting – Is a traditional art form  The minimum paid-up capital requirement for
that originated in the Basohli town of Jammu payments bank is Rs 100 crore.
and Kashmir, known for its rich cultural
heritage.  For the first 5 years, the stake of the promoter
should remain at least 40%.
 It is a distinct style of miniature painting that
has a unique blend of Hindu mythology, folk  The foreign shareholding will be allowed in
art and Persian influences. these banks as per the rules for FDI in private
banks in India.
 Pichwai paintings – Are a traditional Indian
art form that originated in the town of  These banks were created on the basis of
Nathdwara in Rajasthan, India over 400 years the Nachiket Mor Committee.
ago.
37. b
 Thangka paintings – Of Arunachal Ilsenhohle Cave Site
Pradesh are famous in the Tawang, West
Kameng and Upper Siang districts.  The researchers re-excavated the Ilsenhohle
cave site in Ranis, Germany.
 The process involves a picture panel of paint
which is put on a silk border or cover.  The site provides evidence for the first dispersal
of Homo sapiens across the higher latitudes of
35. a
Europe.
C-CARES
 The stone artefacts that were thought to be
 It is the web portal for Coal Mines produced by Neanderthals were in fact part of
Provident Fund Organization the early H. sapiens tool kit.
(CMPFO), working under aegis of Ministry of
 During the re-excavation a 1.7-metre-thick rock
Coal.
and human fossils were also found.

CHENNAI |SALEM| MADURAI | COIMBATORE DELHI | BANGALORE | THIRUVANANTHAPURAM


www.shankariasacademy.com 317
www.iasparliament.com

 The site now has the oldest H. sapiens fossils. Parasitism

 Bones and stone artefacts from the cave showed  In parasitism, one species (the parasite) lives
that these people hunted large mammals with, on, or in a host species, at the expense of
including reindeer, horses, bison and woolly the host species.
rhinoceroses.
 Examples of common parasites includes
 The new study’s revelations mean that Homo- nematodes, leeches and barnacles.
sapiens reached Northern Europe before
Neanderthals went extinct.  Example - Barnacles & Swimming Crabs

38. b Mutualism

Jigarthanda  In a mutualistic relationship, both species


benefit.
India's First High Hypersonic Test Facility, nick
named ‘Jigarthanda’ was recently unveiled at IIT  Examples - Clownfish & Sea Anemones
Kanpur.
Commensalism
 The S2 is the India's First High
Hypersonic Test Facility that was  Commensalism happens when one species lives
nicknamed as Jigarthanda. with in another species, known as the host.

 It is a 24-meter-long facility located at IIT  The host species neither benefits nor is harmed
Kanpur’s Hypersonic Experimental from the relationship.
Aerodynamics Laboratory (HEAL).
 Examples - Barnacles & Humpback
 It is capable of generating flight speeds Whales
between 3-10 km/s, simulating the hypersonic
conditions encountered during atmospheric  The huge whales transport the tiny barnacles to
entry of vehicles, asteroid entry, scramjet plankton-rich waters, where both species feast
flights and ballistic missiles. upon the abundant microorganisms that live
there.
 This makes it a valuable test facility for ongoing
missions of ISRO and DRDO including Competition
Gaganyaan, RLV, and hypersonic cruise
missiles.  It happens between the members of the same
species (intraspecific competition) and between
39. b different species (interspecific competition).

Symbiotic Relationship  Example - Corals & Sponges

Scientists have recently found a mushroom growing  Sponges are very abundant in coral reefs. If they
out of a frog, indicating a symbiotic relationship become too successful they take the needed
between a frog and the mushroom. food and other resources from the corals that
make up the reef.
 Symbiosis is a term describing any relationship
or interaction between two dissimilar
organisms.

 The specific kind of symbiosis depends on


whether either or both organisms benefit from
the relationship.

 Symbiotic relationships can be useful measures


of an ecosystem’s health.

 There are 4 main symbiotic relationships:

1. Mutualism

2. Commensalism

3. Parasitism

4. Competition

CHENNAI |SALEM| MADURAI | COIMBATORE DELHI | BANGALORE | THIRUVANANTHAPURAM


www.shankariasacademy.com 318
www.iasparliament.com

40. c  Russian backed rebels – In the 2 regions


of Donbass region, Donetsk and Luhansk
Super Pollutants regions in eastern Ukraine, they act as
independent republics which aggravated the
The Climate and Clean Air Conference 2024 in Nairobi, issue.
Kenya began with a call for international
collaboration to phase-out short-lived climate
pollutants.

 Super pollutants, also sometimes referred to


as short-lived climate pollutants (SCLPs),
derive their name from their relatively short life
in the atmosphere as compared to carbon
dioxide.

 Common super pollutants - Black


Carbon, Methane. Hydrofluorocarbons,
and Ground-level ozone.

 Nitrogen dioxide (NO2) is not a super 42. b


pollutant.
Interpol Notice system

Recently concerns have been raised about the misuse of


Interpol’s notice system, especially the issuance of blue
corner notices, which are less scrutinized than their red
corner notices.

 Interpol – International Criminal Police


Organisation

 Interpol Notices – They are


international requests for cooperation or
alerts allowing police in member countries
to share critical crime-related
information.

 Most Notices are for police use only and are not
available to the public.

 However, an extract of the Notice can be


41. b published on this site if the requesting country
History of conflict between Russia and Ukraine wishes to alert the public or seek their help.

 All United Nations Special Notices are public.


 Crimea’s annexation – Russia took over
Crimea, a part of Ukraine, by force in 2014 as
 The notices are generally issued by the General
Ukraine wanted to join the European Union
Secretariat at the request of
which Russia sternly objected.
o A member country’s INTERPOL
 Sea of Azov clash – Post Crimea’s
National Central Bureau.
annexation, Russia and Ukraine fought over the
Sea of Azov.

 Russia built a bridge over the Kerch strait and


blocked Ukraine’s ships from passing through.

 Kerch Strait – It is a strait in Eastern Europe.

o It connects the Black Sea and the


Sea of Azov, separating the Kerch
Peninsula of Crimea in the west from
the Taman Peninsula of Russia's
Krasnodar Krai in the east.

CHENNAI |SALEM| MADURAI | COIMBATORE DELHI | BANGALORE | THIRUVANANTHAPURAM


www.shankariasacademy.com 319
www.iasparliament.com

o International Criminal Tribunals and  Indian elephant – Elephas maximus, occurs


the International Criminal Court in the central and southern Western Ghats,
North-east India, eastern India and northern
o The United Nations in relation to the India and in some parts of southern peninsular
implementation of sanctions imposed India.
by the Security Council.
 It occurs in 16 of the 28 states in the country.
Interpol
 Protection Status
 It is an inter-governmental body founded in
1923. o Schedule I of the Indian Wildlife
(Protection) Act, 1972
 Headquarters – Lyon, France.
o Appendix I of the CITES.
 Members – 196, India joined in 1949.
o IUCN – Endangered.
 Role – It provides investigative support,
expertise and training to law enforcement  Project Elephant – It was launched by the
worldwide, focusing on 3 major areas of Government of India in the year 1992 as a
transnational crime (Terrorism, cybercrime Centrally Sponsored Scheme with following
and organized crime). objectives:

 CBI – Central Bureau of Investigation is o To protect elephants, their habitat &


the national central bureau of India to liaison corridors
with Interpol.
o To address issues of man-animal
 Interpol UN Cooperation- INTERPOL has conflict
enjoyed a special role (Permanent Observer) at
the United Nations since 1996. o Welfare of captive elephants

43. b  The Ministry of Environment, Forest and


Climate Change provides the financial and
Minimum Selling Price (MSP) for Sugarcane technical support to major elephant range
states in the country through Project Elephant.
 MSP – It is the minimum price at which sugar
mills can sell sugar to the market. Elephant Reserves in India

 It was introduced by the Centre under  There are 33 elephant reserves in 14 major
the Sugar Price (Control) Order, 2018. elephant States.

 Aim – To ensure that the industry gets at least  They overlap with Tiger Reserves, Wildlife
the minimum cost of production of sugar. Sanctuaries and Reserved Forests which are
protected under Wild Life (Protection) Act,
 Calculation – By taking FRP and adding 1972, Indian Forest Act, 1927 and other local
minimum conversion cost incurred by sugar State Acts.
mills running at highest efficiency.

 It is announced to arrest the constant slide of


sugar and to keep the demand and supply ratio
to a safe limit.

 The Centre had also fixed mill-wise sales quota.

 The mills breaching either of the conditions


were liable for action under the Essential
Commodities Act, 1955.

 Fair and Remunerative Price – It is the


minimum price that sugar mills have to pay
to farmers for sugarcane.

44. a

Elephant Reserves in India

CHENNAI |SALEM| MADURAI | COIMBATORE DELHI | BANGALORE | THIRUVANANTHAPURAM


www.shankariasacademy.com 320
www.iasparliament.com

 Tamil Nadu and Assam has the  States are encouraged to inform the
maximum number of Elephant reserves (5) international community in advance about any
in India. space activities that could cause harmful
interference with the activities of other states.
45. b
 States are encouraged to consult with each
Outer Space Treaty (OST), 1967 other and coordinate their activities in outer
space to prevent harmful interference.
Recently US accused Russia of developing a space
based nuclear weapon that could destroy or damage 46. a
other satellites in the orbit.
Taj Trapezium Zone
 OST, often called as the magna carta of
space law, is formally known as the “Treaty Recently, the Supreme Court of India asked the
on Principles Governing the Activities of States Centrally Empowered Committee to find out an
in the Exploration and Use of Outer Space, alternative solution to cutting down 3,874 trees for the
including the Moon and Other Celestial construction of a road project in the Taj Trapezium
Bodies.” Zone.

 Adopted by – UN General Assembly in  A defined area of 10,400 sq km around the Taj


1967. Mahal, named so, as is located around the Taj
Mahal.
 India signed the treaty in
1967 and ratified in 1982.  Aim – To protect the monument from
pollution.
 The provisions of the treaty are binding on the
signatories.  Geographical spread – Over 5 districts
in Uttar Pradesh and 1 district
 Peaceful use of outer space – For peaceful in Rajasthan.
purposes, and the exploration of outer space
shall be carried out for the benefit of all.  Coverage – Over 40 protected monuments
including 3 World Heritage Sites – Taj
 Outer space including the Moon and other Mahal, Agra Fort and Fatehpur Sikri.
celestial bodies, is not subject to national
appropriation by any means.  Regulations – Bans on the use of coal/ coke
industries located in the TTZ and mandates for
 International cooperation in the exploration relocating them outside the TTZ.
and use of outer spac.

 Prohibition of weapons of mass destruction and


military bases on celestial bodies is prohibited.

 The exploration and use of outer space shall be


carried out for the benefit of all countries, and
there shall be free access to all areas of
celestial bodies.

 States are internationally liable for any


damage caused by their space activities to other
states or their space objects.

 The private entities and other non-government


organizations are required to avoid harmful
contamination of outer space and celestial
bodies.
47. a
 States are obligated to register space objects
launched into outer space with the UN Registry Paruveta Utsavam
of Objects Launched into Outer Space.
The Indian National Trust for Art and Cultural
 States are obligated to render assistance to Heritage (INTACH) is making efforts to secure
astronauts in distress and to return them safely UNESCO recognition for the annual 'Paruveta'
to Earth. festival as an 'intangible cultural heritage'.

CHENNAI |SALEM| MADURAI | COIMBATORE DELHI | BANGALORE | THIRUVANANTHAPURAM


www.shankariasacademy.com 321
www.iasparliament.com

 It is an annual mock hunting festival,  Native Speakers – Borok people


celebrated at Sri Narasimha Swamy temple in or Tripuris.
Ahobilam in Andhra Pradesh.
 It is spoken by 9 sub-tribe or clan of Borok
 Chenchu tribes consider Ahobila Narashima community.
as their brother-in-law and the tradition is
inviting him home on Makar Sankranti.  Lingua franca – For Tripura’s 19 tribal
communities.
 Festival – It starts with the tribals shooting 2
arrows at the palanquin, which is taken to the  Language family – Tibeto-Burman, also has
32 Chenchu tribal villages surrounding close affinity with Bodo, Garo, Dimasa etc.
Ahobilam town for a duration of 40 days.
 Written form – It started in 1897 as Doulot
 Unlike other Paruveta, Ahobilam Utsav is the Ahammad a Muslim scholar wrote the 1st
only tradition conducted for a ‘mandala’ (40 Kokborok Grammar ‘Kokboroma and Tripura –
days). Vyakaran Grammer’.

 The temple staff stay in these hamlets for the  Kokborok and Bengali are the official
entire period, indicating the presence of a languages of Tripura.
casteless society in the past, with no hint of
untouchability. Badaga tribes

 Tribal link – By ‘Guru Parampara’, the temple  It is a unique ethnic group in the state of Tamil
is governed by the 600-year-old Ahobila Mutt, Nadu.
which encouraged this tribal festival to spread
Srivaishnavism among the folk tribes.  Badaga language – It is a 2,300-year-old
Dravidian language spoken by the Badaga
48. b people in the Nilgiris bearing striking
similarities to Kannada.
Santali Language
 It is categorised as a “definitely endangered
Recently, a book ‘10 Indian Languages and How They language” by UNESCO.
Came to Be’ was published that traced the evolution,
transformation and development of Santali language. 49. c

 It is the predominant language of Santal tribes, Martial Arts


India’s largest tribal community.
Post the Galwan violent clash between India and
 Native speakers – Spread across 4 states China, the Army has decided to add martial arts to the
with about 44% in Jharkhand followed regular training of troops.
by West Bengal, Odisha and Bihar.
Martial Arts Description
 Also found in Nepal and Bangladesh.
 It is an effective, modern, and
 Family – Austro-Asiatic language. dynamic self-defence and fighting
system.
 Jeremiah Phillips, an American missionary, Krav Maga
devised a writing system for Santali using the  It was originally taught to
Bangla script. the Israeli Army, and
instruction for civilians began in
 Ol Chiki (Alchiki) script – It was devised by 70s.
Pandit Raghunath Murmu (Guru Gomke) in
1925 which used signs and symbols from  Developed in Kerala, it is
environment like hills, rivers, trees, plough, and considered among the oldest and
sickle. most scientific martial arts in the
world.
 Recognition – It was included in the 8th
Schedule in 2003 and is an additional official  It includes exercises to develop
Kalaripayattu
language in Jharkhand and West Bengal. sharp reflexes for unarmed combat
and skillful fight using sticks,
Kokborok language dagger, sword, shield, Urumi etc.

 The Tripura Board of Secondary Education  It is noted for its high-flying


(TBSE) has allowed both the Roman and acrobatics and the movements of
Bengali scripts for Kokborok exam in Tripura. Kalari are based on the

CHENNAI |SALEM| MADURAI | COIMBATORE DELHI | BANGALORE | THIRUVANANTHAPURAM


www.shankariasacademy.com 322
www.iasparliament.com

mannerisms and fighting 51. a


techniques of animals.
Global Mercury Partnership
 It is a style of stick fighting
between two or more practitioners, The United Nation's Global Mercury Partnership has
with wooden sticks (called Soti) raised concerns about the increasing mercury levels
intended to simulate swords. affecting the marine mammals.
Gatka
 It was one of basic technique of  Aim - To protect human health and the
self-defence for Sikh warriors environment from the releases of mercury to
during the martial period of great air, water and land.
Sikh Gurus in Punjab.
 Launched by - United Nations
 It is a Gurkha folk dance that Environment Programme (UNEP).
originated in the state of Sikkim.
 It consists of stakeholders from governments,
 It is a small knife which symbolises intergovernmental organizations, NGOs,
Khukuri private sector, academia and scientific
victory, robustness and power.
Dance community.
 The dance is also performed as a
tribute to soldiers for the security  There is no financial grant
they provide. provided under the Global Mercury
Partnership.
50. b
 It was initiated at the 23rd session of UNEP
Kaleshwaram Irrigation Project Governing Council/Global Ministerial
Environment Forum in 2005.
 Kaleshwaram is a town on Godavari right bank
in Telangana, about 300 km downstream of  It establishes a Partnership Advisory
major dam, Sriram Sagar project (SRSP). Group, which plays a key role in guiding and
encouraging the work of the Partnership and its
 At Kaleshwaram, the major areas of work.
tributary Pranahita, which merges with
Godavari brings large quantity of water, the 52. c
place is also known as Sangam and Dakshin
Genome India Project
Ganga.
The Department of Biotechnology (DBT) said that the
 The project proposes to lift about 180 TMC
exercise to sequence 10,000 Indian human genomes
(about 5 billion cubic meters) of water.
and create a database under the Centre-backed
Genome India Project is about two-thirds complete.
 Besides, it involves constructions of network
storage dams, tunnels, canals water passing
 Genome – The human genome is the entire
through several stages of pumping and reverse
set of deoxyribonucleic acid (DNA) residing in
flow into Godavari River.
the nucleus of every cell of each human body.
Nokhra Solar Project
 It carries the complete genetic information
 The project is located in Bikaner district responsible for the development and
of Rajasthan. functioning of the organism.

 The project works under the aegis of  Genome sequencing - The process
the Ministry of New and Renewable Energy. of deciphering the order of base pairs, to
decode the genetic fingerprint of a human is
Kiru Hydel Project called genome sequencing.

 It is a run-of-river scheme on the Chenab River  While the sequence or order of base pairs is
in Jammu and Kashmir to be completed in identical in all humans, compared to that of
2025. other species, there are differences in the
genome of every human being that makes them
 It is being constructed between the existing unique.
Kirthai II and Kwar hydel power projects.
Applications
 It will be developed near the convergence point
of river Chenab and the Singad and Bela  Evaluate rare disorders, preconditions for
streams. disorders and cancer.

CHENNAI |SALEM| MADURAI | COIMBATORE DELHI | BANGALORE | THIRUVANANTHAPURAM


www.shankariasacademy.com 323
www.iasparliament.com

 Tool for prenatal screening, to investigate 53. b


whether the foetus has genetic disorders or
anomalies. Unified Payments Interface (UPI)

 Liquid biopsies, where a small amount of blood  It is a system that powers multiple bank
is examined for DNA markers. accounts into a single mobile
application.
Human Genome Project (HGP)
 It is a real-time payment system that helps in
 Carried out from 1990–2003, the Human instant and quick transfer of funds between two
Genome Project was a landmark global bank accounts.
scientific effort to generate the first sequence of
the human genome.  It also caters to the “Peer to Peer” collect
request which can be scheduled and paid as per
 The first results of the complete human genome requirement and convenience.
sequence were given in 2003, while the HGP
released the latest version of the complete  The fund transfer through UPI is faster than
human genome in 2023, with a 0.3% error National Electronic Fund Transfer
margin. (NEFT).

Genome India Project (GIP)  The idea of UPI was developed by the National
Payments Corporation of India (NPCI) and is
 Goal – To collect 10,000 genetic samples from controlled by the Reserve Bank of India
citizens across India, to build a reference (RBI) and the IBA (Indian Bank
genome grid. Association).

 It is based on Next-Generation Sequencing 54. c


(NGS) platform.
Kamakhya temple
 The basic next-generation sequencing process
involves fragmenting DNA/RNA into multiple Recently Prime Minister Inaugurated the Kamakhya
pieces, adding adapters, sequencing the temple corridor project in Guwahati, Assam.
libraries, and reassembling them to form a
genomic sequence.  The Kamakhya temple is located on
the Nilachal hills, at a distance of 7 km from
 Launch - In 2020; Partner organisations Guwahati, Assam.
- 20
 It is one of the biggest Shakti shrines in the
 Initiative by - Indian Institute of Science’s country and an important pilgrimage centre for
(IISC) Centre for Brain Research. tantric worshippers and Hindus.

 Funded by – Department of  It is also considered important as it is the


Biotechnology. temple where the beliefs and practices of the
Aryan communities coincide with non-Aryan
communities.

 It was reconstructed in 1565 by Chilarai,


who was the reigning king of the Koch
dynasty.

 This temple is dedicated to different forms


of Mother Shakti namely Sundari, Tripura,
Tara, Bhuvaneshvari, Bagalamukhi and
Chinnamasta.

 Ambubachi Mela is one of the major festivals


of this temple.

 The festival is held every year to commemorate


the yearly menstruation of Goddess Kamakhya.

 It is also said that during the month of mid-


June, which is also an Ahar, there is a natural
spring which flows through the yoni.

CHENNAI |SALEM| MADURAI | COIMBATORE DELHI | BANGALORE | THIRUVANANTHAPURAM


www.shankariasacademy.com 324
www.iasparliament.com

 It is also dedicated to different forms of Lord 56. c


Shiva, there are 5 temples within the complex of
Kamakhya temple. MakhaBucha

 In addition, the temple complex also houses MakhaBucha Ceremony was organized at
three temples of Lord Vishnu, present in the SanamLuang Pavilion that houses holy relics from
form of Kedara, Gadadhara and Pandunath. India.

55. c  The MakhaBucha (Magh Puja) ceremony is one


of the 5 most revered events for Buddhists in
Central Asian Flyway (CAF) Thailand.

Central Asian Flyway recognised to protect over 600  It was conducted by the esteemed Somdet and
migratory bird species. other senior monks of Thailand.

 A flyway is a geographical region within which  MakhaBucha Day (Magha Puja) is a religious
a single or a group of migratory species celebration marking Lord Buddha’s teachings
completes its annual cycle i.e. breeding, to his disciples.
moulting, staging and non-breeding.
 The term makha comes from the word Magha
 The Central Asian Flyway (CAF) covers a large in Pali and it refers to the 3rd lunar month,
continental area of Eurasia between the Arctic while bucha can translate as to worship, both of
and Indian Oceans and the associated island which are derived from the Pali language used
chains. in Buddhist scripture.

 CAF extends from the northernmost breeding  The term MakhaBucha refers to a day intended
grounds in Siberia to the southernmost non- for worshiping on the 3rd lunar month.
breeding (wintering) grounds in West and
South Asia, the Maldives and the British Indian  It takes place in nations including Thailand,
Ocean Territory. Laos and Cambodia, where the majority of
Buddhists practice Theravada Buddhism, also
 The Central Asian Flyway is the shortest known as Buddhism of the south.
flyway in the world.
57. c
 Lying entirely within the Northern
Hemisphere, it connects a large swathe of the Micellar Water
Palaearctic with the Indian subcontinent.
 Micellar water is a product that is commonly
 370 species of migratory bird from 3 flyways used to remove make-up.
(CAF, East Asian Australasian Flyway and
Asian East African Flyway) are reported to visit  It’s a very effective cleanser and many
the Indian sub-continent during their annual people use in their skincare routine.
cycle.
 Micellar water products contain micelles, a
 India is one among the countries in Central cluster of molecules that are very effective at
Asian Flyway. removing oily substances.

 The micelles in micellar water are formed by


special molecules known as surfactants.
Surfactant stands for surface active agent.

 Surfactants can be found in dishwashing


detergent, body wash, shampoo, toothpaste and
even many foods.

 In all of these cases, they are there to help the


water interact with the dirt and oils and
micellar water is no different.

 Black water – An alkaline solution that is


infused with fulvic acid and contain several
nutrients.

 Grey water – Domestic wastewater with less


nitrogen & phosphorus than black water.

CHENNAI |SALEM| MADURAI | COIMBATORE DELHI | BANGALORE | THIRUVANANTHAPURAM


www.shankariasacademy.com 325
www.iasparliament.com

 Alkaline water – A water that has a higher pH 59. b


level than that of plain tap water and can
neutralize acid in your bloodstream. Kala Azar (visceral leishmaniasis)

58. d  It is a parasitic infection transmitted


by sandflies.
Indian Skimmer (Rynchops albicollis)
 Symptoms – It causes fever, weight loss,
India and Bangladesh submitted the joint proposal spleen and liver enlargement. Left untreated, it
to list Indian skimmer under the Convention on can be fatal in 95% of cases.
Migratory Species (CMS) of Wild Animals at the
14th COP meeting of CMS.  Measures taken by India – Government of
India launched a centrally sponsored Kala-azar
 Indian skimmer – It can ‘skim’ over water to control Programme in 1990-91.
snap up fish, is characterised by a bright orange
bill.  The National Health Policy (2002)
originally set a target of 2010, and later
 Habitat – It is almost completely revised it to 2015.
restricted to India as a breeding bird,
with only occasional breeding in western  Now India's Kala-Azar Elimination
Bangladesh. Programme aims to eliminate the disease by
2023.
 It occurs primarily on larger, sandy,
lowland rivers, around lakes and  In October 2023, Bangladesh became the
adjacent marshes and in the non-breeding first country in the world to be officially
season, estuaries and coasts. validated by the WHO for eliminating
Kala Azar as a public health problem.
 It can be found in the coastal estuaries of
western and eastern India.  Elimination for Kala Azar is defined as no block
in the country reporting more than one case per
 Tree line - The tree line is the edge of a habitat 10,000 people.
at which trees are capable of growing and
beyond which they are not. It is found at high  India is yet to eliminate the Kala-Azar disease
elevations and high latitudes. Beyond the tree officially.
line, trees cannot tolerate the environmental
conditions.  PKDL or Post Kala-azar dermal leishmaniasis,
is a well-recognised complication which causes
 The recent population decline of Indian scaly skin patches and nodular lesions in people
skimmer has been accompanied by a range who have been apparently cured of Kala Azar up
contraction and there are no recent confirmed to 2 years of the initial infection.
breeding records from Pakistan or Myanmar.
Nesting success is currently very low. 60. b

 Distribution – In India the species remains IBSA Fund


widely distributed, but breeding areas are now
highly restricted along the rivers such as Recently, India has contributed 1 million USD to IBSA
Chambal, Ganges, Yamuna, Mahanadi and Fund.
Son.
 IBSA Fund – India, Brazil and South Africa
 They breed between February to June and raise Facility for Poverty and Hunger Alleviation
one to three chicks per clutch. Fund.

Conservation status  Created in – 2004, but became operation in


2006.
 IUCN Red List - Endangered
 Established by – India, Brazil and South
 WPA 1972 - Schedule I Africa, commonly called as IBSA countries.

 It is not listed under the Convention on  Aim – To identify replicable and


International Trade in Endangered scalable projects that can be
Species of Wild Fauna and Flora disseminated to interested developing
(CITES). countries as examples of good practices in
the fight against poverty and hunger.

CHENNAI |SALEM| MADURAI | COIMBATORE DELHI | BANGALORE | THIRUVANANTHAPURAM


www.shankariasacademy.com 326
www.iasparliament.com

 To support for Southern-led, demand-driven, a wide range of Avionics Systems for diversified
transformational projects in developing defense and aerospace applications.
countries.
62. d
 Fund Manager and Secretariat –United
Nations Office for South-South Cooperation Planum Boreum
(UNOSSC), established by UN General
Assembly in 1974 and hosted by UN Recently Mars Express, a European space craft
Development Programme (UNDP). captured massive dunes colliding with several layers of
dusty ice covering the north pole of Mars.
 Fund – Each of the IBSA countries
contribute 1 million USD annually.  The Northern Pole of Mars is known as
Planum Boreum.
 India has contributed more than 18 million
USD in total.  It is covered in layers of fine dust and water ice
that are several kilometers thick and stretch out
 Projects – Through partnerships with for around 1000 kilometers.
local governments, national
institutions and implementing  It was formed as a mix of dust, water ice and
partners. frost settled on the Martian ground over time.

 Fund allocation – 50.6 million USD to  In Martian winter, the layers are topped by a
date, supporting 45 projects across 37 thin cap of dry ice (carbon dioxide ice) a couple
countries of the global South. of meters thick.

61. d  This cap completely disappears into the


atmosphere each Martian summer.
Very Short-Range Air Defence System
(VSHORADS) Missile  Mars Express of European space agency
(ESA) – It has been orbiting the Red Planet
Recently the DRDO conducted two successful flight since 2003.
tests of Very Short-Range Air Defense System
(VSHORADS) missiles off the coast of Odisha from  It is imaging Mars’s surface, mapping its
integrated test range. minerals, identifying the composition and
circulation of its tenuous atmosphere, probing
 VSHORADS – Very Short- beneath its crust.
Range Air Defence System.
63. b
 The VSHORADS is a Man Portable Air Defence
System (MANPAD) indigenously Agalega Island
developed by Research Centre Imarat (RCI)
in collaboration with other DRDO Recently Prime Minister Narendra Modi attended the
inauguration ceremony of several India-assisted
laboratories and other industry partners.
development projects at the Agalega Island in
 It is designed to destroy hostile aircraft, drones Mauritius.
and helicopters at very short ranges. The
 Agalega Islands is a two-island dependency of
range of the missile is up-to 6km.
Mauritius, in the Western Indian Ocean.
 The design of the missile including launcher has
 It consists of North Island and South Island.
been highly optimized to ensure easy
portability.
 Its latitudinal position is between Maldives
 The missile incorporates many novel and Mauritius.
technologies including miniaturized Reaction
 It is located in the southern hemisphere to the
Control System (RCS) and integrated avionics.
north of Mauritius and east to the Madagascar.
 Reaction control system (RCS) – It is a
 It is located south to Seychelles Latitudinally.
spacecraft system that uses thrusters to control
attitude and translation. It can also provide
 India - India signed a Memorandum of
torque to control rotation.
Understanding for the improvement in sea and
air transportation facilities at Agalega Island in
 Research Centre Imarat (RCI) - It is a
2015.
premier laboratory of Dr APJ Abdul Kalam
Missile Complex, DRDO spearheading R&D in

CHENNAI |SALEM| MADURAI | COIMBATORE DELHI | BANGALORE | THIRUVANANTHAPURAM


www.shankariasacademy.com 327
www.iasparliament.com

 Recently Prime Minister Narendra Modi  Gallium – Semiconductors, Integrated


inaugurated the India-built airstrip in the Circuits and LEDs.
island.
 Germanium – Optical fibres, satellites,
 The airstrip contributes to the India’s strategic solar cells.
significance in the Indian Ocean.
 Graphite – Batteries, Lubricants, fuel cells for
EVs, Electric Vehicle.

 Hafnium – Super alloy, Catalyst precursor,


Semiconductors, Oxide for Optical and Nuclear
reactors.

 Indium – Electronics (Laptops, LED


Monitors/TVs, Smartphones), and semi-
conductors.

 Lithium – Electric Vehicle, Batteries,


glassware, ceramics, fuel manufacturing and
Lubricant.

 Molybdenum – Steel alloys, Pigment and


Dyes, Catalyst, Electrical and Electronic.

 Niobium – Construction, transportation

 Nickel – Stainless steel, solar panels, batteries,


aerospace, defence applications and Electric
Vehicle.

 Rhenium – Super-alloys, aerospace and


machinery uses, Catalysts in petroleum
industry.

 Tellurium – Solar power, thermoelectric


devices and Rubber vulcanising.

65. a

BHISHM Cube Initiative


64. d
The Indigenous Mobile Hospital (BHISHM) or Arogya
Critical Minerals Maitri Cube was recently deployed at the Pran
Pratishtha ceremony in Ayodhya to mitigate any
 Critical minerals are elements that are the unexpected disaster events.
building blocks of essential modern-day
technologies. To know more about the critical  BHISHM Cube is an Arogya Maitri Disaster
minerals click here. Management Cube.

Major applications of Critical Minerals  It is the world’s first portable disaster


hospital.
 Bismuth – Chemicals, Pharmaceuticals and
Casting of Iron.  This cube is a part of the broader initiative
named “Project BHISHM” – Bharat
 Cadmium – Batteries, Pigments and Health Initiative for Sahyog, Hita and
Coatings. Maitri.
 Cobalt – Electric Vehicle, Batteries, corrosion  Arogya Maitri Cube is equipped with cutting-
resistant alloys, aerospace applications and edge technology which is tailored to treat up to
Pigments and Dyes. 200 casualties, emphasizing rapid response
and comprehensive care.
 Copper – Electrical and electronics products,
Electrical Wiring, Solar Panel and Automotive  Features – Arogya Maitri Cube is a completely
industry. made-in-India and each cube cage has three
frames.

CHENNAI |SALEM| MADURAI | COIMBATORE DELHI | BANGALORE | THIRUVANANTHAPURAM


www.shankariasacademy.com 328
www.iasparliament.com

 It consists of mini cubes and master cubes.

 These cubes are robust, waterproof, and light,


designed for various configurations, making
them ideal for diverse emergency scenarios.

 India is already supplying the BHISHM Cube to


neighbouring nations like Sri Lanka, Myanmar
and Bangladesh.

66. b

Hangul (Cervus elaphus hanglu)

 Hangul - Also known as the Kashmir stag

 It is a subspecies of the Central Asian red


deer.

 They are, by nature, high-altitude forest


dwellers.
67. b
 The hangul is the only surviving Asiatic
member of the red deer family and Swachhata Green Leaf Rating (SGLR) Initiative
the state animal of Jammu and Kashmir.
The Swachhata Green Leaf Rating initiative to boost
 The Hangul was widely found in the hilly and tourism hygiene was launched recently.
mountainous regions of Kashmir in the early
 Aim - To improve sanitation practices in
20th century with their numbers estimated to
India's tourism sector.
be around 5000.
 It aids to develop an economically viable,
 Threats – Due to hunting and encroachment
responsible, and resilient tourism industry.
of their natural habitat, the number fell to just
around 150 in 1970.
 Launched by - The Department of Drinking
Water & Sanitation (DDWS), Ministry of Jal
 Multiple disturbances, including stress on
Shakti, in collaboration with the Ministry of
forest areas and human intervention, disturb
Tourism.
the habitat of hangul.
 The Rating is not just a recognition but a
 Distribution – Their distribution had earlier
commitment to shaping the future of our
spanned over Dachigam National Park and its
tourism industry.
nearby areas, Rajparian Wildlife Sanctuary,
Overa Aru, Sind Valley, and the forests of
 The SGLR initiative seamlessly aligns with the
Kishtwar and Bhaderwah.
Travel for LiFE (TFL) program under Mission
LiFE, fostering awareness about sustainable
 But now, they are mostly spotted in the
tourism.
protected areas of Dachigam National Park and
Tral Wildlife Sanctuary in Jammu and
 The Bison Resort in Madhya Pradesh's
Kashmir.
Narmadapuram has emerged as a pioneer by
securing the inaugural Five Swachhata Green
 Population trend – Since 1947, the
Leaf Rating Certificate of Recognition.
population of hangul witnessed a sharp decline.
68. d
 According to Wildlife department, the hangul
population since 2015 the population of hangul ADITI Scheme
is steadily increasing and in 2021 it was
261. Recently the Ministry of Defence launched the ADITI
Scheme to promote innovations in critical and
Conservation status strategic defence technologies.
 IUCN – Critically Endangered  ADITI – Acing Development
of Innovative Technologies with iDEX.
 CITES – Appendix I
 Lunched - During the DefConnect 2024.
 WPA, 1972 – Schedule I

CHENNAI |SALEM| MADURAI | COIMBATORE DELHI | BANGALORE | THIRUVANANTHAPURAM


www.shankariasacademy.com 329
www.iasparliament.com

 Aim - To develop about 30 deep-tech northern and central parts of the Red Sea are
critical and strategic technologies also distinctive features of the region.

 It also envisages to create a ‘Technology Watch  Red Sea is among the top areas of high fish
Tool’ to bridge the gap between the endemism in the world.
expectations and requirements of the modern
Armed Forces and the capabilities of the  Endemism is a term used in biology to talk
defence innovation ecosystem. about the distribution of a taxon limited to a
small geographic area and which can therefore
 Grants - Under the scheme, start-ups are be found naturally in this place.
eligible to receive grant-in-aid of up to 50% of
product development budget (PDB)  Malpelo is a small oceanic island in the eastern
with maximum limit of Rs. 25 Crore for Pacific Ocean, located west of the Colombian
their research, development, and innovation mainland.
endeavours in defence technology.
70. d
 The scheme will nurture the innovation of
youth, and help the country leap forward in the Orans
field of technology.
Following the orders of the Supreme Court, the
 Control - It falls under the iDEX (Innovations Rajasthan state government recently released a
for Defence Excellence) framework of notification to classify Orans (sacred groves) as
Department of Defence Production deemed forests.
(DDP), Ministry of Defence.
 Orans are sacred groves in Rajasthan.
Eligibility Criteria for ADITI
 These are lands where herders have been
 Startups, as defined and recognized by the grazing their cows, sheep, and goats for
Department for Promotion of Industry and centuries.
Internal Trade (DPIIT), Ministry of
Commerce and Industry.  These lands usually features a temple and a
waterbody, these community-conserved
 Any Indian company incorporated under pastures are rich in biodiversity.
the Companies Act 1956/2013, primarily a
Micro, Small and Medium Enterprises (MSME)  Significance – The Rajasthan orans
as defined in the MSME Act, 2006. are home to the endangered great Indian
bustard (GIB), chinkara and the Indian
 Individual innovators are also encouraged desert fox.
to apply (research & academic institutions can
use this category to apply).  They are oases of traditional flora and fauna.

 However, post-selection individual innovators  Named after deities, they are considered sacred
need to register as Startup/MSME. by the local people and protected as such. The
richest biodiversity in the deserts is in the
69. d orans.

Red Sea and Gulf of Aden (RSGA) Ecosystems  Threats – Orans are sought to be used for
generating “green energy”, solar in particular.
 The Red Sea and Gulf of Aden (RSGA)
ecosystems supports a rich biological diversity  Many of the orans, are classified as “wasteland”
with a distinct high proportion of endemic in revenue records.
species.
 White cheeked Macaque - It is present in
 The RSGA ecosystems is home to the remote Anjaw district in central Arunachal
three UNESCO Natural Heritage Sites Pradesh. (Not in Orans).

o Socotra Archipelago 71. a

o Dungonab Bay and Gas Exporting Countries Forum (GECF)

o Senganeeb Atoll Recently the 7th summit of the Gas Exporting Countries
Forum was held at the historic city of Algiers, Algeria.
 Farsan and Dahlak Archipelagos in the
southern Red Sea and coral reefs in the  GECF – It is an intergovernmental
organisation that provides a framework for

CHENNAI |SALEM| MADURAI | COIMBATORE DELHI | BANGALORE | THIRUVANANTHAPURAM


www.shankariasacademy.com 330
www.iasparliament.com

exchanging experience and information among Orunodoi Scheme – Assam


its member countries.
 A monthly assistance of Rs. 830 is transferred
 It seeks to build a mechanism for a meaningful to women members of marginalised families of
dialogue between gas producers and consumers Assam.
in order to improve the stability and security of
supply and demand in gas markets around the  A Direct Benefit Transfer scheme, the money is
world. credited directly to the bank account of the
woman head of a family.
 Significance – Together, they represent 69%
of the world’s gas reserves, 39% of the marketed Indira Gandhi Pyari Behna Sukh Samman Nidhi
production, and 40% of global gas exports. Yojana – Himachal Pradesh

 GECF member countries collectively account  To give Rs. 1500 per month to women from 18-
for more than half of the world LNG exports 60 years of age in the state.
(51%).
Gruha Lakshmi scheme – Karnataka
 Algiers Declaration - The central focus of the
7th summit, articulated through the “Algiers  To provide financial assistance of Rs.2,000
Declaration” is to reaffirm the commitment of every month to eligible women.
member countries to the responsible
stewardship of natural gas resources.  Women government employees are not eligible
to get the scheme’s benefits.
 Full-fledged Members – 12 including UAE,
Russia, Iran, Venezuela, Egypt, etc. Mukhyamantri Ladli Bahna Yojana – Madhya
Pradesh
 India, Iraq and USA are not full-fledged
members of GECF.  Women in 23-60 age group will be given Rs
1,000 per month subject to certain conditions,
72. b including that they are not income tax payees
and their families’ annual income is below Rs
IRIS 2.5 lakh.

A school in Thiruvananthapuram, Kerala, recently Mahalakshmi Scheme – Telangana


introduced AI teacher, IRIS which is the first humanoid
robot teacher in INDIA.  Offers women and transgender persons free
travel on state-run non-AC buses.
 IRIS is India’s first generative Artificial
Intelligence teacher.  The scheme has two more components that
includes:
 It is an initiative by the Kaduvayil Thangal
Charitable Trust (KTCT) Higher Secondary  Financial assistance of Rs 2,500 to eligible
School in Thiruvananthapuram, Kerala. women

 IRIS is part of the Atal Tinkering Lab (ATL)  An LPG cylinder at Rs 500.
project, a 2021 NITI Aayog initiative designed
to boost extracurricular activities in schools. Lakshmir Bhandar Scheme – West Bengal

 Atal Tinkering Lab (ATL) is a program that  The scheme received the Skoch award.
aims to encourage innovation and experiential
learning among school children.  Goal – Financial empowerment for women
(25-60 years old) from economically
73. d disadvantaged backgrounds.

Women Schemes in India  Eligibility – Enrolled in Swasthya Sathi


program.
Jagananna Ammavodi Scheme – Andhra
Pradesh  Amount – Rs 1,000 for women from SC/ST
households. Rs500 for women from other
 Provides Rs. 15,000 annually to mothers or households.
guardians from poor families (below poverty
line) to support their children's education (from Vidiyal Payanam Scheme – Tamil Nadu
Class 1 to 12) in any government, aided, or
unaided school/junior college in the state.  Free bus travel scheme for women.

CHENNAI |SALEM| MADURAI | COIMBATORE DELHI | BANGALORE | THIRUVANANTHAPURAM


www.shankariasacademy.com 331
www.iasparliament.com

74. a  Thuvayal Panthy programme – To teach


vegetarianism and discipline to followers, who
Ammonium Phosphate Sulphate spread these teachings across Tamil Nadu.
Recently Rubymar, a cargo ship that was carrying  Nizhal Thangals – A community worship
21,000 tonnes of ammonium phosphate sulphate was spaces, which did not have any idol or deity and
sunk due to the attack by the Houthi missiles. only Tamil was used for worship.
 Ammonium phosphate sulphate is  These community worship centres also had
a fertilizer made by neutralizing sulphuric community kitchens and even basic schools.
acid and phosphoric acid with ammonia.
 Samapanthi-bhojana or community
 It's a complex fertilizer that contains nitrogen, eateries – For people from all backgrounds.
phosphorus, and sulphur. He would send his disciples to the homes of
lower castes to eat with them.
 Urea is the most important nitrogenous
fertilizer in the country because of its high N  Thottu Namam – Inspired priests to apply
content (46%N). the sacred paste on devotees’ forehead,
irrespective of their caste.
 Urea is a white crystalline organic chemical
compound with neutral pH.  The paste would be applied in the form of a
lamp, indicating the soul and God, representing
 DAP is the 2nd most commonly used fertiliser the form of God inside every life.
in India after urea and farmers normally apply
this fertiliser just before or at the beginning of 76. d
sowing.
Golden Langur (Trachypithecus geei)
 Impact on Environment – Ammonium
phosphate sulphate is a highly soluble fertilizer. Recently, the Primate Research Centre NE India
(PRCNE) and few other organisations in their latest
 Its release into the Red Sea could stimulate the survey of the primate indicated that an estimate of 7,
growth of algae, using up oxygen that other 396 golden langurs live in India.
marine life depends upon.
 The golden langur, also known as the golden
 It could harm fish and living organisms such as leaf monkey, is an endangered primates
coral reefs and seaweed. endemic to north-western Assam and
southern Bhutan.
75. b
 In India, the entire
Ayya Vaikundar distribution range of
the golden langur
Recently Indian Prime Minister paid tributes to Ayya covers the Manas
Vaikundar on his birth anniversary (March 3). Biosphere
Reserve and all
 Ayya Vaikundar, born in 1809, is revered as a fragmented forests
social reformer and the founder of the in the western part of
Ayyavazhi sect, primarily in southern Tamil Assam.
Nadu.
 They are arboreal and diurnal creatures.
 His teachings focused on equality, fraternity,
and the eradication of caste-based  It is listed as Endangered in the International
discrimination, challenging the established Union for Conservation of Nature (IUCN) red
religious and social hierarchies of the time. list.
 He encouraged all devotees to wear turbans  Fossorial are species that are adapted
and dhotis, promoting equality. to burrowing which lives primarily
underground.
 He pioneered education for the lower castes
and opposed discriminatory taxes. 77. c
 One of his significant interventions was the Pushpak
introduction of simplified, inclusive marriage
customs without a Brahmin priest or Sanskrit The Indian Space Research Organisation is all set to
mantras. conduct the second landing test of the Reusable Launch
Vehicle, Pushpak.

CHENNAI |SALEM| MADURAI | COIMBATORE DELHI | BANGALORE | THIRUVANANTHAPURAM


www.shankariasacademy.com 332
www.iasparliament.com

 Pushpak is the reusable launch vehicle  Raghanesda Solar Power Station – It is


of Indian Space Research Organisation one of the largest power stations located
(ISRO). in Gujarat.

 The vehicle is designed as an all-rocket, fully  The power generated by the Raghanesda
reusable single-stage-to-orbit (SSTO) Solar Power Station will be supplied to
vehicle. Gujarat Urja Vikas Nigam Limited (GUVNL)
for 25 years.
 The reusable launch vehicle promises to
revolutionise the sector by slashing launch 80. c
costs by nearly 80%, thereby making space
exploration more accessible and sustainable. Gulf of Mannar Biosphere Reserve (GoMBR)

 The winged RLV prototype serves as a versatile  Gulf of Mannar Biosphere Reserve (GoMBR) is
test platform, facilitating the evaluation of located in the south-eastern coastline of India
hypersonic flight, autonomous landing, and extending between Thoothukudi and
powered cruise capabilities. Ramanathapuram districts of Tamil Nadu.

78. c  It came into existence on February 1989, by a


joint declaration of the Government of India
Kasarkod Tonka and the Government of Tamil Nadu.

Recently, 700 eggs of sea turtles were destroyed by  It has been recognized by the Man and
feral dogs in Kasarkod Tonka, Karnataka. Biosphere (MAB) Programme of UNESCO.

 Kasarkod Tonka is one of the important nesting  GoMBR is the first marine biosphere reserve in
sites for the sea turtle including olive ridley south & south-east Asia.
turtle located in the Uttara Kannada district
of Karnataka.  Supports more than 1,10,000 families from 268
fishing villages.
 In Tonka, there are 36 plus nesting sites, of
which seven have been destroyed by stray dogs.  It is an International bird and biodiversity area
and part of central asian bird migratory flyway.
 Dogs are natural predators of the turtle eggs
that are laid on the beaches.  The Reserve is also home to several globally
important and highly threatened species like
 Kasarkod Beach has received Blue Flag the Dugong, whale shark, sea horses,
certification. Balanoglossus, green sea turtle, Hawksbill
turtle, dolphins, sacred chanks, etc.
79. c
81. a
Catalhoyuk, Nandankanan & Raghanesda
Elections in Rajya Sabha
 Çatalhoyuk – A Neolithic site located in
Turkiye (Turkey). The recent Rajya Sabha election in several States
raised concerns about the sanctity of election process
 Recently the ‘oldest bread’ in the world was as it witnessed cross-voting by MLAs.
discovered in this site.
 As per Article 80, the members of Rajya Sabha
 It was declared a UNESCO World Heritage Site elected indirectly by the elected members of the
in 2012. Legislative Assembly of each State and the
respective Union Territories.
 Nandankanan Zoological Park
(NZP) – Nandankanan is the first zoo in  Cross voting – It occurs when members vote
the World to breed White tiger and Melanistic for a candidate from a party different from the
tiger. one to which they belong which questions the
integrity of election process.
 It is located in Odisha, India.
 Open ballot system – To prevent cross-
 It is the only conservation breeding centre of voting, an amendment to the Representation of
Indian Pangolins in the world. the People Act, 1951 in 2003 introduced an
open ballot system for Rajya Sabha elections.
 First zoo in India to become a member of the
World Association of Zoos and Aquariums  Members are required to show their ballot
(WAZA). paper to the authorized agent of their party.

CHENNAI |SALEM| MADURAI | COIMBATORE DELHI | BANGALORE | THIRUVANANTHAPURAM


www.shankariasacademy.com 333
www.iasparliament.com

Failure to do so may result in disqualification of  Elimination of all trade-distorting domestic


the vote. subsidies

 Kuldip Nayar versus Union of India  Elimination of export subsidies and clear rules
(2006) – The Supreme Court upheld the to prevent circumvention of export subsidy
system of open ballot for Rajya Sabha commitments.
elections.
 Member countries – Argentina,
 The court is of the view that if secrecy in voting Australia, Brazil, Canada, Chile, Colombia,
became a source of corruption, transparency Costa Rica, Guatemala, Indonesia, Malaysia,
had the potential to eliminate it. New Zealand, Pakistan, Paraguay, Peru, the
Philippines, South Africa, Thailand, Ukraine,
 However, the Election Commission specified Uruguay and Vietnam.
that the provisions of the Tenth Schedule, with
respect to voting against the instruction of the  India is not a member of Cairns Group.
party, will not be applicable for a Rajya
Sabha elections.

 The elections to Rajya Sabha are not treated


as a proceeding within the Legislative
Assembly.

 Furthermore, political parties cannot issue any


‘whip’ to its members for such elections.

82. d

World Trade Organization’s types of subsidies


84. c

Three stages of India’s nuclear program

 Formulation – By Homi Bhabha who is


credited as “father of India’s nuclear program”.

 Launch year – 1954.

 Aim – To achieve complete self-sufficiency in


nuclear energy by leveraging India's significant
thorium resources.

 The ultimate goal was to capitalize on


India’s vase thorium reserves while
accounting for its low uranium reserves.

 India has only about 2% of the global uranium


reserves but 25% of the world’s thorium
reserves which is found in the monazite sands
of coastal regions of South India.

83. c  Closed fuel cycle – The three stages feed into


each other in such a way that the spent fuel
Cairns Group generated from one stage of the cycle is
reprocessed and used in the next stage of the
 Cairns Group is a coalition of 20 agricultural cycle to produce power.
fair-trading countries which together account
for almost 30% of the world’s agricultural  The closed fuel cycle helps to breed fuel and
exports. minimise the nuclear waste.

 It was launched in 1986 at Cairns,  Thorium – It is viewed as the fuel of the


Australia just before the beginning of the future, Stage-III aims at using thorium as fuel
Uruguay Round. for power generation on a commercial scale.

 Vision – Deep cuts to all tariffs including tariff


peaks and removal of tariff escalation

CHENNAI |SALEM| MADURAI | COIMBATORE DELHI | BANGALORE | THIRUVANANTHAPURAM


www.shankariasacademy.com 334
www.iasparliament.com

Types of nuclear Low altitude,


Stages By-Product Traject High altitude, parabolic tr
reactor level trajectory —
ory ajectory — hard to detect
hard to detect
Pressurized Heavy
Stage-I Water Reactors Plutonium-239 High, up to a few
Low precision, roughly a
(PHWRs Precisio metres — fit
few 100 m — fit for larger,
n for small,
Stage- Energy,Uranium-233 stationary targets
Fast Breeder Reactor moving targets
II and Plutonium-239
Subsonic (Mach Can hit targets at >25,000
Advanced Heavy 5) — slower than km/h or >Mach 20 — very
Stage- Energy and Uranium-
Water Reactors Speed ballistic missiles, fast, extremely hard to
III 233
(AHWR) possible to intercept even with state of
intercept art technology
85. a
87. c
Prototype Fast Breeder Reactor (PFBR)
Missile Technology Control Regime (MTCR)
The vital second stage of India’s three-stage nuclear
programme got a boost with the commencement of  Aim – To limit the spread of ballistic missiles
‘core loading’ at the country’s first indigenous Fast and other unmanned delivery systems that
Breeder Reactor (FBR) at Kalpakkam, Tamil could be used for chemical, biological, and
Nadu. nuclear attacks.
 Prototype Fast Breeder Reactor (PFBR) -  Launch year – 1987.
It is the vital 2nd stage of India’s 3-stage
nuclear programme.  Establishment – By the G-7 industrialized
countries (Canada, France, Germany, Italy,
 Core loading - It is the process of placing Japan, the UK, and the United States).
nuclear fuel assemblies inside the core of a
nuclear reactor.  Member countries – 35.

 The Fast Breeder Reactor (FBR) will initially  India – It is a part of the regime since
use the Uranium-Plutonium Mixed Oxide 2016.
(MOX) fuel.
 Restrictions – The regime urges the member
 PFBR is an advanced third generation reactor countries to restrict their exports of missiles
with inherent passive safety features ensuring a and related technologies capable of carrying
prompt and safe shut down of the plant in the a 500-kilogram payload at least 300
event of an emergency. kilometers or delivering any type of weapon of
mass destruction.
 PFBR has been fully designed and constructed
indigenously by Bhartiya Nabhikiya 88. c
Vidyut Nigam Ltd (BHAVINI).
Shanan Hydropower Project
 Once commissioned, India will only be the
second country after Russia to have The Centre has ordered that status quo be maintained
commercial operating Fast Breeder Reactor. on the Shanan hydropower project, over which Punjab
and Himachal Pradesh have made competing claims.
86. b
 Shanan Hydropower Project is a British-era
Cruise 110-MW hydel project which has been a point of
Ballistic missiles
missiles contention between Punjab and Himachal
Pradesh.
Powered
Powered only in the
Operati throughout its
first phase of flight, not  It is located at Uhl (a tributary of the Beas
on flight, manoeuv River) in Himachal Pradesh.
manoeuvrable
rable
 It is controlled by the Punjab Power
Typically 1,000 From <1,000 km to Corporation Limited (PSPCL).
km, can be as >10,000 km, missiles are
Range
much as 4000 classified according to  PSPCL is a profit-making entity, generating a
km range revenue of Rs 200 crore per annum.

CHENNAI |SALEM| MADURAI | COIMBATORE DELHI | BANGALORE | THIRUVANANTHAPURAM


www.shankariasacademy.com 335
www.iasparliament.com

 It was originally leased to Punjab in 1925 signed objective to ensure livelihood opportunities for
by Raja Joginder Bahadur (then ruler of Mandi) the poor.
and Col BC Batty, a British representative and
Chief Engineer of Punjab. 90. c

 The 99-year-old lease came to an end on March Cervical Cancer


2, 2024.
India’s interim Union Budget 2024-25 has taken a
 Himachal Pradesh contends that the project significant step by supporting the vaccination of girls
should remain under its control once the lease aged 9 to 14 against cervical cancer, marking a new
expires, it stated that they would not allow era in women’s health.
Punjab to stake claim on the project after the
lease period.  Cervical cancer – It is a type of cancer that
occurs in the cells of the cervix -the lower part
89. c of the uterus that connects to the vagina.

Self Help Groups (SHGs)  It is a common sexually transmitted infection


which is caused mainly due to the long-lasting
 A Self-Help Group is an informal infection with certain types of Human
association of individuals who come together papillomavirus (HPV).
to improve their living conditions.
 Almost all cervical cancer cases (99%) are
 It can be formal (registered) or informal. caused due to high-risk HPV, a highly common
Registration under any Societies Act, State virus transmitted through sexual contact.
cooperative Act or a partnership firm is not
mandatory.  It is preventable as long as it is detected early
and managed effectively.
 It is a self-governed and peer-controlled
group comprising people with similar socio-  Symptoms:
economic backgrounds and a shared desire to
achieve a common purpose. o Vaginal bleeding after intercourse,
between periods or after menopause,
 Members of SHG agree to save regularly and
contribute to a common fund. o Pelvic pain or pain during intercourse

o Watery, bloody vaginal discharge that


 The members agree to use this common fund
may be heavy and have a foul odour.
and such other funds (like grants and loans
from banks), which they may receive as a group,
 Global Strategy for Cervical Cancer
to give small loans to needy members as per the
Elimination – In 2020, the WHO adopted
decision of the group.
this strategy in order to eliminate cervical
cancer as a public health problem.
 As per National Rural Livelihood Mission
(NRLM) data, there are around 9 million SHGs
 90-70-90 Targets – These are a set of goals
with nearly 100 million women members,
set by the World Health Organization (WHO) in
averaging 14 SHGs per village.
2020 to eliminate cervical cancer by 2030.
 The concept of SHGs began in the 1970s,
 Two vaccines licensed globally that are
notably with the formation of the Self-
available in India are:
Employed Women’s Association (SEWA) in
Gujarat. o A quadrivalent vaccine (Gardasil, from
Merck) and
 In 1992, SHGs were linked to banks for small
loans, leading to the SHG Bank Linkage o A bivalent vaccine (Cervarix, from
Project (SHG-BLP) for setting up livelihood GlaxoSmithKline).
opportunities.
 Cervavac – It is India’s first indigenous
 Swarna Jayanti Gram Swarojgar Yojana was quadrivalent human papillomavirus vaccine
launched in 1999 to recognize the potential of (qHPV) vaccine and intended to protect women
SHGs in promoting self-employment in rural against cervical cancer.
areas.
 It is developed by the Serum Institute of India
 In 2011, SHGs were integrated into the National in collaboration with the Department of
Rural Livelihoods Mission (NRLM), becoming Biotechnology.
core implementers of the government's

CHENNAI |SALEM| MADURAI | COIMBATORE DELHI | BANGALORE | THIRUVANANTHAPURAM


www.shankariasacademy.com 336
www.iasparliament.com

 Bhutan was the first Low Middle Income recommendations from bodies like Finance
Country to launch a nationwide HPV Commission.
vaccination program for girls aged 12 to 18 in
2010.  It can influence the State’s borrowing limits
through legislative changes, amendments to
 It achieved an initial coverage of 95%, research existing laws like the FRBM Act, or by
shown a decrease in the prevalence of HPV exercising discretion in granting additional
transmission. funds or relaxing borrowing constraints in
exceptional circumstances.
91. d
92. a
Regulation of State’s borrowing in India
Cinematograph (Certification) Rules, 2024
 The State’s borrowing in India in India are
regulated by: Ministry of Information and Broadcasting,
Government of India has notified the Cinematograph
o Article 293 (Certification) Rules, 2024 in accordance with the
Cinematograph (Amendment) Act, 2023.
o 7th Schedule
Cinematograph (Certification) Rules, 2024
o Fiscal Responsibility and Budget
Management Act, 2003
Key aspects About
o Finance Commission

o State’s own Fiscal Responsibility Acts. To streamline and modernise the


film certification process for the
o Union Government Aim digital age, keeping pace with the
emerging technologies and
 Article 293 – It grants fiscal autonomy to advancement in the film sector
states, allowing them to borrow only from
within the territory of India on guarantee from
Cinematograph (Certification)
the Consolidated Fund of the State.
Rules, 1983 have been
Comprehensive comprehensively overhauled in
 7th Schedule – Fiscal matters are delineated
revision order to improve and contemporize
in the 7th Schedule of the Constitution, with
the entire process of certification of
"Public Debt of the State" falling under the State
films for public exhibition.
List, giving states exclusive jurisdiction over
this matter.
It is adopted to enhance
 FRBM Act 2003 – The Fiscal Responsibility Online
transparency, efficiency, and
and Budget Management Act 2003 was enacted certification
ease of doing business for the
to ensure intergenerational equity in fiscal process
film industry.
management, it sets limit on fiscal deficit and
borrowing for both the Central and State
governments.
Implementation of complete digital
 Finance Commission – It periodically processes is to eliminate
Time-effective
makes recommendations regarding fiscal transactional time and reduce the
matters, including borrowing limits for States, time taken for film certification.
it is crucial for determining the borrowing
Movies/feature films to have
ceilings for states, taking into account factors
accessibility features for
such as economic conditions, fiscal health, and
Accessibility certification to make it inclusive for
developmental needs.
features disabled persons, as stipulated in
 State Fiscal Responsibility Acts – Each the guidelines issued in this regard
State may have its own Fiscal Responsibility from time to time.
Act, which further defines the limits and
The UA category is classified
guidelines for borrowing and fiscal
into UA+, UA13+ and UA16+,
management within the State.
these would serve as
Age- based
 Role of Centre – It plays a significant role in recommendations for parents
certification
overseeing fiscal matters, including approving or guardians to decide
borrowing limits for States based on whether the content is suitable
for their children.

CHENNAI |SALEM| MADURAI | COIMBATORE DELHI | BANGALORE | THIRUVANANTHAPURAM


www.shankariasacademy.com 337
www.iasparliament.com

Ensuring greater representation of implemented in an integrated manner through


women in the Central Board of Film 4 institutes.
Certification (CBFC) Board and
Women  Access, Watch and Reserve (AWaRe) – It
Advisory Panels, with one-third of
representation is a classification tool of World Health
the members on the Board
being women, and preferably half Organization (WHO) for monitoring
on the Panels. antibiotic consumption.

It is provided to expedite the  It defines the targets and monitors the effects of
certification process in case of stewardship policies that aim to optimize
Priority urgency felt by filmmakers due to antibiotic use and curb antimicrobial
screening prior commitments, this would resistance.
enhance transparency and promote
ease of doing business. 94. a

DIANA Initiative
Removal of the restriction on the
Perpetual Recently Finland joined the DIANA Initiative of North
validity of certificates for only 10
validity of Atlantic Treaty Organization (NATO).
years for perpetual validity of CBFC
certificates
certificates
 DIANA – Defence Innovation Accelerator for
the North Atlantic.
Recertification of the edited film for
 It aims to address future challenges in defence
Recertification Television broadcast, as only
and security by harnessing technological
for TV Unrestricted Public Exhibition
solutions.
broadcast category films can be shown on
television.  Initiative of - North Atlantic Treaty
Organization (NATO).
93. b  Aim - It seeks to accelerate the development
and deployment of innovative solutions with
Atmospheric Research Testbed
applications in both civil and defence sectors.
Recently the 1st phase of India’s Atmospheric Research
Testbed in Central India (ART-CI) was inaugurated at  DIANA has a network of more than 200
Bhopal in Madhya Pradesh affiliated accelerator sites and test centres.

 About - It is an open-field observational and  All NATO nations are members of


analytical research program. DIANA.

 Aim - To study vital cloud processes associated  The DIANA Board of Directors is responsible
with the Indian monsoon, particularly focusing for governance and comprises representatives
on the Monsoon Core Zone (MCZ) in from every Allied country.
central India.
 DIANA accelerator programme – Aimed
 MCZ is the core monsoon zone is a region in at helping companies and countries develop
India that stretches from Gujarat in the west to deep technologies and innovations applicable
West Bengal in the east. to both commercial and defence sectors.

95. c
 Funded by - The Ministry of Earth Sciences
(MoES). SheRNI Portal
 Implemented by - The Indian Institute of Recently the SheRNI Portal was launched by the
Tropical Metrology, Pune. University Grants Commission (UGC) to connect,
support women scientists and faculty members across
 ART-CI - It is a component under Atmosphere India.
& Climate Research-Modelling Observing
Systems & Services (ACROSS).  SheRNI - She Research Network in India.
 ACROSS Scheme - It is a Central Sector  Aim - To create a national-level expert
Scheme, pertains to the atmospheric science platform for women faculty
programs of MoES. members and facilitate the exchange of
expertise, insights and experiences across
 The sub-schemes under the ACROSS scheme diverse fields.
are multi-disciplinary in nature and will be

CHENNAI |SALEM| MADURAI | COIMBATORE DELHI | BANGALORE | THIRUVANANTHAPURAM


www.shankariasacademy.com 338
www.iasparliament.com

 Launched by - The Information and Library  A 1.08% per annum growth in sample area from
Network (INFLIBNET) of the University 2018.
Grants Commission (UGC).
 The largest growth rate of 1.5%, in Central
PM-SURAJ Portal India & Eastern Ghats.

 PM-SURAJ – Pradhan Mantri Samajik  A 3.4% decline per annum in Shivalik hills and
Utthan evam Rozgar Adharit Jankalyan. Gangetic plains.

 Aim - To offer credit support to the  Only 4 States reported over a 1,000 animals,
marginalized segments of society. where Madhya Pradesh houses the
largest followed by Maharashtra, Karnataka
 Launched by - The Ministry of Social and Tamil Nadu.
Justice and Empowerment.
 Tiger Reserves or sites with highest leopard
 It provides financial assistance directly to population
beneficiaries, eliminating middlemen and
commissions.  Nagarajunasagar Srisailam (Andhra Pradesh),
followed by Panna (Madhya Pradesh), and
 The portal will help empower people deprived Satpura (Madhya Pradesh).
of opportunities.
 Threats – Habitat loss, human-wildlife
 It will provide information about various conflict, and poaching.
government schemes at one place.
 Human-wildlife conflict likely reason for
96. b decline in the big cat population by 22% in
Uttarakhand.
Status of Leopards in India Report
 Significance – The findings underscore the
Recently, the 5th cycle of leopard population critical role of Protected Areas in conserving
estimation (2022) was carried out and ‘the Status of leopard populations.
Leopards in India’ report was released.
Indian Leopard
 Aim – To reveal crucial insights into leopard
distribution and conservation challenges.  Scientific Name - Panthera pardus fusca
 Conducted by – National Tiger  Conservation Status
Conservation Authority (NTCA), Wildlife
Institute of India (WII) in collaboration with o IUCN - Near Threatened
State Forest Departments.
o CITES - Appendix I
 It is under quadrennial “Monitoring of
Tiger, Co-predators, prey and their o WPA, 1972 - Schedule I
habitat” exercise in tiger range States.
97. c
 Coverage – 20 States of India, and focussed
on about 70% of the animals’ expected habitat, Kulasekarapattinam Spaceport
which are India’s tiger reserves and protected
forest areas. The Rohini Sounding Rocket was launched from the
mobile launch pad to mark the commencement of
o Forested habitats within 18 tiger states, physical work on the Kulasekarapattinam spaceport.
covering 4 major tiger conservation
landscapes.  Geographical location – Thoothukudi
district of Tamil Nadu.
o Non-forested habitats, arid, and high
Himalayas, total of about 30% area  Kulasekaranpattinam is a town better known
were not sampled for leopard. for its Dussehra celebrations at Sri
Mutharamman Temple.
 Scientific methodologies – A meticulous
process combining camera trapping, habitat  Role – It is ISRO’s 2nd satellite launch
analysis, and population modelling. facility, would be completed within 2 years to
launch SSLVs in order.
Key Findings of the report
 It would have the capacity to launch 24
 8% rise in numbers from 2018 to 2022 satellites per year.
with 13,874 individuals

CHENNAI |SALEM| MADURAI | COIMBATORE DELHI | BANGALORE | THIRUVANANTHAPURAM


www.shankariasacademy.com 339
www.iasparliament.com

 Need – The rockets launched from ISRO’s  In India – It might have reached Cuttack from
Sriharikota facility in Andhra Persia through Indonesia some 500 years ago by
Pradesh 1st heads east, and then turn south to sea trade, also evidences shows the exchange of
avoid the Sri Lanka’s airspace, costing more jewellery and gems between Kalinga and Indonesia.
fuel and reduce payload’s capacity.
The Banglar muslin – Handloom Craft
 Geographical advantages – It can head
straight in the southern direction without a  Geography – Bengal.
dogleg manoeuvre and closer to
equator than Sriharikota thereby reducing time  Qualities – This finest muslin is made of cotton,
and fuel wastage. which are spun to create threads that
maintained tensile strength higher than any other
 Economic significance – It is cotton products.
relatively closer to the ISRO Propulsion
Research Complex located in Mahendragiri, in Narasapur crochet lace products
Tirunelveli district, Tamil Nadu.
 Geography – Narsapur in Andhra Pradesh.
 It makes it easier to transport the rocket
components safely in shorter time, thus  Qualities – Intricate craftsmanship, unique
reducing the costs involved. designs and good quality crochet craft.

 Safety considerations – Proximity to the  In India – It was brought by missionaries to


eastern coastline of India are safer, as any Narsapur.
failure of launches allow the debris to fall
harmlessly in Bay of Bengal or Indian Ocean. Ratlam Riyawan Lahsun – Garlic variety

 Geography – It is named after Riyawan village in


Ratlam district of Madhya Pradesh

Ambaji White Marble

 Geography – Gujarat.

 It is formed when limestone is re-crystallised under


the earth’s crust due to intense pressure and heat.

 Qualities – Pure white Colour with light grey


veins.

Majuli mask

 Geography – Assam

 They are made in different variety and sizes as they


are mainly divided into different categories

 Mukha bhaona face mask – It covers the face.

98. d  Lotokoi hanging mask – It is bigger in size


extends to the chest.
Cuttack Rupa Tarakasi - Jewellery
 Cho Mukha huge mask – It is a head and body
 It is a silver filigree, a form of metalwork in mask.
jewellery.
Majuli Manuscript Paintings
 Geography – Odisha.
 Geography – Assam.
 Qualities – Fine craftsmanship and luxurious
design.  It has illustrated numerous stories and chapters
taken from the great Hindu epics Ramayana,
 Origin – As early as 3500 BCE in Mesopotamia, it Mahabharata and above all subjects from the
was incorporated and even today it is done as Bhagavata Purana.
Telkari work.

CHENNAI |SALEM| MADURAI | COIMBATORE DELHI | BANGALORE | THIRUVANANTHAPURAM


www.shankariasacademy.com 340
www.iasparliament.com

Risa Textile  Childcare – Addressing the childcare gap


would immediately lead to a 1% increase in
 Geography – Tripura women’s participation in the labour force.

 It is common in almost all 19 indigenous tribal  Legal gap – Countries on average, established
communities of Tripura. less than 40% of the systems needed for full
implementation.
 However, each community has its own design.
100. c
 Hyderabad Lac Bangles and Kutch rogan craft are
the other products that got GI tag recently Asiatic Lions

99. a The International Union for Conservation of Nature


(IUCN) recently recategorized the Asiatic lion status
Women, Business and the Law Report 2024 as vulnerable from endangered indicating a positive
shift in its conservation status.
The 10th edition of the women, business and the law
report was published recently. Nomenclature change

 An annual studies measuring the laws that  Due to genetic similarity between Central
affect women’s economic opportunity. African, West African and Asiatic Lions, IUCN
has clubbed all three under Panthera leo leo.
 Published by – World bank
 Earlier, the Asiatic lions were classified
 Coverage – 190 economies. as Panthera leo persica.

 For the 1st time – It analysed the impact of Asiatic Lions


childcare & safety policies on women’s
participation in the labour market.  About - The Asiatic lion is a subspecies of lion,
slightly smaller than African lions.
 It presents a new approach to measure the
implementation gap between laws (de jure)  They are also known as Persian lions or
and how they function in practice (de facto). Indian lions.

 10 indicators – Safety, Mobility, Workplace,  Scientific Name - Panthera leo leo.


Pay, Marriage, Parenthood, Childcare,
Entrepreneurship, Assets, and Pension.  Distribution - They used to range from
Turkey, across Asia, to eastern India.
Key Findings of the Report
 Habitat - At present Gir National Park,
 Workplace - No country in the world affords Gujarat is the only abode of the Asiatic lion.
women the same opportunities as men in the
workforce.  They are one of the 5 big cats found in India and
are better known as ‘the king of beasts’.
 Global gender gap - It was far wider than
previously thought and closing this could raise Conservation Status
global GDP by more than 20%.
IUCN Vulnerable
 Pay - Globally, women earned just 77 cents of
each dollar earned by a man. WPA Schedule I of Wildlife Protection Act, 1972

 Pension – In 81 countries, a woman’s pension CITES Appendix I


benefits do not account for periods of work
absences related to childcare.
Project Lion
 Parenthood – Less than 50% the countries
had financial support or tax relief for parents of  The Project Lion envisages landscape ecology-
young children. based conservation of the Asiatic Lion in
Gujarat by integrating conservation and eco-
 Safety – 151 countries had laws against sexual development.
harassment in the workplace but only 40
countries had laws that covered abuse in  The Project is being implemented in the Gir
public areas or on public transport. landscape in Gujarat which is the last home of
the Asiatic lion.

CHENNAI |SALEM| MADURAI | COIMBATORE DELHI | BANGALORE | THIRUVANANTHAPURAM


www.shankariasacademy.com 341
www.iasparliament.com

TEST – VII 5. Barbados

1. Consider the following statements with respect to the Select the correct answer using the code given below:
Traditional Medicine Global Summit, 2023
a. 1, 2 and 3 only
1. It is the first-ever Global Summit on Traditional
b. 1, 2 and 4 only
Medicine which was held at New Delhi, India.
c. 1, 3 and 5 only
2. It was jointly hosted by the World Health
Organization (WHO) and the Government of d. 3, 4 and 5 only
India.

Which of the statement(s) given above is/are correct?


5. Which of the following best describes the
a. 1 only term ‘Alpenglow phenomenon’?
b. 2 only a. It is a natural northern lights or southern lights
in the Earth's sky that appears as curtains, rays,
c. Both 1 and 2
spirals or flickers.
d. Neither 1 nor 2
b. The change of colour due to a combination of
high salinity and the presence of algae.

2. W12+ Blueprint, a platform that hosts city profiles c. It is a reddish hue seen near mountain summits
and case studies of programs, technologies, policies that during sunrise or sunset.
addresses common water security challenges, is an
d. It is a natural phenomenon that occurs during
initiative of?
the spring and summer periods in the middle
a. UNESCO latitudes.

b. UN Water

c. UN DESA 6. Consider the following.

d. World Bank 1. Sahyadri Mountains

2. Konkan Coast

3. Which of the following best describes the 3. Deccan Plateau


term ‘SNAP-10A’?
4. Eastern Ghats
a. A space mission tasked to return samples of
5. Coromandel Coast
Jupiter to Earth
Which of the following are integrated part of Maratha
b. The world’s 1st operational nuclear reactor in
military landscape?
space
a. All five
c. An instrument to measure pollution and air
quality across greater North America b. All except 3 & 4
d. Only spacecraft that have ever operated outside c. All except 4 & 5
the heliosphere, the border of our solar system
d. All except 5

4. Which of the following islands are part of the Pacific


Island Countries (PICs)? 7. Global Tree Assessment (GTA), sometimes seen in the
news, is an initiative of?
1. Fiji
a. Global Environment Facility (GEF)
2. Solomon Islands
b. United Nations Environment Programme
3. Papua New Guinea (UNEP)
4. Comoros c. Intergovernmental Panel on Climate Change
(IPCC)

CHENNAI |SALEM| MADURAI | COIMBATORE DELHI | BANGALORE | THIRUVANANTHAPURAM


www.shankariasacademy.com 342
www.iasparliament.com

d. International Union for Conservation of Nature 11. Consider the following statements with respect to
(IUCN) Red Sand Boa

1. It is a venomous species inhabiting the arid


regions of the Indian subcontinent.
8. Consider the following statements with respect to
Model Code of Conduct (MCC) 2. They are ovoviviparous and nocturnal and
spends the majority of its time under the
1. The code comes into force immediately when ground.
the election dates are declared and revoked
when the elections are completed. 3. It is protected under the Schedule 1 of the
Wildlife (Protection) Act, 1972.
2. It derives its statutory backing from the
Conduct of Election Rules, 1961. How many of the statement(s) given above is/are
correct?
3. MCC was first introduced in the state assembly
elections in Kerala. a. Only one

How many of the statement(s) given above is/are b. Only two


correct?
c. All three
a. Only one
d. None
b. Only two

c. All three
12. Consider the following statements with respect to
d. None Household Consumption Expenditure Survey, 2022-23

1. It was conducted by the National Sample


Survey Office (NSSO).
9. Global Innovation Index (GII), 2023 was launched
by? 2. According to the report, Consumption
expenditure growth is faster in rural India than
a. European Patent Office urban India.

b. World Economic Forum 3. India's per capita monthly consumption


expenditure increases by more than 50% when
c. The US Chamber of Commerce compared to 2011-12 level.
d. World Intellectual Property Organization How many of the above statements is/are correct?

a. Only one
10. Consider the following pairs. b. Only two
Places in news Regions c. All three

Almora Fault North-eastern India d. None

Julian Felipe Reef South China Sea

Kopili Fault Nepal 13. Consider the following statements regarding Nazool
Lands
How many of the pair(s) given above are incorrect?
1. It is owned by the respective state governments
a. Only one but most often not directly administered as
state property.
b. Only two
2. The government generally uses Nazool land for
c. All three public purposes like building schools, hospitals,
etc.
d. None
Which of the statement(s) given above is/are correct?

a. 1 only

CHENNAI |SALEM| MADURAI | COIMBATORE DELHI | BANGALORE | THIRUVANANTHAPURAM


www.shankariasacademy.com 343
www.iasparliament.com

b. 2 only c. All three

c. Both 1 and 2 d. None

d. Neither 1 nor 2

17. Windsor Framework, sometimes seen in the news


recently, is a deal between?
14. ‘Horizon 2047’, sometimes seen in the news
recently, refers to? a. Israel and Palestine

a. An earth sized exoplanet discovered by the b. Russia and Ukraine


Hubble telescope.
c. Azerbaijan and Armenia
b. A roadmap set by India and France to set the
course for the bilateral relationship. d. U.K and European Union

c. An interplanetary space probe launched as a part


of NASA's New Frontiers program.
18. Consider the following statements with respect to
d. India’s initiative to enrich the soil with humus to Sigur Landscape
supports plant growth and water retention.
1. It is a part of the World Network of Biosphere
Reserves of UNESCO.

15. Consider the following statements with respect to 2. Sigur plateau is the last southernmost viable
UN Human Rights Council (UNHRC) breeding population for the White-rumped
vulture in India.
1. It is one of the specialized agencies of the
United Nations. 3. The Sigur corridor protects the contiguity of
elephant habitats between the Eastern and
2. Its resolutions are not legally binding on its Western Ghats.
members.
How many of the statements given above
3. It has no criminal enforcement or sanctioning are incorrect?
powers.
a. Only one
How many of the statement(s) given above is/are
correct? b. Only two

a. Only one c. All three

b. Only two d. None

c. All three

d. None 19. Consider the following passage:

They spend most of their lives in the open ocean and


only return to land to breed and raise young. They must
16. Consider the following statements with respect to be able to withstand salt water, wind and waves. The
Rat-Hole Mining examples of this type include Frigate birds, Fulmars,
Shearwaters and Tropicbirds.
1. Rat-hole mining is a method of extracting coal
from narrow, horizontal coal beds. The above passage best describes which of the
following?
2. This practice of mining is prevalent in
Meghalaya. a. Pelagic birds

3. The practice of rat-hole mining is banned in b. Demersal species


India.
c. Perching birds
How many of the statements given above are correct?
d. Waterfowl
a. Only one

b. Only two

CHENNAI |SALEM| MADURAI | COIMBATORE DELHI | BANGALORE | THIRUVANANTHAPURAM


www.shankariasacademy.com 344
www.iasparliament.com

20. Consider the following statements with respect to 1. It is a statutory body established through an act
the Indus Water Treaty (IWT): of parliament.

1. It was signed between India and Pakistan, 2. It is one of the recommendations of the
brokered by the World Bank. National Education Policy (NEP), 2020.

2. It gave the waters of the rivers Indus, Ravi and 3. It replaced the National Research Development
Chenab to Pakistan and waters of rivers Corporation (NRDC).
Jhelum, Beas and Sutlej to India.
4. It is responsible for conducting national level
3. It does not permit the construction of dams for exams for admission in to engineering, medical
any purpose by either parties on these rivers. colleges.

How many of the statement(s) given above is/are How many of the statement(s) given above is/are
correct? correct?

a. Only one a. Only one

b. Only two b. Only two

c. All three c. Only three

d. None d. All four

21. Kampala Declaration, sometimes seen in the news 24. Management Effectiveness Evaluation (MEE) is a
recently, is associated with? framework developed by?

a. Artificial Intelligence a. World Wide Fund for Nature

b. Neglected Tropical Diseases b. United Nations Environment Program

c. Combat violence against women c. Intergovernmental Panel on Climate Change

d. Environment and Climate Change d. International Union for Conservation of Nature


and Natural Resources

22. Consider the following statements with respect to


Bharat NCAP 25. Consider the following statements regarding Global
Gender Gap Report 2023
1. It is India's autonomous program for evaluating
vehicle safety features and technologies. 1. It is an annual report published by the World
Economic Forum (WEF).
2. Vehicles are assessed and assigned a rating
ranging from one star to five stars based on an 2. Political empowerment is one of the four
evaluation of three parameters. dimensions considered for the report.

3. All cars manufactured in India after 2024 are 3. India had attained parity in enrolment across
mandated to obtain Bharat NCAP certification. all levels of education.

How many of the statement(s) given above is/are How many of the statement(s) given above is/are
correct? correct?

a. Only one a. Only one

b. Only two b. Only two

c. All three c. All three

d. None d. None

23. Consider the following statements regarding 26. Consider the following statements with respect to
National Research Foundation (NRF). Lakhpati Didi Initiative

CHENNAI |SALEM| MADURAI | COIMBATORE DELHI | BANGALORE | THIRUVANANTHAPURAM


www.shankariasacademy.com 345
www.iasparliament.com

1. The program is being implemented by the b. Only two


Ministry of Women and Child Development
with Ministry of Skill Development and c. All three
Entrepreneurship.
d. None
2. A Lakhpati Didi is a self-help group member
who earns a monthly income of one lakh or
more.
29. Dakar Declaration deals with the reduction of
3. In Union budget 2024-25, the target for global emission. Dakar is the capital of which of the
Lakhpati Didi, which was initially set at 2 crore following countries?
women, has been enhanced to 3 crore women.
a. Guinea
Which of the above statement(s) is/are correct?
b. Ghana
a. Only one
c. Senegal
b. Only two
d. Gambia
c. Only three

d. None
30. The terms ‘Bokmal and Nynorsk’, sometimes seen
in the news recently, are associated with?

27. Consider the following statements with respect to a. Towns located in the Ukraine-Russia border
Wildlife Protection (Amendment) Act, 2022
b. Chemical compounds highly reactive with
1. The Act removes the schedule for vermin oxygen
species and reduces the total number of
schedules from six to five . c. Distinct standards of writing in Norwegian
language
2. The Act empowers the central government to
regulate or prohibit the import, trade, d. Elusive subatomic particles detected by neutrino
possession or proliferation of invasive alien observatories
species.

3. The Act empowers both the central and state


31. Consider the following statements with respect to
government to notify a conservation reserve.
Competition Commission of India (CCI)
How many of the statement(s) given above is/are
1. It is a statutory body established under the
correct?
Competition Act, 2002.
a. Only one
2. It works under the aegis of Ministry of
b. Only two Commerce and Industry.

c. All three 3. The decision of CCI can be challenged only


before High courts and Supreme Court.
d. None
How many of the statement(s) given above is/are
correct?

28. Consider the following pairs. a. Only one

GI Products States b. Only two

1. Ayyampalayam Nettai – Kerala c. All three

2. Mahua Liquor – Maharashtra d. None

3. Udangudi Panangkarupatti – Tamil Nadu

How many of the pair(s) given above is/are correctly 32. Consider the following statements with respect to
matched? Uniform Civil Code

a. Only one 1. It is defined in Article 44 of part III of Indian


Constitution.

CHENNAI |SALEM| MADURAI | COIMBATORE DELHI | BANGALORE | THIRUVANANTHAPURAM


www.shankariasacademy.com 346
www.iasparliament.com

2. Goa is the first state in India to introduce and 2. Volcanoes help to stabilize the heat of the core
implement Uniform Civil Code after India’s part of our planet.
independence.
3. Volcanoes form new land forms after the drying
Which of the statement(s) given above is/are correct? process of liquid lava.

a. 1 only 4. The lava from volcano contains different


mineral which enriches the existing soil.
b. 2 only
Select the correct answer using the codes given below:
c. Both 1 and 2
a. None
d. Neither 1 nor 2
b. Only two

c. Only three
33. Consider the following pairs.
d. All four
Paintings Location

1. Subika – Manipur
36. Consider the following statements with respect to
2. Basohli – Madhya Pradesh Indian Pangolins.

3. Pichwai – Gujarat 1. Indian Pangolin has the ability to survive in any


conditions and can be found in all landforms
4. Thangka – Arunachal Pradesh including the arid region.
5. Pattachitra – Odisha 2. When threatened, they uses keratin scales as
armour to defend itself against predators.
How many of the pair(s) given above is/are correctly
matched? 3. Indian Pangolin has been classified as
‘Critically Endangered’, by the IUCN Red List of
a. Only two Threatened Species.
b. Only three How many of the statement(s) given above is/are
correct?
c. Only four
a. None
d. All five
b. Only one

c. Only two
34. Which of the following statements regarding Large
Hadron Collider (LHC) is incorrect? d. All three
a. It is a single stage, proton-proton machine.

b. A hadron is a subatomic particle made of two or 37. Graded Response Action Plan (GRAP), sometimes
more quarks held together by the strong seen in the news recently, is implemented by which of
interaction. the following organisations?
c. Higgs boson, often dubbed as the god’s particle, a. Central Pollution Control Board (CPCB)
was first discovered in the Large Hadron Collider in
2012. b. Dust and Control Management Centre (DCMC)
d. It is built by the European Organization for c. Commission for Air Quality Management
Nuclear Research (CERN). (CAQM)

d. Centre Carbon Conversion Energy Management


(CCCEM)
35. Which of the following are the advantages of
volcanoes?

1. Volcanoes leads to the formation of geysers


which are sources of geothermal electricity.

CHENNAI |SALEM| MADURAI | COIMBATORE DELHI | BANGALORE | THIRUVANANTHAPURAM


www.shankariasacademy.com 347
www.iasparliament.com

38. Consider the following pairs: a. A tax to be paid by the importers of certain goods
that reflects the carbon content of those goods.
Terms in News States
b. A tax levied on the investments made by an
1. Gupteswar Forest – Karnataka investor in startups and early-stage companies.

2. Umiam Wetland – Odisha c. A tax levied on the profits of a company that is


derived from an external or unprecedented event.
3. Pangolakha Wildlife Sanctuary – Tripura
d. A tax imposed on the gains realized by
How many of the pair(s) given above is/are correctly shareholders or owners when a company is
matched? liquidated or wound up.
a. Only one

b. Only two 42. With reference to Office of Deputy Chief Minister,


consider the following statements.
c. All three
1. It is mentioned in the Article 75 of Indian
d. None Constitution.

2. A state cannot have more than one deputy Chief


Minister.
39. Yak Churpi, Khamti Rice & Tangsa Textile are
Geographical Indication (GI) Products that are 3. She/he is equivalent to the rank of cabinet
indigenous to? minister in that state.
a. Assam How many of the statement(s) given above is/are
correct?
b. Manipur
a. Only one
c. Nagaland
b. Only two
d. Arunachal Pradesh
c. All three

d. None
40. Consider the following statements with respect to
Right to be forgotten

1. It is the right to remove or erase content from 43. Researchers have unveiled an alarming insights into
internet records so that it is not accessible to the the potential impact of clethodim on male reproductive
public. health. Clethodim is widely used as a?
2. Information Technology Rules, 2021 explicitly a. Herbicide
defines the right to be forgotten.
b. Refrigerant
3. In K.S.Puttaswamy vs Union of India Case,
2017, Supreme Court has recognized the right c. Polymer resins
to be forgotten as a right under the right to
privacy. d. Food preservative

How many of the statement(s) given above is/are


correct?
44. Consider the following statements with respect to
a. None Cott-Ally App

b. Only one 1. It aims to educate the farmers about MSP rates,


nearest procurement centers, payment tracking
c. Only two and best farming practices.
d. All three 2. It is a free mobile-based application owned by
the ICAR-Central Institute of Cotton Research
(CICR).
41. Which of the following best describes the Which of the statement(s) given above
term, “Windfall Tax”? is/are incorrect?

CHENNAI |SALEM| MADURAI | COIMBATORE DELHI | BANGALORE | THIRUVANANTHAPURAM


www.shankariasacademy.com 348
www.iasparliament.com

a. 1 only c. All three

b. 2 only d. None

c. Both 1 and 2

d. Neither 1 nor 2 48. “Qanat” is an ancient system associated with which


of the following?

a. Market system
45. The practice of ‘veli’ among the Vaddi community of
Andhra Pradesh, refers to? b. Irrigation system

a. An equitable distribution of land rights among c. Land revenue system


indigenous or tribal communities
d. Military administration system
b. The isolation of tribal or indigenous
communities refers from mainstream society

c. A practice of slash-and-burn agriculture where 49. Which of the following countries are part of the Chip
land is cleared and cultivated for a short duration 4 Initiative?

d. A form of social ostracisation and the boycotted 1. US


people are not allowed to speak to other villagers
2. China

3. Russia
46. Consider the following passage:
4. India
It is located on the banks of the Godavari River. The
temple derives its name from a black statue of the Lord. 5. Japan
The Babasaheb Ambedkar led a landmark agitation
demanding temple entry rights for Dalits in this temple. Select the correct answer using the codes given below:
The sanctum sanctorum has statues of Ram, Sita and
a. 1 and 5 only
Lakshman.
b. 1, 3 and 4 only
The above passage best describes which of the following
temples? c. 1, 2 and 4 only
a. Brihadishvara Temple d. 2, 3 and 5 only
b. Kalaram temple

c. Chennakeshava Temple 50. Consider the following statements with respect to


Primary Agricultural Credit Societies (PACS)
d. Simhachalam Temple
1. It constitutes the lowest tier of the long-term
cooperative credit structure in India.
47. With reference to Chiral Bose-Liquid state, consider
2. Individual farmers are members of the PACS,
the following statements
and office-bearers are elected from within
1. In this state, electrons can be frozen into them.
predictable patterns.
3. A village can have multiple Primary
2. In this state, electrons can be made resilient to Agricultural Credit Societies (PACS).
changes in spin.
How many of the statement(s) given above is/are
3. In this state, electrons can even synchronise correct?
their movements.
a. None
How many of the above statements are correct?
b. Only one
a. Only one
c. Only two
b. Only two
d. All three

CHENNAI |SALEM| MADURAI | COIMBATORE DELHI | BANGALORE | THIRUVANANTHAPURAM


www.shankariasacademy.com 349
www.iasparliament.com

51. Plastiglomerates, which was recently discovered in 54. Which of the following is incorrect with respect to
Trindade Islands refers to ______? Quantum Dots?

a. A naturally evolving plastic eating bacterial a. They are man-made nanoscale crystals.
species that breaks the plastics and digests it.
b. When light is shined on, it absorbs and then re-
b. A plastic rock, where plastic hold together a emits the light at a different frequency.
mixture of sedimentary granules and other debris.
c. By manipulating their size, we can make them
c. A group of plastics which has the ability to emit light of specific colours when they are excited
withstand the effects of repeated wearing, rubbing, by light.
scraping.
d. Smaller dots emits red light while relatively
d. None of the above larger dots emits blue light.

52. Consider the following statements with respect to 55. Consider the following pairs.
the Indo-Pacific Economic Framework (IPEF)
Art forms Regions
1. All the IPEF partners must join the four pillars
of IPEF. 1. Pichwai Painting – Mysore

2. India is one of the IPEF partner and it has 2. Dogra architecture – Jammu
joined all the four pillars of the IPEF.
3. Vajra Mushti Kalaga – Rajasthan
3. Only member countries of Regional
Comprehensive Economic Partnership (RCEP) How many of the pair(s) given above is/are correctly
are allowed to join IPEF. matched?

How many of the statement(s) given above is/are a. Only one


correct?
b. Only two
a. Only one
c. All three
b. Only two
d. None
c. All three

d. None
56. Consider the following statements with respect to
Cricket

53. Consider the following statements with respect to 1. The Los Angeles Olympics, 2028 will be the first
Special Category Status. Olympics to feature the Cricket sport.

1. It is a classification granted by the Centre to 2. In commonwealth games, it was featured for


assist the development of States that face the first time in the Commonwealth Games,
geographical or socio-economic disadvantages. 2022.

2. Special category status to states was granted by Which of the statement(s) given above is/are correct?
the National Development Council (NDC).
a. 1 only
3. Special category status to states empowers
them with political and legislative powers. b. 2 only

Which of the statement(s) given above is/are correct? c. Both 1 and 2

a. Only one d. Neither 1 nor 2

b. Only two

c. All three 57. Consider the following pairs with respect to


evacuation of Indians.
d. None
Operations Countries

1. Ajay – Israel

CHENNAI |SALEM| MADURAI | COIMBATORE DELHI | BANGALORE | THIRUVANANTHAPURAM


www.shankariasacademy.com 350
www.iasparliament.com

2. Kaveri – Sudan 60. Which of the following statements with respect to


Society for Worldwide Interbank Financial
3. Ganga – Ukraine Telecommunications (SWIFT) is incorrect?

4. Devi Shakti – Afghanistan  a. It creates a bank account that a bank holds


with a foreign bank in the currency of the
How many of the pair(s) given above is/are correctly country where the funds are held.
matched?
 b. It follows the LIBOR, an interest rate used by
a. Only one major banks in the market for short-term loans.
b. Only two
 c. It assigns each financial organization a
c. Only three unique code known as a bank identifier code.

d. All four  d. It uses a pattern of recognition system to


identify a cheque’s originality.

58. Who among the following are part of the


composition of Delimitation Commission? 61. Consider the following statements with respect to
Surrogacy rules in India
1. A retired Supreme Court judge
1. The child to be born through surrogacy must
2. A retired High Court judge have at least one gamete from the intending
parents.
3. Union Home Minister
2. Single women opting for surrogacy should be
4. Chief Election Commissioner of India either a divorcee or a widow.

5. Respective State Election Commissioners 3. Single women must use self-eggs and donor
sperms to avail surrogacy procedure.
Select the correct answer using the codes given below.
How many of the above statements is/are correct?
a. All except 3
a. Only one
b. All except 5
b. Only two
c. All except 2 & 3
c. All three
d. All five
d. None

59. Consider the following statements.


62. Which one of the following is incorrect regarding
1. It is the only Nano fertilizer approved by the the Large Language Model (LLM)?
Ministry of Agriculture and included in the
Fertilizer Control Order. 1. They are large general-purpose language
models that can be pre-trained and then fine-
2. It comes only in powdered form with 8% tuned for specific purposes.
Nitrogen and 16% Phosphorus by volume.
2. It has high parameter count that encodes the
3. It was launched by the Indian Council of knowledge and memories the model acquires
Agricultural Research (ICAR). during training.

How many of the above statement(s) with respect to 3. Gemini and Claude are examples of Large
Nano DAP fertilizers is/are correct? Language Models.

a. Only one How many of the statement(s) given above is/are


correct?
b. Only two
a. Only one
c. All three
b. Only two
d. None
c. All three

CHENNAI |SALEM| MADURAI | COIMBATORE DELHI | BANGALORE | THIRUVANANTHAPURAM


www.shankariasacademy.com 351
www.iasparliament.com

d. None b. 2 only

c. Both 1 and 2

63. Consider the following pairs. d. Neither 1 nor 2

Tribal Leaders Relevance

1. Tirot Sing – Hero of the Khasi Hills 66. Consider the following statements with respect to
Climate Change Performance Index (CCPI), 2024
2. Komaram Bheem – Jal, Jangal, Zameen
1. It is an annual report published by German
3. Alluri Sitharama Raju – Manyam Veerudu Watch since 2005.

How many of the pair(s) given above is/are correctly 2. It ranks countries based on four categories
matched? which includes Energy Use and Net Zero.

a. Only one Which of the statement(s) given above is/are correct?

b. Only two a. 1 only

c. All three b. 2 only

d. None c. Both 1 and 2

d. Neither 1 nor 2

64. Consider the following statements with respect to e-


Courts Project
67. Consider the following passage.
1. The project aims for universal computerisation
of district and subordinate courts in the He popularised and improved the musical instrument
country. ‘plucked rabab’. He was among the Navaratnas (9
jewels) at the court of the Mughal Emperor Akbar. A
2. The e-Courts Project is based on hybrid model festival is celebrated every year in the month of
that allows for both physical and virtual courts December to pay tribute to this person.
to co-exist.
The above passage best describes who among the
3. The e-Courts Project is monitored and funded following personality?
by the Ministry of Law and Justice.
a. Tansen
How many of the statement(s) given above is/are
correct? b. Amir Khusrau

a. Only one c. Abd al-Samad

b. Only two d. Abdul Rahim Khan-I-Khana

c. All three

d. None 68. Consider the following pairs of substance used in


doping and their effects.

Substances Effects
65. With reference to Methanotrophs, consider the
following statements. 1. Stimulant – Build more muscles

1. They are methane utilising bacteria that require 2. Diuretic – Hides fatigue
methane as a source of carbon and energy for
their metabolism. 3. Blood doping – Boost oxygen levels

2. They are gram-negative bacteria that are 4. Gene doping – Manipulates body weight
capable to grow both aerobically or
anaerobically. How many of the pair(s) given above is/are correctly
matched?
Which of the statement(s) given above is/are correct?
a. Only one
a. 1 only

CHENNAI |SALEM| MADURAI | COIMBATORE DELHI | BANGALORE | THIRUVANANTHAPURAM


www.shankariasacademy.com 352
www.iasparliament.com

b. Only two c. Both 1 and 2

c. Only three d. Neither 1 nor 2

d. All four

72. “Dar-Es-Salaam Declaration”, sometimes seen in


the news recently, refers to?
69. Consider the following statements with respect to
China Pakistan Economic Corridor (CPEC) a. A declaration to reduce to road crash fatalities.

1. It is a part of Belt and Road Initiative (BRI) of b. A declaration on ending AIDS in children by
China. 2030.

2. It aims to circumvent the Strait of Malacca and c. A declaration to reduce hazardous e-waste
the South China Sea. substances.

3. It enters Pakistan Occupied Kashmir (PoK) d. A declaration for protection of geo-heritage sites
through the Karakoram Highway in Gilgit of national importance.
Baltistan.

How many of the above statements is/are incorrect?


73. Consider the following pairs.
a. Only one
GI Tag products States
b. Only two
1. Kapdaganda shawl - Odisha
c. All three
2. Kalonunia Rice - Bihar
d. None
3. Adi Kekir - Sikkim

4. Wancho Wooden Craft - Arunachal Pradesh


70. Pirola, sometimes seen in the news recently, is
related to? How many of the pair(s) given above is/are correctly
matched?
a. A new variant of Coronavirus
a. All four
b. A species of Bamboo which can be used as
reusable straw b. Only two

c. A Nano compound that has ability to trace c. Only three


counterfeit currency
d. None
d. A species of dancing frogs that are endemic to
the Western Ghats
74. World Employment and Social Outlook Report, is
published by?
71. Recently, scientists have observed that an
ineffectiveness of the bivalent or variant-specific a. World Bank
boosters is due to immune imprinting. Consider the
following statements with respect to Immune b. World Economic Forum
Imprinting.
c. International Labour Organisation
1. It is a tendency of the body to repeat its immune
d. UN Department of Economic and Social Affairs
response based on the first variant of a
pathogen it encountered.

2. It can occur through both infection and 75. Consider the following statements with respect to
vaccination. the Thirty Meter Telescope (TMT) Project:
Which of the above statement(s) is/are correct? 1. It is a space based telescope designed to observe
the universe in ultraviolet wavelengths to
a. 1 only
measure the history of star formation in the
b. 2 only universe.

CHENNAI |SALEM| MADURAI | COIMBATORE DELHI | BANGALORE | THIRUVANANTHAPURAM


www.shankariasacademy.com 353
www.iasparliament.com

2. The project will be set up in Mauna Kea, an 78. A replica of wheel with 24 spokes served as the
inactive volcano on the island of Hawaii. backdrop of Indian Prime Minister's welcome
handshake with G20 leaders during 2023 summit. This
3. India is not a participatory in this project. wheel is a replica of which one of the following?

How many of the statement(s) given above a. Lion Capital of Sarnath pillar
is/are correct?
b. Konark Sun Temple of Odisha
a. Only one
c. Hoysaleswara Temple of Karnataka
b. Only two
d. Pillars of Ashoka in Bodh Gaya
c. All three

d. None
79. AITIGA Agreement, sometimes seen in the news, is
a trade deal between India and?

76. Consider the following statements with respect to a. BRICS


the World Cities Culture Forum
b. ASEAN
1. It is an initiative of UNESCO to strengthen
cultural activities, goods, services and c. Arab League
international cooperation for sustainable
development. d. Asia Pacific Forum

2. It covers seven creative fields including


Gastronomy, Literature and Music.
80. Consider the following statements with respect to
3. Kozhikode is the first Indian city to get City of PM-DAKSH Yojana
Literature tag under this forum.
1. It is a centrally sponsored scheme that aims to
How many of the statements given above are correct? improve the skill level of marginalized
individuals.
a. Only one
2. It is being implemented by the Ministry of
b. Only two Micro, Small and Medium Enterprises.

c. All three 3. There is no income threshold for eligibility


under this scheme.
d. None
How many of the statement(s) given above is/are
correct?

77. Consider the following statements with respect to All a. Only one
India Tiger Estimation Report, 2022
b. Only two
1. Bandipur Tiger Reserve holds the highest tiger
population among all tiger reserves in India. c. All three

2. Western Ghats has experienced localized d. None


declines in the tiger population.

3. Madhya Pradesh has the largest tiger


population among all states in India. 81. Consider the following statements with respect to
Graphene
How many of the statement(s) given above is/are
correct? 1. It is a single layer of carbon atoms arranged in
a hexagonal lattice.
a. Only one
2. It is used in touch screens, light panels and
b. Only two solar cells.

c. All three 3. Graphene-Aurora Program in India is


implemented by the Ministry of Science and
d. None Technology.

CHENNAI |SALEM| MADURAI | COIMBATORE DELHI | BANGALORE | THIRUVANANTHAPURAM


www.shankariasacademy.com 354
www.iasparliament.com

How many of the statement(s) given above is/are Select the correct answer using the codes given below:
correct?
a. Only two
a. Only one
b. Only three
b. Only two
c. Only four
c. All three
d. All five
d. None

85. Consider the following statements with respect to


82. Consider the following statements with respect to Voter Verifiable Paper Audit Trail (VVPAT)
Indian Forest & Wood Certification Scheme
1. VVPAT is a device linked to the Electronic
1. It aims to offer voluntary 3rd party certification Voting Machine (EVM) that provides voters
to promote sustainable forest management and with feedback through a ballot-less voting
agroforestry in the country. system.

2. The Scheme was launched by the Ministry of 2. It features a transparent window, displaying the
Commerce and Industry. Name, Serial number and Election symbol for 7
seconds to the voter.
3. Indian Institute of Forest Management (IIFM)
is the operating agency for the overall 3. VVPATs were first used in 1982 in the by-
management of the scheme. election to the North Paravur Assembly
constituency in Kerala.
4. National Accreditation Board for Certification
Bodies (NACB) will accredit the certification How many of the statement(s) given above is/are
bodies. correct?

Which of the above statement(s) is/are correct? a. Only one

a. Only one b. Only two

b. Only two c. All three

c. Only three d. None

d. All four

86. “The Global Fund” is a worldwide movement to


defeat which of the following diseases?
83. Essequibo region, sometimes seen in the news
recently, is a disputed region between? 1. Malaria

a. Russia and Ukraine 2. Tuberculosis

b. Israel and Palestine 3. Human Papilloma Virus

c. Somalia and Ethiopia 4. Human Immunodeficiency Virus

d. Guyana and Venezuela Select the correct answer using the codes given below:

a. 1, 2 and 3 only

84. How many of the following countries are part of b. 1, 2 and 4 only
the Five Eyes Alliance?
c. 1, 3 and 4 only
1. United States
d. 2, 3 and 4 only
2. Australia

3. United Kingdom
87. India has decided to scrap the Free Movement
4. France Regime (FMR), a mutual pact to allow tribes dwelling
along the border to access the other side without a visa.
5. Germany FMR is an agreement signed between India and?

CHENNAI |SALEM| MADURAI | COIMBATORE DELHI | BANGALORE | THIRUVANANTHAPURAM


www.shankariasacademy.com 355
www.iasparliament.com

a. Nepal How many of the pair(s) given above


is/are incorrectly matched?
b. Bhutan
a. Only one
c. Myanmar
b. Only two
d. Bangladesh
c. Only three

d. All four
88. Which of the following statements regarding
Pradhan Mantri Kisan Samman Nidhi (PM-KISAN)
scheme is incorrect?
91. Consider the following statements regarding UN
a. It is a centrally sponsored scheme with 60% from High Seas Treaty
the central government and 40% from the
respective state governments. 1. The treaty was also called as Paris Agreement
for the Ocean.
b. It aims to provide income support of Rs. 6,000
per year to farmers in 3 equal instalments. 2. UN High seas treaty is legally binding in nature.

c. It includes all land-holding farmer families, 3. The treaty was adopted under the framework of
irrespective of the size of their landholdings. United Nations Convention on Laws of the Sea
(UNCLOS).
d. It will supplement financial needs for procuring
various inputs related to agriculture and allied 4. High Seas are the parts of the sea that are not
activities as well as domestic needs. included in the territorial waters of a country.

How many of the statement(s) given above is/are


correct?
89. Consider the following statements with respect to
Wetland City Accreditation (WCA) a. Only one

1. It is a voluntary scheme that recognizes cities b. Only two


which have taken exceptional stapes to
safeguard their urban wetlands. c. Only three

2. It is an initiative under Ramsar Convention. d. All four

3. Indore is the only accredited city in India under


this program.
92. Methane Global Tracker is an annual report
Which of the above statement(s) is/are correct? released by _____?

a. Only one a. Germanwatch

b. Only two b. International Energy Agency

c. All three c. United Nation Environment Programme

d. None d. International Methane Emissions Observatory

90. Consider the following pairs: 93. Consider the following statements with respect to
the United Nations Forum on Forests (UNFF)
Terms in News Entities
1. It was established by United Nations
1. AWaRe Tool – World Health Organization Environment Programme (UNEP) as a
subsidiary body.
2. Prerana Program – Ministry of Health and Family
Welfare 2. It has universal membership, and is composed
of all Member States of the United Nations.
3. Operation AMRITH – Narcotics Control Bureau
Which of the statement(s) given above is/are correct?
4. e-SAKSHI App –Ministry of Electronics &
Information Technology a. 1 only

CHENNAI |SALEM| MADURAI | COIMBATORE DELHI | BANGALORE | THIRUVANANTHAPURAM


www.shankariasacademy.com 356
www.iasparliament.com

b. 2 only How many of the statements given above are correct?

c. Both 1 and 2 a. Only one

d. Neither 1 nor 2 b. Only two

c. All Three

94. Consider the following statements regarding the d. None of the above
Pygmy Hogs

1. They are an indicator species.


97. Consider the following statements with respect to
2. They live only in the wet high grasslands at the Pradhan Mantri Anusuchit Jaati Abhuyday Yojana
foothills of the Himalayas.
1. It aims to reduce poverty of the Schedule Caste
3. They are herbivores and feeds on roots, plants (SC) communities by generation of additional
and tubers. employment opportunities through skill
development.
How many of the above statements are correct?
2. It is a merged scheme of 3 Centrally Sponsored
a. Only one Scheme with Adarsh Gram being one of the
components of the yojana.
b. Only two
3. The scheme functions under aegis of the
c. All three Ministry of Skill Development and
Entrepreneurship.
d. None
How many of the statement(s) given above is/are
correct?
95. Consider the following statements with respect to a. Only one
Parliamentary Committees.
b. Only two
1. An allegation of corruption against a Member of
Parliament (MP) can be sent to either Ethics c. All three
Committee or Privileges Committee.
d. None
2. An allegation of corruption against a non-
Member of Parliament can be sent only to
Privileges Committee and not to Ethics
Committee. 98. The 2023 Nobel Prize for Physics which awarded for
experimental methods generating attosecond pulses,
Which of the above statement(s) is/are correct? that corresponds to which one of the following?
a. 1 only a. They are sound pulses, which used to unravel
dynamical processes in matter with unprecedented
b. 2 only time resolution.
c. Both 1 and 2 b. They are extremely short light pulses which are
used to study superfast processes.
d. Neither 1 nor 2
c. It is one trillionth of a second, the timescale at
which the properties of an electron change.
96. Consider the following statements with respect to d. It is a beam of sound consists of oscillating
the Dancing Frogs electric and magnetic fields.
1. They are endemic to Western Ghats.

2. Foot-flagging is a unique behaviour to it that 99. Abhyudaya, a hindi weekly magazine, was authored
serves the dual purpose of attracting females by?
and warning other males in the vicinity.
a. Madan Mohan Malaviya
3. According to the second edition of the Global
Amphibian Assessment, dancing frogs are the b. Gopal Krishna Gokhale
most threatened amphibian genus of India.

CHENNAI |SALEM| MADURAI | COIMBATORE DELHI | BANGALORE | THIRUVANANTHAPURAM


www.shankariasacademy.com 357
www.iasparliament.com

c. Usha Mehta

d. Nanaji Deshmukh

100. Consider the following pairs with respect to the


Chola Dynasty

Terms Meaning

1. Kalam – Sailing vessels

2. Salai – Army training centre

Which of the pair(s) given above is/are correctly


matched?

a. 1 only

b. 2 only

c. Both 1 and 2

d. Neither 1 nor 2

CHENNAI |SALEM| MADURAI | COIMBATORE DELHI | BANGALORE | THIRUVANANTHAPURAM


www.shankariasacademy.com 358
www.iasparliament.com

Answer Key - Test - VII


1 2 3 4 5 6 7 8 9 10

B A B A C D D A D B

11 12 13 14 15 16 17 18 19 20

A B C B B C D D A A
21 22 23 24 25 26 27 28 29 30
D B C D C A B A C C
31 32 33 34 35 36 37 38 39 40
A D B C D B C D D C
41 42 43 44 45 46 47 48 49 50
C A A B D B C B A C
51 52 53 54 55 56 57 58 59 60
B D B D A D D C A C
61 62 63 64 65 66 67 68 69 70
C C C C C A A A D A
71 72 73 74 75 76 77 78 79 80
C B B C A D B B B B
81 82 83 84 85 86 87 88 89 90
B C D B B B C A B C
91 92 93 94 95 96 97 98 99 100
D B B B C C B B A C

EXPLANATION  Global Centre for Traditional Medicine


(GCTM) – Headquarter is located
1. b
in Jamnagar, Gujarat.
Traditional Medicine Global Summit, 2023
2. a
 It is the first WHO Traditional Medicine W12+ Blueprint
Global Summit held in
Gandhinagar, Gujarat, India. The W12+ Blueprint committed knowledge sharing for
water security challenges at Un Water Conference
 It was held alongside the G20 health 2023.
ministerial meeting.
 About - An informational database
 Aim - To mobilize political commitment and for solutions to urban water challenges.
evidence-based action on traditional medicine
 Launched by – 2 NGOs (SOS and ECOCIV) in
 Host - Co-hosted by the WHO and the partnership with UNESCO IHP.
Government of India, which holds the
presidency of the G20 in 2023.  Aim - To create a tool that provides insight on
how to address common urban water
 The stakeholders will share best practices and challenges through easy to read case studies.
game-changing evidence, data and innovation
on the contribution of traditional medicine to  It is dynamic, interactive knowledge-sharing
health and sustainable development. database of case studies and best-practice water
solutions at city-level.

CHENNAI |SALEM| MADURAI | COIMBATORE DELHI | BANGALORE | THIRUVANANTHAPURAM


www.shankariasacademy.com 359
www.iasparliament.com

 2 cities from India are listed in this  All these islands are located at the crossroads of
Blueprint strategically important maritime trade
corridors.
o Bengaluru – Community Leadership
for Water Source Rejuvenation.  Of the 14 PICs, Fiji and Papua New Guinea
(PNG) are the ones with the biggest populations
o Chennai – Rainwater Harvesting and the most heft.
Policy for Enhanced Supply.

3. b

SNAP-10A

 It is the world’s first operational nuclear


reactor in space.

 Launched in – 1965 by the U.S.

 It was built as a result of the System for Nuclear


Auxiliary Power (SNAP) programme or
SNAPSHOT for Space Nuclear Auxiliary Power
Shot.

 Objective – To develop compact, lightweight,


reliable atomic devices that could then be
employed in space, sea, and land.
5. c
 Reactor – Enriched uranium fuel with
zirconium hydride as a moderator, and liquid Alpenglow
Na-K alloy as the coolant.
Alpenglow phenomenon was recently seen near the
 A thermoelectric converter was used to directly Hindu Kush mountain range.
convert heat from the reactor into electricity.
 Alpenglow is a reddish hue seen near
 Russia has sent quite a few of them, including mountain summits during sunrise or
one that crashed and scattered radioactive sunset.
debris over Canada in 1978.
 It refers to the indirect sunlight reflected or
 NASA's TEMPO – The Tropospheric diffracted by the atmosphere before sunrise or
Emissions: Monitoring of Pollution. after sunset, giving a horizontal reddish glow
near the horizon opposite the Sun.
 An instrument to measure pollution and air
quality across greater North America on an  It occurs when direct sunlight around sunrise
hourly basis during the daytime. or sunset is reflected off airborne precipitation,
ice crystals or particulates in the lower
 Voyager 2 and Voyager 1 – They are the only atmosphere, as the sunlight has no direct path
spacecraft that have ever operated outside the to reach a mountain.
heliosphere, which is considered to be the
border of our solar system.  After sunset, if there are no mountains the
aerosols in the eastern sky can be lit up by the
4. a remaining red light scattered above the edge of
Earth's shadow.
Pacific Island Countries (PICs)
 It produces a pinkish band on the opposite side
 PICs is a cluster of 14 island nations dotting the of the Sun's direction, named the Belt of Venus.
South-western Pacific. They are
6. d
 The Cook Islands, Fiji, Kiribati, the Marshall
Islands, Micronesia, Nauru, Niue, Samoa, Maratha Military Landscapes
the Solomon Islands, Palau, Papua New
Guinea, Tonga, Tuvalu and Vanuatu. India nominates 12 forts of Marathas as ‘Maratha
Military Landscapes’ for inclusion in the UNESCO
 Comoros and Barbados are not the World Heritage list for 2024-25.
part of the Pacific Island Countries (PICs).
 Maratha Military Landscapes – A
fortification and military system envisioned by

CHENNAI |SALEM| MADURAI | COIMBATORE DELHI | BANGALORE | THIRUVANANTHAPURAM


www.shankariasacademy.com 360
www.iasparliament.com

Maratha rulers and developed between 17th 7. d


and 19th centuries.
Global Tree Assessment (GTA)
 Integrated
physiography – Sahyadri Mountains, Kon  Launched in - 2015
kan Coast, Deccan Plateau and the Eastern
Ghats.  Coordinated by - Botanic Gardens
Conservation International (BGCI) and
 They vary in hierarchies, scales and typological the International Union for
features. Conservation of Nature (IUCN) Species
Survival Commission Global Tree Specialist
 Maratha Military Landscapes in World Group (IUCN-SSC GTSG).
Heritage List – Only 12 forts are nominated
as Maratha landscapes (11 are in Maharashtra  It is a comprehensive list of species and critical
and 1 is from Tamil Nadu). health indicator of global world bio-diversity.

 Earlier, it was included in the Tentative List of  The 6 major threats categorised by the Global
World Heritage sites in 2021. Tree Assessment are:

 It is the 6th cultural property nominated from 1. Crop agriculture expansion


Maharashtra.
2. Tree cover decline as a proxy for
 Out of 6 criteria for cultural category in WHS, overexploitation in all vegetation types
the landscape has 3 criteria.
3. Urbanisation
Twelve Forts
4. Deforestation as a proxy for land-
 Hill forts - Salher fort, Shivneri fort, Lohgad, use change threats in forested areas,
Raigad, Rajgad and Gingee. changes in burnt area descaled as fire
and fire suppression
 Hill-forest fort – Pratapgad.
5. Climate change measures in terms
 Hill-plateau fort – Panhala. of changes in annual variation in
figures of temperatures and
 Coastal fort – Vijaydurg. precipitation

 Island forts – Khanderi fort, Suvarnadurg 6. Seasonality and vapour pressure deficit
and Sindhudurg. (VPD) and VPD seasonality

UNESCO World Heritage Sites 8. a

Model Code of Conduct (MCC)


 At present in India there are 42 World
Heritage sites, out of which 34 are
 About - A code that contains some general
cultural sites, 7 are natural sites whereas
precepts for model behaviour during elections
one is mixed site.
conducted by Election Commission (EC).
 In Maharashtra there are 6 World Heritage
 The code comes into force immediately
Sites (5 cultural and 1 natural). They are:
when the election dates are
o Ajanta Caves (1983) declared and remains till the results are
announced.
o Ellora Caves (1983)
 It has 8 chapters, with one dedicated to what
o Elephanta Caves (1987) the party in power can and cannot do once
elections are announced by the EC.
o Chhatrapati Shivaji Maharaj Terminus
(formerly Victoria Terminus) (2004)  MCC evolved as part of the ECI’s drive to ensure
free and fair elections, and was the result of a
o Victorian Gothic and Art Deco consensus among major political parties.
Ensembles of Mumbai (2018) and
 It has no statutory backing.
o Western Ghats of Maharashtra,
Karnataka, Tamil Nadu and Kerala is  Simply put, this means anybody breaching the
serial property in natural category MCC can’t be proceeded against under any
(2012). clause of the Code. Everything is voluntary.

CHENNAI |SALEM| MADURAI | COIMBATORE DELHI | BANGALORE | THIRUVANANTHAPURAM


www.shankariasacademy.com 361
www.iasparliament.com

 A version of the MCC was first introduced in 10. b


the state assembly elections in Kerala in
1960. Almora Fault

 The code’s purpose, emphasising it only  Almora Fault – Located in Western


prohibits new projects, programmes, financial Nepal.
grants or promises that could influence the
electorate in favour of the ruling party.  Nepal and the adjoining northern part of India,
are highly seismically active areas prone to
 The MCC restrictions are solely applicable to earthquakes due to collision tectonics, where
states undergoing elections. the Indian plate subducts beneath the Eurasian
Plate.
 It forbids use of official machinery and
personnel for the political gains of the party in  It separates the inner lesser Himalayas in the
power. north from outer lesser Himalayas in the south.

9. d

Global Innovation Index (GII)

 It is a leading reference for measuring an


economy’s innovation ecosystem performance.

 Published by - The World Intellectual


Property Organization.

 For the 13th year in a row, Switzerland is the


most innovative economy in 2023 followed by
Sweden, the United States, the United Kingdom
and Singapore.
Kopili Fault
 In 2023, Sweden has overtaken the United
States and has climbed to second position.  Location – North Eastern Region of
India
 Singapore has entered the top five and has
taken the leading position among South East  Range – Extends from the western part of
Asia, East Asia and Oceania (SEAO) region Manipur to the tri-junction of Bhutan,
economies. Arunachal Pradesh, and Assam.

India  It is one of the active fault that has experienced


large earthquakes and falls into the Highest
 India retains 40th rank out of Seismic Hazard Zone V.
132 economies in the Global Innovation Index
2023 rankings.

 India has topped the lower-middle income


group category in the index.

 India has been on a rising trajectory, over the


past several years in the GII.

 The rank of India in the index was 81 in 2015,


which improved to 40 in 2023.

 The consistent improvement in the GII ranking


is owing to:

o Immense knowledge capital.

o Vibrant start-up ecosystem.


Julian Felipe Reef
o Work done by the public and private
research organizations.  Location - Spratly Islands of the South
China Sea.

CHENNAI |SALEM| MADURAI | COIMBATORE DELHI | BANGALORE | THIRUVANANTHAPURAM


www.shankariasacademy.com 362
www.iasparliament.com

 It lies in the Exclusive Economic Zone of  Protection status


Philippines.
o Wildlife (Protection) Act, 1972 –
 It's also known as Whitsun Reef, Whitson Reef, Schedule IV
and Whitsum Reef.
o CITES – Appendix II

o IUCN Status – Near Threatened

12. b

Household Consumption Expenditure Survey

Ministry of Statistics & Program Implementation has


released Household Consumption Expenditure Survey
2022-23.

 Conducted by – National Sample Survey


Office (NSSO) under Ministry of Statistics and
Program Implementation.

 NSSO merged with the Central Statistical Office


(CSO) to form the National Statistical Office
(NSO) since 2019.

 Publication – It is supposed to be held every


5 years.

 2017-18 survey – Not released citing data


quality issues.
11. a
 Aim – To generate estimates of household
Red Sand Boa (Eryx johnii)
Monthly Per Capita Consumption Expenditure
(MPCE) and its distribution for
 It is commonly called as the Indian Sand Boa.
o Rural and urban sectors
 It is a non-venomous
species found throughout the dry parts of the o States and Union Territories, and
Indian subcontinent.
o Different socio-economic groups.
 The sand boa is a small burrowing snake.
 Information collected in HCES is useful for
 The nine species inhabit arid lands in Africa, understanding
southeastern Europe, the Middle East and
India. o Consumption and expenditure pattern,

 tis a primarily reddish-brown and thick-set o Standard of living and


snake that grows to an average length of 75 cm.
o Well-being of the households.
 Unlike most snakes, the tail is almost as thick as
the body and gives the reptile the appearance of o To update Consumer Price Inflation
being double-headed. (Retail inflation) basket.

 They are the largest of the sand boas in Household Consumption Expenditure Survey
the world. 2023

 The red sand boa is now acknowledged as one  Covered whole of the Indian Union except a few
of the most traded reptile species in the inaccessible villages in the Andaman and
illegal trade market, due to its demand in Nicobar Islands.
the pet trade, as well as for use in black magic.
 3 broad categories of consumption
 They are ovoviviparous and basket – Food items, consumables & services
nocturnal and spends the majority of its time items and durable goods.
under the ground.
 Findings – Consumption
expenditure has increased by over 2.5

CHENNAI |SALEM| MADURAI | COIMBATORE DELHI | BANGALORE | THIRUVANANTHAPURAM


www.shankariasacademy.com 363
www.iasparliament.com

times from 2011-12 levels in current prices, and  The state generally allots such land to any entity
the growth is faster in rural India than urban on lease for a fixed period.
India.
 The government generally uses Nazool land for
 MPCE – A measure of the average spending of public purposes like building schools,
a person in a month on various goods and hospitals, etc.
services.
 The Nazool Lands (Transfer) Rules, 1956 is the
 It is based on Modified Mixed Reference Period law mostly used for Nazool land adjudication.
(MMRP).
14. b
 India's per capita monthly consumption
expenditure was 33-40% higher in 2022- Horizon 2047
23 than 2011-12.
The French President was the chief guest at India’s
 The average monthly per capita consumption Republic Day 2024, making it his third visit to India,
expenditure (MPCE) of both urban and rural after his 2018 state visit and G-20 2023 summit hosted
households doubled in the 11-year period with by India.
both spending lesser on food items.
 Diplomatic relation – In 1947, both
 Of this 46% of the expenditure was on food countries established diplomatic relation which
items in rural households and 39% in urban upgraded to strategic partnership in 1998.
homes in 2022-23.
 Horizon 2047: Charting the future of
India-France Strategic Partnership
Key aspect High/low State/ UT
 Signed in – 2023, to mark the 25th
Both rural High Sikkim anniversary of the Indo-French partnership.
and urban
areas  Both countries agree to adopt a roadmap to
Low Chhattisgarh set the course for the bilateral relationship
up to 2047.
Meghalaya (83%)
 Three pillars - ‘Security, planet and people’
High in States Chhattisgarh guides the bilateral ties for the next 25 years.
(82%)
 New Horizons – It is an interplanetary space
Rural-
High in UT Chandigarh probe launched as a part of NASA's New
urban
Frontiers program.
difference
in average 15. b
MPCE Ladakh- In rural
areas UN Human Rights Council (UNHRC)
Low in UT
Lakshadweep- In Russia has failed to regain a seat on the U.N. Human
urban areas Rights Council after a majority of countries in the UN
General Assembly voted against it.
13. c
 It is an intergovernmental body within the
Nazool Land United Nations system.

Violence erupted in Uttarakhand after the demolition  It is a subsidiary of UN General Assembly


drive at the site of a mosque and madrasa, allegedly on (UNGA).
Nazool land.
 It was established in 2006, by replacing the
 Emergence – During the British rule where Commission on Human Rights.
after defeating the opponents, the British would
often take their land away from them.  Membership – 47 States who are elected from
193 members of UN General Assembly by
 After Independence – British vacating these absolute majority through secret ballot system.
lands and became vacant.
 In order to ensure equitable geographical
 Owned by – The respective state distribution, its seats are distributed among
governments but most often not directly regional groups of States that is Africa (13),
administered as state property. Asia-Pacific (13), Eastern European (6), Latin

CHENNAI |SALEM| MADURAI | COIMBATORE DELHI | BANGALORE | THIRUVANANTHAPURAM


www.shankariasacademy.com 364
www.iasparliament.com

American and Caribbean (8), and Western  The 2 types of rat-hole mining includes:
European and others (7).
o Side-cutting – In this method the
 India - India is a member. narrow tunnels are dug on the hill
slopes and workers go inside until they
 Term – 3 years with a maximum of 2 find the coal seam.
consecutive terms.
o Box-cutting – A rectangular opening
 Meeting – It meets 3 times a year at the is made, varying from 10 to 100 sqm
United Nations Office at Geneva (UNOG). and through that a vertical pit is dug,
100 to 400 feet deep.
 Functions
 Rat hole mining poses significant safety and
o Universal Periodic Review – This environmental hazards.
review by UNHRC assesses the human
rights situations in all UN Member  The mines are typically unregulated, lacking
States. safety measures such as proper ventilation,
structural support, or safety gear for the
o Suspension – The General Assembly, workers.
via a 2/3rd majority, can suspend the
rights and privileges of any Council  The mining process can cause land
member that it decides has persistently degradation, deforestation, and water
committed gross and systematic pollution.
violations of human rights during its
term of membership.  They often persist due to economic factors and
the absence of viable alternative livelihoods for
 Leading entity – Office of the High the local population.
Commissioner for Human Rights (OHCHR) is
the leading UN entity on human rights.  Ban - The National Green Tribunal
(NGT) banned the practice of rat-hole
 Not binding - Its resolutions are not legally mining in 2014, and retained the ban in
binding but carry moral authority. 2015.

 Powers – It has no criminal enforcement 17. d


or sanctioning powers.
Windsor Framework
 The council can undertake investigations that
help to influence a country’s behaviour. UK government reached a landmark deal on post-
Brexit trade rules that will govern Northern Ireland.
 Russia lost the election to UNHRC against
Albania and Bulgaria in the Eastern European  Great Britain – England + Scotland + Wales
Region.
 United Kingdom – Great Britain + Northern
 Russia’s membership was suspended in the Ireland
wake of its full-scale invasion of Ukraine.
 Windsor framework - Agreement
16. c between UK and European Union.

Rat-Hole Mining  It will replace the Northern Ireland Protocol.

 Rat-hole mining is a method of extracting  The framework has two crucial aspects:
coal from narrow, horizontal coal beds
which is prevalent in Meghalaya. o Introduction of Green and red
lane system – For goods that will stay
 The term “rat hole” refers to the narrow pits dug in Northern Ireland and those that will
into the ground, typically just large enough for go to the EU respectively.
one person to descend and extract coal.
o Stormont Brake – Allows Northern
 Once the pits are dug, miners descend using Ireland lawmakers and London to veto
ropes or bamboo ladders to reach the coal any EU regulation they believe affects
seams. the region adversely.

 The coal is then manually extracted using  The two lane system– British goods meant
primitive tools such as pickaxes, shovels, and for Northern Ireland will use the green lane at
baskets.

CHENNAI |SALEM| MADURAI | COIMBATORE DELHI | BANGALORE | THIRUVANANTHAPURAM


www.shankariasacademy.com 365
www.iasparliament.com

the ports, and will be allowed to pass with  It connects the Western and the Eastern Ghats.
minimal paperwork and checks.
 It has the Nilgiri Hills on its south-western side
 Physical checks will be conducted if the goods and the Moyar River Valley on its north-eastern
are deemed suspicious, in place of the routine side.
checks now.
 It is bounded to the northwest by the Bandipur
 Goods destined for Ireland or the rest of the EU National Park, to the east by the
will have to take the red lane, with the Sathyamangalam Wildlife Sanctuary and to the
attendant customs and other checks. west by the Wayanad Wildlife Sanctuary.

 The Stormont Brake – It means the  It sustains elephant populations and their
democratically elected Northern Ireland genetic diversity.
Assembly can oppose new EU goods rules that
would have significant and lasting effects on  It is a part of the World Network of
everyday lives in Northern Ireland. Biosphere Reserves of UNESCO.

 For this, they will need the support of 30  The elephants cross the plateau in search of
members from at least 2 parties and the British food and water.
government can then veto the law.
 Sigur plateau, Nilgiris is the last
southernmost viable breeding
population for the White-rumped
vulture in India.

 The Sigur plateau is home to the Irular


community tribals.

19. a

Pelagic Birds

 Pelagic means relating to or living in


the open ocean, especially the upper layers
far from the shore.

 Pelagic birds spend most of their lives in the


Open Ocean and only return to land to breed
and raise young.

 The decline in fish populations has a direct


effect on the density of pelagic birds.

 Examples – Sooty Shearwater, Brown Skua,


Brown Booby, Streaked Shearwater and
Masked Booby, Frigate birds, Fulmars,
Shearwaters and Tropicbirds.

20. a

Indus Water Treaty (IWT)

 IWT – It was signed in 1960 under


the World Bank's supervision, between
18. d
India and Pakistan.
Sigur Landscape
 Aim – To regulate the use and distribution of
 Sigur Plateau is located in the Nilgiri Hills of the Indus River system between India and
Tamil Nadu. Pakistan.

 Sigur landscape is a “critical node of  India maintains absolute control over the
central connection” between elephant waters of the eastern rivers, the Ravi, Sutlej,
populations in the Western Ghats. and Beas.

CHENNAI |SALEM| MADURAI | COIMBATORE DELHI | BANGALORE | THIRUVANANTHAPURAM


www.shankariasacademy.com 366
www.iasparliament.com

 Pakistan’s jurisdiction – It has unrestricted  IOM is dedicated to promoting humane and


use of the western rivers, the Indus, Jhelum, orderly migration for the benefit of all.
and Chenab.
 India – India was granted an observer's status
 Respective countries can construct dams on to IOM in the year 1991 and became a member
the rivers which are under their control. state in 2008.

22. b

Bharat New Car Assessment Programme


(Bharat NCAP)

 The Bharat New Car Assessment Programme


(Bharat NCAP) is modelled on the Global New
Car Assessment Programme (Global NCAP).

 It is India’s independent vehicle


assessment on safety features and
technologies.

 The new programme will be applicable to


passenger vehicles with not more than 8 seats
in addition to the driver’s seat with gross vehicle
weight not exceeding 3,500 kg.

 It will be based on the soon-to-be published


Automotive Industry Standard 197, which lays
down testing protocols.

 A rating from one star to five stars will


be assigned to a vehicle after an
evaluation of 3 parameters that include:
21. d 1. Adult occupant protection
Kampala Declaration 2. Child occupant protection
A total of 48 African countries have agreed to adopt the 3. Safety assist technologies present in
Kampala Ministerial Declaration on Migration, the car
Environment and Climate Change (KDMECC) to
address the nexus of human mobility and climate  The programme is voluntary except in
change in the African continent. certain cases such as a base variant of a popular
vehicle model (minimum clocked sale of
 The agreement was signed at the conference 30,000 units), or when the Ministry
hosted by the Governments of Kenya and recommends a model for testing based on
Uganda. market feedback or in the interest of public
safety.
 The conference was supported by International
Organization for Migration (IOM) and the 23. c
United Nations Framework Convention on
Climate Change (UNFCCC). Anusandhan National Research Foundation
(NRF) Act, 2023
 The Kampala declaration is the first
comprehensive, action-oriented framework led The Union Cabinet has recently approved the
introduction of the National Research Foundation
 by Member States to address climate-induced (NRF) Bill, 2023.
mobility in a practical and effective manner.
 Established in – 2023
International Organization for Migration (IOM)
 Objectives – To seed, grow and promote
 Established – 1951 research and development (R&D) and foster a
culture of research and innovation throughout
 About - A leading intergovernmental India’s universities, colleges, research
organization in the field of migration and works institutions, and R&D laboratories.
closely with governmental, intergovernmental
and non-governmental partners.

CHENNAI |SALEM| MADURAI | COIMBATORE DELHI | BANGALORE | THIRUVANANTHAPURAM


www.shankariasacademy.com 367
www.iasparliament.com

 Powers – It will act as an apex body to  It annually benchmarks the gender parity of 146
provide high-level strategic direction countries since its adoption in 2006.
of scientific research in the country as
per recommendations of the National  4 key dimensions
Education Policy (NEP), 2020.
o Economic Participation and
 It replaced the Science and Engineering Opportunity,
Research Board (SERB), established in
2008. o Educational Attainment,

 Functions – It will forge collaborations o Health and Survival,


among the industry, academia, and government
departments and research institutions, and o Political Empowerment.
create an interface mechanism for participation
 Range - The index provides scores between 0
and contribution of industries and State
and 1, where 1 shows full gender parity and 0 is
governments in addition to the scientific and
complete imparity.
line ministries.
 Findings – Overall, the Southern Asian region
 National Testing Agency is responsible for
has achieved 63.4% gender parity.
conducting national level exams for admission
in higher educational institutions related to
 No country has yet achieved full gender parity.
engineering, medical.
 Iceland is the most gender-equal country in the
24. d
world for the 14th consecutive year and the only
Management Effectiveness Evaluation (MEE) one to have closed more than 90% of its gender
gap.
 MEE is defined as the assessment of how
well protected areas (PAs) are being  India - It had attained parity in
managed, primarily, whether they are enrolment across all levels of
protecting their values and achieving the goals education.
and objectives agreed upon.
 It had closed 64.3% of the overall gender gap
 MEE is the framework of the World but had reached only 36.7% parity in economic
Commission on Protected participation.
Areas of International Union for
Conservation of Nature and Natural  There was an increase in parity in wages and
Resources (IUCN). income but the shares of women in senior and
technical roles dropped slightly compare to
 India is the only nation in the world to 2022.
have institutionalised and effectively
completed five cycles of MEE of Tiger Reserves  On political empowerment, India has registered
in the country. 25.3% parity, with women representing 15.1%
of parliamentarians, the highest for the country
5th cycle of MEE of Tiger Reserves since the inaugural report in 2006.

 It is jointly conducted by the National Tiger  In sex rate at birth it is increased by 1.9% points
Conservation Authority (NTCA) and the compared to the 2022, index.
Wildlife Institute of India (WII).
 Neighbouring Countries that ranks ahead of the
 India has a network of 55 Tiger Reserves. India were Nepal, Srilanka, China, Bhutan and
Bangladesh.
 Periyar Tiger Reserve achieved the top score of
94.53. 26. a

25. c Lakhpati Didi Initiative

Global Gender Gap Report, 2023 The Ministry of Rural Development has signed a MoU
with the Ministry of Ayush to collaborate in making
India climbs 8 places from 135 to 127 in the recently Lakhpati Didis from the SHGs.
released Global Gender Gap Report 2023.
 Launched by – Ministry of Rural
 Published by – World Economic Forum Development.

 Aim – To enable rural SHG women to earn at


least Rs.1 lakh per annum.

CHENNAI |SALEM| MADURAI | COIMBATORE DELHI | BANGALORE | THIRUVANANTHAPURAM


www.shankariasacademy.com 368
www.iasparliament.com

 Lakhpati Didis are women with annual o No compensation will be paid to the
earnings of Rs. 1 lakh and more. person for surrendering and such items
become property of the State
 In Union budget 2024-25, the target for government.
Lakhpati Didi, which was initially set at 2 crore
women, has been enhanced to 3 crore  Extended functions of Zoos – It will also
women. serve as the areas for ex-situ
conservation, rescue centers and breeding
27. b centers apart from exhibition centres.

Wildlife Protection (Amendment) Act, 2022  Dilution of conservation – It allows low


intensity, small-scale fishing in
 Enacted in – 1972. rivers and other water bodies in and
around protected areas.
 Objectives – To prohibit the hunting of wild
animals and to protect and manage of wildlife  Relaxes the norms on grazing of cattle and use
areas and the species in these areas. of drinking water by the local communities
living inside the protected areas until they are
 To establish new protected areas such as settled elsewhere.
National Parks and Wildlife Sanctuaries.
 Allows transfer or transport of a captive
 Importance – It paved the way for elephant for a religious or any other
establishment of The National and State Board purpose subject to such terms and conditions as
for Wildlife, Central Zoo Authority and may be prescribed by the Central Government.
National Tiger Conservation Authority (NTCA).
28. a
 Earlier Amendments – 1982, 1991, 1993,
2002 and 2006. GI Tagged products

2022 Amendment Recently, Goan Cashews, Mahua liquor and Udangudi


Panangkarupatti have received GI tag while
 It empowers both the states and Centre to Ayyampalayam Nettai have applied for the tag.
notify a conservation reserve.
Goan Cashews - Goa
 Rationalised Schedules – Reduces the
number from 6 to 4  Name derived from the Portuguese ‘caju’ or
‘kaju’ in Konkani.
o Schedule I — Animal species with
highest level of protection including  It has unique identity, flavour and taste due to
those which are critically endangered. overall climate of Goa and traditional farming
practices.
o Schedule II – Animal species with a
lesser degree of protection  Cashew feni (brew) was awarded GI in 2009 as
a speciality alcoholic beverage from Goa.
o Schedule III – Protected Plant
species Udangudi Panangkarupatti – Tamil Nadu

o Schedule IV – Specimens listed in the  Karuppukatti is a hard and golden brown


Appendices under CITES (scheduled coarse sugar which is added to food items as
specimens) sweetener.

 Implementation of CITES – Introduced of  It is prepared from the sap (Pathaneer),


new chapter for this and enables Centre to extracted from the Palmyra tree.
designate a management and scientific
authority.  It is prepared in traditional way without any
additional modern strategies and chemical
 Prohibit invasive alien species – additives like Triple super phosphate and
Empowers Centre to regulate or prohibit the phosphoric acid.
import, trade, possession or proliferation of
such species.  It is unique due to presence of red sand dune
soil of this region.
 Surrender of captive animals – Persons
shall voluntarily surrender any captive animals  Low groundwater and atmospheric moisture
or its products to the Chief Wild Life Warden. enriches high sucrose content and is suitable for
long term storage.

CHENNAI |SALEM| MADURAI | COIMBATORE DELHI | BANGALORE | THIRUVANANTHAPURAM


www.shankariasacademy.com 369
www.iasparliament.com

Mahua liquor – Madhya Pradesh  IPCC’s Synthesis Report for the 6th Assessment
Reports cycle (IPCC AR6), shows that global
 It is a traditional sweetened liquor with strong greenhouse gas (GHG) emissions have
floral notes that has been made for ages by the continued to rise and that global warming is
tribal people of India. rapidly approaching 1.5°C.

 It is a pungent, potent drink that is usually  Climate Finance – Developed countries shall
brewed in unorganised, small-scale backyard deliver at least doubling adaptation finance
stills. delivered by 2025 through public, grant-based
financing.
 It is the only pot-distilled and fermented spirit
in the world made from naturally sweet flowers.  To create a new Collective Quantified Goal on
Climate Finance, greater than the current $100
Ayyampalayam Nettai – Tamil Nadu (yet to billion per year floor.
receive GI tag)
 Operationalising Loss and Damage Fund –
 It is a coconut variety is grown in coconut To operationalise UNFCCC centralised carbon
farms just above the dam, bordering the market mechanism by 2024.
catchment area in Dindigul district of Tamil
Nadu.  To implement the Article 6 of the Paris
Agreement of capacity building programme.
 These century-old trees that have more than
60% oil content and an extremely sweet  Global Stocktake – To prepare a roadmap for
the period after 2023 to implement the
 kernel have still not lost their crop yielding recommendations from the first Global
ability. stocktake.
 It has oval shaped nuts that are drought 30. c
resistant and disease resistant.
Bokmal and Nynorsk
 This variety also promotes sustainable
intercropping.  The Nobel Prize in Literature 2023 is awarded
to the Norwegian author Jon Fosse for his
29. c innovative plays and prose.
Dakar Declaration on Climate Change 2023  Fosse was born in 1959 in Norway and writes
in Norwegian Nynorsk. (Bokmal and
Recently, Ministers from 46 Least Developed Countries Nynorsk are the two different standards
(LDC) issued a joint Dakar Declaration on Climate of writing in Norwegian language).
Change 2023.
 His writings spans a variety of genres consisting
 Aim – To outline the LDC’s expectation and of plays, novels, poetry collections, essays,
priorities for 28th Conference of Parties children’s books and translations.
(COP28) to the UN Framework Convention on
Climate Change that will be held in 2023  His style of writing novels famously known
at Dubai, UAE. as ‘Fosse minimalism’.
 Dakar is the capital of Senegal, a  His work has been translated into more than
Western African country. 40 languages.
 Dakar Declaration calls for  Notable works of Jon Fosse:
o Urgent global emissions reductions. o I Am the Wind
o Increased climate finance. o Melancholy
o To operationalise Loss and Damage o Boathouse
Fund.
o The Dead Dogs
 Global Stocktake to close the gaps in global
climate action. o A New Name: Septology VI-VII (finalist
of the International Booker Prize,
 Global Emission Reduction – To revisit 2022)
and strengthen the 2030 targets in their NDCs
to limit warming to 1.5°C.

CHENNAI |SALEM| MADURAI | COIMBATORE DELHI | BANGALORE | THIRUVANANTHAPURAM


www.shankariasacademy.com 370
www.iasparliament.com

31. a  Cartelisation – It is one of the horizontal


agreements that shall be presumed to have
Competition Commission of India appreciable adverse effect on competition.

 Established in – 2003 32. d

 Nodal ministry – Ministry of Corporate Uniform Civil Code


Affairs
The Uttarakhand passed the Uniform Civil Code (UCC)
 Statutory body – Under Competition Act, 2024 Bill in the Assembly.
2002.
 UCC – It seeks to create a uniform set of laws
 Aim – To establish a competitive environment to replace the distinct personal laws of every
in the Indian economy by engaging with all religion pertaining to subjects such as marriage,
stakeholders, the government, and divorce, adoption, and inheritance.
international jurisdiction
 It is defined in Article 44 as part of the
 Raghavan committee – Based on the Directive Principles of State Policy (DPSP), in
recommendations the Monopolies and part of Part IV of the Constitution – “The State
Restrictive Trade Practices Act, 1969 (MRTP shall endeavour to secure for the citizens a
Act) was repealed and replaced by the uniform civil code throughout the territory of
Competition Act, 2002. India”.

 Composition – A Chairperson and not more  Shah Bano Begum case, 1985 – The Court
than 6 Members. observed that “it is a matter of regret that

 Appointed by – Central Government.  Article 44 has remained a dead letter” and


called for its implementation.
 Appellate mechanism – Competition
(Amendment) Act 2009 provided for  Goa – It is the 1st state in India that had a
the Competition Appellate Tribunal uniform civil code regardless of religion, gender
(COMPAT) to hear and decides appeals and caste.
against the orders of the CCI.
 But it was inherited from the Portuguese
 COMPAT has been replaced with Civil Code, 1867 when it joined the Indian
the National Company Law Appellate Union in 1961.
Tribunal (NCLAT) in 2017.
 Uttarakhand – It became the first Indian
 Role – It acts as a Quasi-judicial body. state to introduce and implement
UCC after India’s Independence.
 Competition Commission Act, 2002 – It
aims to remove all impediments to competition  However, according to section 2 of the code, the
law does not apply to Scheduled Tribes of the
 among strong groups, whether they be private state.
or public.
33. b
Terms associated with CCI
Paintings in News
 Gun jumping – If the combining parties close
a notified transaction before the approval, or  Subika paintings – They are
have consummated a reportable transaction from Manipur and are based on stories and
without bringing it to the Commission’s events from the Hindu epic and Puranas.
knowledge, it is seen as gun-jumping.
o It is intricately linked to
 The penalty for gun-jumping was a total of 1% the Meitei community’s.
of the asset or turnover.
 Basohli painting – A traditional art form that
 Cartel – An association of producers, sellers, originated in Basohli, Jammu and Kashmir.
distributors, traders or service providers who,
by agreement amongst themselves, limit, o Basohli painting of Kathua district is
control or attempt to control the production, the 1st independent GI tagged product
distribution, sale or price of, or, trade in goods from Jammu region.
or provision of service.
o Basohli and Kangra 2 contrasting
schools of Pahari, is a miniature
painting

CHENNAI |SALEM| MADURAI | COIMBATORE DELHI | BANGALORE | THIRUVANANTHAPURAM


www.shankariasacademy.com 371
www.iasparliament.com

 Pichwai paintings – Originated in 17th o To search for possible new generations


century in Rajasthan, a unique blend of of quarks or leptons.
Rajasthani, Mughal, and Hindu motifs.
o To confirm/disprove the elementary
o The devotees of the Vallabhacharya nature of quarks/leptons.
sect in the early 20th century revived
the art. o To discover direct evidence for the
particle responsible for the dark matter
o Kamal Kunj is one of the world’s largest in the Universe.
Pichwai paintings.
 Higgs Boson - Sometimes referred to as the
 Thangka paintings - Belongs to Arunachal God particle Higgs Boson was first discovered
Pradesh, Sikkim, Ladakh and Himachal. by the ATLAS and CMS detectors in 2012
and is considered as the last remaining piece of
o It is deriving from the pattachitra the standard model of particle physics.
painting as well as the Buddhist
practice of drawing mandalas on the 35. d
ground.
Advantages of Volcanoes
 Pattachitra – It means "picture on cloth" was
born in Odisha and West Bengal both holding  Volcanism creates new landforms.
separate GI tag.
 Volcanic rocks yield very fertile soil upon
34. c weathering and decomposition.

Large Hadron Collider  The Kimberlite rock of South Africa, the


source of diamonds, is the pipe of an ancient
 LHC is single stage proton- volcano.
proton machine.
 In the vicinity of active volcanoes, waters in the
 It energizes the protons by accelerating them depth are heated from contact with hot magma
through a narrow circular pipe. giving rise to springs and geysers.

 Large - It is the world’s largest powerful  They help to stabilize the heat of the core
particle accelerator (27 km long circular part of our planet.
pipe).
 They form new land forms after the drying
 Hadron - The particles used here are Hadrons process of liquid lava.

 Collider - It accelerates 2 beams of particles in  The lava contains different minerals which
opposite directions and smashes them head on enriches existing soil.

 Hadrons - A hadron is a subatomic  It facilitate moderation of climate and receive


particle that contains quarks, antiquarks, higher rainfall than flat areas.
and gluons.
36. b
 Built by – European Organisation for Nuclear
Research (CERN) Pangolins

 A proton is made to move by turning on one  They are among the most trafficked wild
hemisphere of magnets and turning off the mammals globally.
other, such that the magnetic field acting on the
proton causes it to move clockwise.  Features – When threatened, they uses
keratin scales as armour to defend itself
 Eventually, the protons move at 99.999999% of against predators.
the speed of light.
 2 species in India – Indian Pangolin, found
 Goals across the subcontinent and the Chinese
Pangolin.
o To understand the fundamental
structure of matter  Distribution - Indian Pangolin is widely
distributed in India, except the arid region,
o To discover Higgs boson and determine high Himalayas and the North-East.
how particles acquire mass.
 The species also occurs in Bangladesh,
Pakistan, Nepal and Sri Lanka.

CHENNAI |SALEM| MADURAI | COIMBATORE DELHI | BANGALORE | THIRUVANANTHAPURAM


www.shankariasacademy.com 372
www.iasparliament.com

 Protection status of both species governments (Delhi, Haryana, Punjab,


Rajasthan, and Uttar Pradesh).
o Schedule I - Wildlife Protection Act,
1972. o Planning and executing plans to
prevent and control air pollution in the
o Appendix I- Convention on National Capital Region (NCR).
International Trade in Endangered
Species of Wild Flora and Fauna o Preparing various action plans such as
(CITES). increasing plantation and addressing
stubble burning.
o IUCN Status
o Investigate, conducts research and
 Indian Pangolin - issue directions related to
Endangered environmental pollution impacting air
quality.
 Chinese Pangolin - Critically
Endangered  The Commission is required to form sub-
committees on
37. c
o Monitoring and identification,
Commission for Air Quality Management
(CAQM) o Safeguarding and enforcement and

 Commission for Air Quality Management o Research and development.


(CAQM) was setup under the Commission for
Air Quality Management in National Capital  These sub-committees will be headed by a
Region and Adjoining Areas Ordinance, 2021. member, the chairperson and a technical
member respectively.
 Commission for Air Quality
Management (CAQM) has recently 38. d
announced the implementation of the
Stage 3 of Graded Response Action Plan Gupteswar Forest
(GRAP).
 Gupteswar forest in Odisha declared as Bio-
 The GRAP has 4 categorises that includes: Diversity Heritage site (BHS).

1. Stage I - ‘poor’ (AQI 201-300).  The forest has been declared as the 4th BHS.
The other 3 are Mandasaru BHS, Mahendragiri
2. Stage II - ‘very poor’ (AQI 301-400). BHS, and Gandhamardan BHS.

3. Stage III - ‘severe’ (AQI 401-450).  The limestone caves of Gupteswar are adorned
with eight species of bats out of the total 16
4. Stage IV - ‘severe plus’ (AQI above species found in southern Odisha.
450).
 Among them, two species Hipposideros
 The Commission will be the sole authority with galeritus and Rhinolophus rouxii are under the
jurisdiction over matters defined in the near-threatened category of the International
Ordinance (such as air quality management). Union for Conservation of Nature.

 In case of any conflict, the orders or directions Umiam Wetland


of the Commission will prevail over the orders
of the following:  As per the Asian Waterbird Census, 2024, there
are over 155 water birds found at the wetland
o Respective state governments, areas of Umiam Lake.
o Central Pollution Control Board  Umiam Lake (Barapani Lake) is the largest
(CPCB), artificial lake in Meghalaya.
o State PCBs and  The name, Umiam, which means ‘water of
tears’, derives from a local legend of 2 sisters
o State-level statutory bodies.
who wished to descend from their abode in
heaven.
 Functions of the Commission includes:

o Co-ordinating actions taken under the


Ordinance by concerned state

CHENNAI |SALEM| MADURAI | COIMBATORE DELHI | BANGALORE | THIRUVANANTHAPURAM


www.shankariasacademy.com 373
www.iasparliament.com

Pangolakha Wildlife Sanctuary Pradesh are famous for their exotic designs
and colours.
 It is situated at the tri-junction of Sikkim,
Bhutan and West Bengal. 40. c

 The Pangolakha range in the east separates Right to be forgotten


Sikkim from Bhutan.
 Right to be Forgotten – It is the right to
 It is home to various species, including red remove or erase content so that it’s not
pandas, snow leopards, Himalayan musk, deer, accessible to the public at large.
goral and black bears.
 It empowers an individual to have information
 The area includes Lake Tsongmo, a in the form of news, video, or photographs
biodiversity hotspot for migratory birds. deleted from internet records so it doesn’t show
up through search engines, like Google.
 Major rivers like Rangpo and Jaldhaka
originate from nearby lakes within the  Origin - The Right to be Forgotten originates
sanctuary. from the 2014 European Court of Justice.

39. d  Thereafter, it was included in the EU’s General


Data Protection Regulation (GDPR) in addition
 Recently 3 indigenous products of Arunachal to the right to erasure.
Pradesh such as Yak churpi, Khamti rice &
Tangsa textile received GI tag.  Section 43A of the IT Act, 2000 – It says
that organizations who possess sensitive
 Yak churpi – It is the first ever yak milk personal data and fail to maintain appropriate
product, Arunachal Yak Churpi, to receive security to safeguard such data, may be
the Geographical Indication. obligated to pay damages to the affected person.

 Churpi is prepared from the milk of Arunachali  IT Rules, 2021 does not include this
yak, a unique breed found in West Kameng and right.
Tawang districts of the state.
 However, it lays down the procedure for filing
 It is reared by tribal yak pastoralists known as complaints with the designated Grievance
Brokpas who migrate along with their yaks to Officer to have content exposing personal
higher reaches during summers and descend to information about a complainant removed
mid-altitude mountainous regions in winters. from the internet.

 Yaks cannot survive in lower altitude during  Supreme Court – In 2017 Puttaswamy case,
summer and thus, they need to be taken beyond it held that the right to be forgotten was
13,000 ft. inherent to the right to privacy under
Article 21 of the Constitution by referring to
 The naturally fermented yak milk product, will the European Union Regulation of 2016.
give a major boost to the pastoral production
system and yak rearing in the country.  However, the court also recognized that such a
right can be restricted by the right to freedom of
 Churpi is an excellent source of proteins and is expression and information or for compliance
frequently used as a substitute for vegetables by with legal obligations.

 tribal yak herders in the vegetation-starved cold 41. c


and hilly mountainous regions of the state.
Windfall tax
 It is also mixed with vegetable or meat curry
and is consumed with rice as a staple food in  It is a tax on the profits of a company that
tribal. is derived from an external or
unprecedented event.
 Khaw Tai – Is a chewy sticky rice variety of
Namsai region being cultivated by traditional  In other words, when a company benefits from
Khampti tribal farmers in Arunachal something that they are not responsible for, the
Pradesh. financial gain that ensues is called windfall
profits.
 It is also called as Khamti rice.
 Governments, typically, levy a one-time tax
 Tangsa textile – Textile products of the over and above the normal rates of tax on such
Tangsa tribe of Changlang district, Arunachal profits and that is called windfall tax.

CHENNAI |SALEM| MADURAI | COIMBATORE DELHI | BANGALORE | THIRUVANANTHAPURAM


www.shankariasacademy.com 374
www.iasparliament.com

 For example, the energy price rises as a result of  Clethodim is an oxime O-ether resulting from
the Russia-Ukraine Conflict. the formal conversion of the acyclic keto group
of oxime with chloroallyl group.
 Since, energy companies are gaining profit not
because of any improvement in their processes  It is used as a selective post
emergence herbicide for the control of annual
 but because of the geopolitical situation, many and perennial grasses in numerous crops.
governments are considering to impose such
tax.  The crops includes alfalfa, celery, clover,
conifers, cotton, cranberries, garlic, onions,
 This will boost the government’s finances, and ornamentals, peanuts, soybeans,
help fund efforts to protect vulnerable sections strawberries, sugar beet, sunflowers, and
from rampant inflation. vegetables.

42. a  Effects – Exposure to the widely used


herbicide Clethodim can significantly impact
Office of Deputy Chief Minister male reproductive function and early
embryonic development.
 Constitutional provisions - Deputy CM is a
political post, and it’s not a constitutional  It causes a reduction in testicular weight, a
post like Vice President of India. decrease in germ cell population, lower levels of
serum testosterone, abnormalities in sperm,
 Its origin can be traced to the post of Deputy and compromised pre-implantation embryo
Prime Minister which was appointed in 1947 development.
post-independence, Sardar Vallabhai
Patel is the first Deputy PM of India. 44. b

 Anugrah Narayan Sinha from Bihar was the Cott-Ally App


1st leader to hold the post of deputy CM post-
Independence.  About - A free mobile-based
application to farmers cultivating cotton.
 Appointment and removal - It is entirely at
the discretion of Chief Minister, who  Owned by - Cotton Corporation of India.
can appoint more than one Deputy CM.
 Features - Provides information
o For example: Maharashtra has 2 regarding payment status, purchase
Deputy CMs and Andhra Pradesh has 5 centers available across all cotton growing
Deputy CMs. states in the country, MSP rates, and variety
of cotton, latest news, and notifications.
 Tenure - No fixed tenure as Chief Minister
may reshuffle the portfolio or remove a deputy  This application is developed to maintain the
CM at any point of time. transparency about the business between the
CCI (Cotton Corporation of India Ltd) team and
 Rank - Equivalent to the rank of cabinet the farmers.
minister in the state, and has same pays and
perks as Cabinet Minister. Cotton Corporation of India (CCI)

 It is considered as second highest ranking  It is a Public Sector Undertaking under the


Executive officer of the State Council of administrative control of Ministry of
Ministers. Textiles, Government of India.

 He has no authority to see the files earmarked  CCI operates in all the cotton growing States.
for CM and the portfolios allotted to Deputy CM
are routed to CM via Deputy CM.  The major role of the CCI is to undertake
Minimum Support Price (MSP) operations on
43. a behalf of the Government of India and outreach
with the cotton farmers.
Clethodim
 CCI has developed a mobile app Cott-Ally
Researchers have unveiled alarming insights into the exclusively for the cotton farmers.
potential impact of the widely
used herbicide clethodim on male reproductive 45. d
health.
Veli

CHENNAI |SALEM| MADURAI | COIMBATORE DELHI | BANGALORE | THIRUVANANTHAPURAM


www.shankariasacademy.com 375
www.iasparliament.com

In a fishing village of Andhra Pradesh, the practice of extremely low temperatures approaching
social boycott among the Vaddi community has absolute zero.
adverse social and economic consequences.
 Chiral Bose-liquid state – A new phase
 Veli is the practice of social boycott. discovered by physicists in a frustrated
quantum system, where infinite possibilities
 Those subjected to social ostracisation are not result from the interaction of particles.
allowed to speak to other villagers and in some
cases, compelled to abandon their homes and  Setup – A bilayer semiconductor device or a
seek refuge elsewhere. frustration with the top layer made to be
electron-rich, in which the electrons can freely
 An informal grama peddalu (village elders), move about, while the bottom layer only had
also called a kula (caste) panchayat, of six or holes which are slots an electron can occupy.
seven members nominated for a year by the
community (they are never re-installed), takes  Then the 2 layers are brought extremely close
a call on veli. together.

 The grama peddalu may also enforce tappu  The machine is then triggered to create a local
(wrong), a monetary penalty against imbalance resulting in electrons not having
‘wrongdoers’. enough holes to fill (similar to the game of
musical chairs).
46. b
 This kicks off the novel state called the chiral
Kalaram Temple Bose-liquid state.

 Location – Nashik, Maharashtra.  Significance - In this state,

 Built - It was built in 1792 with the efforts o Electrons can be frozen into
of Sardar Rangarao Odhekar, a Maratha predictable patterns.
nobleman.
o Electrons can be made resilient to
 The Kalaram temple derives its name from changes in spin (a defining
a black statue of the Lord, which is Kala Ram characteristic of subatomic particles)
that translates to Black Ram.
o Electrons can even synchronise
 The sanctum sanctorum has statues of Ram, their movements.
Sita and Lakshman.
48. b
 A black idol of Hanuman is located at the main
entrance of the temple. Qanat System

 The main temple has 14 steps, which represent  The Persian Qanat or Qanat is an ancient
the 14 years of Ram’s exile. irrigation system for transporting water
from underground sources to the surface.
 It is located on the banks of the Godavari
River.  The system originated in Iran around 3,000
years ago.
 Babasaheb Ambedkar – In 1930, B R
Ambedkar led an agitation to demand access for  It can be built in arid regions of Iran,
Dalits to Hindu temples. agricultural and permanent settlements.

47. c  It’s known by a variety of names such


as foggara in North Africa, falaj in Oman
Chiral Bose-Liquid State and qarez in parts of Asia.

A recent research shows that the chiral Bose-liquid  Qanats can only be built where there’s a slope,
state may be an entirely new state of matter. like a mountain or a valley.

 3 classical states of matter – Solid, liquid  Advantages – The qanat is sustainable as it


and gas. works with gravity and no electricity is needed.

 But, exotic or quantum states of matter, such as  Qanats are multiple kilometres long and once
plasma, time crystals, and Bose-Einstein this water hits a floodplain, it can irrigate
condensate also exist at the atomic scale, and at multiple hectares of land.

CHENNAI |SALEM| MADURAI | COIMBATORE DELHI | BANGALORE | THIRUVANANTHAPURAM


www.shankariasacademy.com 376
www.iasparliament.com

49. a  Functions – Other than short-term credit,


they can also provide other input services, like
Chip 4 Initiative seed, fertilizer, and pesticide distribution to
member farmers.
 It includes world’s top semiconductor
producing countries namely the U.S., Japan,  Significance – They ensure financial
Taiwan and South Korea. inclusion of the most vulnerable rural
population.
 It is also known as “Fab 4” initiative. (Fab
refers to an industry term for the fabrication  It account for 41 % of the KCC loans given by all
plants). entities in the Country and 95 % of these KCC
loans are to the Small and Marginal farmers.
 It represents more than 70% of the value of the
global semiconductor industry. 51. b

 Need- To help ensure a stable supply of chips Plastiglomerates


as a bulwark against China's semiconductor
industry. Researchers have found plastic rocks on Trindade
Island in the state of Espirito Santo, Brazil.
 Taiwan – It is the global epicenter of
semiconductor manufacturing with the  Plastic was found in the rocks called
manufacture of over 60% of the world’s plastiglomerates.
chips.
 Plastiglomerates – A mixture of
 Japan – It is dominant in production of critical sedimentary granules and other debris
manufacturing equipment and materials such held together by plastic.
as photoresists.
 This intertwining of plastic with rocks is
 South Korea is a global leader in memory evidence of humans' pollution reaching earth’s
chip production. geological cycles.

50. c  Causes – Plastic Debris like the fishing nets


are dragged by the marine currents and
Primary Agricultural Credit Societies (PACS) accumulate on the beach.

The Cabinet cleared a plan for setting up 2 lakh  When the temperature rises, this plastic melts.
Primary Agricultural Credit Societies (PACS), dairy,
 Over the time,
and fisheries cooperatives in the country in the next 5 the melted
years. plastic became
intertwined
 PACS – It constitute the lowest tier of with rocks on
the short-term cooperative credit (STCC) the island.
structure in the country.
 Trindade Island – It is an archipelago
 Individual farmers are members of the located in the South Atlantic
PACS, and office-bearers are elected Ocean administered by Brazil.
from within them.
 It is an important conservation spot for one of
 A village can have multiple PACS. the largest sea turtles, Green Turtles (Chelonia
Mydas) whose IUCN Status is Endangered.

52. d

Indo-Pacific Economic Framework (IPEF)

 Launched by - USA in 2022.

 Aim - To reassert U.S. economic engagement


and to provide a U.S.-led alternative to China’s
economic statecraft in the region.

 Member countries (14) - USA, Australia,


Brunei Darussalam, Fiji, India, Indonesia,
Japan, the Republic of Korea, Malaysia, New

CHENNAI |SALEM| MADURAI | COIMBATORE DELHI | BANGALORE | THIRUVANANTHAPURAM


www.shankariasacademy.com 377
www.iasparliament.com

Zealand, Philippines, Singapore, Thailand, and Special Category


About Special Status
Vietnam excluding China. Status

 No tariffs - IPEF proposal completely The Constitution


removes the tariff element of typical trade It is granted by (Article 371 to 371-
deals. the National J) provides special
Development status through an
 Four pillars Provision Council, which is Act that has to be
an administrative passed by
o Trade body of the 2/3rd majority in
government both the houses of
o Supply chains Parliament
o Clean energy, decarbonization, and Deals only with Empowers them
infrastructure economic, with legislative
administrative and and political
o Tax and anti-corruption Powers
financial aspects. rights.
 The IPEF is flexible as IPEF partners are not
required to join all four pillars.
11 States - Assam, 12 States-
 India has joined 3 pillars except the trade Sikkim, Manipur, Maharashtra,
pillar. Nagaland, Gujarat, Nagaland,
Himachal Pradesh, Goa, Assam,
 Regional Comprehensive Economic
Applicable Mizoram Manipur, Andhra
Partnership (RCEP) is a Free Trade Agreement
States Meghalaya, Pradesh, Arunachal
(FTA) between China, Japan, South Korea,
Tripura, Arunachal Pradesh,
Australia, New Zealand and the 10 ASEAN
Pradesh, Telangana, Sikkim,
member.
Uttarakhand and Mizoram and
Telangana. Karnataka.
 The 10 ASEAN member are Brunei, Cambodia,
Indonesia, Laos, Malaysia, Myanmar, the
Philippines, Singapore, Thailand and Vietnam.
54. d
 Except USA, India and Fiji all members
of IPEF are members countries of RCEP. Quantum Dots
 The 14 IPEF partners represent 40 % of global Nobel Prize for Chemistry 2023 has been awarded to
GDP and 28 % of global goods and services Alexei I. Ekimov, Louis E. Brus, and Moungi G.
trade. Bawendi for the discovery and synthesis of quantum
dots.
53. b
 Quantum dots – They are man-made
Special Category Status Vs Special Status nanoscale crystals that exhibit unique
optical and electronic properties, including the
 In light of the “Bihar Caste-based Survey, 2022” ability to transport electrons and emit light of
which showed that almost 33% of Bihar’s various colors when exposed to UV light.
people are poor, the State government seeks
Special Category Status.  Effect of light– When light is shined on a
quantum dot, it absorbs and then re-emits
 Special Category Status – It is a it at a different frequency.
classification granted by the Centre to assist the
development of States that face geographical or  By manipulating their size, one can
socio-economic disadvantages. precisely control their properties and make
them emit light of specific colours when
 Launched in – 1969, on the recommendation they are excited by light
of the 5th Finance Commission (FC) and based
on Gadgil formula. o Smaller dots – Emits bluer light
 It was 1st accorded to Jammu and Kashmir, o Larger dots – Emits redder light
Assam and Nagaland in 1969.
 Principle – This happens because light shone
 Discontinued in – 2015, after the 14th on the dot energises some electrons to jump
Finance Commission recommendations. from one energy level to a higher one, before

CHENNAI |SALEM| MADURAI | COIMBATORE DELHI | BANGALORE | THIRUVANANTHAPURAM


www.shankariasacademy.com 378
www.iasparliament.com

 jumping back and releasing the energy at a  Different schools of Pichwai — Nathdwara,
different frequency. Kishangarh and Bundi in Rajasthan and Deccan
school
 Alexei Ekimov – He added different amounts
of copper chloride to a glass before heating it to  Materials used – Natural colors made from
different temperatures for different durations, minerals and plant extracts.
tracking the dopants’ structure and properties.
 Stone pigments are used for gold and silver
 They found that the glass’s colour changed tones.
depending on the size of the copper chloride
nanocrystals.  Vegetable dyes are used for brighter orange,
red, chrome yellow and kesari colours.
 Louis Brus – He and his colleagues prepared
similar crystals in a liquid solution, rather  Usage – As a backdrop for hindu deities in
than in a glass. temples.

 These crystals also interacted with light  They are typically hung behind the idol of
differently depending on small variations in Shrinathji, a local form of Krishna and the
their size. centre of Pushtimarg worship.

 Moungi Bawendi – He devised a simple way  Popularity – It is known for their intricate
to make quantum dots with just the right details, ornate borders, and fine brushwork.
properties using hot-injection method.
Dogra architecture
 Quantum theory – The motion and
behaviour of very small particles are radically  Srinagar Smart City Limited and the Kashmir
different, and strange, when compared with any chapter of INTACH have joined hands to
familiar object in normal human experience. conserve vernacular elements of Kashmiri
architecture recently.

 Dogra architecture is a distinct culture


that formed in Jammu, where cultures
mixed together.

 Dogra Dynasty – Dynasty of Hindu Rajputs


who ruled Jammu & Kashmir from 1846 to
1947.

 Examples – Some examples of Dogra


architecture include:

 Mubark mandi – A hub of Dogra culture until


1947, with a jharokha style of balcony.

 Maharaj Gunj – A market area with a variety


of colonial and vernacular architecture.
55. a  Rani Charak Mahal – A mahal with
phenomenal architecture and interiors.
Pichwai Painting
Vajra Mushti Kalaga
 Origin – It is a 400-year-old art whose roots
are from Nathdwara town in Rajasthan.  The ‘’vajra mushti kalaga’’ is a form of
wrestling or a traditional Indian martial
 Theme – Various stages of Lord Krishna’s art that incorporates various hand-to-hand
life, including his childhood, youth, and combat techniques, such as grappling,
adulthood. wrestling, and striking.
 Paintings – It is typically done on cloth,  It entails two jettys taking a swipe at each
usually khadi but now being produced on other’s head with a small metal weapon called a
knuckleduster.
 various mediums such as paper, canvas, and
silk.  Vajra Mushti matches are still held during
the annual Mysore Dasara festival at
Mysore Palace.

CHENNAI |SALEM| MADURAI | COIMBATORE DELHI | BANGALORE | THIRUVANANTHAPURAM


www.shankariasacademy.com 379
www.iasparliament.com

 The tradition dates back to the Wadiyar  T20 cricket is the sport’s shortest international
dynasty in 1610. format.

 Modern combatants use knuckle-dusters with 57. d


blunt studs. The weapon is sometimes called
Indra-musti. Evacuation Operations

 Whosoever draws the blood from the  Operation Ajay - It is a major operation to
opponent’s head first is declared the winner. evacuate its citizens who wish to return from
conflict-hit Israel.
 Medieval travellers from Portuguese noticed
this form of wrestling during the Navaratri  Operation Kaveri - It brought back several
celebrations in Vijayanagar Empire and have thousand Indian citizens from conflict-
left detailed accounts of it. torn Sudan.

56. d  Operation Ganga – Is an operation executed


by the Indian government to rescue Indians in
Cricket in Olympics the war hit Ukraine.

International Olympic Committee approves cricket  Operation Devi Shakti – Evacuation of


among other 5 sports for 2028 Los Angeles Olympics Indians and Afghans and shipment of
whose final voting will be held soon. humanitarian assistance for the people
of Afghanistan.
 The 5 new sports under consideration are:
58. c
1. Twenty20 cricket
Delimitation
2. Baseball/softball
The delimitation of constituencies for the Lok Sabha
3. Flag football (non-contact American and State Legislative Assemblies is to be carried out on
football) the basis of the first Census after 2026 (Census in
2031).
4. Squash
 It is the act of redrawing boundaries of Lok
5. Lacrosse Sabha and state Assembly seats to represent
changes in population.
 1900 Paris Olympics – Cricket was last
featured at this event.  Constitution – Article 82 and 170 of the
Constitution provide that the number of seats
 Britain beat a side representing France. in the Lok Sabha and State Legislative
assemblies as well as its division into territorial
 It would tap into the lucrative south Asian constituencies shall be readjusted after each
market, attracting fans in countries such as Census.
India and Pakistan.
 Delimitation Commission – It performs
 Roadmap of Cricket in Global stage the delimitation processes under provisions of
the Delimitation Commission Act.
o In 2019, International Cricket Council
(ICC) took a decisive move to make
 It was carried out after the 1951, 1961 and 1971
cricket a global sport.
Census.
o In 2022, it returned to the
 Exemption - Several States such as Assam,
Commonwealth Games 2022, when
Arunachal Pradesh, Nagaland and Manipur are
women's T20s team played.
left out from the exercise due to “security risks.”
 Cricket is an optional sport at the quadrennial
 Appointed by – President of India, and it
Commonwealth Games. It first appeared at
works in collaboration with the Election
the 1998 Commonwealth Games, with a
Commission of India.
men's tournament.
 Composition
 BCCI agreed to come under the ambit of India's
National Anti-Doping Agency, an affiliate of the
o A retired Supreme Court judge
World Anti-Doping Agency (WADA).
o Chief Election Commissioner of
o IOC mandates that every global sport
India
body must be WADA compliant.

CHENNAI |SALEM| MADURAI | COIMBATORE DELHI | BANGALORE | THIRUVANANTHAPURAM


www.shankariasacademy.com 380
www.iasparliament.com

o Respective State Election used fertiliser in Agriculture and


Commissioners. India after urea. included in
the Fertilizer
 Bar on court – Commission’s orders are final Control Order.
and cannot be questioned before any court.
It comes in liquid
 Reserve seats – It is also tasked with form, it has an
identifying seats reserved for Scheduled Castes advantage in terms
and Scheduled Tribes. of surface area to
Formation Granular form.
volume, as its
 42nd Amendment Act, 1976 – It froze the particle size is less
number of Lok Sabha seats as per the 1971 than 100 Nanometre
Census and put off delimitation for 25 years (nm).
until the 2001 Census under Article 82.
Benefits in terms of
 84th Amendment Act, 2001 – Extended the surface area to
deadline for delimitation to 2026. Relatively less volume as the
Efficiency
efficient particle size is less
 Readjustment – The boundaries of territorial than 100
constituencies were readjusted (without nanometres.
changing the number of seats) and seats for SC
and ST were determined as per the 2001 50 kg of DAP is Rs 500 ml of Nano DAP
Census and will again be carried out after 2026. 1350 which is is equivalent to 50 kg
Cost
more than twice bag of conventional
59. a effective
the amount DAP priced at only
of Nano DAP. Rs 600.
Nano DAP Fertilizers
Pocket friendly and
During Interim Budget 2024 the Central government Relatively less
relatively easier to
announced the expansion of the application of Nano Logistics convenient to
store, transport &
DAP on various crops in all agro climatic zones. store and use
use.
 Nano DAP – It is the first in world, launched 60. c
in 2023 by IFFCO
Society for Worldwide Interbank Financial
 IFFCO – Indian Farmers Fertiliser Telecommunications
Cooperative.
 SWIFT is a vast messaging network used by
 It a multi-state cooperative society which is financial institutions to quickly, accurately, and
wholly owned by Cooperative Societies of India, securely send and receive information, such as
established in 1967. money transfer instructions.
 Headquarters – New Delhi.  It is the largest and most streamlined method
for international payments and settlements.
 Liquid nano urea – IFFCO commissioned
the Kalol liquid nano urea plant at Gujarat, the  Established- In 1973.
country’s first in 2021.
 Headquarters- La Hulpe, Belgium.
Conventional
Key aspects Nano DAP
DAP  SWIFT is overseen by the Group of Ten
countries' G-10 central banks.
Di-Ammonium
Phosphate is high  These countries are Belgium, Canada, France,
in phosphorus (P) Germany, Italy, Japan, Netherlands, Sweden,
that stimulates Contains 8% Switzerland, the United Kingdom, and the
root Nitrogen and 16% United States.
About
establishment Phosphorus by
and development, volume.  Bank Identifier Code- SWIFT works by
it is applied just assigning each member institution a unique ID
before or at the code (a BIC number) that identifies the bank
time of sowing. name and the country, city, and branch.
It is the only Nano  It has been used to impose economic sanctions
Usage It is the second
fertilizer approved on Iran, Russia, and Belarus.
most commonly
by the Ministry of

CHENNAI |SALEM| MADURAI | COIMBATORE DELHI | BANGALORE | THIRUVANANTHAPURAM


www.shankariasacademy.com 381
www.iasparliament.com

 Nostro is a bank account that a bank holds  They are large general-purpose language
with a foreign bank in the currency of the models that can be pre-trained and then
country where the funds are held. fine-tuned for specific purposes.

 Magnetic Ink Character Recognition  These models are trained to solve common
(MICR) is a pattern recognition system used language problems such as text classification,
mostly by the bank industry to identify the question answering, and text generation across
document's originality and to enable the industries, document summarization, etc.
processing and approval of cheques and other
papers.  Deep learning techniques – It involves the
training of artificial neural networks, which are
61. c mathematical models which are believed to be
inspired by the structure and functions of the
Surrogacy human brain.
The Centre has allowed married couples with medical  The neural network learns to predict the
complications to use donor gametes for surrogacy, probability of a word or sequence of words
revoking a previous ban. given the previous words in a sentence.
 Surrogacy – A practice where a woman gives
birth to a child for an intending couple with the
intention to hand over the child after the birth
to the intending couple.

 Surrogacy Act 2021 – It prohibits


commercial surrogacy (carrier is paid),
but allows altruistic surrogacy. Types of LLMs
 Eligibility criteria – A mandate certificate of  Autoregressive model – GPT 3 is an
essentiality and certificate of eligibility. example of this model as they predict the
next word in a sequence based on previous
 2023 amendment – The donor eggs could not
words.
be used for gestational surrogacy (In-Vitro
Fertilization).  Transformer
model –
Surrogacy (Regulation) Amendment Rules,
LaMDA or
2024
Gemini are
 The married couples to use a donor based on this as
gamete on the condition that a District they use specific
Magistrate Board certifies that either the type of neural
husband or the wife suffers from a medical network
condition. architecture for
language
 It outlines that the child to be born through processing.
surrogacy must have at least one gamete
from the intending parents.  Encoder-
decoder model – It encode input text into a
 If both partners are unable to use their representation and then decode it into another
gametes due to an existing medical language or format.
condition cannot opt for surrogacy.
 Next word prediction engines – Once
 Other rules – Single women opting for trained, an LLM can predict the most likely next
surrogacy should be a divorcee or a widow. word or sequence of words based on inputs also
known as prompts.
 They must use self-eggs and donor
sperms to avail surrogacy procedure.  Features – LLMs has primary features like
large, parameter and general purpose.
62. c
 Large - This means
Large Language Model (LLM)
o The enormous size of training data
The ability of Generative AI models to converse with
humans and predict the next word or sentence is due to o High parameter count – It encode
LLMs. the knowledge and memories the
model acquires during training.

CHENNAI |SALEM| MADURAI | COIMBATORE DELHI | BANGALORE | THIRUVANANTHAPURAM


www.shankariasacademy.com 382
www.iasparliament.com

 General purpose – It means the model is  He launched the Rampa Rebellion of


sufficient to solve general problems that are 1922 against the British Madras Forest Act of
based on the commonality of human language 1882 which restricted the land rights of the
regardless of specific tasks, and resource tribal.
restrictions.
64. c
 Parameter – It play a crucial role in
determining the model’s performance, it e-Courts Project
defines the model’s ability to solve specific task.
 The e-Court project is an integrated Mission
63. c Mode Project under implementation since
2007 for ICT development of the Indian
U Tirot Sing Judiciary as part of the National e-Governance
Plan.
A statue of U Tirot Sing was unveiled recently at Indira
Gandhi Cultural Centre, Dhaka, Bangladesh  Aim - Universal computerisation of
district and subordinate courts in the
 Tirot Sing is a tribal chief known as ‘the Hero country.
of the Khasi Hills' of Meghalaya against the
British.  The e-Courts Project was conceptualized on the
basis of the “National Policy and Action Plan for
 He was born in 1802 and died in Dhaka on July Implementation of Information and
17, 1835. Communication Technology (ICT) in the Indian
Judiciary in 2005.
 Every year on July 17, Meghalaya
commemorates U Tirot Sing Day.  Based on - Hybrid model that allows for
both physical and virtual courts to co-exist.
Komaram Bheem
 Monitored and funded by - The
 He was born on October 22, 1901, in Sankepalli Department of Justice, Ministry of Law and
in Hyderabad. Justice for the District Courts across the
country.
 He hailed from the Gond tribal community and
led a rebellion against the Nizams of  The project envisages:
Hyderabad.
o To provide efficient & time-bound
 Bheem grew up in the forests without citizen centric services delivery as
education. detailed in e-Court Project Litigant’s
Charter.
 Along with his Gond tribe, he kept moving as
zamindars and businessmen exploited them. o To develop, install & implement
decision support systems in courts.
 He gave the slogan ‘Jal, Jangal,
Zameen’ (Water, Forest, Land), which was o To automate the processes to provide
used in the Aadivasi movement. transparency in accessibility of
information to its stakeholders.
 As the British government levied heavy taxes on
the tribals, nearly 12 villages’ in Adilabadu were o To enhance judicial productivity, both
gearing up to fight the British. qualitatively & quantitatively, to make
the justice delivery system affordable,
 Bheem came back and formed a guerrilla army. accessible, cost effective, predictable,
He operated the battle from Jodgehat forest. reliable and transparent.

 The Nizams were initially surprised by their  The National Judicial Data Grid (NJDG) is an
attack and charted out plans to take them on. online platform under the e-Courts Project.

 During one of those battles, Komaram Bheem 65. c


was killed by the Nizam army.
Methanotrophs
Alluri Sitharama Raju
Methylotuvimicrobium buryatense 5GB1C is a
 He was fondly called Manyam Veerudu, bacterial strain that consumes methane
which roughly translated to 'hero of the forest.' (methanotrophs).

CHENNAI |SALEM| MADURAI | COIMBATORE DELHI | BANGALORE | THIRUVANANTHAPURAM


www.shankariasacademy.com 383
www.iasparliament.com

 Methanotrophs are methane-utilizing  According to the index, no country was strong


bacteria that require methane as a source of enough in all categories to achieve an overall
carbon and energy for their metabolism. very high rating.

 They are gram-negative bacteria that are  India –India receives a high ranking in the
capable in utilizing methane as a carbon energy GHG Emissions and Energy Use categories, but
source. a medium in Climate Policy and Renewable
Energy.
 They are able to grow both aerobically or
anaerobically which only need single-carbon  While India is the world’s most populous
compound to live on. country, it has relatively low per capita
emissions.
 It can grow at low methane concentrations
ranging from 200-1,000 ppm.  Performance of G20 nations – India (7th),
Germany (14th) and the EU (16th) are only 3
 Bacteria produce biomass after consuming G20 countries/regions are among the high
methane, which can be used as feed in performers in CCPI 2024.
aquaculture.

66. a

Climate Change Performance Index (CCPI)

India ranks 7th in CCPI, 2024 which has moved up one


spot from the previous CCPI and remains among the
highest performers.

 CCPI - An instrument to enable transparency


in national and international climate politics.

 An independent monitoring tool for tracking


the climate protection performance of 59 67. a
countries and the European Union.
Tansen
 Published by - Germanwatch, the
NewClimate Institute, and Climate Action  Tansen Samaroh is a musical extravaganza
Network International. celebrated every year in the month of
December, near his tomb at Gwalior district,
 The CCPI uses a standardized framework to Madhya Pradesh.
compare the climate performance of 63
countries and the EU, which together account  The festival is organized to pay tribute to the
for over 90% of global greenhouse gas Great Indian Musical Maestro Tansen.
emissions.
 Tansen popularised and improved the
 Categories – The climate mitigation musical instrument ‘plucked rabab’ (of
performance is assessed in 4 categories: Central Asian origin).

1. GHG emissions  He was among the Navaratnas (9 jewels)


at the court of the Mughal Emperor
2. Renewable energy Akbar who gave him the title Mian, an
honorific, meaning learned man.
3. Energy use
68. a
4. Climate policy
Substance used in doping
 Net Zero is not a category under the
index.  Doping – It is the consumption of certain
prohibited substances by athletes to enhance
 Rating – Each country are given a rating in performance.
each category that includes Very High, High,
Medium, Low, Very Low and Not included. Doping substances

 Denmark remains the top-ranked country but  Androgenic agents like anabolic
does not perform well enough to earn an overall steroid – Allow athletes to train harder and
very high rating. build more muscles.

CHENNAI |SALEM| MADURAI | COIMBATORE DELHI | BANGALORE | THIRUVANANTHAPURAM


www.shankariasacademy.com 384
www.iasparliament.com

 Stimulants – Make athelets hide fatigue by  Aim – To connect China with Europe by land
increasing heart rate and blood flow. and rail, and to Africa and Asia by sea.

 Diuretics & masking agents – To remove  3 major undertakings under Project South
fluids from the body, which can hide other drug Asia
use and can also help in manipulating weight.
o China-Myanmar Economic Corridor
 Gene doping – Manipulating gene cell or (CMEC)
body cells to improve performances.
o Nepal-China Trans-Himalayan Multi-
 Blood doping – Blood is removed from the dimensional Connectivity Network
body and injected back to boost oxygen levels.
o China Pakistan Economic
Monitoring of Doping in sports Corridor (CPEC)

 Globally – World Anti-Doping Agency 70. a


(WADA) monitors doping in all kinds of
competitive sports. Pirola

 At national level (In India) – National  Pirola or BA2.86 is one of the lineages
Anti-Doping Agency (NADA) that is of highly-mutated Omicron variant of
provided statutory backing by National Anti- Covid-19 virus.
Doping Act, 2022.
 Pirola has around 30 genetic changes
69. d (mutations) from its parent Omicron variant.

China Pakistan Economic Corridor (CPEC)  This gives it the capacity to be immune
evasive and highly transmissible.
 It is 3000km long economic corridor.
 Symptoms – Stuff nose, fever, fatigue, cough
 Part of – Belt and Road Initiative (BRI) of and sneezing, rashes, eye redness and
China. diarrhoea.

 Objectives – To circumvent the Straits of  Pirola might have a higher affinity for infecting
Malacca and the South China Sea. individuals who have either previously
contracted COVID-19 or have been vaccinated
 To improve infrastructure within Pakistan for against it.
better trade
with China 71. c
and to
further Immune Imprinting
integrate
the  It is a tendency of the body to repeat its
countries of immune response when it comes across a
South Asia newer or slightly different variant of the same
pathogen.
 To connect
the deep-sea  The immune response of the body is based on
Pakistani the first variant it encountered through
ports of infection or vaccination.
Gwadar and
Karachi to  It was first observed in 1947, when scientists
noted that in flu patients and termed as
China’s
Xinjiang province and beyond by overland ‘original antigenic sin’.
routes.
 Role – It acts as a database for the immune
 Contention – It enters Pakistan system, helping it put up a better response to
Occupied Kashmir (PoK) through the repeat infections.
Karakoram Highway in Gilgit Baltistan.
 Issue - The immune system rather than
Belt and Road Initiative (BRI) generating new B cells, activates memory B

 A connectivity initiative to revive the ancient  cells, when a similar, not identical, variant of
Silk Road across Eurasia and Africa. the virus is encountered by the body.

CHENNAI |SALEM| MADURAI | COIMBATORE DELHI | BANGALORE | THIRUVANANTHAPURAM


www.shankariasacademy.com 385
www.iasparliament.com

 These cross-reactive antibodies do offer some  Kalonuina Rice is a traditional black-husked


protection but aren’t as effective as the ones non-basmati tyoe scented rice of Tarai region
produced by the B cells. (foothills of eastern Himalayas) of West
Bengal, India.
72. b
 Adi Kekir is a variety of ginger produced in
Dar-Es-Salaam Declaration East Siang, Siang and Upper Siang districts
of Arunachal Pradesh known for its size and
 A declaration on ending AIDS in children taste.
by 2030.
 Wancho Wooden Craft is from Arunachal
 It was announced at the first ministerial Pradesh, they are unique as they feature
meeting of the Global Alliance to end AIDS in tobacco pipes with head-shaped bowls and
Children, which brings together the 12 African drinking mugs showing warriors carrying
countries with UNAIDS and other health heads.
agencies.
74. c
 Jaipur Declaration – It was adopted at the
World Security Congress, jointly organized by World Employment and Social Outlook Report,
Railway Protection Force (RPF) and 2023
International Union of Railways (UIC).
 It details the impact of economic slowdown on
 Aim – To explore innovative approaches to the global labour market.
help global Railway organizations to achieve
their longterm goal of safety and security.  Released by- International Labour
Organisation
 UIC committed to provide safe and secure
rail network across globe, by activating Asia-  Time period – Annual report
Pacific, Latin America and African regional
assemblies by 2025.  Theme- The value of essential work.

73. b  2023 report- Calls for a revaluation of the


work of key workers to reflect their social
GI Tag contribution and greater investment in key
sectors.
 Geographical indications assign products to the
place of its origin, conveying an assurance of  Key workers- Food system workers, health
quality and distinctiveness for such goods. workers, retail workers, security workers,
manual workers, cleaning and sanitation
 GI is defined under Article-23 and 24 of workers, transport workers, and technicians
the Trade-Related Aspects of Intellectual and clerical workers.
Property Rights (TRIPS) Agreement of
the World Trade Organization (WTO). Aspects Trend
 Under the Paris Convention for the Protection
of Industrial Property, GIs are covered as an Global
5.1%, a modest improvement on
element of Intellectual Property Rights unemployment
2022
rate
 India is a member of the WTO and thus enacted
the Geographical Indications of Goods Global labour It is expected to decline further in
(Registration & Protection) Act, 1999 that came force 2023, reaching 60.5% the lowest
into force from 2003. participation rate level since 1990.

 GI tagged products can be both Natural and


Man-made and is valid for ten years. It has fallen by 3.3% points since
Global labour
2004 indicating worsening of
income share
 The first product in India to be accorded income inequality
with GI tag was Darjeeling Tea in the year
2004-05. Number of
workers in It is estimated to increase by 34
GI tagged products in India
extreme or million in 2023, reversing the
moderate progress made in the past decade.
 Kapdaganda Shawl is from Odisha, the
poverty
traditional knitted shawls of the hill tribe
Dongaria Kondh are unique.

CHENNAI |SALEM| MADURAI | COIMBATORE DELHI | BANGALORE | THIRUVANANTHAPURAM


www.shankariasacademy.com 386
www.iasparliament.com

They account for 38% of all key  WCCF - It is a global network of cities
Women that share research and intelligence, and
workers globally, though they are
workforce explore the role of culture in future
the majority in health and retail.
prosperity.
75. a  The network currently has 40 cities spanning
six continents.
Thirty Meter Telescope (TMT) Project
 Bengaluru is set to join the league of cities like
 It will be the world’s most advanced and New York, London, Paris, Tokyo and Dubai
capable ground-based optical, near- among others.
infrared, and mid-
infrared observatory. (not ultraviolet)  Bengaluru is the only Indian city in this
forum.
 The TMT has been conceived as a 30-metre
diameter primary-mirror optical and infrared  WCCF will work in collaboration with
telescope that will enable observations into Unboxing BLR Foundation in Bengaluru.
deep space.
 The annual world cities culture summit is
 It is proposed as a joint collaboration involving hosted on a rotating basis by city partners.
institutions in the U.S., Japan, China,
Canada and India.  The summit allows city leaders to share ideas
and knowledge about the role of culture as an
 The project will be set up in Mauna Kea, organising principle for the sustainable city of
an inactive volcano on the island of the future.
Hawaii, United States.
 Helsinki (Capital of Finland) hosted the
India’s participation
World Cities Culture Summit 2022.
 Indian participation in the project was UNESCO Creative Cities Network (UCCN)
approved by the Union Cabinet in 2014.
UNESCO has published the new list of 55 creative cities
 The Aryabhatta Research Institute for on World Cities Day (Oct 31st), in which 2 Indian cities
Observational Sciences (ARIES), Nainital, The Kozhikode and Gwalior were included.
Indian Institute of Astrophysics (IIA),
Bangalore and the Inter-University Center for  Launch Year - 2004
Astronomy and Astrophysics (IUCAA), Pune
are the three main institutes constituting TMT-  Eligibility - UNESCO’s member states and
India. associate members
 The activities of TMT-India will be coordinated  Updation Time - Once every 2 years
by the India TMT Coordination Centre (ITCC)
 Total Cities (2023) - 350 in more than 100
 set up by the Department of Science and countries
Technology, with IIA as the nodal agency
of ITCC.  Objective - It aims to strengthen cultural
activities, goods, services and international
 India to provide the project with hardware, cooperation for sustainable development.
instruments, software worth Rs.200 million.
 The UNESCO Creative Cities Network covers
 India TMT will be jointly funded by the seven creative fields: Crafts and Folk Art,
Departments of Science and Technology and Design, Film, Gastronomy, Literature, Media
Atomic Energy. Arts, and Music.
76. d  Kozhikode is the first Indian city to get
City of Literature tag.
World Cities Culture Forum
7 Creative Fields of
Bengaluru has become the first Indian city to become Indian Cities in UCCN
UCCN
part of the World Cities Culture Forum (WCCF)
Jaipur (2015), Srinagar
 World Cities Culture Forum (WCCF) was Crafts and Folk Arts
(2021)
founded in 2012 by Justine Simons OBE,
London’s Deputy Mayor for Culture & the
Design -
Creative Industries.

CHENNAI |SALEM| MADURAI | COIMBATORE DELHI | BANGALORE | THIRUVANANTHAPURAM


www.shankariasacademy.com 387
www.iasparliament.com

Film Mumbai (2019)  Small tiger population – Mizoram,


Nagaland, Jharkhand, Goa, Chhattisgarh, and
Gastronomy Hyderabad (2019) Arunachal Pradesh.

Literature Kozhikode (2023)  Approximately 35% of the tiger reserves


urgently require enhanced protection
Media Arts - measures, habitat restoration, ungulate
augmentation, and subsequent tiger
reintroduction.
Chennai (2017), Varanasi
Music
(2015), Gwalior (2023)  Corbett Tiger reserves has the maximum
tigers in the tiger reserve in India.
77. b Tigers within Tiger reserves
All India Tiger Estimation Report, 2022
Corbett (260) Kanha (105)
 India currently harbors almost 75% of the
world’s wild tiger population. Bandipur (150) Kaziranga (104)
 India categorized tiger habitats into 5 major
Nagarhole (141) Sundarbans (100)
landscapes based on biogeography and
interconnectivity, enabling effective ecological
and management-based strategies. Bandhavgarh (135) Tadoba (97)

 The report is released by the Ministry of Dudhwa (135) Sathyamangalam (85)


Environment, Forest and Climate Change.
Mudumalai(114) Pench-MP (77)
 An increase in unique tiger sightings from 2461
in 2018 to 3080 in 2022, now more than 3/4th
78. b
of the tiger population is found within protected
areas. Konark wheel
 It reflects a commendable annual growth rate of A replica of Wheel served as the backdrop of Indian
6.1% per annum. Prime Minister's welcome handshake with G20
leaders.
 Increasing trend – Central India and the
Shivalik Hills and Gangetic Plains, particularly  It is a replica of Konark Wheel from the Sun
in the states of Madhya Pradesh, Uttarakhand, temple in
and Maharashtra. Odisha.
 Declining trend – Western Ghats has  Built in – 13th
experienced localized declines in the century, under the
tiger population. reign of King
Narasimhadeva-
 Large tiger population – Madhya I of the Eastern
Pradesh, followed by Karnataka (563) & Ganga dynasty.
Uttarakhand (560), and Maharashtra (444).
 Features – It
has 24 spokes representing the wheels of Lord
Surya's sun chariot.

 Its rotating motion symbolises time,


Kaalchakra, as well as progress and continuous
change.

 Konark sun temple, an example of Kalinga


architecture, is a UNESCO World Heritage Site.

79. b

AITIGA Agreement

CHENNAI |SALEM| MADURAI | COIMBATORE DELHI | BANGALORE | THIRUVANANTHAPURAM


www.shankariasacademy.com 388
www.iasparliament.com

 ASEAN-India Trade in Goods Agreement  The program aims to foster innovation of


(AITIGA) is a trade deal between the ten graphene technologies.
member states of ASEAN and India.
 The program is launched by Ministry of
 ASEAN and India signed the Agreement at the Electronics & Information Technology
7th ASEAN Economic Ministers-India (MeitY).
Consultations in Bangkok, Thailand in 2009.
 The program shall be implemented by Digital
 The Agreement, which came into effect in 2010, University Kerala with joint funding from
is also referred to as the ASEAN-India Free MeitY, Government of India and Government
Trade Agreement. of Kerala and Industry partners.

 The Agreement has led to steadily increasing  India Graphene Engineering and
trade between ASEAN and India since its Innovation Centre (I-GEIC) is set up under
signing. the program.

80. b  It shall fill the gap between R&D and


commercialization by providing a complete
Pradhan Mantri Dakshata Aur Kushalata facility to startup and industry.
Sampanna Hitgrahi (PM-DAKSH) Yojana
 Creation of a commercialization eco-system for
 PM-DAKSH Yojana empowers millions from graphene as an emerging technology would
India's marginalized communities. help India take a pole position in the world’s
new material market.
 Aim - To enhance competency level of the
target groups to make them employable both in  India's first Graphene Centre, the India
self- employment and wage-employment for Innovation Centre for Graphene (IICG), is
their socio-economic development. located at Maker Village in Kochi, Kerala
 Type - It is a Central Sector Scheme. Graphene

 Launched by – Ministry of Social Justice &  Graphene is a single layer of carbon atoms
Empowerment. arranged in a hexagonal lattice.

 Implemented by – Ministry of Skill  It is often referred to as a wonder material for


Development & Entrepreneurship. its extraordinary electrical and electronic
properties.
 Target Groups – SCs, OBCs, EBCs, DNTs
Safai Karamcharis including waste pickers etc.  Discovered by - Andre Geim and Konstantin
Novoselov, discovered in 2004.
 Age criterion – 18-45 years.
 It is stronger than steel, very stretchable and
 Income criteria can be used as a flexible conductor.
o No income limit for SCs,  Its thermal conductivity is much higher than
SafaiKaramcharis Including that of silver.
wastepicker and DNT.
 It is an ultimately thin, mechanically very
o The annual family income should strong, transparent and flexible conductor.
be below Rs.3 lakh for OBCs and
 Applications - Touch screens, light panels
o Annual family income should be and solar cells.
below Rs.1 lakh for EBCs
(Economically Backward 82. c
Classes).
Indian Forest & Wood Certification Scheme
 The cost of training is as per common norms
issued by Ministry of Skill Development & India has launched its 1st Indian Forest and Wood
Entrepreneurship and it varies as per duration certification scheme.
of the course.
 Launched by – MoEFCC (Environment,
81. b Forest & Climate Change)
Graphene-Aurora Program  Aim – To offer voluntary 3rd party
certification to promote sustainable

CHENNAI |SALEM| MADURAI | COIMBATORE DELHI | BANGALORE | THIRUVANANTHAPURAM


www.shankariasacademy.com 389
www.iasparliament.com

forest management and agroforestry in  1899 arbitration demarcated their boundaries


the country. which was formally accepted in 1905
agreement.
 To incentivise entities like State forest
departments, individual farmers, or Farmer  While Venezuela challenged the 1899
Producer Organizations, farm forestry and arbitration in 1962, the 1966 Geneva
other wood-based industries in the value chain. Agreement aimed at maintaining the status
quo.
 Certification Types
 Recent contention – Through referendum,
o Forest management (FM) certification Venezuelans approved the claim of sovereignty
o Tree outside forest management  over Essequibo and were about to immediately
certification begin exploration in the disputed region.
o Chain of custody (CoC) certification  International response – Guyana
approached the International Court of Justice
 Overseen by – Indian Forest and Wood (ICJ) which refused to ban the referendum but
Certification Council. urged Venezuela not to alter the status quo.
 Operating agency – Indian Institute of
Forest Management (IIFM) in Bhopal,
responsible for overall management.

 The National Accreditation Board for


Certification Bodies under the Quality
Council of India (QCI) will accredit the
certification bodies.

 Certification bodies – It will carry out


independent audits and assess entities on their
adherence to the prescribed standards.

 Indian Forest and Wood Certification


Council – It is a multi-stakeholder advisory
body.

 Composition – Representatives from Indian 84. b


Council of Forestry Research and Education,
FSI, QCI, IIFM, Union Ministry (Agriculture Five Eyes Alliance
and Commerce), State Forest Departments,
Forest Development Corporations and Wood-  Five Eyes refers to an intelligence-sharing
based industries. alliance of the United States, United
Kingdom, Australia, Canada and New
83. d Zealand.

Essequibo region  Origin – The alliance originated during


the Second World War.
 Essequibo region is a 61,600-square-mile area
located in the heart of the Guiana Shield, a  In 1943, the Britain-USA (BRUSA) agreement
geographical region in the northeast of South laid the foundations for what became the UK-
America. USA (UKUSA) agreement, signed in 1946.

 It is a disputed territory between  Canada joined it in 1949, and New Zealand and
Guyana and Venezuela. Australia did so in 1956, forming the alliance.

 It accounts for two-thirds of Guyana.  Working – The Five Eyes members use
communications methods, including signals
 The area is one of the 4 last pristine tropical intelligence (SIGINT), to monitor the citizens of
forests in the world and it has vast oil and other member countries.
mineral resources.
 SIGINT is intelligence derived from electronic
 Guyana is set to surpass the oil production of signals and systems used by foreign targets,
Venezuela, and by 2025, it would become the such as communications systems, radars, and
world’s largest per-capita crude producer. weapons systems that provides a vital window

CHENNAI |SALEM| MADURAI | COIMBATORE DELHI | BANGALORE | THIRUVANANTHAPURAM


www.shankariasacademy.com 390
www.iasparliament.com

for our nation into foreign adversaries'  The Global Fund pools the world’s resources to
capabilities, actions, and intentions. invest in ending AIDS, tuberculosis and malaria
as epidemics.
 In 2016, the Five Eyes Intelligence Oversight
and Review Council came into being.  It is a partnership of governments, civil
society, technical agencies, the private
sector and people affected by the
diseases.

 The Global Fund raises funds on a 3 year


cycle, bringing longer term predictability in
the fight against AIDS, TB and malaria.

 The fund provides the advanced pill known as


TLD for under 45 USD per person per year.

87. c

85. b Free Movement Regime (FMR)

Voter verifiable paper audit trail (VVPATs) The Ministry of Home Affairs (MHA) has
recommended the immediate suspension of FMR.
Recently the Election Commission of India (ECI)
mandates the counting of all VVPAT slips in 5  Free Movement Regime – It is a mutual
randomly selected polling stations per Assembly pact between India and Myanmar to
constituency or segment in the upcoming 2024 Lok allow tribes dwelling along the border on either
Sabha election. side to travel up to 16 km inside the other
without a visa.
 VVPAT is a machine that is attached to the
ballot unit (BU) Electronic Voting Machine  Eligibility – It can be used by either a citizen
(EVM) that provides feedback to voters using a of India or a citizen of Myanmar with the
ballot less voting system. production of a border pass

 It is also called as verifiable paper record  Usually valid for a year, and can stay for up to 2
(VPR). weeks per visit.

 When a vote is cast, a slip is printed containing  Challenges – Myanmar’s military coup in
the serial number, name and symbol of 2021 prompted an influx of undocumented
the candidate and remains exposed through migrants, who took shelter in Mizoram, and
a transparent window for 7 seconds. also entered Manipur.

 Thereafter, this printed slip automatically gets  The migrants belonging to the Kuki-Chin-Zo
cut and falls in the sealed drop box of the ethnic group share ethnic ties with
VVPAT. communities in Mizoram and Manipur.

 VVPAT runs on a power pack Battery.

 It was used for the first time in all 21 polling


stations of the Noksen Assembly constituency
of Nagaland in 2013.

 The 2019 Lok Sabha elections became the first


general election to have 100% of EVMs being
attached to VVPATs.

86. b

The Global Fund

 The Global Fund is a worldwide


movement to defeat HIV, TB and
Malaria and ensure a healthier, safer, more
equitable future for all.

CHENNAI |SALEM| MADURAI | COIMBATORE DELHI | BANGALORE | THIRUVANANTHAPURAM


www.shankariasacademy.com 391
www.iasparliament.com

88. a -India’s -Draft City


Smart City Development
PM-KISAN Scheme Award 2023. Plan 2031

 Pradhan Mantri Kisan Samman Nidhi is a Criteria for getting WEA


Central Sector scheme with 100% funding from
Government of India.  It has one or more Ramsar Sites, or other
significant wetlands which provide(s) a range
 Launched in – 2019, retrospectively of ecosystem services to the city;
operational from 2018.
 It has adopted measures for conservation of
 Feature– Provides income support of Rs. wetlands.
6,000 per year to farmers in 3 equal
instalments of Rs.2000 each.  It has implemented wetland
restoration measures.
 Beneficiaries – All land-holding farmer
families, irrespective of the size of their  It considers the challenges and
landholdings. opportunities of integrated spatial/land-use
planning for wetlands under its jurisdiction.
 Direct Benefit Transfer- The amount is
directly transferred to the bank accounts of the  It has raised public awareness about the values
beneficiaries without intermediaries. of wetlands and enabled public participation in
decision-making processes.
89. b
 It has established a local committee to
Wetland City Accreditation (WCA) support the preparation work and the
implementation of measures.
Recently Ministry of Environment Forest and Climate
Change has submitted first 3 Indian cities Indore,  Significance – Cities can gain
Bhopal & Udaipur for Wetland City Accreditation international recognition and positive
(WCA) under the Ramsar Convention on Wetlands. branding opportunities for their efforts
and provides sustainable socio-economic
 WCA – A voluntary scheme that recognizes
benefits for local populations.
cities which have taken exceptional steps to
safeguard their urban wetlands.  It will generate public awareness about
wetland conservation and also help in
 Establishment – During the Ramsar
implementation of Amrit Dharohar in India
Convention COP12, 2015.
which aims to foster sustainable ecosystem
development with the help of local
 Objectives – To recognize the importance of
communities.
wetlands in urban and peri-urban
environments and to take appropriate
Indore Bhopal Udaipur
measures to conserve and protect these
wetlands. Madhya
Location Bihar Rajasthan
Pradesh
5
wetlands 90. c
–Pichola
Bhoj Wetland
Fateh Access, Watch and Reserve (AWaRe)
with Jal
Ramsar Site Sirpur Lake, a Sagar
Tarang
(or) bird Rang  Access, Watch and Reserve (AWaRe) is a
(wetland
Wetland sanctuary Sagar classification tool for monitoring antibiotic
interpretation
Swaroop consumption that was developed by World
centre)
Sagar Health Organization in 2017.
Doodh
Talai  The tool defines the targets and monitors the
effects of stewardship policies that aim to
-Wetland -Wetland optimize antibiotic use and curb antimicrobial
mitras for mitras for resistance.
Efforts & Sarus Crane. Sarus Crane.
-
Recognitions -Lake  This classification is a helpful tool for assessing
-Cleanest city Conservation the results of stewardship policies that seek to
in India. Cell maximize antibiotic use and reduce
antimicrobial resistance.

CHENNAI |SALEM| MADURAI | COIMBATORE DELHI | BANGALORE | THIRUVANANTHAPURAM


www.shankariasacademy.com 392
www.iasparliament.com

 Antibiotics are categorized into 3 groups to e-SAKSHI Mobile Application


emphasize the significance of their appropriate
use while also taking into account the impact of  The MPLAD Scheme, designed to empower
various antibiotics. Members of Parliament (MPs) to recommend
developmental projects, has undergone a
 The 3 groups are: Access, Watch, and Reserve. transformation with the introduction of a
revised fund flow procedure.
 The National Centre for Disease Control
(NCDC) is the nodal agency for India’s  The primary objective is to facilitate the
national programme on AMR containment, of recommendation of works that focus on
which one of the key components is the creating lasting community assets based on
surveillance of antibiotic usage. locally perceived needs.

Prerana Program  The e-SAKSHI mobile application provides


real-time access, enhancing decision-
 Prerana Program aims to offer a meaningful, making processes for MPs.
unique and inspiring experience to all
participants, thereby empowering them with  It was launched by the Ministry of
leadership qualities. Statistics and Programme
Implementation.
 It is a week-long residential program for 20
selected students of class IX to XII.  This innovative technology solution aims to
enhance transparency, accessibility, and
 It is an Experiential Learning Program to efficiency in the execution of MPLAD scheme.
empower students with leadership qualities.
 The mobile app will promote transparency by
 It was launched by the Department of providing MPs with instant updates on the
School Education & Literacy, Ministry status and progress of their proposed projects.
of Education.
91. d
 Prerana program will run from a Vernacular
School which was established in 1888 at UN High Seas Treaty
Vadnagar, Gujarat. Vadnagar is one of the
oldest living cities of India. The High Seas Treaty has been adopted at the 5th UN
Intergovernmental Conference on Marine Biodiversity
 The Prerana stands as a tribute to Vadnagar's of Areas Beyond National Jurisdiction.
indomitable spirit, a living city that has
triumphed over challenges like earthquakes  Under the framework of – United Nations
and natural calamities. Convention on Laws of the Sea (UNCLOS).

 The curriculum of Prerana School prepared by  Objective – It will ensure the conservation
IIT Gandhi Nagar. and sustainable use of marine biological
diversity in areas beyond national
Operation AMRITH jurisdiction through international cooperation
and coordination.
 The Operation AMRITH conducted by
the Kerala state government to tackle  It is also referred to as the ‘Paris Agreement
antimicrobial resistance. for the Ocean’ or ‘Biodiversity of Areas
Beyond National Jurisdiction treaty’.
 Operation AMRITH – Antimicrobial
Resistance Intervention for Total Health to  Draft agreement – It recognises the need to
prevent the overuse of antibiotics in the state. address biodiversity loss and degradation of
ecosystems of the ocean.
 It is aimed at conducting surprise raids in retail
medical shops for detecting OTC sale of  It places 30% of the world’s oceans into
antibiotics that was launched by the Kerala protected areas, for marine conservation and
state government. covers access to and use of marine genetic
resources.
 Operation Sesha – To curb the illegal trade
of Timber, including Red Sanders.  Legality – It is legally binding in nature but
will only enter into force once 60 countries have
 Operation Nanhe Faristey – Reunification ratified it.
of children in need of care and protection with
their families.  High seas – According to the 1958 Geneva
Convention on the High Seas, they are the parts

CHENNAI |SALEM| MADURAI | COIMBATORE DELHI | BANGALORE | THIRUVANANTHAPURAM


www.shankariasacademy.com 393
www.iasparliament.com

of the sea that are not included in the  It is 80 times more potent at warming than
territorial waters or the internal waters of a carbon dioxide.
country.
 It is responsible for around 30% of the rise in
 It is the area beyond a country’s Exclusive global temperatures since the preindustrial era.
Economic Zone which extends up to 200
nautical miles or 370 km from the coastline & 93. b
till where a nation has jurisdiction on living &
non-living resources. United Nations Forum on Forests (UNFF18)

 No country is responsible for management and UNFF18, the 18th edition of UNFF held in New York,
protection of resources on the high seas. discussed the contributions of SFM to energy,
livelihoods and the SDGs.

 Established by – UN ECOSOC in 2000, as


subsidiary body.

 The UN General Assembly adopted the 1st


ever UN Strategic Plan for Forests 2017-2030.

 The Strategic Plan provides a global framework


for actions at all levels to sustainably manage all
types of forests and trees outside forests and
halt deforestation and forest degradation.

 There are 6 Global Forest Goals and 26


associated targets to be achieved by 2030 in the
92. b Strategic Plan.

Methane Global Tracker Report  These goals and targets are voluntary and
universal.
2023 edition of Methane Global Tracker Report was
released.  Membership – It has universal
membership, and is composed of all
 Released by – International Energy Member States of the United Nations
Agency (IEA) and specialized agencies.

 Aim – To bring down methane emissions from  Meeting – It happens annually and will focus
across the energy sector and implement the new on discussions on implementation, technical
Global Methane Pledge. advice and exchange of experiences.

 2023 Report – The energy sector accounts for  UNFF18 – It discussed the contributions of
around 40% of the total average methane sustainable forest management (SFM) to
emissions from human activity. energy, livelihoods and the SDGs.

 More than 260 billion cubic metres (bcm) of  India presented a case of a UNFF country-led
natural gas (mostly composed of methane) is initiative on long-term SFM.
wasted through flaring and methane leaks
globally.  India also shared concerns on wildfires and the
problems associated with current forest
 80% of the available options to curb the release certification schemes.
of methane could be implemented by the fossil
fuel industry at net zero cost. 94. b

 Ultimately, reducing 75% of the wastage of Pygmy Hog


natural gas could lower global temperature rise
by nearly 0.1 degree Celsius by mid-century. According to a research article, the African swine fever
could fatally affect the population of pygmy hogs, the
Methane world’s rarest and smallest pigs.

 It is far more powerful than CO2 at trapping  Scientific name – Porcula salvania
heat in the atmosphere but relatively short-
lived.  Features – They are smallest and rarest wild
piggy.

CHENNAI |SALEM| MADURAI | COIMBATORE DELHI | BANGALORE | THIRUVANANTHAPURAM


www.shankariasacademy.com 394
www.iasparliament.com

 Their skin is dark brownish-black in colour and  Committee of Privileges recommended the
their hair is dark. constitution of an Ethics Committee during the
13th Lok Sabha.
 Native to alluvial grasslands in the foothills of
the Himalayas.  Aim – To oversee the moral and ethical
conduct of members and examine cases of
 They live only in the wet high grasslands misconduct referred to it.
at the foothills of the Himalayas.
 The rules applicable to the Committee of
 Distribution – Once found all the way from Privileges also apply to the ethics panel.
Uttar Pradesh to Assam, but vanished by the
early 1960s. Ethics Committee

 Now their population is confined to Assam and About Lok Sabha Rajya Sabha
southern Bhutan.
Established 2000 1997
 Diet – Omnivores and thus feed on roots,
tubers, insects, rodents, and small reptiles. Should not 10 members
Members contain more than including
 Significance – They are an indicator 15 members Chairman
species.
Nominated by Speaker Chairman
 Their presence ensures a healthy habitat
for other rarities such as the one-horned It is appointed by
It is from the
rhinoceros, hog deer, Eastern barasingha, tiger, Speaker amongst
Chairperson largest party in
water buffalo, lesser florican and the hispid the Committee
the House
hare. members

 Threats – Loss and degradation of habitat due Not exceeding 1 Not exceeding 1
Term
to agricultural encroachments, human year year
settlements, livestock grazing, etc.
 It enforces the code of conduct of
 Conservation Efforts – Pygmy Hog members of Parliament.
Conservation Programme in 1995.
 It examines the cases of
95. c misconduct and recommends
Functions
appropriate action.
Ethics committee
 It is engaged in maintaining
The Lok Sabha Ethics Committee inquired regarding a discipline and decorum in
complaint of cash for query against a sitting Member Parliament.
of Parliament.
96. c
 An allegation of corruption against an
MP – It can be sent to either Ethics or Dancing Frogs
Privileges Committee, but usually more
serious accusations go to the latter. The Wildlife Trust of India after analysing the second
edition of the Global Amphibian Assessment and said
 Privileges Committee – It can take up cases the dancing frogs are endemic to the Western Ghats
of misconduct that involve both MPs and are the most threatened amphibian genus of India.
non- MPs.
 Dancing frogs are members of the Micrixalus
 Ethics Committee – It can take up only genus and there are about 24 frog species in this
cases of misconduct that involve MPs. family.

 History – Vohra committee report submitted  Their preferred habitats are shola
in 1995 pointed out the nexus between criminal grasslands, myristica swamps and
gangs, police, bureaucracy and politicians have evergreen forests within the Western
come clearly out in various parts of the country.
 Ghats, where they mainly reside near slow-
 Recommendations – A Presiding Officers’ moving perennial streams.
Conference held in Delhi in 1996 1st mooted the
idea of ethics panels for the two Houses.  The dancing frogs that are found near the
streams do a unique display to mate.

CHENNAI |SALEM| MADURAI | COIMBATORE DELHI | BANGALORE | THIRUVANANTHAPURAM


www.shankariasacademy.com 395
www.iasparliament.com

 The males stretch up their hind legs one at a 2. Special Central Assistance to
time and wave their webbed toes in the air in a Scheduled Castes Sub Plan (SCA
rapid motion akin to a dance. to SCSP) – Provides 100% grant to the
States/UTs as an additive to their
 The signature dance move is called ‘foot- Scheduled Castes Sub Plan (SCSP).
flagging’ and serves the dual purpose of
attracting a female while also sending out a 3. Babu Jagjivan Ram Chhatrawas
warning signal to other male frogs in the area. Yojana(BJRCY) – Construction of
hostels for students belonging to
 According to the second edition of the Global Scheduled Castes (SC).
Amphibian Assessment the dancing frogs that
are endemic to the Western Ghats are the most  PM- AJAY aims to reduce poverty of the
threatened amphibian genus of India. SC communities by generation of additional
employment opportunities through Skill
 The factors that are threating the Dancing frogs development, income generating schemes and
are: other initiatives.

o Invasive species like the mosquito and  It aids to improve socio-economic


fish, developmental indicators by ensuring adequate
infrastructure and requisite services in the SC
o Land use change, and variation in dominated villages.
temperature and humidity,
 The scheme functions under aegis of the
o Extreme weather events such as floods Ministry of Social Justice and
and excess rainfall, Empowerment.
o Infectious diseases, water pollution,  The 3 components of PM- AJAY are:
light pollution and
1. Development of SC dominated villages
o Infrastructure projects such as dams. into an “Adarsh Gram”.
o They are also impacted by human 2. Grants-in-aid to State/Districts.
activities that alter the flow of wild
streams in the forest. 3. Construction/Repair of Hostels.

o Of the 24 species of the frogs belonging  The objective of Adarsh Gram is to ensure
to the Micrixalus genus that were integrated development of SC majority villages.
assessed, 2 were found to be critically
endangered and 15 were endangered.  All requisite infrastructure necessary for socio-
economic development needs are to be
o It is also the 5th most threatened genus provided under the Scheme.
in the world with 92 % of its species in
the threatened category.  The PM- AJAY aims at socio-economic
development of SCs through grants for
o Frogs are valuable in the food chain following types of projects:
and also provide other ecological
services. o Comprehensive Livelihood Projects.
o Protecting the natural habitats and o Skill Development.
preserving their optimal living
conditions is vital to save the last of o Grants for creation/acquisition of
these species. assets for beneficiaries/households.

97. b o Infrastructure development.


Pradhan Mantri Anusuchit Jaati Abhuyday 98. b
Yojana
Attosecond Science
 Pradhan Mantri Anusuchit Jaati Abhuyday
Yojana (PM- AJAY) is a merged scheme of 3 2023 Nobel Prize for Physics was awarded to Anne
Centrally Sponsored Scheme such as: L’Huillier, Pierre Agostini, and Ferenc Krausz for
experimental methods that generate attosecond pulses
1. Pradhan Mantri Adarsh Gram of light for the study of electron dynamics in matter.
Yojana (PMAGY) – Aims at
integrated development of Scheduled
Castes majority villages.

CHENNAI |SALEM| MADURAI | COIMBATORE DELHI | BANGALORE | THIRUVANANTHAPURAM


www.shankariasacademy.com 396
www.iasparliament.com

 Attosecond – It is one quintillionth of a  Uraiyur (now Tiruchchirappalli) was its oldest


second or 10-18 seconds, the timescale at capital.
which the properties of an electron change.
 Chola kings and emperors bore the titles
 It can also be referred as one-billionth of a Parakesharivarman and Rajakesharivarman.
nanosecond.
 Rajaraja I, Rajendracola Deva I, Rajadhiraja
 These light pulses are used to unravel and Kulottunga I were notable rulers of Chola
dynamical processes in matter with dynasty.
unprecedented time resolution.
 To know more about Imperial Cholas click here.
 Attosecond science – It is a branch of
physics that deals with light-matter interaction  Terms in Chola dynasty
phenomena, production of extremely short
light pulses and using them to study o Kalam - Sailing vessels
superfast processes.
o Salai - Army training centre

99. a

Abhyudaya

 Abhyudaya is a Hindi weekly magazine which


was authored by Madan Mohan Malaviya,
in 1907.

 MM Malaviya is the founder of the Banaras


Hindu University and he himself edited the
paper Abhyudaya.

 It remained a weekly until 1915, when it was


converted into a daily.

100. c

Chola Dynasty

 Chola dynasty antedates the early Sangam


poems (200 CE).

 The dynasty originated in the rich Kaveri


(Cauvery) River valley.

CHENNAI |SALEM| MADURAI | COIMBATORE DELHI | BANGALORE | THIRUVANANTHAPURAM


www.shankariasacademy.com 397
www.iasparliament.com

BONUS QUESTIONS How many of the above countries have had already
conducted Conference of Parties (COP) under UNFCCC
COP-28 framework?

a. All except 1
1. Which of the following statements is/are correct with
respect to the status of Greenhouse Gas (GHG) b. All except 3
Emissions?
c. All except 4
1. In last 15 years, USA is the world biggest emitter
and responsible for 30% of global emissions d. All except 1 & 5
every year.

2. China, the UK and the EU are considered to be


responsible for 50% of all emissions. 4. Which of the following is correct with respect to
Panchamrit goals adopted in CoP26 held at Glasgow?
3. India is responsible for only about 4% of
historical emissions. 1. To achieve net zero by 2070.

Select the correct answer using the codes given below: 2. Bring down carbon intensity by more than 45%
by 2030.
a. 1 only
3. 500 GW of non-fossil fuel installed power
b. 2 only generation capacity by 2030.
c. 3 only 4. To meet 50% of its energy requirement through
renewable energy by 2030.
d. 1 and 3 only
5. Cutting net projected carbon emission by one
billion tonne from now until 2030.
2. Consider the following statements. Select the correct answer using the codes given below:
1. The COP is the main decision-making body of a. Only two
the United Nations Framework Convention on
Climate Change (UNFCCC). b. Only three
2. The COP secretariat is headquartered in Bonn, c. Only four
Germany.
d. All five
3. Azerbaijan and Brazil will host the COP29 and
COP30 respectively.

How many of the statement(s) given above is/are 5. Consider the following statements with respect to the
correct? Loss and Damage (L&D) Fund

a. Only one 1. The concept of Loss & Damage fund was first
suggested by Vanuatu on behalf of the Alliance
b. Only two of Small Island States (AOSIS) in 1991.
c. All three 2. World Bank will act as the permanent host for
the L&D facility as a financial intermediary
d. None fund (FIF).

Which of the statement(s) given above is/are correct?


3. Consider the following countries. a. 1 only
1. India b. 2 only
2. Germany c. Both 1 and 2
3. Australia d. Neither 1 nor 2
4. Japan

5. Egypt

CHENNAI |SALEM| MADURAI | COIMBATORE DELHI | BANGALORE | THIRUVANANTHAPURAM


www.shankariasacademy.com 398
www.iasparliament.com

6. With reference to the Global Stocktake (GST) process, 3. India is not a part of the Global Methane
which of the statements given below is incorrect? Pledge.

a. The global stocktake process takes place for 4. India is among the top five methane emitters
every five years. globally.

b. It was established under Article 14 of the Paris How many of the above statement(s) is/are correct?
Agreement adopted in 2015.
a. Only one
c. It was designed to assess the collective progress
towards achieving the Bali Roadmap adopted in b. Only two
2007.
c. Only three
d. The first-ever global stocktake process
culminated at the CoP-28 held in Dubai, UAE. d. All four

7. Consider the following. 10. Consider the following.

1. Global Cooling Pledge 1. Green Credit Initiative

2. Declaration on Climate and Health 2. Global River Cities Alliance

3. Global Renewables and Energy Efficiency 3. Waste to Zero Coalition


Pledge
4. The Buildings and Cement Breakthrough
4. Emirates Declaration on Resilient Food
Systems, Sustainable Agriculture, and Climate Which of the following initiatives were launched by
Action India during COP28 Climate Summit in United Arab
Emirates?
To which of the above initiatives, India is not a
signatory? a. Only one

a. Only one b. Only two

b. Only two c. Only three

c. Only three d. All four

d. All four
11. Consider the following statements regarding Global
Cooling Pledge
8. Podong Indigenous Peoples Initiative, which aims to
recognise and support Indigenous people’s 1. It commits the countries to reduce their cooling
contributions to the conservation of biodiversity and emissions by at least 68% by 2050.
climate solutions through their Indigenous knowledge
2. It is a joint initiative of Egypt and the UN
systems, was launched by?
Environment Program led Cool Coalition.
a. UN Forum on Forests
Which of the statement(s) given above
b. Convention of Migratory Species is/are incorrect?

c. UN Permanent Forum on Indigenous Issues a. 1 only

d. International Union for Conservation of Nature b. 2 only

c. Both 1 and 2

9. Consider the following statements with respect to d. Neither 1 nor 2


Global Methane Pledge

1. It was signed at COP26 in Glasgow, UK.


12. Which of the statements with reference to Global
2. It aims to cut methane emissions by at least Renewables and Energy Efficiency Pledge
30% by 2030 from the 2020 levels. is incorrect?

CHENNAI |SALEM| MADURAI | COIMBATORE DELHI | BANGALORE | THIRUVANANTHAPURAM


www.shankariasacademy.com 399
www.iasparliament.com

a. It was jointly led by the European Union, China 3. UAE


and the UAE.
4. France
b. The pledge calls for the phase down of unabated
coal power and an end to the financing of new coal- 5. European Union
fired power plants.
Select the correct answer using the codes given below:
c. It aims to triple worldwide installed renewable
energy generation capacity to at least 11,000 GW. a. 3 only

d. It aims to double global average annual rate of b. 1, 3 and 5 only


energy efficiency improvements to more than 4% by
2030. c. 2, 3 and 4 only

d. 1, 2, 3, 4 and 5

13. The CHAMP Pledge, signed at Conference of Parties


(COP28) of UNFCCC, deals with which of the following?
16. Which of the following countries is/are not the
a. A commitment by national governments to work member(s) of G7 climate club?
in partnership with their subnational governments
1. Chile
for climate action.
2. India
b. A commitment to reduce the degradation of
forest land and glaciers. 3. Germany
c. A commitment to accelerate the reliance on Select the correct answer using the codes given below:
renewable energy to power the economy and be
effectively fossil fuel-free by 2070. a. 1 only
d. A commitment to reduce global methane b. 2 only
emissions from 2020 levels by 2030 by 30%.
c. 3 only

d. None
14. With reference to Santiago Network, consider the
following statements.

1. It aims to provide technical knowledge and 17. The actions undertaken in the field of afforestation,
resources to developing countries on averting, particularly through the Jal-Jeevan-Hariyali Abhiyan
minimising and addressing loss and damage. was praised by the international community at COP-28.
Jal Jivan Hariyali Abhiyan is a scheme launched by
2. At COP28, UN Office for Disaster Risk which of the following states?
Reduction (UNDRR) and the UN Office for
Project Services (UNOPS) were announced as a. Bihar
the hosts of the Santiago network secretariat.
b. Rajasthan
Which of the statement(s) given above is/are correct?
c. Chhattisgarh
a. 1 only
d. Madhya Pradesh
b. 2 only

c. Both 1 and 2
18. In light of the Indian Prime Minister’s request to
d. Neither 1 nor 2 host the 33rd edition of Conference of Parties (COP)
Summit under UNFCCC in India in 2028, which one of
the following statement (s) is/are correct?
15. ALTERRA, sometimes seen in the news, is a 1. A proposal to host the COP must be approved
privately managed fund of USD 250 billion to catalyse by other signatories to the UNFCCC.
private sector climate investments globally by 2030. It
was announced by? 2. Venues for future COP are only decided 2 years
in advance.
1. UK
Which of the statement(s) given above is/are correct?
2. USA

CHENNAI |SALEM| MADURAI | COIMBATORE DELHI | BANGALORE | THIRUVANANTHAPURAM


www.shankariasacademy.com 400
www.iasparliament.com

a. 1 only Which of the statement(s) given above is/are


incorrect?
b. 2 only
a. 1 only
c. Both 1 and 2
b. 2 only
d. Neither 1 nor 2
c. 1 and 3 only

d. 3 only
19. With reference to Coal Transition Accelerator,
consider the following statements.

1. It aims to facilitate just transitions from coal to 22. Consider the following statement regarding
clean energy. protected lists of different species under the Bonn
Convention.
2. It was jointly launched by India and Powering
Past Coal Alliance (PPCA). 1. It consists of three appendices.

Which of the statement(s) given above 2. Birds occupied the first place followed by
is/are incorrect? terrestrial mammals.

a. 1 only 3. Appendix II covers endangered migratory


species.
b. 2 only
4. Appendix III requires international agreements
c. Both 1 and 2 for their conservation and management.

d. Neither 1 nor 2 How many of the statement(s) given above is/are


correct?

a. Only one
20. Consider the following statements with respect to
Buildings and Cement Breakthrough b. Only two

1. It aims to advance the decarbonisation of the c. Only three


construction sector by 2030.
d. All four
2. It was launched by Canada and the United Arab
Emirates.

Which of the statement(s) given above 23. Consider the following statements with respect to
is/are incorrect? Convention on Migratory Species COP14:

a. 1 only 1. It is the first COP of any global environmental


treaty to take place in Central Asia.
b. 2 only
2. It held in Samarkand, the capital of Kazakhstan.
c. Both 1 and 2
Which of the statement(s) given above is/are correct?
d. Neither 1 nor 2
a. 1 only
CMS-14
b. 2 only
21. Consider the following statements with respect to
c. Both 1 and 2
Convention of Migratory Species (CMS)
d. Neither 1 nor 2
1. It was conceived at the 1972 Stockholm
Conference on the Human Environment.

2. Its secretariat is provided by United Nations 24. Samarkand Strategic Plan (2024-2032) is
Environment Program (UNEP). associated with which of the following?
3. Conference of Parties to CMS meets every two a. Biodiversity
years.
b. Migratory species

CHENNAI |SALEM| MADURAI | COIMBATORE DELHI | BANGALORE | THIRUVANANTHAPURAM


www.shankariasacademy.com 401
www.iasparliament.com

c. Plastics Select the correct answer using the codes given below:

d. Waste management a. Both Statement-I and Statement-II are correct


and Statement-II is the correct explanation for
Statement-I

25. Consider the following statements with respect to b. Both Statement-I and Statement-II are correct
the State of the World Migratory Species report, and Statement-II is not the correct explanation
for Statement-I
1. It is the first ever report that provides a
comprehensive overview and analysis of the c. Statement-I is correct but Statement-II is
conservation status of migratory species. incorrect

2. One in five Convention of Migratory Species d. Statement-I is incorrect but Statement-II is


listed species are threatened with extinction correct
globally.

3. Global extinction risk is reduced marginally due


to various international efforts. 28. Consider the following statements with respect to
Interim budget
Which of the statement(s) given above is/are correct?
1. Article 112 (2) of Indian Constitution refers to
a. 1 and 3 only the provision of an interim budget.
b. 2 and 3 only 2. It is presented by the outgoing government
ahead of the Lok Sabha polls.
c. 1 and 2 only
3. It includes estimates for government
d. 1, 2 and 3 expenditure, revenue, and fiscal deficit with
minimum major policy announcements.

How many of the above statements is/are correct?


26. Consider the following
a. Only one
1. Atlas of Animal Migration
b. Only two
2. Action Plan for African-Eurasian Migratory
Land birds Action Plan c. All three
3. Action Plan for Great Bustard in Asia d. None
4. Action Plan for Butterfly

5. Central Asian Mammals Initiative 29. Consider the following statements


Which of the following is/are part of Convention of Statement-I: Interim budget receives less scrutiny due
Migratory Species? to its limited scope and temporary nature.
a. All Statement-II: Interim budget is usually valid for 2–4
months, but remains valid until the new government
b. All except 1 & 3 presents its full budget.
c. All except 4 Select the correct answer using the codes given below:
d. All except 5 a. Both Statement-I and Statement-II are correct
and Statement-II is the correct explanation for
Interim Budget 2024-25 Statement-I

27. Consider the following statements b. Both Statement-I and Statement-II are correct
and Statement-II is not the correct explanation
Statement-I: Article 112 in Constitution of India for Statement-I
mentions that ‘the President shall in respect of every
financial year cause to be laid before both the Houses of c. Statement-I is correct but Statement-II is
Parliament a budget for that year’. incorrect

Statement-II: The annual financial statement is a d. Statement-I is incorrect but Statement-II is


statement of the estimated receipts and expenditure. correct

CHENNAI |SALEM| MADURAI | COIMBATORE DELHI | BANGALORE | THIRUVANANTHAPURAM


www.shankariasacademy.com 402
www.iasparliament.com

30. Consider the following schemes How many of the above corridors were proposed in
Interim Budget 2014-25 as major economic railway
1. PM-Jan Dhan Yojana corridor programmes under PM Gati Shakti?

2. PM-SVANidhi a. Only one

3. PM-JANMAN Yojana b. Only two

4. PM-Vishwakarma Yojana c. Only three

Haw many of the above are schemes meant for poverty d. All four
alleviation?

a. All except 4
34. Which of the following statements regarding
b. All except 1 Interim Budget 2024-2025 is/are correct?

c. All except 3 1. In included changes in direct taxation laws.

d. All four 2. It maintained same tax rates for direct and


indirect taxes.

Select the answer using the code given below:


31. Consider the following
a. 1 only
1. There is a continuous increase in capital
expenditure in the last five financial year. b. 2 only

2. The Foreign Direct Investment (FDI) has been c. Both 1 and 2


doubled in 2014-2023 compared to 2005-2014
levels. d. Neither 1 nor 2

Which of the statement(s) given above is/are


incorrect?
35. Arrange the following receipts from North to South
a. 1 only according to latest financial year:

b. 2 only 1. Income tax

c. Both 1 and 2 2. GST and other tax

d. Neither 1 nor 2 3. Customs

4. Borrowing and other liabilities

32. Deep Tech Scheme as announced in Interim Budget 5. Non-tax receipts


2024-25, will be launched for strengthen which of the
following sectors? Select the correct answer using the code below:

a. Food industry a. 1-4-5-2-3

b. Defence b. 4-1-5-2-3

c. Cyber crime c. 1-4-2-5-3

d. Space sector d. 4-1-2-5-3

33. Consider the following

1. Energy, mineral and cement corridors

2. Port connectivity corridors

3. High traffic density corridors

4. Cargo carriage corridors

CHENNAI |SALEM| MADURAI | COIMBATORE DELHI | BANGALORE | THIRUVANANTHAPURAM


www.shankariasacademy.com 403
www.iasparliament.com

Answer Key - BONUS QUESTIONS

1 2 3 4 5 6 7 8 9 10
C C B D A C D D D B
11 12 13 14 15 16 17 18 19 20
D A A C A B A C B D
21 22 23 24 25 26 27 28 29 30
D A A B C C D A A D
31 32 33 34 35
D B C B D

EXPLANATION
1. c

Status of Greenhouse Gas (GHG) Emissions

 G20 countries represent around 75% of global


GHG emissions.

 The US, the UK and the EU are considered


to be responsible for 50% of all
emissions.

 India is responsible for only 4% of


historical emissions.

 China, the world’s biggest emitter in the


last 15 years, is responsible for 30% of global
emissions every year.

2. c
4. d
Climate Summits of COP under UNFCCC

 COP – Conference of Parties, is the main


decision-making body of the United Nations
Framework Convention on Climate Change
(UNFCCC).

 It brings together the 198 Parties (including the


European Union) to negotiate and agree on how
to tackle climate change, reduce emissions and
limit global warming.

 Inaugural meeting in – Berlin, Germany, in


1995.

 Secretariat – Headquartered in Bonn. 5. a

Loss and Damage (L&D) Fund


 Presidency – It rotates among the 5
recognized UN regions - Africa, Asia, Latin
 The Loss and Damage (L&D) Fund was first
America and the Caribbean, Central and
suggested by Vanuatu (on behalf of the
Eastern Europe and Western Europe and
Alliance of Small Island States) in 1991.
others.
 The proposal was rejected, and the issue of loss
3. b
and damage was not mentioned when the text
Australia has never hosted a United Nations of the Framework Convention was adopted in
climate conference (COP). 1992.

CHENNAI |SALEM| MADURAI | COIMBATORE DELHI | BANGALORE | THIRUVANANTHAPURAM


www.shankariasacademy.com 404
www.iasparliament.com

 Loss and damage first appeared in a negotiated 7. d


outcome of the UN climate talks in 2007 as
part of the Bali Action Plan. Emirates Declaration on Resilient Food
Systems, Sustainable Agriculture, and Climate
 First Announced at – COP27 of UNFCCC, Action
in Sharm el-Sheikh, Egypt.
 Aim – To promote food security while
 Adopted at – CoP 28, UNFCCC, Dubai, combatting climate change.
UAE.
 Promoted by – The UAE
 It is a global financial package that rich nations,
whose industrial growth has resulted in global  Partner - Melinda Gates Foundation
warming, must pay to poor nations, who are
facing the cascading effects of climate change.  Signatories – 134 nations which make up 76%
of food-based GHGs and produce 70% of food
 Initial funding – 475 million USD, maximum globally
is pledged by European Union followed by
UAE, US and Japan.  India refrained from signing as it
conflicted with its longstanding policy of not
 Administration – World Bank will host committing agriculture, livestock and the
the L&D facility as a financial intermediary millions of livelihoods that depend on them in
fund (FIF) for an interim period of 4 years. climate action

 Fund allocation – It is based on the available Declaration on Climate and Health


evidence and with a minimum percentage
allocated to least developed countries and  About - It is a non-binding declaration that
Small Island Developing States. calls for nations to reduce their greenhouse
gases (GHG) emissions in the health sector
6. c swiftly, sustainably, and substantially.

Global Stocktake (GST) Process  Signatories – 144 countries till now.

 About - It is a process for countries and  India have not signed the
stakeholders to see where they’re collectively declaration citing the lack of practicality in
making progress towards meeting the curbing GHG use for cooling in the health
goals of the Paris Climate Change sector.
Agreement.
 December 3 was celebrated as World Health
 Established - under Article 14 of the Paris Day.
Agreement, 2015.
The Global Renewables and Energy Efficiency
 The stocktake takes place every five years, Pledge
with the first-ever stocktake concluded
at COP28, Dubai, UAE.  Aim – To triple the installed renewable energy
generation capacity to at least 11,000 GW by
 It is intended to inform the next round of 2030
climate action plans under the Paris Agreement
(nationally determined contributions, or  Doubling global average annual rate of energy
‘NDCs’) to be put forward by 2025. efficiency improvements to more than 4% by
2030

 It is not legally binding.

 Spearheaded by – EU, the US and the UAE

 Signatories – As many as 130 countries


signed on to a pledge, with the China and
India, absent from the list of signatories.

 China has the world’s largest installed


renewable energy capacity followed by the
United States, Brazil and India.

CHENNAI |SALEM| MADURAI | COIMBATORE DELHI | BANGALORE | THIRUVANANTHAPURAM


www.shankariasacademy.com 405
www.iasparliament.com

Global Cooling Pledge  Methane Finance Sprint – It is a USD 1


billion by Governments and private sector for
 Aim – To commit the countries to reduce their methane reduction, which is due to be
cooling emissions by at least 68% by 2050 from administered by World Bank and the Global
2022 levels and outlines several strategies to Methane Hub.
tackle them.
 An Eye on Methane is a report published by the
 Conventional cooling, such as air-conditioning International Methane Emissions Observatory
is responsible for over 7% of global GHG (IMEO) which works under the UNEP.
emissions.
10. b
 Signatories - 66 countries, including the US
and Canada.. Role of India in COP 28

 India has not signed the pledge as strict  The actions of Bihar government in the field of
adherence to it could restrict access to afforestation, particularly through the Jal-
affordable cooling and also requires substantial Jeevan-Hariyali Abhiyan (Rural Development
investments. Department), was praised by the international
community at COP-28.
 India has its own Cooling Action Plan,
announced in 2019 to reduce power  Prime Minister of India offered to host the 33rd
consumption for cooling across sectors by 20- edition of the annual summit due in 2028 in
25% by 2038. India.

8. d  Earlier, India had hosted the 8th edition in


2002.
Podong Indigenous Peoples Initiative
 Launched 2 Initiative - Green Credit
 Launched in – 2023 at COP28 of UNFCCC in Initiative & Global River Cities Alliance
UAE
Green Credit Initiative
 Launched by
 Unveiled in 2023, by the Ministry of
o International Union for Environment, Forest and Climate Change.
Conservation of Nature
 An initiative within the Lifestyle for
o International Indigenous Forum Environment or Life Movement.
on Biodiversity
 Working – Green credits will be assigned to
o IUCN Indigenous Peoples specific environmental activities, and will be
Organisations members. treated as tradable commodities.
 Aim – To provide funding directly to  These credits will be able to be sold on domestic
indigenous peoples, ensuring no less than 85% market platforms.
of funds reach indigenous territories and
communities.  Administration – By Indian Council of
Forestry Research and Education (ICFRE)
9. d
 Priorities – The 2 main priorities include
Climate Action for Methane emission water conservation and afforestation.
At COP28, Summit on Methane and Non-CO2 Global River Cities Alliance (GRCA)
Greenhouse Gases was convened by USA, China, and
UAE to accelerate actions to cut methane and other  Launched by - National Mission for Clean
non-CO2 GHGs. Ganga (NMCG)
 Global Methane Pledge – An initiative  Aim - To scale up the concept of collaborative
from COP26 Glasgow, to cut methane knowledge sharing to facilitate the transfer of
emissions by at least 30% by 2030 from the good experiences.
2020 levels.
 Members – 9 countries (Indian, Denmark,
 India is not a part of the Global Methane Cambodia, Japan, Bhutan, Australia,
Pledge, is among the top 5 methane Netherlands, Egypt, and Ghana)
emitters globally.

CHENNAI |SALEM| MADURAI | COIMBATORE DELHI | BANGALORE | THIRUVANANTHAPURAM


www.shankariasacademy.com 406
www.iasparliament.com

Other International Initiatives  Pursue the life cycle management of


fluorocarbons through the Initiative on
 Waste to Zero Coalition – It was Fluorocarbons Life Cycle Management.
launched by UAE Ministry of Climate
Change & Environment to decarbonizing the  Support initiatives such as United Nations
waste sector and accelerating the shift towards Environment Programme-led Cool Coalition, to
circular and regenerative modes of industry and advance global cooperation and domestic
production. actions.

 The Buildings and Cement  Support collaborative research, innovation, and


Breakthrough – It was launched by deployment activities at the local and
Canada and UAE to advance the international level.
decarbonisation of the construction sector by
2030. 12. a

11. d Global Renewables and Energy Efficiency


Pledge
Global Cooling Pledge
 Led by- US, European Union and UAE.
 Joint initiative- United Arab Emirates as
host of COP28 and the UNEP-led ‘Cool  Aim- To achieve net zero emission by mid-
Coalition’. century (2050).

 It is the world's first collective focus on energy  Coal phase-out- The pledge calls for “the
emissions from the cooling sector. phase down of unabated coal power” and an
end to the financing of new coal-fired power
 Passive cooling strategies- It outlines plants.
actions such as insulation, natural shading,
ventilation and reflective surfaces, higher  It is seen as a crucial step to limit global
energy efficiency standards and a rapid phase warming to 1.5°C above pre-industrial levels, as
down of climate-warming hydrofluorocarbon agreed in the Paris Agreement.
(HFC) refrigerants.
 Target-
 Aim- It commits the countries to reduce their
cooling emissions by at least 68% by 2050. o To triple worldwide installed
renewable energy generation capacity
 Outcome- It could reduce the projected 2050 to at least 11,000 GW.
emissions from business-as-usual cooling by
around 3.8 billion tons of CO2 equivalent. o To double global average annual rate of
energy efficiency improvements
Commitments pledged in Global Cooling Pledge to more than 4% by 2030.

 Ratify Kigali Amendment by 2024.  Signed- At COP 28 in Dubai.

 Support robust action through the Montreal  Major non-signatory


Protocol Multilateral Fund for early action to countries- India and China
reduce HFC consumption.
 According to IEA projections, the global share
 Establish Minimum Energy Performance of renewables in power generation is
Standards (MEPS) for air conditioning by anticipated to jump from 28% in 2021 to 80 %
2030. by 2050, while coal’s contribution will dwindle
to 3%.
 Include cooling emissions in countries overall
climate action plans, called Nationally  Primary renewable energy sources with their
Determined Contributions. share in global power generation-

 Publish own national cooling action plans by o Hydroelectric- 16.1 %


2026.
o Wind- 5.6%
 Support the deployment of highly efficient air
conditioning technologies. o Solar-3.8%

 Establish national model building energy codes 13. a


by 2030.
CHAMP Pledge

CHENNAI |SALEM| MADURAI | COIMBATORE DELHI | BANGALORE | THIRUVANANTHAPURAM


www.shankariasacademy.com 407
www.iasparliament.com

The CHAMP was recently taken at the COP 28 of o At COP25, 2019 (Madrid, Spain) –
UNFCCC at Abu Dhabi. Establishment

 CHAMP – Coalition for High Ambition o At COP26, 2021 (Glassgow, UK) –


Multilevel Partnerships Decision on functions

 CHAMP Pledge – It is a commitment o At COP27, 2022 (Sharm El-Sheik,


taken by national governments on a new Egypt) – Adopted terms of references &
way of working in partnership with their established advisory body
subnational governments.
 Function – To contribute for the
 It is a new way of approaching the development implementation of the functions of the Warsaw
and implementation of their next Nationally international mechanism.
Determined Contributions (NDCs) in time for
COP30 in 2025.  Hosts – At CoP 28, the United Nations for
Disaster Risk Reduction (UNDRR)
 Established in – 2023 and United Nations Office for Project
Services have been selected as hosts.
 Aim – To collectively pursue efforts to limit the
temperature increase to 1.5°C above pre- 15. a
industrial levels, and increase adaptation and
resilience. ALTERRA

 Signatories – 71 countries, India is yet to sign The United Arab Emirates recently announced the
this pledge. ALTERRA, a privately managed fund of USD 250
billion to catalyse private sector climate investments
 Coverage globally by 2030.

o Nationally Determined Contributions  ALTERRA – Alterra Mountain Company


(NDCs)
 It is the World's largest private
o National Adaptation Plans (NAPs) investment vehicle for Climate Action.

o National Biodiversity Strategies and  It aims to revolutionise international climate


Action Plans (NBSAPs) finance by fostering a fairer system, with a focus
on enhancing funding accessibility for the
o Long Term Low-Emission Global South.
Development Strategies (LT-LEDS)
 Established by – Lunate, an independent
14. c global investment manager, and is domiciled in
the Abu Dhabi Global Market.
Santiago Network
 The 4 verticals of the ALTERRA includes:
The draft text on the Santiago Network has been
adopted by the Parties and sent to the Conference of the o Energy Transition
Parties (COP) of UNFCCC at COP28 in UAE.
o Industrial Decarbonisation
 It is a collaborative
framework established as part of the o Sustainable Living
Warsaw International Mechanism for Loss
and Damage (WIM) under the UNFCCC. o Climate Technologies

 Aim – To catalyze the technical 16. b


assistance for reducing Loss &
Damage in developing countries that are G7 Climate Club
particularly vulnerable to the adverse effects of
 Origin - It first arose at a Group of Seven (G7)
climate change
summit 2022.
 Santiago is the capital of Chile
 Launched - It was formally launched at the
 Genesis CoP 28, Dubai, UAE.

o At COP19, 2013 (Warsaw, Poland) –  Led by – Germany and Chile.


Proposal to establish
 Members - 36 countries including Kenya,
European Union and Switzerland.

CHENNAI |SALEM| MADURAI | COIMBATORE DELHI | BANGALORE | THIRUVANANTHAPURAM


www.shankariasacademy.com 408
www.iasparliament.com

 India is not a member of G7 Climate club.  Aim - To advance the decarbonisation of the
construction sector by 2030.
 Aim - To ambitiously tackle industrial
emissions.  To focus the industry on speeding up
decarbonization by sharing 3 best practices,
17. a working on policy and standards, and
supporting new innovation in areas like carbon
Jal-Jivan-Hariyali Abhiyan capture and storage or utilization, or circular
economy.
 It is an initiative of the Bihar government in
order to protect water resources and plants. 21. d

 The Mission is being implemented in Convention of Migratory Species (CMS)


coordination with various departments and the
Rural Development Department has been  Conception – It began at the 1972
designated as the Nodal Department. Stockholm Conference on the Human
Environment.
18. c
 Launched in – 1979 in Bonn, Germany (called
Venues for COP Summit under UNFCCC as Bonn Convention)
 A proposal to host the COP must be approved  Entered into force in – 1983
by other signatories to the UNFCCC.
 Secretariat – Provided by United Nations
 Venues for future COP are only decided 2 Environment Program, based in Bonn in
years in advance. Germany
o COP 29 – Baku, Azerbaijan  Depository – Germany
o COP 30 – Belem, Brazil  Out-post office – Abu Dhabi, United Arab
Emirates.
19. b

Coal Transition Accelerator  Aim – To provide a global platform for the


conservation and sustainable use of migratory
 Launched by - France, together with animals and their habitats.
Canada, European Commission, Indonesia,
Malaysia, Senegal, United Kingdom, United  To lay the legal foundation for internationally
States, Vietnam, and several organizations coordinated conservation measure throughout
including the Powering Past Coal Alliance a migratory range.
(PPCA). India is not a part of it.
 Parties – 133 as of 2022, Jamaica has signed
 Aim - It aims to share expertise, design new but not ratified to become a party yet.
policies including through best practices and
 India has been a part of Bonn convention since
lessons learned and unlock new sources of
1983.
public and private financing to
facilitate just transitions from coal to
 Conference of the Parties (COP) – It is the
clean energy.
decision-making body of the Convention which
Powering Past Coal Alliance (PPCA) meets at three-yearly intervals.

 It is a coalition of national and subnational  CMS Trust Fund – Each Party shall
governments and organisations working to contribute to this fund according to a scale to be
advance the transition from unabated coal agreed upon by the COP.
power generation to clean energy.
22. a
 USA is a member while India and China are Species protection under Bonn Convention
not members.
 It is also called as Conservation of Migratory
20. d
Species (CMS).
The Buildings and Cement Breakthrough
 Listed species – It has about 1,200 species
 Launched by - Canada and the United Arab listed in 2 Appendix.
Emirates

CHENNAI |SALEM| MADURAI | COIMBATORE DELHI | BANGALORE | THIRUVANANTHAPURAM


www.shankariasacademy.com 409
www.iasparliament.com

 Birds occupied the first place followed by  New Global Partnership on Ecological
terrestrial mammals. Connectivity – The report is a global
partnership which includes Convention on
 Appendix I – Endangered migratory Biological Diversity, Ramsar Convention,
species. IUCN, UNCCD, World Wide Fund for Nature,
UNEP etc.,
 It prohibits the taking of such species, with very
restricted scope for exceptions Key findings of the report

 Conserve and where appropriate restore their  Two greatest threats to both CMS-listed and all
habitats migratory species are overexploitation and
habitat loss due to human activity.
 Prevent, remove or mitigate obstacles to their
migration and control other factors that might  50% of Key Biodiversity Areas identified as
endanger them. important for CMS-listed migratory animals do
not have protected status.
 Appendix II – Migratory species conserved
through Agreements.  58% of the monitored sites recognized as being
important for CMS-listed species are
 It covers migratory species that have an experiencing unsustainable levels of human-
unfavourable conservation status and that caused pressure.
require international agreements for
their conservation and management.  Global extinction risk is escalating for
both CMS-listed and all migratory species.
23. a
 One in five CMS-listed species are
CMS COP 14 threatened globally.
The 14th Meeting of the Conference of the Parties to the
Convention on the Conservation of Migratory Species
of Wild Animals (CMS) was held in 2024.

 Location – In the historic city of Samarkand,


Uzbekistan.

 It is the 1st COP of any global environmental


treaty to take place in Central Asia, a
region home to many migratory species
including Saiga Antelope, Snow Leopard, and
many migratory birds.

 Slogan – Nature Knows No Border.

24. b

Samarkand Strategic Plan (2024-2032)

 It was prepared at CMS COP14 that held in


Samarkand, Uzbekistan.

 It is a framework for the conservation and


sustainable use of migratory species and 26. c
their habitats.
Key takeaways of CMS COP14
 It is based on the Kunming-Montreal Global
 Atlas of Animal Migration
Biodiversity Framework (GBF) adopted in
2022.  Agreement on an Initiative for the
Central Asian Flyway – It spans 30 Range
25. c
States with a coordinating unit in India.
The State of the World Migratory Species
 Action Plan for African-Eurasian
 It is the first ever report that provides a Migratory Landbirds Action Plan
comprehensive overview and analysis of the (AEMLAP)
conservation status of migratory
species.  Action Plan for Great Bustard in Asia

CHENNAI |SALEM| MADURAI | COIMBATORE DELHI | BANGALORE | THIRUVANANTHAPURAM


www.shankariasacademy.com 410
www.iasparliament.com

 Transboundary Jaguar initiative – Listed  Article 112 – The President shall in respect of
in CMS Appendices I and II. every financial year cause to be laid before both
the Houses of Parliament a statement of the
 Central Asian Mammals Initiative (CAMI) estimated receipts and expenditure of the
– A framework for the conservation of 15 Government of India which is also referred as
migratory mammal species and their habitats in Annual financial statement.
the Central Asian region adopted by CMS in
2014. 28. a

 Global Partnership on Ecological Interim budget


Connectivity – The objective is to ensure that
ecological connectivity is maintained,  It is a short-term financial statement that
enhanced, and restored in places of importance allows for the smooth functioning of the
for migratory species of wild animals. government until a new administration can
present a full budget for the entire fiscal year.
 Global guidelines on light pollution – It
covers marine turtles, seabirds and migratory  It is presented by the outgoing government
shorebirds as endorsed by COP 13. ahead of the Lok Sabha polls.

 New international guidelines focusing on  There is no constitutional provision for an


migratory land birds and bats are currently interim budget.
being developed under CMS.
 Coverage – Estimates for government
 Migratory Aquatic species – To address the expenditure, revenue, fiscal deficit, and
impacts of deep-seabed mineral exploitation on financial performance, but cannot include
migratory species, their prey, and their major policy announcements.
ecosystem.
29. a
 3 new Action Plans for aquatic species –
Atlantic Humpback Dolphin, the Hawksbill Annual Budget Vs. Interim Budget
Turtle and the Angelshark.
Annual Interim
 Wildlife health – A Review of Migration and
budget budget
Wildlife Disease Dynamics, and the Health of
Migratory Species, within the context of ‘One
Health’. Presented in
an election
 Vessel strike – A new resolution on reducing year,
the risk of vessel strikes for marine megafauna. Presented on
typically
February 1st
around
 Amend Convention’s Appendices – The of each year,
February, to
proposals for amendment of CMS Appendices outlines the
keep essential
were discussed and recommended by the government's
Timing government
Committee of the Whole (COW) or countries for financial
operations
adoption. roadmap for
funded until
the entire fiscal
the newly
 Inclusion of 14 migratory species – year (April 1 to
elected
Eurasian lynx, Peruvian pelican, Pallas’s cat, March 31).
government
guanaco, Laulao catfish, Balkan lynx, Lahille’s
presents its full
bottlenose dolphin, harbour porpoise,
budget.
Magellanic plover, bearded vulture, Blackchin
guitarfish, Bull ray, Lusitanian cownose ray and Covers all Focusses
Gilded catfish. aspects of primarily on
government maintaining
27. d
finances, essential
Budget including spending on
revenue ongoing
Scope
 A budget is an estimation of revenue and generation, schemes and
expenses over a specified future period of time. expenditure critical public
allocations and services until
 In the Constitution of India, the term policy the new
‘budget’ is not used. announcement government
s takes charge.

CHENNAI |SALEM| MADURAI | COIMBATORE DELHI | BANGALORE | THIRUVANANTHAPURAM


www.shankariasacademy.com 411
www.iasparliament.com

Due to its  There is also increase in physical infrastructure


transitory in many sectors.
nature, an
Policy Major policy
interim budget
Announcemen pronouncemen
avoids major
ts ts are made in
policy
the annual
pronouncemen
budget.
ts or significant
changes to tax
structures.

Receives less
Parliamentary Faces scrutiny due
scrutiny rigorous to its limited
debate and scope and
analysis temporary
nature.

Usually 2–4 32. b


months, but
Validity remains valid Research and Innovation – Interim Budget
1 year until the new 2024-25
government
presents its  Sunrise domain - A corpus of 1 lakh crore will
full budget. be established with 50-year interest free loan to
encourage the private sector to scale up
30. d research and innovation in sunrise domains.

Schemes for Poverty alleviation  Deep Tech Scheme - A new scheme will be
launched for strengthening deep-tech
 PM-Jan Dhan Yojana - A financial inclusion technologies for defence sector.
program open to Indian citizens, that aims to
expand affordable access to financial services 33. c
such as bank accounts, remittances, credit,
Railway Corridors – Interim Budget 2024-25
insurance and pensions
 3 major economic railway corridor
 PM-SVANidhi – A Special Micro-Credit
programmes will be implemented under PM
Facility launched by Ministry of Housing and
Gati Shakti.
Urban Affairs, for providing affordable loans to
street vendors. o Energy, mineral and cement corridors
 PM-JANMAN Yojana – A government o Port connectivity corridors
scheme that aims to bring tribal communities
into the mainstream. o High traffic density corridors

 PM-Vishwakarma Yojana – A central sector 34. b


scheme launched by the Ministry of Micro,
Small, and Medium Enterprises, which offers Tax proposals in Interim Budget 2024-25
services like market linkage support, skill
training, and incentives for digital transactions  In keeping with the convention, no changes
to artisans and craftspeople engaged in were made relating to taxation.
specified trades.
 Same tax rates for direct and indirect
31. d taxes.

India’s Performance  Import duties were retained.

 There is continuous increase in capital 35. d


expenditure from FY18 till FY25.
Receipts and Expenditure
 FDI inflow has been doubled during the
time interval of 2005-14 to 2014-23.

CHENNAI |SALEM| MADURAI | COIMBATORE DELHI | BANGALORE | THIRUVANANTHAPURAM


www.shankariasacademy.com 412
www.iasparliament.com

Direct taxes

 Achievements - Over the last 10 years, the


direct tax collections have more than tripled
and the return filers swelled to 2.4 times.

 Average processing time of returns has been


reduced from 93 days in 2013-14 to a mere 10
days this year.

 Corporate tax rate - It was decreased from


30% to 22% for existing domestic companies
and to 15% for certain new manufacturing
companies.

 No tax liability for taxpayers with income up to


Rs. 7 lakh under the new tax regime.

 Direct Tax Demands - The Interim Budget


proposes to withdraw outstanding direct tax
demands up to Rs. 25000 pertaining to the
period up to financial year 2009-10 and up to
Rs. 10,000 for financial years 2010-11 to 2014-
15.

Indirect Taxes

 Tax base of GST more than doubled this year.

 The average monthly gross GST collection has


almost doubled to 1.66 lakh crore, this year.

 States’ SGST revenue, including compensation


released to states, in the post-GST period of
2017-18 to 2022-23, has achieved a buoyancy of
1.2

CHENNAI |SALEM| MADURAI | COIMBATORE DELHI | BANGALORE | THIRUVANANTHAPURAM

You might also like